Está en la página 1de 611

Universidad de Santiago de Chile.

Facultad de Ciencia
Departamento de Matematica y C. C.

Calculo Avanzado

Miguel Martinez - Carlos Silva - Emilio Villalobos


Derechos de Autor

Autor: Universidad
c de Santiago de Chile
Se autoriza la reproduccion parcial o
total de esta obra, con fines academicos,
por cualquier forma, medio o procedimiento,
siempre y cuando se incluya la cita
bibliografica del documento.
Agradecimientos
Este texto fue financiado en el marco de los proyectos concursables de
innovacion docente que promueve anualmente la Universidad de Santiago de
Chile a traves de la Vicerrectora Academica por intermedio de la Direccion
de Docencia.
El centro motor que motivo a los autores a emprender tan significativo
desafo fue su compromiso con el proceso de ensenanza aprendizaje que se
lleva semestre a semestre en la Universidad de Santiago, con los estudiantes
de ingeniera quienes tienen el imperativo de mejorar sus aprendizajes y elevar
sus estandares de competencias con la finalidad de que puedan asumir con
propiedad el desafo de sus asignaturas profesionales y de especialidad con
un mayor empoderamiento en el contexto de: teora, practica y aplicaciones
a problemas en las areas de sus distintas especialidades.
En general, cada captulo comienza con una presentacion de definiciones,
principios y teoremas, junto con material ilustrativo. Los problemas resueltos
sirven para ilustrar la teora y suministrar herramientas de analisis de los
principios basicos tan importantes en el aprendizaje activo de los estudiantes.
El gran numero de problemas resueltos y aplicaciones sirven para encauzar el
aprendizaje del material, as como las autoevaluaciones propuestas al fin de
cada unidad. Hemos escogido un enfoque y nivel de profundidad de acuerdo
con lo que se espera del curso de Calculo Avanzado, asignatura que se imparte
durante el tercer semestre del Plan Comun de la Carrera de Ingeniera Civil
de la Facultad de Ingeniera de Universidad de Santiago de Chile.
El objetivo del primera parte de este texto es presentar los conceptos
basicos y las aplicaciones de las series de Fourier, y las funciones integrales,
como asimismo, ilustrar su utilizacion en la resolucion de problemas de ecua-
ciones en derivadas parciales y aplicaciones en el campo de la fsica e inge-
niera.
En la segunda parte se abordan los temas de funciones vectoriales y calcu-
lo diferencial de funciones de dos o mas variables y sus aplicaciones, incluyen-
do aplicaciones e interpretaciones geometricas y fsicas que contribuyan a la
comprension de los estudiantes.
Unido a lo anterior, en la tercera parte se incluyen los temas de integracion
multiple, integral de lnea , superficie y los teoremas de Green, Gauss y Stokes
por sus multiples aplicaciones en los campos de la fsica y ciencias de la
ingeniera
Finalmente, queremos aprovechar la ocasion para expresar nuestro es-
pecial agradecimiento hacia nuestros colegas de la Coordinacion de Calculo
Avanzado que con sus crticas constructivas y opiniones han ayudado a el

i
enriquecimiento del material que conforma este texto. Deseamos tambien
agradecer la participacion directa e indirecta de nuestros estudiantes con los
cuales pusimos a prueba el material que se estaba generando incluyendolos
en la pagina web de la asignatura de Calculo Avanzado.
Agradecemos tambien muy especialmente la colaboracion del profesor
Omar Ramos por la confeccion de diagramas, figuras e imagenes de fun-
ciones bi y tridimensionales que ilustran conceptos y problemas. Tambien
se encargo de la version Latex de los distintos archivos que conforman el
manuscrito del texto.
No obstante lo anterior, la responsabilidad por los eventuales errores o
inexactitudes que se puedan encontrar en el texto corresponde a los autores,
quienes estaran atentos para recibir cualquier comentario o sugerencia que
permita mejorar su contenido en las siguientes direcciones:
miguel.martinez@usach.cl, carlos.silva.c@usach.cl,emilio.villalobos@usach.cl.

Los Autores:
Miguel Martnez
Concha Carlos Silva Cornejo
Emilio Villalobos Marn

ii
Prefacio
El material presentado en el texto contiene los temas tratados en el curso
de Calculo Avanzado, asignatura semestral para las carreras de Ingeniera
de la Universidad de Santiago de Chile, correspondiente al area de Ciencias
Basicas, tiene por prerrequisitos las asignaturas de Calculo I y Calculo II de
primer ano. Proporciona los conceptos, habilidades y tecnicas que permiten
adquirir las competencias matematicas alineadas con el perfil de competen-
cias profesionales, necesarias para cursar con exito las asignaturas de ciencias
basicas de la ingeniera e ingeniera aplicada. Los temas tratados por el texto
y en el orden de aparicion son los siguientes: Series e Integrales de Fourier,
que forma parte de este temario porque por razones de tiempo no se incluye
en el Captulo de Series de primer ano, este tema resulta necesario en la
formacion basica de alumnos de ingeniera sobre todo cuando necesiten re-
solver ecuaciones diferenciales parciales usando el metodo de separacion de
variables o bien en aplicaciones en el campo de la ingeniera. Este tema bien
podra formar parte de un texto de ecuaciones diferenciales. El tema de fun-
ciones vectoriales de una variable real trata la importancia de la derivada de
este tipo de funciones, interpretacion geometrica y analtica, y su aplicacion
a problemas de movimiento, comportamiento de curvas, especialmente en lo
que tiene que ver con caractersticas geometricas de ellas. Las funciones es-
calares son tratadas en detalle, se analiza el concepto de lmite y continuidad
considerando funciones de dos variables, generalizando en aquellos casos que
lo amerita, se ve el tema de la diferenciacion con todas sus potencialidades
que garantizan la derivacion tanto la derivacion parcial, como la derivacion
direccional y derivacion implcita, este tema termina con maximos y mni-
mos y problemas aplicados de optimizacion. Este ultimo captulo tratan los
temas de integracion, integrales dobles y triples en coordenadas cartesianas
y generalizando con cambios de coordenadas, integral de lnea para funciones
escalares y vectoriales, propiedades de los campos gradientes y el teorema de
Green; integral de superficie para funciones escalares y vectoriales finalizan-
do con el estudio de los teoremas de Gauss y Stokes. Los temas tratados de
acuerdo con los objetivos generales los podemos describir como sigue: Series
e integrales de Fourier
i) Analizar los conceptos asociados a la definicion de la Serie de Fourier, sus
propiedades y procedimientos de calculo, y aplicarlos a la resolucion de
problemas de ingeniera.
ii) Formular el concepto de Integral de Fourier, sus propiedades y metodos
de calculo y aplicar esta informacion en la solucion de problemas de
ingeniera.

iii
Funciones vectoriales

i) Analizar el concepto de diferenciacion de funciones vectoriales, sus propiedades,


procedimientos para realizar calculos y aplicarlos a la resolucion de
problemas de Ciencia e Ingeniera

ii) Utilizar los conceptos de vector tangente, normal , binormal, curvatura


y torsion e identidades de Frenet sus propiedades y procedimientos de
calculos para emplearlos en la resolucion de problemas.

Diferenciacion parcial

i) Definir los conceptos de lmite, continuidad y describir las caractersticas


graficas de las funciones de varias variables en IR2 en IR.

ii) Analizar criterios para reconocer y evaluar la diferenciabilidad de una


funcion escalar de varias variables, usar su propiedades, metodos de
calculos para su aplicacion a la resolucion de problemas de Ingeniera.

iii) Generalizar el concepto de diferenciacion para funciones compuestas e


implcitas de varias variables, sus propiedades, metodos de calculos y
su aplicacion a la resolucion de problemas de Ingeniera

iv) Aplicar diferentes metodos para determinar maximo y mnimos de una


funcion de varias variables y utilizarlos en la resolucion de problemas
de optimizacion

Integracion

i) Analizar el concepto de integral doble sus propiedades y procedimientos


de calculo , y su aplicacion a problemas de fsica e ingeniera

ii) Analizar el concepto de integral triple sus propiedades y procedimientos


de calculo, y su aplicacion en problemas de fsica e ingeniera.

iii) Analizar los conceptos de integral de trayectoria e integral de lnea, y


utilizar sus propiedades en la resolucion de problemas matematicos, de
fsica e ingeniera

iv) Analizar los conceptos de integral de superficie, y utilizar sus propiedades


en la resolucion de problemas matematicos, de fsica e ingeniera
Estructura de cada unidad Cada unidad en su desarrollo teorico y
fundamentacion matematica enfatiza los conceptos, los teoremas que
avalan los procedimientos y las tecnicas de resolucion de problemas.

iv
Unido a lo anterior, en cada tema hay una unidad de problemas resuel-
tos, otra de problemas propuestos, algunos con soluciones y una unidad
de aplicaciones a los temas de ingeniera. Finalmente, se incluye tam-
bien un instrumento de autoevaluacion que consiste en un test con
problemas de desarrollo que mide el nivel de las competencias cogniti-
vas alcanzado por el estudiante. Este material pretende ser una fuente
de motivacion que haga que los estudiantes perseveren en sus estudios
y puedan vencer las dificultades de aprendizaje, y alcanzar niveles que
le permitan rapidamente conectarse con los temas en que esta inmerso
un problema y estructurar un a respuesta al problema usando diversas
herramientas matematicas, esto le dara desde luego un plus durante
toda su vida profesional.

v
Indice general

1. Serie de Fourier 1
1.1. Introduccion . . . . . . . . . . . . . . . . . . . . . . . . 1
1.2. Propiedades Generales . . . . . . . . . . . . . . . . . . 2

1.2.1. Lema Elemental . . . . . . . . . . . . . . . . . . 2


1.3. La serie de Fourier de una funcion . . . . . . . . . . . . 4
1.3.1. Coeficientes de Fourier . . . . . . . . . . . . . . 6
1.3.2. Atributos de la funcion . . . . . . . . . . . . . . 7
1.3.3. Convergencia de las series de Fourier . . . . . . 9
1.3.4. La integral de funciones pares e impares . . . . 15

1.3.5. Teorema de las funciones pares y de las impares 16


1.4. Desarrollos llamados de medio rango . . . . . . . . . . 17
1.4.1. Extension impar: . . . . . . . . . . . . . . . . . 17
1.4.2. Extension par . . . . . . . . . . . . . . . . . . . 19
1.5. Diferenciacion e Integracion de la series de Fourier . . . 21
1.5.1. Derivacion . . . . . . . . . . . . . . . . . . . . . 21
1.5.2. Integracion . . . . . . . . . . . . . . . . . . . . 23
1.5.3. Identidad de Parseval . . . . . . . . . . . . . . . 25
1.6. Integral de Fourier . . . . . . . . . . . . . . . . . . . . 27
1.6.1. Criterio de convergencia de la integral de
Fourier . . . . . . . . . . . . . . . . . . . . . . . 29
1.6.2. Integrales de Fourier de cosenos y senos . . . . . 31
1.7. Aplicaciones de Series de Fourier . . . . . . . . . . . . 38

vi
1.7.1. Onda cuadrada alta frecuencia . . . . . . . . . 38
1.7.2. Rectificador de onda completa. . . . . . . . . . 39
1.7.3. Ecuacion de calor unidimensional . . . . . . . . 40
1.7.4. Ecuacion de calor: barra aislada . . . . . . . . 41
1.7.5. Ecuacion de Onda . . . . . . . . . . . . . . . . 42
1.7.6. Deflexion de una viga . . . . . . . . . . . . . . 44
1.8. Problemas Propuestos . . . . . . . . . . . . . . . . . . 45
1.9. Ejercicios Resueltos . . . . . . . . . . . . . . . . . . . . 50
1.9.1. Serie de Fourier . . . . . . . . . . . . . . . . . . 50
1.9.2. Integral de Fourier . . . . . . . . . . . . . . . . 63
1.10. Ejercicios propuestos . . . . . . . . . . . . . . . . . . . 70
1.10.1. Respuestas . . . . . . . . . . . . . . . . . . . . . 74
1.11. Auto evaluaciones . . . . . . . . . . . . . . . . . . . . 77

2. Funciones Vectoriales de una variable real 92


2.1. Introduccion . . . . . . . . . . . . . . . . . . . . . . . . 92
2.2. Funciones Vectoriales . . . . . . . . . . . . . . . . . . 93
2.3. Lmite de una funcion vectorial. . . . . . . . . . . . . . 95
2.3.1. Teorema del lmite . . . . . . . . . . . . . . . . 95
2.3.2. Operaciones con funciones vectoriales . . . . . 98
2.3.3. Teoremas del algebra de lmites . . . . . . . . . 98
2.3.4. Teorema: producto de funcion escalar por vec-
torial . . . . . . . . . . . . . . . . . . . . . . . 99
2.4. Continuidad . . . . . . . . . . . . . . . . . . . . . . . 99
2.5. La Derivada . . . . . . . . . . . . . . . . . . . . . . . 100
2.6. Regularidad de una curva . . . . . . . . . . . . . . . . 102
2.6.1. Camino regular . . . . . . . . . . . . . . . . . . 103
2.6.2. Propiedades de la Derivada . . . . . . . . . . . 103
2.7. Parametrizacion . . . . . . . . . . . . . . . . . . . . . 104
2.7.1. Ejemplos de reparametrizaciones . . . . . . . . 106
2.8. Longitud de Arco . . . . . . . . . . . . . . . . . . . . . 107

vii
2.8.1. La Longitud de Arco como Parametro . . . . . 109
2.8.2. Parametrizacion por Longitud de Arco . . . . . 111
2.9. Trayectorias y curvas . . . . . . . . . . . . . . . . . . . 113
2.10. Vectores Unitarios . . . . . . . . . . . . . . . . . . . . 115
2.10.1. Vector Tangente unitario . . . . . . . . . . . . 115
2.10.2. Vector Normal . . . . . . . . . . . . . . . . . . 116
2.10.3. Vector Binormal . . . . . . . . . . . . . . . . . 116
2.11. Curvatura . . . . . . . . . . . . . . . . . . . . . . . . . 118
2.11.1. Calculo de curvatura usando parametro t cualquiera
en R3 . . . . . . . . . . . . . . . . . . . . . . . 120
2.12. Planos por un punto de la curva . . . . . . . . . . . . . 123
2.12.1. Plano Osculador . . . . . . . . . . . . . . . . . 124
2.12.2. Plano Normal . . . . . . . . . . . . . . . . . . . 124
2.12.3. Plano Rectificante . . . . . . . . . . . . . . . . 124
2.12.4. Recta Tangente . . . . . . . . . . . . . . . . . . 125
2.12.5. Recta Normal . . . . . . . . . . . . . . . . . . . 125
2.12.6. Recta Binormal . . . . . . . . . . . . . . . . . . 125
2.13. Torsion . . . . . . . . . . . . . . . . . . . . . . . . . . . 127
2.13.1. Calculo de la torsion usando parametro t cualquiera
(en R3 ) . . . . . . . . . . . . . . . . . . . . . . 128
2.14. Formulas de Frenet . . . . . . . . . . . . . . . . . . . . 130
2.15. Aplicaciones de Funciones Vectoriales y Curvas . . . . 131
2.15.1. Problemas . . . . . . . . . . . . . . . . . . . . . 132
2.16. Ejercicios resueltos . . . . . . . . . . . . . . . . . . . . 140
2.17. Ejercicios propuestos . . . . . . . . . . . . . . . . . . . 169
2.17.1. Respuestas . . . . . . . . . . . . . . . . . . . . . 173
2.18. Auto Evaluaciones . . . . . . . . . . . . . . . . . . . . 175

viii
3. Funciones de varias variables 189
3.1. Introduccion . . . . . . . . . . . . . . . . . . . . . . . . 189
3.2. Funciones Escalares de Variable Vectorial . . . . . . . . 193
3.2.1. Conceptos Topologicos . . . . . . . . . . . . . . 193
3.2.2. Aspectos Geometrico de las Funciones Escalares 197
3.2.3. Grafica de una Funcion . . . . . . . . . . . . . . 197
3.2.4. Curvas y Superficies de Nivel . . . . . . . . . . 198
3.2.5. Lmite . . . . . . . . . . . . . . . . . . . . . . . 199
3.2.6. Continuidad . . . . . . . . . . . . . . . . . . . . 205
3.2.7. Derivadas Parciales . . . . . . . . . . . . . . . . 207
3.3. Diferenciabilidad en dos variables . . . . . . . . . . . . 209
3.3.1. Derivada Direccional . . . . . . . . . . . . . . . 211
3.3.2. Plano tangente y recta normal . . . . . . . . . . 215
3.3.3. Funcion Compuesta. La Regla de la Cadena. . . 218
3.3.4. Funcion Implcita . . . . . . . . . . . . . . . . . 222
3.3.5. Jacobiano . . . . . . . . . . . . . . . . . . . . . 226
3.3.6. Maximos y Mnimos . . . . . . . . . . . . . . . 228
3.3.7. Extremos Restringidos . . . . . . . . . . . . . . 233
3.4. Problemas Resueltos . . . . . . . . . . . . . . . . . . . 245
3.4.1. Continuidad y diferenciabilidad . . . . . . . . . 245
3.4.2. Regla de la cadena . . . . . . . . . . . . . . . . 248
3.4.3. Derivacion Implcita . . . . . . . . . . . . . . . 251
3.4.4. Plano Tangente a una Superficie . . . . . . . . . 256
3.4.5. Derivadas Direccionales . . . . . . . . . . . . . . 258
3.4.6. Valores extremos . . . . . . . . . . . . . . . . . 260
3.4.7. Multimplicadores de Lagrange para extremos re-
stringidos . . . . . . . . . . . . . . . . . . . . . 264
3.4.8. Aplicacion al calculo de errores . . . . . . . . . 275
3.5. Ejercicios Propuestos . . . . . . . . . . . . . . . . . . 276
3.5.1. Lmites . . . . . . . . . . . . . . . . . . . . . . . 276
3.5.2. Diferenciabilidad, continuidad . . . . . . . . . . 277

ix
3.5.3. Derivadas parciales . . . . . . . . . . . . . . . . 278
3.5.4. Derivadas Direccionales . . . . . . . . . . . . . . 280
3.5.5. Puntos crticos maximos y mnimos . . . . . . . 284
3.6. Aplicaciones Derivada Direccional . . . . . . . . . . . . 286
3.7. Aplicaciones de Maximos y Mnimos . . . . . . . . . . 289
3.7.1. Aplicacion al campo de la mecanica . . . . . . . 289
3.7.2. Aplicaciones a la geometra . . . . . . . . . . . 292
3.7.3. Aplicacion al campo de la economa . . . . . . 297
3.7.4. Problemas Propuestos de Aplicaciones . . . . . 302
3.8. Auto evaluaciones . . . . . . . . . . . . . . . . . . . . . 304

4. Integracion Multiple 315


4.1. Integrales dobles y triples . . . . . . . . . . . . . . . . 315
4.1.1. Integrales Dobles . . . . . . . . . . . . . . . . . 315
4.1.2. Integrales sobre conjuntos acotados de R2 . . . 320
4.1.3. Teorema de Fubini . . . . . . . . . . . . . . . . 322
4.1.4. Areas y Volumenes . . . . . . . . . . . . . . . . 327
4.1.5. Cambio de variable . . . . . . . . . . . . . . . . 329
4.2. Aplicaciones de la integral doble . . . . . . . . . . . . 333
4.2.1. Masa de una region plana de densidad variable. 333
4.2.2. Momentos y centroide de una region plana . . . 334
4.3. Integrales triples . . . . . . . . . . . . . . . . . . . . . 337
4.3.1. Ideas preliminares . . . . . . . . . . . . . . . . . 337
4.3.2. Teorema de Fubini . . . . . . . . . . . . . . . . 337
4.3.3. Teorema de la integral triple (Para dominios mas
generales) . . . . . . . . . . . . . . . . . . . . . 338
4.3.4. Cambio de variable para integrales triples . . . 342
4.3.5. Formula del cambio de variable . . . . . . . . . 345
4.3.6. Masa, Momentos, y Centroide de una Region del
Espacio . . . . . . . . . . . . . . . . . . . . . . 348
4.4. Ejercicios resueltos integrales triples y dobles . . . . . . 350

x
4.4.1. Calculo de integrales dobles en coordenadas rectangu-
lares cartesianas . . . . . . . . . . . . . . . . . . 350
4.4.2. Cambios de orden de Integracion . . . . . . . . 361
4.4.3. Cambios de variables: Coordenadas polares . . . 363
4.4.4. Cambios de variables. Coordenadas curvilneas . 367
4.4.5. Calculo de integrales triples en coordenadas rectangu-
lares cartesianas . . . . . . . . . . . . . . . . . . 373
4.4.6. Coordenadas esfericas . . . . . . . . . . . . . . 377
4.4.7. Coordenadas Cilndricas . . . . . . . . . . . . . 380
4.5. Ejercicios propuestos integrales dobles y triples . . . . . 389
4.5.1. Integrales dobles . . . . . . . . . . . . . . . . . 389
4.5.2. Calculo de Integrales dobles usando transforma-
cion de coordenadas . . . . . . . . . . . . . . . 392
4.5.3. Integrales triples . . . . . . . . . . . . . . . . . 394
4.5.4. Integrales triples iteradas . . . . . . . . . . . . . 394
4.5.5. Integrales triples en coordenadas rectangulares
cartesianas. . . . . . . . . . . . . . . . . . . . . 396
4.5.6. Calcular las integrales dadas usando las coorde-
nadas adecuadas: . . . . . . . . . . . . . . . . . 397
4.5.7. Resolver las integrales usando coordenadas esferi-
cas: . . . . . . . . . . . . . . . . . . . . . . . . . 399
4.6. Aplicaciones integrales dobles y triples . . . . . . . . . 401
4.6.1. Volumenes de cuerpos en el espacio . . . . . . . 401
4.6.2. Area de figuras planas. . . . . . . . . . . . . . . 404
4.6.3. Momentos y centros de masa para placas planas
delgadas . . . . . . . . . . . . . . . . . . . . . . 406
4.6.4. Centroide de figuras geometricas . . . . . . . . . 407
4.6.5. Momentos y Centros de masa de un solido . . . 412
4.6.6. Masa de un solido . . . . . . . . . . . . . . . . . 413
4.6.7. Determinacion del centroide dee un solido . . . 424
4.7. Autoevaluacion Integrales dobles y triples . . . . . . . 426

xi
5. Integral de Linea 436
5.1. Campos vectoriales . . . . . . . . . . . . . . . . . . . . 441
5.2. Cambio de parametrizacion . . . . . . . . . . . . . . . 446
5.2.1. Reparametrizacion . . . . . . . . . . . . . . . . 447
5.3. Independencia de trayectoria . . . . . . . . . . . . . . . 449
5.4. Campos Conservativos . . . . . . . . . . . . . . . . . . 451
5.4.1. Campo gradiente . . . . . . . . . . . . . . . . . 451
5.4.2. Teorema de Green . . . . . . . . . . . . . . . . 456
5.5. Aplicaciones de la integral de trayectoria . . . . . . . . 461
5.5.1. Area de una pared . . . . . . . . . . . . . . . . 465
5.6. Aplicaciones de la integral de lnea . . . . . . . . . . . 467
5.7. Problemas Resueltos . . . . . . . . . . . . . . . . . . . 477
5.7.1. Campo conservativo . . . . . . . . . . . . . . . 481
5.7.2. Teorema de Green . . . . . . . . . . . . . . . . 485
5.8. Problemas propuestos . . . . . . . . . . . . . . . . . . . 492
5.8.1. Integral de trayectoria . . . . . . . . . . . . . . 492
5.8.2. Integral de lnea . . . . . . . . . . . . . . . . . . 493
5.8.3. Campos conservativos . . . . . . . . . . . . . . 494
5.8.4. Teorema de Green . . . . . . . . . . . . . . . . 495
5.9. Autoevaluaciones . . . . . . . . . . . . . . . . . . . . . 496

6. Integrales de superficie 504


6.1. Superficie orientada . . . . . . . . . . . . . . . . . . . . 507
6.1.1. Integral de flujo. . . . . . . . . . . . . . . . . . 508
6.1.2. Superficies Parametrizadas. . . . . . . . . . . . 510
6.1.3. Vector normal a S : . . . . . . . . . . . . . . . . 510
6.1.4. Area de una superficie parametrizada . . . . . . 513
6.1.5. Integral de una funcion escalar sobre una super-
ficie. . . . . . . . . . . . . . . . . . . . . . . . . 516
6.1.6. Integral de Superficie de campos vectoriales . . 517
6.1.7. Aplicacion al campo de la fsica: . . . . . . . . . 519

xii
6.2. Teoremas de Gauss y de Stokes . . . . . . . . . . . . . 519
6.2.1. Divergencia . . . . . . . . . . . . . . . . . . . . 519
6.2.2. Teorema de la divergencia de Gauss. . . . . . . 520
6.2.3. Teorema de Stokes. . . . . . . . . . . . . . . . . 524
6.3. Problemas Resueltos . . . . . . . . . . . . . . . . . . . 525
6.3.1. Integrales de superficie . . . . . . . . . . . . . . 529
6.3.2. Integral de Flujo de un campo vectorial . . . . . 532
6.3.3. Teorema de la divergencia de Gauss . . . . . . . 537
6.3.4. Teorema de Stokes . . . . . . . . . . . . . . . . 543
6.4. Ejercicios Propuestos . . . . . . . . . . . . . . . . . . . 550
6.4.1. Area de una superficie . . . . . . . . . . . . . . 550
6.4.2. Integrales de funciones escalares sobre superficie 553
6.4.3. Integral de Flujo . . . . . . . . . . . . . . . . . 555
6.4.4. Teorema de la divergencia de Gauss . . . . . . . 557
6.4.5. Teorema de Stokes . . . . . . . . . . . . . . . . 561
6.5. Aplicaciones . . . . . . . . . . . . . . . . . . . . . . . . 564
6.5.1. Aplicaciones Integral de Flujo . . . . . . . . . . 564
6.5.2. Aplicacion del teorema de Gauss . . . . . . . . 568
6.5.3. Aplicacion teorema de Stokes . . . . . . . . . . 573
6.5.4. Aplicacion teorema de Green . . . . . . . . . . 576
6.5.5. Aplicaciones al electromagnetismo . . . . . . . 580
6.6. Auto evaluaciones . . . . . . . . . . . . . . . . . . . . . 587

xiii
Captulo 1

Serie de Fourier

En el presente captulo se estudiaran los conceptos basicos , metodos


de calculo de los coeficientes y condiciones de convergencia para repre-
sentar funciones mediante series e integrales de Fourier .

1.1. Introduccion
Las funciones periodicas se presentan frecuentemente en una gran var-
iedad de problemas de fsica e ingeniera, tales como propagacion de
ondas en un medio, conduccion del calor a lo largo de una varilla ,
resonancia nuclear magnetica ,en consecuencia, abordar la solucion de
tales problemas, requiere del estudio de la serie de Fourier.
La serie de Fourier es la representacion de una funcion en terminos
de una serie trigonometrica infinita cuyas bases son las funciones seno
y coseno. Algunas de las ventajas de esta representacion sobre otras
representaciones, tales como, las series de Taylor, son:
a) primero, se puede representar funciones periodicas en terminos de
las bases seno y coseno que tienen diferentes frecuencias;
b) segundo, se puede representar funciones discontinuas en un punto o
seccionalmente continuas en un numero finito de puntos;
c) tercero, permite encontrar la respuesta de un sistema que es pertur-
bado por una funcion periodica, en terminos de una frecuencia funda-
mental y cada una de las frecuencias armonicas.

1
1.2. Propiedades Generales

Para problemas con condiciones de frontera periodicas en el intervalo


L x L, nos preguntamos si es posible expresar una funcion como
una combinacion lineal de funciones seno y coseno de frecuencias cada
vez mayores, como la siguiente serie infinita (conocida como serie de
Fourier de f (x)):

   
X nx nx
f (x) = a0 + (an cos + bn sin ) (1,1,1)
n=1
L L

Obviando la igualdad, vale preguntarse converge esta serie infinita?,que condi-


ciones debe cumplir f para que se de la convergencia?,cuando converge
a f (x)?
Estas preguntas no tienen una respuesta sencilla. Sin embargo, las series
de Fourier normalmente funcionan bastante bien.
Supongamos que f admite desarrollo en serie de Fourier, como se ob-
tienen los coeficientes a0 , an y bn en terminos de f (x) ?. Para responder
esta ultima pregunta necesitaremos del siguiente lema.

1.2.1. Lema Elemental

Lema 1.2.1. i) Si m y n son numeros enteros no negativos distintos,


entonces:
Z L  nx   mx  Z L  nx   mx 
cos cos dx = sin sin dx = 0
L L L L L L
(1.2.1)
ii) Para cualquier par de enteros no negativos m y n,entonces:
Z L  nx   mx 
cos sin dx = 0 (1.2.2)
L L L

iii)Para cualquier entero positivo n, entonces:


Z L  nx  Z L  nx 
2
cos dx = sin2 dx = L (1.2.3)
L L L L

2
Demostracion:
Se prueba integrando directamente: usando la identidad cos cos =
cos( ) + cos( + )
2
Z L
1 L 1 L
   
(n m)x
 nx   mx  Z Z
(n + m)x
i) cos cos dx = cos dx+ cos d
L L L 2 L L 2 L L

  L
1 L (n m) x
= sin
2 (n m) L
L
  L
1 L (n + m) x
+ sin
2 (n + m) L
L
=0

Ademas, si m = 0 y n 6= 0 es facilmente verificable que la integral

es cero.

Z L  nx   mx 
En forma similar se prueba que sin sin dx = 0
L L L
sin( ) + sin( + )
ii) Usando la identidad trigonometrica sin cos =
2

L ZL   ZL  
(n m) x
Z  nx   mx  1 1 (n + m) x
cos sin dx = sin dx + sin dx
L L L 2 L 2 L
L L
 
1 L (n m) x L
= cos |L
2 (n m) L
 
1 L (n + m) x L
sin |L
2 (n + m) L
= 0

A estas formulas integrales se les llama relaciones de ortogonalidad


n  nx   mx o
y diremos que en tal caso el conjunto de las funciones cos , sin
L L
n = 0, 1, 2, ..., y m = 1, 2, ..., son ortogonales en [L, L]
iii) La demostracion queda como ejercicio para el lector, se prueba

3
integrando directamente.En sntesis, se puede puntualizar que:
Z L  
1  nx   mx  0, si m 6= n
cos cos dx = = m,n
L L L L 1, si m = n
Z L  
1  nx   mx  0, si m 6= n
sin sin dx = = m,n
L L L L 1, si m = n
donde m,n se define como el delta de Kroneker.

1 L
Z  nx   mx 
cos sin dx = 0 m, n
L L L L
Z L  nx 
cos dx = 0 m, n ; y
L L
Z L  mx 
sin dx = 0 m, n
L L

1.3. La serie de Fourier de una funcion


Se debe distinguir entre f (x) y su serie de Fourier en el intervalo L
x L:
Serie de Fourier de f (x)

X   nx   nx 
a0 + an cos + bn sin
n=1
L L

La serie trigonometrica puede incluso no converger y si converge, puede


que no lo haga a f (x). Partiendo del supuesto que la serie converge
podramos determinar los coeficientes de Fourier a0 , an y bn , usando
las relaciones de ortogonalidad.
Sea f (x) definida en el intervalo L x L:

X  nx   nx 
f (x) = a0 + an cos + bn sin (1.3.4)
n=1
L L

Integrando la identidad ( 1.3.4) se tiene:


Z L Z L X  Z L  nx  Z L  nx  
f (x)dx = a0 dx+ an cos dx + bn sin dx
L L n=1 L
L L L

4
Como todas las integrales de la derecha valen cero, excepto la primera,
ZL
se deduce de aqu el valor de a0 , suponiendo que la f (x)dx existe, as.
L

ZL
1
a0 = f (x)dx
2L
L
 mx 
Para el calculo de an multiplicamos la identidad ( 1.3.4) por cos
L
e integramos la serie termino a termino, queda
Z L  mx  L Z  mx 
f (x) cos dx = a0 cos dx+
L L L L
X  Z L  nx   mx  Z L  nx   mx  
an cos cos dx + bn sin cos dx =
n=1 L
L L L L L
X
=0+ an Ln,m + 0 = Lam
n=1

Por lo tanto, al evaluar n,m , queda un solo termino:


Z L  mx 
f (x) cos dx = am L,
L L

as el valor de am es
Z L
1  mx 
am = f (x) cos dx, m 1
L L L

Cambiando el ndice libre m por n , en ambos lados de la ecuacion,


queda
1 L
Z  nx 
an = f (x) cos dx, n 1
L L L
 mx 
Ahora, multiplicando ( 1.3.4) por sin e integrando de manera
L
similar y por el lema se tiene
1 L
Z  nx 
bn = f (x) sin dx, n 1
L L L

Hemos determinado los coeficientes a0 , an y bn ,claro que, bajo muchos


supuestos. Estos calculos sugieren la siguiente definicion.

5
1.3.1. Coeficientes de Fourier
Definicion 1.-
i) Sea f una funcion Riemann integrable en [L, L], las constantes

ZL
1



a0 = f (x)dx

2L


ZLL


1  nx  2,1,1
an = f (x) cos dx para n = 1, 2, 3, ...
L ZL L


1 L  nx 


bn = f (x) sin dx para n = 1, 2, 3, ...

L L L

se denominan los coeficientes de Fourier de f en [L, L].


ii) La serie:

X h  nx   nx i
f (x) a0 + an cos + bn sin
n=1
L L

es la serie de Fourier de f en el intervalo [L, L] , cuando los coefi-


cientes estan dados por (2,1,1). Para no hablar de convergencia todava,
escribimos el signoque significa que a la derecha se tiene la serie de
Fourier de f en L x L.

Observese que, la serie de Fourier de f , se puede interpretar como una


generalizacion de una combinacion lineal en una base ortogonal seno,
coseno, que es aplicada a una funcion en lugar de un vector estandar
en Rn .
El siguiente ejemplo ilustra como dada una funcion periodica f (x), de
perodo 2, se calculan los coeficientes de Fourier y expresa la serie
trigonometrica de Fourier correspondiente.

Ejemplo 1: Determinar la serie de Fourier de f (x) = x si x [, ]

6
Solucion: La grafica de la funcion es:

Los coeficientes de Fourier de f en [, ] , son:


1
R
a0 = 2 xdx = 0

1
R  1 x

an =
x cos (nx) dx = n2
cos(nx) + n
sin(nx) = 0

1
R  1 x

bn =
x sin(nx)dx = n2
sin(nx) n
cos(nx)

bn = n2 (1)n+1 n1
Por tanto, la serie de Fourier de f en[, ] es:

X 2
(1)n+1 sin(nx)
n=1
n

1.3.2. Atributos de la funcion


Lo anteriormente expuesto es valido para cierto tipo de funciones, nos
referimos a las funciones f (x) que son seccionalmente continuas.

Definicion 2.- Sea f (x) definida en [a, b]. Entonces f es seccional-


mente continua en [a, b] si:
a) f es continua en [a, b] ,excepto quizas en un numero finito de puntos.
b) Ambos lmxa+ f (x) y lmxb f (x) existen y son finitos.
c) f no es continua en x0 , x0 (a, b) y los lmites lmxx+0 f (x) y lmxx0 f (x)
existen y son finitos .

7
Definicion 3.- f (x) es seccionalmente suave en [a, b] si f y f 0 son

seccionalmente continuas en [a, b] .

1
Ejemplo 2: Muestre que f (x) = x 3 no es seccionalmente suave en
ningun intervalo cerrado que contenga en su interior al cero.

Solucion: En efecto, se tiene que


1 2 1 2
f 0 (x) = x 3 = lmf 0 (x) = lm x 3 = ,
3 x0 x0 3

no existe. Por tanto, la funcion no es seccionalmente suave.


Observacion:
Las funciones seno y coseno, que aparecen como bases en la serie de
2L
Fourier, tienen perodos diferentes los que son iguales a para n 1.
n
Por otra parte, un multiplo entero del perodo de una funcion perodica
es tambien un perodo , podemos afirmar entonces, que 2L es el perodo
comun para las funciones seno y coseno del desarrollo de la serie. Por
lo anterior, la serie de Fourier no solo representa a f en el intervalo
L x L , sino que, proporciona una extension perodica de f en
todos los reales.

8
Ejemplo 3: Encontrar el perodo de la funcion f (x) = 100 cos2 x.

Solucion: Utilizando la identidad trigonometrica cos2 = 12 (1+cos 2)


se tiene
1
f (x) = 100 cos2 x = 100 (1 + cos 2x)
2
luego queda

f (x) = 50 + 50 cos 2x

como el perodo de cos 2x es y una funcion constante tiene cualquier


perodo, entonces f (x) es de perodo .

1.3.3. Convergencia de las series de Fourier


A continuacion vamos a establecer las condiciones de suficiencia que
debe cumplir una funcion f (x) para que pueda ser representada por
medio de una serie de Fourier.

Teorema 1.3.1. Si f (x) es seccionalmente suave en el intervalo


[L, L], entonces la serie de Fourier de f (x) converge. i) A la ex-
tension perodica de f (x), en los puntos que la extension perodica sea
continua. ii) Al promedio de los lmites laterales 12 (f (x+ ) + f (x )) en
los puntos donde la extension perodica tenga una discontinuidad de
salto.

En el siguiente ejemplo, se evalua si la serie de Fourier resultante de


una funcion f (x) en un punto x0 dado converge o no en ese punto

9

0 si 3 x 0
Ejemplo 4: Sea f (x) = .Construir la serie de
x si 0 x 3
Fourier y analizar la convergencia en todo R

Solucion: Representemos la grafica de la funcion

Los coeficientes de la serie de Fourier de f (x),son:


Z3 Z3
1 1 3
a0 = f (x)dx = xdx =
6 6 4
3 0
Z3
1  nx 
an = f (x) cos dx
3 3
3
3

nx
  3
nx
9 cos 3x sin
Z
1  nx  1

3 3
= x cos dx = +

3 3 3 n 2 2 n

0
0
3 3
= (cos(n) 1) = ((1)n 1)
n2 2 n2 2

Z3
1  nx 
bn = f (x) sin dx
3 3
3
 3
1 9 sin nx nx
3

3x cos
Z
1  nx 
3 3
= x sin dx = +

3 3 3 n 2 2 n

0
0
3 3
= cos(n) = (1)n
n n
Por consiguiente, la serie de Fourier la podemos escribir
  nx 
3 X 3 n
 nx  3 n
+ ((1) 1) cos (1) sin
4 n=1 n2 2 3 n 3

10
Tenemos que f es continua en [3, 3] ,por lo tanto su extension perod-
ica es seccionalmente continua en R , con discontinuidad de salto en
los puntos x = 3 6n, n Z
Por lo tanto,
 de acuerdo al teorema la serie converge a
f (x) si x 6= 3 6n
fE (x) = 3 nZ
2
si x = 3 6n
entonces
  nx 
3 3 X 1 n
 nx 
n
fE (x) = + 2 ((1) 1) cos (1) sin
4 n=1 n2 3 n 3

los coeficientes ((1)n 1) son nulos, si n es numero par e iguales a


2, si n es numero impar. Entonces

    nx 
3 6 X 1 (2n 1)x n
f (x) = 2 cos + (1) sin
4 n=1 (2n 1)2 3 6n 3

Al evaluar la convergencia en x0 = 3, punto de discontinuidad de la

funcion, se obtiene


2

3 3 6X 1 2n1
X 1
= 2 (1) = =
2 4 n=1 (2n 1)2 n=1
(2n 1)2 8

Observese que a partir de la convergencia de la serie de Fourier en un


punto se puede inferir la convergencia de la suma de terminos de la
serie resultante.

Definicion 4.- Una suma parcial de la serie de Fourier es una suma


de la forma:
N
X  n   n 
Sn = a0 + an cos x + bn sin x
n=1
L L

Observacion. Al truncar la serie infinita se obtiene un polinomio de


grado n.

11
Ejemplo 5 Sea f (x) = x + , x [, ] . Determine la serie de
Fourier y obtener la grafica de sumas parciales S1 (x), S3 (x), S1 0(x).

Solucion : La grafica de la funcion es

Los coeficientes de Fourier de f en [, ]


Z 
1 x2

1 1
2 2 =

a0 = (x + )dx = + x =
2 2 2 2

a0 =
Z  
1 1 1 x
an = (x + ) cos (nx) dx = cos(nx) + sin(nx)
n2 n

an = 0
Z  
1 1 1 x
bn = (x + ) sin(nx)dx = sen(nx) cos s(nx)
n2 n

bn = n2 (1)n+1

As la serie de Fourier de f (x) es



(1)n+1
 
X sin 2x sin3x
+2 sin(nx) = + 2 sin x + ..
n=1
n 2 3

Para visualizar la convergencia de esta serie grafiquemos algunas de sus


sumas parciales

n
X (1)n
Sn (x) = + 2 sin(kx)
k=1
k

12
Obtengamos S1 :
1
X (1)n
S1 (x) = + 2 sin(kx)
k=1
k

13
Obtengamos S3

3
X (1)n
S3 (x) = + 2 sin(kx)
k=1
k

Finalmente Obtengamos S10


10
X (1)n
S10 = + 2 sin(kx)
k=1
k

A partir de este ejemplo, podemos inferir que para las series de Fourier
las graficas de las sumas parciales son curvas aproximadas de la grafica
de la funcion perodica representada por la serie. Se puede visualizar
ademas, que en la medida que es mayor el numero de terminos de
las sumas parciales estas convergen de mejor forma a la grafica de la
funcion f .

14
1.3.4. La integral de funciones pares e impares

Lema 1.3.1. (de funciones pares e impares) Sea f una funcion in-
tegrable en [L, L] . a) Si f una funcion par en [L, L], entonces
RL RL
f (x)dx = 2 f (x)dx. b) Si f es funcion impar en [L, L], entonces
L 0
RL
f (x)dx = 0.
L

Demostracion
a) f funcion par, entonces f (x) = f (x) x R. Considerando que
f
es par y el cambio de variable t = x se tiene

Z0 Z0 ZL ZL
f (x)dx = f (x)dx = f (t)dt = f (x)dx
L L 0 0

entonces

ZL Z0 ZL ZL
f (x)dx = f (x)dx + f (x)dx = 2 f (x)dx
L L 0 0

b) f funcion impar, entonces f (x) = f (x) x R. Usando este


hecho y el cambio de variable t = x se tiene

Z0 Z0 Z0 LZ

f (x)dx = f (x)dx = f (t)dt = f (x)dx


L L L 0

entonces
ZL Z0 ZL ZL ZL
f (x)dx = f (x)dx + f (x)dx = f (x)dx f (x)dx = 0
L L 0 0 0

lo que demuestra el lema.


A continuacion, vamos a determinar los coeficientes y la serie de Fourier
coseno (o seno) segun corresponda, dada una funcion f par (o impar)
de periodo 2L.

15
1.3.5. Teorema de las funciones pares y de las im-
pares
Teorema 1.3.2. Sea f una funcion integrable en [L, L], a) Si f es
par, la serie de Fourier de f en [L, L] es

X  nx 
a0 + an cos
n=1
L

ZL ZL
1 2  nx 
con coeficientes a0 = f (x)dx y an = f (x) cos dx,
L L L
0 0
se denomina serie de cosenos. b) Si f es impar, la serie de Fourier de
f en [L, L] es
X  nx 
bn sin
n=1
L

2 L
Z  nx 
con coeficiente bn = f (x) sin dx, se denomina serie de
L 0 L
senos.

Demostracion: Se deja al lector, debe aplicar el Lema 1.3.1 en el calculo


de los coeficientes de Fourier.

Ejemplo 6: Calcule la serie de Fourier de f (x) = 1 |x| en 2


x 2.

Solucion: A partir de la grafica de la funcion podemos inferir que la


funcion es par.

Es decir f (x) = 1 |x| = 1 |x| = f (x) x R, luego se tiene


que f es par.
Los coeficientes del desarrollo de Fourier, son:

16
 2

Z2  2
1 1 x
a0 = (1 x)dx = x =0
2 2 2
0 0
Z 2 Z 2 Z 2
2  nx   nx   nx 
an = (1 x) cos dx = cos dx x cos dx
2 0 2 0 2 0 2
"  #2
4 cos nx nx

2
2x sin 2
= 0 +
n2 2 n
0

por consiguiente

0 si n es par
an = 8
(2n1)2 2
si n es impar
As la serie de Fourier de f (x) = 1 |x| es:
 
8X 1 (2n 1)x
cos
2 n=1 (2n 1)2 2

En muchos problemas se tiene la posibilidad de trabajar con series de


senos o series de cosenos. Por ejemplo , al resolver ecuaciones diferen-
ciales parciales de segundo orden aplicando el metodo de separacion de
variables.

1.4. Desarrollos llamados de medio rango


Sea una funcion f seccionalmente continua que esta definida solo en
el semi-intervalo [0, L], queremos obtener el desarrollo de f en serie de
Fourier x [0, L] . Una forma de hacer lo anterior es extender f al
intervalo [L, L] y por supuesto , puede ser hecho de muchas maneras,
sin embargo, dos extensiones son las mas convenientes e importantes.
Construir una extension impar lo que origina una serie de senos o
construir un extension par lo que determina una serie de cosenos.
Estas se denominan desarrollos de medio rango.

1.4.1. Extension impar:


Supongamos que conocemos f (x) solamente para 0 x L, entonces
podemos extenderla como una funcion impar, obteniendo otra funcion

17
denotada fi (x) definida por:

f (x), 0 x L
fi (x) =
f (x), L x 0

como se muestra en la figura adjunta.

Si f (x) es seccionalmente suave en 0 x L, entonces fi (x) es tambien


seccionamente suave y se puede aplicar el teorema de convergencia de
series de Fourier.

La serie de Fourier de fi (x) es


X  nx 
fi (x) = bn sin , LxL
n=1
L

Como estamos interesados solamente en lo que ocurre entre 0 x L.


En esa region f (x) es identica a fi (x) y la serie de Fourier es


X  nx 
f (x) = bn sin , 0xL
n=1
L
Z L
2  nx 
con coeficiente bn = f (x) sin
L 0 L

Ejemplo 7. Sea la funcion f (x) = x en el interior 0 x L.


Obtener el desarrollo de medio rango considerando una extension im-
par.

18
Solucion. Consideremos la extension impar de f (x) en 0 x L,
la grafica de f muestra que la serie de fourier de senos converge a f (x)
en 0 x L. Sin embargo, en x = L hay una discontinuidad de salto,
luego la serie converge a cero aunque f (L) 6= 0.

Z L Z L
2  nx  2  nx  2L
bn = f (x) sin dx = x sin dx = (1)n+1
L 0 L L 0 L n

Por lo tanto, la serie resultante es:



2L X (1)n+1  nx 
x= sin , 0xL
n=1 n L

1.4.2. Extension par


Supongamos ahora que conocemos f (x) solamente para 0 x L
, entonces la extendemos como funcion par, obteniendo otra funcion
denotada fp (x) definida por:

f (x), 0 x L
fp (x) =
f (x), L x 0
como muestra la figura adjunta:
Si f (x) es seccionalmente continua en 0 x L, entonces su extension
par fp (x) lo sera tambien por lo que se puede aplicar el teorema de
convergencia de series de Fourier.
En el intervalo 0 x L, la funcion f (x) es identica a su extension
par. La serie que se obtiene se denomina serie de Fourier de cosenos de
f (x).


X  nx 
a0 + an cos , 0 x L, con coeficientes
n=1
L

19
Z L Z L
1 2  nx 
a0 = f (x)dx y an = f (x) cos dx
L 0 L 0 L

Ejemplo 8: Construir la serie de Fourier de Cosenos de f (x) = x en

0 x L.

Solucion: Por las caractersticas de la extension en lo que concierne


a la continuidad de la funcion tenemos:

X  nx 
x = a0 + an cos , 0xL
n=1
L
L
L
1 L 1 x2
Z Z
1 L
a0 = f (x)dx = xdx = =
L 0 L 0 L 2 0 2
Z L Z L
2  nx  2  nx 
an = f (x) cos dx = f (x) cos dx
L 0 L L 0 L

0 si n par.
an = 4L
n2 2 si n impar.

20
Finalmente, la serie de Fourier coseno de f (x) = x en 0 x L es:
 
L 4L X 1 (2n 1)x
2 cos
2 n=1 (2n 1)2 L

1.5. Diferenciacion e Integracion de la se-


ries de Fourier
1.5.1. Derivacion
Las series infinitas, aun las convergentes no siempre se pueden derivar
termino a termino. Un caso ilustrativo, es el de la funcion f (x) = x
definida para x , cuya serie de Fourier es

X 2(1)n+1
sin(nx)
n=1
n

que converge para < x < , es decir



X 2(1)n+1
x= sin(nx), x ], [
n=1
n

Si diferenciamos, esta serie termino a termino tenemos:



X
2(1)n+1 cos(nx)
n=1

la cual es una serie que no converge en ] , [ , ya que si an =


2(1)n+1 cos(nx) para cada x ] , [, lm an no existe, como no
n

2(1)n+1 cos(nx) no converge
P
ocurre que an 0 ,concluimos que
n=1
para cada x ] , [.
Por otro lado, f 0 (x) = 1 x ] , [. Esto muestra en este caso que
la derivada termino a termino de la serie, no converge a la derivada de
la funcion que representa.
La dificultad se nos presenta cada vez que la serie de Fourier de f (x)
tiene una discontinuidad de salto, la derivacion termino a termino no
esta justificada en estos casos. Sin embargo, podemos aqu considerar el
siguiente teorema que precisa las condiciones para permitir la derivacion
termino a termino.

21
Teorema 1.5.1. Sea f una funcion continua en [L, L] con f (L) =
f (L), si f 0 es seccionalmente suave en [L, L] donde f 00 (x) existe se
tiene.
0
X n h  nx   nx i
f (x) = an sin + bn cos
n=1
L L L

Demostracion.-
Se deja al lector, se sugiere escribir la serie de Fourier de f 0 (x), con-
siderando que esta serie converge a f 0 (x) siempre que f 00 (x) exista. Use
integracion por partes para relacionar los coeficientes de f 0 (x) con los
correspondientes de f (x).

Ejemplo 9. Dada la funcion f (x) = x2 en x , verifique


si la derivada de esta serie existe.

Solucion Claramente se satisface las hipotesis de la proposicion an-


terior. La serie de Fourier de la funcion f (x) en [, ] es:
(Ver Problema 2 en problemas resueltos)

2 X (1)n
f (x) = +4 cos(nx)
3 n=1
n2

Como f 0 (x) = 2x es continua, y existe f 00 (x) = 2 en todo el intervalo,


entonces para < x <

0
X (1)n+1
f (x) = 2x = 4 sin(nx)
n=1
n

Note que este resultado concuerda con lo establecido en el ejemplo 1


del inciso 2.1.

22
1.5.2. Integracion

La precaucion que se tiene para la derivacion termino a termino de la


serie de Fourier no se requiere para el caso de la integracion .

Teorema 1.5.2. Sea f una funcion seccionalmente suave en [L, L]


con serie de Fourier

X h  nx   nx i
f (x) = a0 + an cos + bn sin
n=1
L L

Entonces para cada x [L, L] .

Z x
LX1h  nx    nx  i
f (t)dt = a0 (x+L)+ an sin bn cos (1)n
L n=1 n L L

Demostracion;
Zx
Sea F (x) = f (t)dt a0 x x [L, L] , as definida F es continua
L
en [L, L] , ademas

L
Z ZL
F (L) = f (t)dta0 (L) = a0 L y F (L) = f (t)dta0 L = 2a0 La0 L = a0 L
L L

Por lo cual F (L) = F (L), asimismo F 0 (x) = f (x) a0 x [L, L]


donde f es continua. Entonces podemos asegurar que F 0 es seccional-
mente continua en [L, L] y por el teorema de convergencia tenemos
que

X h  nx   nx i
F (x) = A0 + An cos + Bn sin (1.5.5)
n=1
L L

donde para n 1.

23
Z L  
1 nt
An = F (t) cos dt integrando por partes
L L L
  L Z L  
1 L nt L 0 nt
= F (t) sin F (t) sin dt
L n L L n L L
Z L  
L nt
= 0 (f (t) a0 ) sin dt
n L L
Z L   Z L  
L nt L nt
= f (t) sin dt + a0 sin dt
n L L n L L
L
An = bn
n
donde bn es el coeficiente correspondiente de la serie de Fourier de f en
[L, L].

De manera analoga se tiene que:


Z L  
1 nt L
Bn = F (t) sin dt = an
L L L n

donde an es tambien el correspondiente coeficiente de la serie de Fourier


de f en [L, L].
Por lo tanto, reemplazando en 1.5.5

LX1h  nx   nx i
F (x) = A0 + bn cos + an sin , x [L, L]
n=1 n L L

para A0 tenemos:


X
X
F (L) = a0 L = A0 + An cos(n) = A0 = a0 L An cos(n)
n=1 n=1

finalmente


LX 1
A0 = a0 L + bn cos(n)
n=1 n

24
ahora sustituyendo A0 se tiene


L1X LX1h  nx   nx i
F (x) = a0 L+ bn cos(n)+ bn cos + an sin
n n=1 n=1 n L L

y reemplazando en la igualdad inicial obtenemos lo que afirma el teo-


rema.

Z x
LX1h  nx    nx  i
f (t)dt = a0 (x+L)+ an sin bn cos (1)n
L n=1 n L L

1.5.3. Identidad de Parseval


Sea f una funcion seccionalmente continua en [L, L] y tal que f 0
es tambien seccionalmente continua en [L, L].
Si h
X  nx   nx i
f (x) = a0 + an cos bn sin
n=1
L L

es la serie de Fourier de f , entonces


ZL
1 2 2
X
(an )2 + (bn )2
 
[f (x)] dx = 2 (a0 ) +
L n=1
L

que se conoce como identidad de Parseval

Prueba: La serie de Fourier de f converge a f (x) para cada x del


intervalo [L, L].
X h  nx   nx i
f (x) = a0 + an cos bn sin
n=1
L L

Multiplicando por f (x) se tiene


h
X  nx   nx i
2
f (x) = a0 f (x) + an f (x) cos bn f (x) sin
n=1
L L

25
podemos integrar termino a termino.

Z L Z L X  Z L  nx  Z L  nx 
2
[f (x)] dx = a0 f (x)dx+ an f (x) cos bn f (x) sin
L L n=1 L L L L

de aqu recordando lo que son los coeficientes de una serie de Fourier


se tiene.

ZL
X
2 2
[f (x)] dx = 2 (a0 ) L + L [an an + bn bn ] =
n=1
L

ZL
1 2
X2
(an )2 + (bn )2
 
[f (x)] dx = 2 (a0 ) +
L n=1
L

Observese que la identidad de Parseval, permite inferir la suma de


una serie infinita, dada una funcion f que tiene una representacion de
Fourier para cada x del intervalo [L, L].


x < x <
Ejemplo 10. Sea f (x) = , perodica de perodo
0 x = ,
P 1 2
2. Pruebe que 2
= .
n=1 n 6

Figura 1.1: grafica funcion perodo 2

Solucion: Como f (x) en es una funcion impar se tiene que :


an = 0 para n = 0, 1, 2, ...y

26
Z Z  
1 2 2x cos(nx)
bn = x sin (n) dx = x sin (n) dx =
n 0
0
 2
n
n = 1, 3, 5, ...
= bn =
n2 n = 2, 4, 6, ...

Por tanto

 
n+1 sin(nx) sin x sin 2x sin 3x
X
f (x) 2 (1) =2 + ...
n=1
n 1 2 3

Aplicando la identidad de Parseval

Z  
1 2 1 1 1 1
x dx = 4 2 + 2 + 2 + 2 + ... =
1 2 3 4

Z  
X 1 1 2 1 x3 2
= x dx = =
n=1
n2 4 4 3 6


X 1 2
=
n=1
n2 6

1.6. Integral de Fourier


Las series de Fourier nos proporcionan una herramienta poderosa para
representar funciones perodicas. Luego, es conveniente generalizar este
metodo para incluir funciones no perodicas.
A continuacion en esta seccion vamos a representar una funcion f no
perodica por medio de la integral de Fourier

Definicion.- Si f (x) definida en (, ) es seccionalmente continua


en cada intervalo finito y tiene derivadas por la derecha e izquierda en

27

R
todo punto y tal que |f (x)| dx converge, entonces la integral de

Fourier de f se define como:
Z
[A(w) cos wx + B(w) sin wx] dw
0

donde:

1
Z
A(w) = f (t) cos wtdt

1
Z
B(w) = f (t) sin wtdt

A(w) y B(w) se llaman los coeficientes de la integral de Fourier de f (x).

Ejemplo 11. Encontrar la representacion por medio de la integral de


Fourier de la funcion:

1 , |x| < 1
f (x) =
0 , |x| > 1

Solucion: Primeramente, determinemos la grafica de la funcion

Ahora, calculemos los coeficientes de la Integral de Fourier


Z Z1  
1 sin wu 1 1 sin w
A(w) = f (u) cos wudu = cos wudu = |1 = 2
w w
1

Z Z1
1
B(w) = f (u) sin wudu = sin wudu = 0

1

28
Por lo tanto, la integral de Fourier de f (x) es:
Z
1 2
sin w cos wxdw
w
0

1.6.1. Criterio de convergencia de la integral de


Fourier
Si f (x) es seccionalmente continua en [L, L] L > 0 y tal que

|f (t)| dt existe, entonces la integral de Fourier converge a 12 [f (x+ ) +
R

f (x )] (Promedio de los lmites izquierdo y derecho de f (x)), x donde


0 0
f (x+ ) y f (x ) existen.

Ejemplo 12. Estudie la convergencia de la Integral de Fourier del


ejemplo 11

Solucion Sea f (x) definida en ejemplo 11, debido a que f (x) es sec-
cionalmente suave, la integral de Fourier de f (x) converge a 21 [f (x+ ) +
f (x )] x. De acuerdo con el criterio de convergencia se tiene:

Z 1 si 1 < x < 1
2 sin w 1
cos wxdw = si x = 1
w 2
0 0 si |x| > 1

En particular, una situacion interesante se da cuando x = 0.

Z
2 sin w
cos 0dw = 1 =
w
0
Z
sin w
dw =
w 2
0

Aunque integrales de este tipo no pueden expresarse en terminos de


funciones elementales, suelen presentarse en matematicas aplicadas con
tal frecuencia , que han recibido un nombre especial y se encuentran

29
tabuladas.
En particular sabemos que:

sin w
lm =1
w0 w

y que

Z
sin w
dw =
w 2
0

En el caso de la integral de Fourier, la grafica de la funcion f se obtiene


mediante aproximaciones sucesivas sustituyendo el lmite superior de
la integral por los numeros x. De aqu que la integral

Zz
sin w
cos wxdw
w
0

es una aproximacion de la integral encontrada anteriormente, y por lo


tanto de f (x) .
Supongamos que solo consideramos las frecuencias w < w0 .En este
caso, nos da como aproximacion de f (x)

Zw0
2 cos wx sin w
f (x) dw
w
0

30
Ahora bien,
sin (wx + w) sin(wx w)
cos wx sin w =
2

y, por consiguiente, podemos escribir la ultima integral en la forma

Zw0 Zw0
1 sin w(x + 1) 1 sin w (x 1)
f (x) dw dw
w w
0 0

Consideremos el cambio de variable u = w (x 1) = du = wdx para


la primera y la segunda de estas integrales. Entonces tenemos
w0 Z
(x+1) w0 Z
(x1)
1 sin u 1 sin u
f (x) du du
u u
0 0
1 1
f (x) Si [w0 (x + 1)] Si [w0 (x 1)]

En terminos fsicos, estas curvas describen la salida de un filtro ideal


de pasa baja, que elimina todas las frecuencias superiores w0 cuando
la senal de entrada es un impulso aislado rectangular.

1.6.2. Integrales de Fourier de cosenos y senos


Sea f (x) una funcion definida en [0, ), podemos extender esta funcion
a una funcion par o impar en (, ) y calcular la integral de Fourier
de esta ultima, que resulta ser de coseno y seno respectivamente, lo
cual es completamente analoga a los desarrollos en cosenos y senos de
una funcion definida en un intervalo [0, L] para el caso de las series de
Fourier.


R
Definicion: Sea f definida en [0, ) y sea |f (u)| du convergente,
0
la integral de Fourier en cosenos de f es
Z
A(w) cos(wx)dw
0

donde el coeficiente es:

31
Z
2
A(w) = f (u) cos(wu)du

0

A su vez, la integral de Fourier en senos de f es

Z
B(w) sin(wx)dw
0

donde el coeficiente es:

Z
2
B(w) = f (u) sin(wu)du

0

En cuanto a la convergencia de la integral de Fourier, en este caso,


si f es seccionalmente suave en todo el intervalo [0, ], entonces esta
integral converge a 21 [f (x+ ) + f (x )] en (0, ).


x2 si 0 x 10
Ejemplo 13: Encontrar la integral de Fourier de f (x) = ,
0 si x > 10

si:

a) se considera una extension par de f (x)

b) se considera una extension impar de f (x); y luego

c) en ambos casos, determinar las convergencias de estas integrales .

Solucion: Consideremos la grafica de la funcion

a) Para obtener la integral de Fourier de cosenos, extendemos f como


una funcion par fP definida en toda la recta real, luego:

32
Z Z10
2 2
A(w) = f (u) cos(wu)du = u2 cos(wu)du

0 0

2 Z10
2 u 2
= sin(wu)|10
0 u sin(wu)du
w w
0
 2  10
2 u 2 1 u
= sin(wu) sin(wu) cos(wu)
w w w2 w 0
 
1 200 4 40
= 3 sin 10w + cos 10w
w w w2

Por tanto, la integral de Fourier de cosenos es:

Z   
1 200 4 40
3 sin 10w + 2 cos 10w cos wxdw
w w w
0

Al aplicar el criterio de convergencia obtenemos:

Z   
1 200 4 40
3 sin 10w + 2 cos 10w cos wxdw
w w w
0
2

x si 0 < x < 10
0 si x > 10

=

0 si x=0
50 si x = 10

33
b) Para obtener la integral de Fourier de senos, extendemos f como
una funcion impar fI definida en toda la recta real.

Z Z10
1 2
B(w) = f (t) sin wtdt = u2 sin wudu

0

 2
10 Z10
2 u cos wu + 2 u cos wudu
=
w 0 w
0
 2  10
2 u 2 1 u
= cos wu + cos wu + sin wu
w w w2 w 0
2
 
2 10 2 20 2
= cos 10w + 3 cos 10w + 2 sin 10w 3
w w w w

entonces la integral de Fourier de senos es:

Z   
1 200 4 40 4
+ 3 cos 10w + 2 sin 10w 3 sin wxdw
w w w w
0

Finalmente, al aplicar el criterio de convergencia obtenemos:

Z   
1 200 4 40 4
+ 3 cos 10w + 2 sin 10w 3 sin wxdw
w w w w
0
2

x si 0 < x < 10
0 si x > 10

=

0 si x=0
50 si x = 10

Ejemplo 14: Encontrar la integral de Fourier de f (x) = eax si

x > 0 y a es una constante tal que a > 0, considerando una extension


a) par de f.
b) impar de f.
c) en ambos casos, determinar las convergencias de estas integrales.
Solucion
Extension Par

34
Extension impar

a) Puesto que f es par , es decir f (x) = f (x) x R se tiene

Z
f (x) = A(w) cos(wx)dw
0

donde el coeficiente es:

Z
2
A(w) = eau cos(wu)du

0

Integrando por partes se tiene

Z
a h  w iR
eau cos(wu)du = lm e au
senwu + cos wu
a2 + w2 R a 0
0
a
=
a2 + w2

35
Por consiguiente,

2 a
A (w) =
a + w2
2

Sustituyendo esta expresion se obtiene:

Z
2a cos(wx)
dw
a2 + w 2
0

para x > 0, a > 0.

Finalmente, como la funcion es continua x > 0, la integral converge


a f (x) , entonces
Z
ax 2a cos(wx)
f (x) = e = dw
a2 + w 2
0
Z
cos(wx) eax
= dw =
a2 + w 2 2a
0

b) Puesto que f es impar , es decir f (x) = f (x) x R se tiene

Z
f (x) = B(w) sin(wx)dw
0

donde el coeficiente es:

Z
2
B(w) = eau sin(wu)du

0

Integrando por partes se tiene

Z
1 R
eau sin(wu)du =
 au
lm e (asenwu w cos wu) 0
a2 + w2 R
0
w
=
a2 + w2

36
Por consiguiente,

2 w
B (w) =
a + w2
2

Sustituyendo esta expresion se obtiene:

Z
2 w sin(wx)
f (x) = eax = dw
a2 + w 2
0

para x > 0, a > 0.


Estos ejemplos ilustran como puede aplicarse la representacion de la
integral de Fourier para evaluar integrales.

37
1.7. Aplicaciones de Series de Fourier
Para dar una vision del uso de las series e integrales de Fourier, se for-
mularan, analizaran y resolveran problemas de sistemas fsicos sujetos
a perturbaciones periodicas y no periodicas.

1.7.1. Onda cuadrada alta frecuencia


Una aplicacion simple de la Serie de Fourier la podemos encontrar
en el analisis de circuitos electronicos que son disenados para manejar
pulsos variables agudos, tales como, una onda cuadrada o un diente
de sierra. Supongamos que una onda cuadrada esta definida por la
funcion:

0, < x < 0
f (x) =
h, 0<x<

Encuentre la serie de Fourier que representa esta senal.


Solucion
Los coeficientes de Fourier son:
Z
1 h
a0 = hdt =
2 0 2
Z
1
an = h cos ntdt = 0, n 1
0
Z
1 h
bn = h sin ntdt = (1 cos n)
0 n
 2h 2h
n
, n impar = bn = (2n1)
bn =
0; n par
As la serie resultante es:
 
X 2h h sin x sin 3x sin 5x
f (x) = sin (2n 1) x = + + + + ...
n=1
(2n 1) 2 1 3 5

38
Es importante decir que el primer termino representa el promedio de
f (x) sobre el intervalo [, ] y que todos los terminos en base coseno
se anulan. Ademas f (x) h2 es una funcion impar, luego ,tenemos
una serie de fourier solo con base seno. Por otra parte, los coficientes
bn decrecen inversamente proporcional con n. Fisicamente esto signifi-
ca que la onda cuadrada debe contener muchos componentes de alta
frecuencia. Si el aparato electronico no deja pasar estos componentes,
la onda cuadrada resultante emerge mas o menos redondeada.

1.7.2. Rectificador de onda completa.


Consideremos ahora la salida de un rectificador de onda completa, que
produce corriente continua pulsante como muestra la figura. El rectifi-
cador se puede modelar como un dispositivo que se alimenta con una
onda senoidal ,que deja pasar los los pulsos positivos, e invierte los
pulsos negativos. Esto produce:


sin x, 0 < x <
f (x) =
sin x, < x < 0
Encuentre la serie de Fourier que respresenta esta senal
Solucion
Puesto que f (x) es una funcion par, es decir f (x) = f (x), la serie
de fourier sera cosenoidal

Z 0 Z  Z
1 2 2
a0 = sin td(t) + sin td(t) = sin td(t) =
2 2 0
Z 0
2
an = sin t cos ntd(t), n 1
0
 2 2
n2 1
, n par = an = 1 4n241
an =
0, n impar
bn = 0, n

39
Por lo tanto, la serie resultante es:


2 4X 1
f (x) = 2
cos (2nx)
n=1 (4n 1)

La frecuencia de oscilacion mas baja es 2.Las componentes de alta


frecuencia decaen inversamente con n2 , lo que muestra que el rectifi-
cador de onda completa hace un buen trabajo para producir un modelo
aproximado de la corriente continua.

1.7.3. Ecuacion de calor unidimensional

El flujo unidimensional de calor en un cuerpo material homogeneo


2 u(x,t)
esta modelado por la ecuacion c2 x 2 = u(x,t)
t
donde u(x, t) es
2
la temperatura del cuerpo y c = 2 la constante de difusion del calor.
Si se considera que 0 < x < 3 y t > 0, y que las temperaturas en
la fronteras son u(0, t) = u(3, t) = 0, limu(x, t) < , entonces la
x0
solucion general de este problema esta dado por:


X 2 2 t nx
u (x, t) = Cn e2n sin , 0<x<3y t>0
n=1
3

Encontrar la temperatura de la barra , si la temperatura inicial es


u(x, 0) = 25o C , 0 < x < 3 .
Solucion:
Evaluemos la solucion general para t = 0, lo que produce:


X nx
u (x, 0) = 25 = Cn sin , 0<x<3
n=1
3

Se obtiene una serie de Fourier seno. As, para determinar


 la constante
25 0<x<3
Cn se debe construir una extension impar f (x) = .
25 3 < x < 0

40
Podemos encontrar entonces:
2 L 2 3
Z Z
nx nx
Cn = f (x) sin dx = 25 sin dx
L 0 L 3 0 3
 3
50 3 nx 50 (1 cos n)
Cn = cos =
3 n 3 0 n

De modo, que la temperatura en la barra queda


X 50 (1 cos n) 2 2 t nx
u (x, t) = e2n sin , 0<x<3y t>0
n=1
n 3

Este problema ilustra la importancia de la serie de Fourier para re-


solver problemas de aplicacion modelados por ecuaciones diferenciales
parciales de segundo orden.

1.7.4. Ecuacion de calor: barra aislada


El flujo unidimensional de calor en un cuerpo material homogeneo
2 u(x,t)
esta modelado por la ecuacion c2 x 2 = u(x,t)
t
donde u(x, t) es
2
la temperatura del cuerpo y c la constante de difusion del calor.
En el caso de una barra aislada, que se prolonga hacia el infinito en
ambos sentidos, la solucion general esta dada por
Z
2 2
u(x, t) = (A (w) cos(wx)+B (w) sin(wx) ) ec w t dw. Si se aplica la
0
condicion inicial u(x, 0) = f (x) , < x < ,donde
R f (x) es la tem-
peratura inicial, se obtiene que u(x, 0) = f (x) = 0 (A (w) cos(wx) +
BZ (w) sin(wx) ) dw es una integral de Fourier con coeficientes A (w) =
1 1
Z
f (v) cos(wv) dv y B (w) = f (v) sin(wv) dv

Determine la integral de Fourier, si la funcion temperatura inicial es
2
f (x) = ex /2 ; < x < , y la solucion general de esta ecuacion.

Solucion:
Z
Como f es una funcion par se tiene Ip = A (w) cos(wx) dw, con
0
, y B (w) = 0 luego

41
Z Z
2 v 2 /2 2 0 2 /2
A (w) = e cos(wv) dv = A (w) = vev sin(wv) dv
0 0

Integrando por partes se tiene


 Z 
0 2 v 2 /2 v 2 /2
A (w) = e sin(wv) + w e sin(wv) dv
0 0

Evaluando la integral y resolviendo EDO(1)

0 2h i 0
A (w) = 0 + w( A (w) = A (w) = wA (w)
2 0
w2 /2
A (w) = Ce , C constante

Luego la integral de Fourier es:


Z
x2 /2 2 /2
e =C ew cos(wx) dw
0

Por tanto, la solucion general queda:


Z
2 /2 2 w2 t
u(x, t) = C (ew cos(wx)) ec dw
0

Este problema ilustra la importancia de la Integral de Fourier para


resolver problemas de aplicacion modelados por ecuaciones de difusion
del calor.

1.7.5. Ecuacion de Onda


Una onda unidimensional que se desplaza en una cuerda elastica ho-
2 u(x,t) 2
mogenea, esta modelado por la ecuacion c2 x 2 = u(x,t)
t2
donde
u(x, t) es el desplazamiento de la cuerda desde el eje x en el tiempo t
y c2 la constante la rapidez de la onda en el medio.
Si los extremos de la cuerda estan fijos en x = 0, x = L , t > 0, es
decir que las condiciones de frontera son u(0, t) = u(L, t) = 0 , entonces
la solucion general de este problema esta dado por:

X nct nct nx
u (x, t) = (An cos + Bn sin ) sin , 0<x<Ly t>0
n=1
L L L

42
Considere que la forma inicial de la cuerda esta dado por f (x) , es
decir u (x, 0) = f (x) , y que la velocidad inicial de la cuerda es cero,
u (x, t)
es decir = 0. Encontrar el desplazamiento u (x, t) de la cuerda
t
en un tiempo posterior.
Solucion.
Determinemos las constantes An y Bn de la solucion general aplicando
las condiciones iniciales.
u (x, t)
Para satisfacer la condicion = 0 , sera necesario derivar la
t
solucion general, entonces

X nc nct nct nx
ut (x, t) = (An sin + Bn cos ) sin
n=1
L L L L

X nc nx
ut (x, t) = Bn sin Bn = 0 n
n=1
L L

De manera que la solucion general se reduce a


X nct nx
u (x, t) = An cos sin ,0 < x < L y t > 0
n=1
L L

Ahora, apliquemos la condicion u (x, 0) = f (x) , para determinar la


constante An . Esto da como resultado


X nx
u (x, 0) = f (x) = An sen ,0 < x < L y t > 0
n=1
L

que corresponde a una serie de Fourier senoidal. As, es necesario con-
f (x) si 0 < x < L
siderar una extension impar de la funcion dada fi (x) = ,
f (x) si L < x < 0
de este modo el coeficiente queda
Z L
2 nx
An = f (x) sin dx
L 0 L
El resultado final es

2 L
Z 
X nx nt nx
u (x, t) = f (x) sin dx cos sin ,0 < x < L y t > 0
n=1
L 0 L L L

43
1.7.6. Deflexion de una viga

Una viga de longitud L , esta soportada desde sus extremos como mues-
tra la figura adjunta . Sobre la viga actua una carga uniformemente dis-
tribuida q por unidad de longitud y su deflexion esta dada por y (x) .
Si se escoge la direccion del eje y apuntando hacia abajo, como indica
la figura, se sabe que la funcion y (x) satisface la ecuacion:

d4 y 1
4
= q (x)
dx EI
donde q (x) es la carga por unidad de longitud en el punto x, I es
el momento de inercia y E el modulo de elasticidad de la viga. Si en
nuestro caso estas tres cantidades son constantes encuente la deflexion
y (x) de la viga.
Solucion.
Puesto que la funcion y (x) debe se nula en los extremos x = 0 y x = L,
la podemos representar mediante una serie de Fourier de senos.


X nx
y (x) = bn sin , x [0, L]
n=1
L

Si suponemos que y (x) es una funcion continua , con derivadas contin-


uas hasta el cuarto orden en [0, L] , entonces

X nx 4 nx
y (4) (x) = bn sin , x [0, L]
n=1
L L

A su vez la carga distrribuida por unidad de longitud q (x) = q, tambien


puede ser desarrollada en serie de Fourier de senos

X nx
q= qn sin , x [0, L]
n=1
L

44
4

1
RL nx

n
n = impar
de donde qn = L 0
q sin L
dx =
0 n = par
Sustituyendo ambas series de Fourier en la ecuacion diferencial


X nx 2 nx X nx
bn sin = qn sin , x [0, L]
n=1
L L n=1
L

Comparando los coeficientes de ambas serie queda


 4  4qL4 1
nx 2

n
n = impar n = impar
L
bn = = bn EI5 n5
0 n = par 0 n = par
Por tanto la deflexion queda determinada por

4qL4 X 1 (2n 1)x
y (x) = 5 5
sin , x [0, L]
EI n=1 (2n 1) L

1.8. Problemas Propuestos


Rectificador media onda
La funcion adjunta sirve para modelar la salida de un rectificador de
media onda:

sin x, 0 x
f (x) =
0, x 0
a) Represente graficamente la senal de salida si esta se extiende peri-
odicamente con periodo 2.
b) Determine la serie de Fourier que la representa.

Solucion:


1 1 2X 1
f (x) = + sin x cos (2nx)
2 n=1 (4n2 1)
Onda triangular
Una onda triangular se representa por la funcion:

x, < x < 0
f (x) =
x, 0 < x < h

45
a) Represente graficamente la funcion.
b) Represente f(x) mediante una serie de Fourier.
c) Estudie la convergencia de la serie en x = , x = 0, y x =

X 1 2
d) Muestre que: =
n=1
(2n 1)2 8

Solucion:

b)

4 X cos (2n 1) x
f (x) =
2 n=1 (2n 1)2
Conduccion del calor.
Consideremos una varilla delgada, aislada, situada a lo largo del eje
x, desde x = 0 hasta x = a,y supongamos que la conduccion de calor
desde la varilla hacia el exterior se da solamente por los extremos de
ella, los cuales se mantienen a temperatura cero. En fsica se muestra
que si en tiempo t = 0 la temperatura u a lo largo de la varilla es igual
a u(x, 0) = bn sin nx, donde bn = cte y n Z+ , entonces para el tiempo
2
t > 0 la temperatura es igual a u(x, t) = bn (sin nx) en t , donde > 0
es una constante positiva. Asimismo, hay un principo de superposicion
que nos permite anadir los efectos de diferentes distribuciones iniciales
de temperatura. Por lo tanto, si la temperatura inicial es:


X
u(x, 0) = f (x) = bn sin nx
n=1

entonces en tiempo t > 0, se tiene:



2t
X
u(x, t) = bn (sin nx) en para 0 x a
n=1

De acuerdo con todo esto, hallar la temperatura para t > 0 para las
siguientes temperaturas iniciales dadas.
a) u(x, 0) = f (x) = 3 sin x + 5 sin 2x. Que tipo de extension de f (x)se
requiere en este caso?
b) u(x, 0) = f (x) = ex sin x.Que tipo de extension de f (x)se requiere
en este caso?

46
Soluciones.

a)
u(x, t) = f (x) = 3 sin xet + 5 sin 2xe4t

b)

4 X n  2
n1
sin nx en t

u(x, t) = 2
(1) e 1
n=1 n + 4

Valor de la raz media cuadratica

Las series de fourier se constituyen en una herramienta poderosa en el


analisis del comportamiento de los sistemas fsicos sujetos a pertuba-
ciones periodicas f (t).
El valor de la raz media cuadratica o RMC de una funcion f (t), sobre
un intervalo (a, b) ,se define como:

sR
b
a
f 2 (t) dt
hf (t)i =
ba

a) Sea f (t) una funcion definida x [a, b] , con un perodo funda-

mental T = b a. Pruebe que aplicando la identidad de Parseval el


valor RMC se reduce a la formula:

v
u
u
2 1X 2
hf (t)i = a0 +
t [a + b2n ]
2 n=1 n

b) Determine RMC de f (t) = E sin t, con E y constantes positi-


vas.
Solucion:

47
2
b) El perodo fundamental de la funcion f (t) = E sin t, es
.
Entonces el valor RMC de f (t) es:

s
Z 2
1 E
hf (t)i = E 2 sin2 (t) dt =
(2/) 0 2

Cuerda vibrante. Extremos fijos


Un cuerda vibra libremente con ambos estremos fijos en x = 0 y x = L.
a) Si su movimiento esta descrito por la ecuacion de onda:

2 u (x, t) 2
2 u (x, t)
= v
t2 x2

con las condiciones iniciales:

u (x, 0)
u (x, t) = f (x) y = g(x)
t

Suponga que la solucion de esta ecuacion es una serie de Fourier de la


forma:

X nx
u(x, t) = bn (t) sin( )
n=1
L

sustituya esta solucion en la ecuacion anterior y determine los coefi-


cientes b (t) .
b) Considere la presencia de un medio resistivo que amortigua las vi-
braciones de acuerdo con la ecuacion

2 u (x, t) 2
2 u (x, t) u (x, t)
2
=v 2
k
t x t

Suponga que rige la solucion anterior con las mismas condiciones ini-
ciales y nuevamente determine el coeficiente b (t) , suponiendo que el
k 2
2
amortiguamiento es pequeno, es decir n

L
2
>0
c) Repita los calculos pero suponiendo que el amortiguamiento es grande
n 2 k 2
 
es decir L
2
< 0.
Soluciones:

48
a)
nt nt
bn (t) = An cos( ) + Bn sin( )
L L
2 L
Z  nx  Z L
2  nx 
An = f (x) sin dx, Bn = g (x) sin dx
L 0 L n 0 L

b)
k
bn (t) = e 2 t (An cos(n t) + Bn sin(n t))
2 L
Z  nx  Z L
2  nx  k
An = f (x) sin dx, Bn = g (x) sin dx + An ,
L 0 L Ln 0 L 2n
 n 2  k 2
2
n = >0
L 2

c)
k
bn (t) = e 2 t (An cosh(n t) + Bn sinh(n t))
2 L
Z  nx  Z L
2  nx  k
An = f (x) sin dx, Bn = g (x) sin dx + An
L 0 L Ln 0 L 2n
 n 2  k 2
2
donde, n = <0
L 2

Distribucion de temperatura en un disco

En una placa circular de radio = 1, cuyas secciones superior e inferior


estan aisladas, se mantiene la mitad de su periferia superior a una
temperatura constante T1 y la otra mitad a una temperatura constante
T2 .Encontrar la temperatura de la placa en condiciones estacionarias.
a) La ecuacion de difusion del calor, en coordenadas polares (, ) ,en
2 2
condiciones estacionarias esta dada por 2 + 1

+ 12 2 = 0,donde
(, ) es la funcion temperatura. Suponga que (, ) ,se puede sepa-
rar como (, ) = M () N ()y pruebe que la ecuacion se transforma
2 00 0 00
en MM + MM = NN .
b) A partir del resultado anterior , haga cada lado de la ecuacion igual
a 2 y encuentre las EDO(2)

N () + 2 N () = 0

49
2 M () + M () + M () = 0

b) Pruebe que N () = A1 cos + A2 sin y M () = B1 +


B2 son soluciones de las correspondientes ecuaciones anteriores.
c) Pruebe que la solucion general es


X T1 + T2 (T T ) (1 cos n) n
(, ) = M () N () = senn
n=1
2 n

1.9. Ejercicios Resueltos


Mediante la inclusion de ejercicios resueltos se espera que
los estudiantes tengan oportunidad de movilizar sus capaci-
dades para buscar, analizar, procesar, representar y comu-
nicar diferentes tipos de informacion, decodificando y tra-
duciendo la informacion contenida en las funciones, graficos,
series de Fourier, integrales de Fourier y sus propiedades.

1.9.1. Serie de Fourier


Problema 1

Sea f (x) periodica de perodo 2 dada por

1

2
x, 0 x 1
f (x) =
x 23 , 1 x 2

a) determinar su serie de Fourier


b) estudie la convergencia de la serie en x0 =
Solucion
La serie de Fourier en este intervalo es

X
a0 + (an cos (nx) + bn sin n (nx)), x [0, 2]
n=1

con coeficientes

50
1
R2 1
R1 R2
a0 = 2
f (x) dx = 2
( 12 x)dx + 1
2
(x 23 )dx
0 0 1
2 1
2
x2 3
  
1 1 x 1
a0 = 2
x + 2
x =0
2 2 0 2 2 1
R2
an = f (x) cos nxdx
0

R1 1
R2
x cos nxdx + (x 23 ) cos nxdx

an = 2
0 1

Integrando por partes, se tiene


 1  1
1 cos nx x sin nx
an = sin nx + +
2n 0 (n)2 n 0
 2  2
cos nx x sin nx 3
+ + sin nx
(n)2 n 1
2n 0

2 (1 (1)n )
an =
(n)2
R2
bn = f (x) sin nxdx
0

R1 1
R2
x sin nxdx + (x 23 ) sin nxdx

bn = 2
0 1

Integrando por partes, se tiene


 1  1
1 sin nx x cos nx
bn = cos nx +
2n 0 (n)2 n 0
 2  2
sin nx x cos nx 3
+ + cos nx = 0
(n)2 n 1
2n 1

La serie de Fourier de f en [0, 2] es



X (1 (1)n ) 4X 1
2 cos nx = 2 cos ((2n 1) x)
n=1
(n)2 n=1 (2n 1)2

b) Como f es continua ,entonces la serie en x = converge a


1 7
f () = f (4 ) = (4 ) =
2 2
51
Problema 2

a) Desarrollar en serie de Fourier la funcion periodica de perodo 2,


definida por:

f (x) = x2 , x

b) A partir del resultado obtenido calcular la suma de:


X 1
n=1
n2


X 1
c) Determine la convergencia de la serie
n=1
n4
Solucion:
a) La funcion f es par por lo cual obtendremos una serie de cosenos,
que tiene la forma:
X
a0 + an cos (nx)
n=1

1
R 1
R 1
h 3 i
x 2
a0 =
f (x)dx =
x2 dx = 3
= 3
0 0 0

2
R 2
R
an =
f (x) cos(nx)dx =
x2 cos(nx)dx
0 0
h i
x2 sin(nx) 2x cos(nx) 4 cos(n) 4(1)n
an = n
+ n2
= n2
= n2
0
Luego, la serie de Fourier de f en [, ]es:

2 X (1)n
+4 cos (nx)
3 n=1
n2

Como la funcion es continua en R ,tenemos:



2 X (1)n
x2 = +4 2
cos (nx) , x R
3 n=1
n

52
b) La serie numerica se puede obtener poniendo x = y f () = 2 ,
2
 
2 1 1 1
= 4 2 2 2 ...
3 1 2 3

de donde
2 2
 
X 1 1
= =
n=1
n2 4 3 6

c) Como la funcion f es seccionalmente suave para x y


f () = f () se cumplen las condiciones de suficiencia de la identidad
de Parseval, entonces:
Z  2 2 X  2
1  2 2 4 (1)n
x dx = 2 + =
3 n=1
n2

 
1 x5 2 4 X 16
= + =
5 9 n=1
n4

X 1 4
=
n=1
n2 90

Problema 3

Sea f (x) = |x| + 1, 1 x 1, la funcion periodica de perodo 2,


determinar:
a) Su serie de Fourier
b) La convergencia de la serie:


X 1
n=1
(2n 1)2

X 1
c) La convergencia de la serie
n=1
(2n 1)4
Solucion
a) f (x) = |x| + 1 es funcion par, con semiperodo L = 1, entonces
tenemos una serie coseno, que tiene la forma:

X
S (x) = a0 + an cos (nx)
n=1

53
Con coeficientes
R1 R1
a0 = 12 f (x)dx = 12 (x + 1) dx = 3
2
0 0
R1 R1
an = 2 f (x) cos(nx)dx = 2 (x + 1) cos(nx)dx
0 0
h i1 h i1
an = 2 sin(nx)
n
+ 2 x sin(nx)
n
+ 2 cos(nx)
(n)2
0 0
h i1
2 cos(nx) 2((1)n 1)
an = (n)2 = (n)2
0

0; si n par
an = 4 ; si n impar
(n)2

Por consiguiente, la serie de Fourier de f en [1, 1] es:



3 4X 1
S (x) = 2 cos ((2n 1) x)
2 n=1 (2n 1)2

b) Como la funcion es continua en R ,considerando el valor x = 0,se


obtiene por el teorema de la convergencia puntual:

3 4 X 1
+ 2 = f (0) = 1
2 n=1 (2n 1)2

X 1 2
=
n=1
(2n 1)2 8

c) Como la funcion f es seccionalmente suave para 1 x 1 y


f (1) = f (1) se cumplen las condiciones de suficiencia de la identidad
de Parseval, entonces:
Z1  2 X  2
2 3 4
2 [x + 1] dx = 2 + 2 =
2 n=1
(2n 1)
0
" #1
(x + 1)3 9 X 16
2 = + =
3 2 n=1 (2n 1)4
2
0

X 1 4
=
n=1
(2n 1)2 96

54
Problema 4

a) Para f (x) = e[x] , 0 x 2 ,obtener su serie de Fourier en cosenos,


de perodo 4.
b) Del resultado determinar la convergencia de:

X (1)n1
n=1
2n 1

Solucion
a) Evaluando la funcion parte entera tenemos

1 si 0 x < 1
f (x) = e1 si 1 x < 2
2
e si x=2
Con extension par fp (x) de f (x) se obtiene la serie:

X nx
a0 + an cos
n=1
2

1
R2

1 1
e dx = 12 [1 + e1 ]
R
a0 = 2
1dx +
0 1
1
R2

nx nx
1 sin sin
nx nx
|10 + e1
R
an = cos 2
dx + e cos 2
dx = n
2
n
2
|21
2 2
0 1
sin n sin nsin n
sin n
= 2 n2 + 2e1 n
2
=2 n
2
[1 e1 ]

Finalmente, la serie es:


1 + e1 X sin n nx
+ 2(1 e1 ) 2
cos
2 n=1
n 2

b) Convergencia de x0 = 2 punto de discontinuidad con lmites laterales


e1 se tiene convergencia:


1 1 + e1 1
X sin n
2
e = + 2(1 e ) cos n
2 n=1
n

55

e1 1 1
X sin n
2
= 2(1 e ) cos n
2 n=1
n

X (1)n1
=
n=1
2n 1 4

Problema 5

Sea f (x) = x2 [x] , para x [0, 2] .


a) Obtener la serie de Fourier coseno de f (x) .
b) Obtener a que valores converge la serie para cada x [0, 2] .

Solucion
a) Si se evalua la funcion parte entera de x tenemos
[x] = 0, x (0, 1) y [x] = 1, x (1, 2) .

x2 , 0x<1
Entonces la funcion queda f (x) = 2
x 1, 1 x < 2

Consideremos ahora una extension par de la funcion f , entonces la serie


coseno de f (x) es

X  nx 
S (x) = a0 + an cos
n=1
2

con coeficientes
R2 R1 R2
a0 = 12 f (x) dx = 12 x2 dx + 21 (x2 1)dx
0 0 1
3 1
2
x3
  
1 x 1 5
a0 = 2
+ 2
x = 6
3 0 3 1
R2
an = f (x) cos nx
2
dx
0
R1 R2 2
an = x2 cos nx
2
dx + (x 1) cos nx
2
dx
0 1
Integrando por partes, se tiene
   1
8x nx 2 2 16 nx
an = cos + x sin
(n)2 2 n (n)3 2 0

56
   2  1
8x nx 2 2 16 nx 2 nx
+ cos + x sin sin
(n)2 2 n (n)3 2 1 n 2 0
 
16 2
an = 2 cos n + sin n
(n) n
Sustituyendo estos resultados, se obtiene la serie de Fourier
 
5 X 16 2 nx
S (x) = + 2 cos n + sin n cos
6 n=1 (n) n 2

Tenemos que f es seccionamente continua en [0, 2] , por lo tanto su ex-


tension perodica es seccionalmente continua en R , con discontinuidad
de salto en los puntos x = 1 y x = 2
Por lo tanto, de acuerdo con el teorema de convergencia, la serie con-
verge a

f (x) si 0 x < 1
1
2
si x=1
S(x) =

f (x) si 1 x < 2
3
2
si x=2

Problema 6

Utilice la serie de Fourier para demostrar la identidad trigonometrica


3 1
sin3 (x) = sin(x) sin(3x)
4 4

Solucion
Se calcula la serie de Fourier de f (x) = sin3 (x) en [, ] . Como f (x)
es impar la serie sera:
X
bn sin n
n=1

con coeficientes:

Z
2
bn = sin3 (x) sin(nx)dx

0

57
En primer lugar, calculemos la integral para n 6= 1
Z Z
3 cos nx 3
h i
3
sin x sin nxdx = sin x |0 + sin2 x cos x cos nxdx
n n
0 0
cos(n 1)x cos(n + 1)x
Usando la identidad trigometrica: cos x cos nx =
2
La ultima integral se puede expesar como
Z
3
= sin2 x [cos(n 1)x cos(n + 1)x] dx (1)
2n
0

En segundo lugar, calculemos el valor del coeficiente b1 para n = 1 en


(1)
Z Z Z
13 3 3 1 cos 4x
b1 = sin2 x cos 2xdx = (1 cos 2x) cos 2xdx = dx
2 4 4 2
0 0 0
23 3
b1 = =
4 2 4
En tercer lugar, para n > 1 en (1)

  Z  
3 2 sin(n + 1)x sin(n 1)x sin(n + 1)x sin(n 1)x
bn = sin x + |0 + sin 2xdx
2n n+1 n1 n+1 n1
0
Z  
3 sin(n + 1)x sin(n 1)x
bn = + sin 2xdx
2n n+1 n1
0
Usando la identidad trigonometrica
Z
3 1 1
bn = (cos(n + 1)x cos(n + 3)x) dx
2n n + 1 2
0
Z
3 1 1
(cos(n 3)x cos(n + 1)x)dx = 0, n 6= 3
2n n 1 2
0

Para n = 3 el calculo directo, produce:


3 2 1
b3 = =
2322 4
Por tanto, la serie de Fourier de f en [, ]es:
3 1
sin(x) sin(3x)
4 4

58
Problema 7

Sea f (x) = x(sin x), para x , entonces:


a) Determine la serie de esta funcion.

b) Pruebe la convergencia de la serie:



X (1)n 1
=
n=1
n2 1 4

c) Pruebe que esta serie se puede diferenciar termino a termino y utilice


este hecho para obtener el desarrollo de Fourier de sin (x) + x cos (x) .
Solucion
a) La funcion f (x) es par, es decir f (x) = f (x) x (, ),
entonces:
bn = 0
Z Z
1 1
a0 = f (x)dx = x sin xdx =

0 0

Z
1
a0 = [x ( cos x)]0 + cos xdx = 1

0
Z Z
2 2
an = f (x) cos(nx)dx = x sin x cos(nx)dx

0 0
Para n 6= 1
Z
1
an = x [sin ((n + 1) x) sin ((n 1) x)] dx

0
Integrando por partes, queda
  
1 cos ((n + 1) x) cos ((n 1) x)
an = x +
 (n + 1) (n 1)
 0
1 sin ((n + 1) x) sin ((n 1) x)
+
(n + 1)2 (n 1)2 0
Evaluando los lmites de la integral produce

59
2 (1)n+1
an =
n2 1

Para n = 1

Z Z
2 1 1
a1 = x sin x cos xdx = x sin(2x)dx =
2
0 0

Por tanto, la serie de Fourier de f para x [, ]es:


1 X (1)n+1
f (x) = 1 cos x + 2 21
cos (nx)
2 n=2
n

b) En x = 0 hay un punto de continuidad de la funcion, entonces la


serie converge a f (0)


1 X (1)n+1
f (0) = 0 = 1 cos 0 + 2 cos (0)
2 n=2
n2 1

Finalmente


X (1)n+1 1
=
n=2
n2 1 4

c) Sea f (x) = x(sin x), para x .


i) Como f (x) = x sin x ,es producto de funciones continuas, es continua
en [, ] .
ii) f 0 (x) = sin x + x cos x ,es producto y suma de funciones continuas,
es continua en [, ] .
00
iii) Existe f (x) = 2 cos x x sin x, y tambien es continua en [, ] .
Ademas f () = () (sin ()) = () ( sin ()) = sin = f ()
Por tanto, se satisfacen las hipotesis del teorema de diferenciacion de
la serie de Fourier, entonces para < x <

60

1 X (1)n+1
f (x) = 1 cos x + 2 21
cos (nx)
2 n=2
n

0 1 X (1)n
= f (x) = sin x + x cos x = sin x + 2 sin (nx)
2 n=2
n2 1

Problema 8

a) Desarrollar en serie de Fourier la funcion perodica de perodo 2.


Representar graficamente y estudiar la convergencia de la serie en R.

0, si x 0
f (x) =
x, si 0 < x

b) A partir del resultado anterior obtenga la suma de la serie:


X 1
n=1
(2n 1)2

c) Pruebe que esta serie se puede integrar termino a termino y obtener


Z x
un desarrollo en serie trigonometrica para f (u) du en [, ] .

Solucion
a) Calculemos los coeficientes de Fourier.
0
Z Z Z Z
1 1 1
a0 = f (x)dx = f (x)dx + f (x)dx = xdx
2 2 2
0 0
 2 
1 x
a0 = =
2 2 0 4
Z Z
1 1
an = f (x) cos(nx)dx = x cos(nx)dx

0

Usando el metodo de integracion por partes se tiene:



(1)n
  
1 x cos(nx) cos(nx) 1 1
an = + = 00+ 2
n n2 0 n2 n
n 
(1) 1 0 si n par
an = = 2
2
n n2 si n impar

61
As:

a2n = 0 n
2
a2n1 = n.
(2n 1)2
Z Z
1 1
bn = f (x) sin(nx)dx = x sin(nx)dx

0
 
1 x cos(nx) sin(nx) cos(n)
= + =
n n2 0 n
luego estos coeficientes son:
(1)n+1
bn =
n
Por lo tanto, la serie de Fourier de f para x [, ] ,es:

" #
X (1)n1 (1)n+1
+ cos nx + sin(nx)
4 n=1 n2 n

Esta serie converge a:


i) f (x) = 0 para < x 0, puesto que, son puntos de continuidad
de f.
ii) f (x) = x para 0 < x < , son puntos de continuidad de f.
f (+) + f ()
iii) = en los puntos de discontinuidad del tipo
2 2
x = + 2n con n Z.

b) Aplicando el criterio de convergencia en x = 0, f (0) = 0 se tiene


 
2 1 1 1
0= + + + ...
4 12 32 52

de donde  
2 1 1 1
= + + + ...
4 12 32 52

y de aqu

X 1 2
=
n=1
(2n 1)2 8

62

0, si x 0
c) Como f (x) = es una funcion seccionalmente
x, si 0 < x
continua en [, ] , con serie de Fourier


" #
X 2 (1)n+1
+ cos ((2n 1) x) + sin(nx)
4 n=1 (2n 1)2 n

se satisface las hipotesis del teorema de integracion de series de Fourier,


luego puede integrase termino a termino. Entonces, para cualquier x
[, ] , se tiene:
Z x 
0, si x 0
Primero, f (u) du = x2 en [, ]
2
si 0<x
Segundo, integrando la serie produce
Z x"
!#
X 2 cos ((2n 1) u) (1)n+1 sin(nu)
+ + du
4 n=1
(2n 1)2 n
"
!#x
X 2 sin ((2n 1) u) (1)n+1 cos(nu)
= u+
4 n=1
(2n 1)3 n2


!
X 2 sin (2n 1) x (1)n+1 cos(nx) 1
= x + 2 + 3 + 2
2
4 n=1
(2n 1) n n

Por tanto, la funcion f queda representada por la serie anteriormente


obtenida.

1.9.2. Integral de Fourier


Problema 9

a)
 Halle la representacion de la integral de Fourier de la funcion f (x) =
x, |x| <
0, |x|
b) De esta representacion deducir que:
Z Z
sin(w) cos(w)
2
sin(wx)dx = sin(wx)dx
0 w 0 w

63
Solucion
a) Como f es una funcion impar, entonces

Z
f (x) = B(w) sin(wu)dw
0

con coeficiente

Z
2
B (w) = u sin(wu)du

0 
2 cos(wu) sin(wu)
= u +
w w2
 0
2 cos(w) sin(w)
B (w) = +
w w2

Por consiguiente

Z  
2 cos(w) sin(w)
f (x) = + sin(wx)dw
w w2
0

Es la integral de Fourier de f para |x| =


6 0
b) En particular cuando |x| > , se tiene f (x) = 0. Entonces la
integral converge a cero

Z  
2 cos(w) sin(w)
0= + sin(wx)dw
w w2
0

Por tanto,

Z Z
sin(w) cos(w)
sin(wx)dw = sin(wx)dw
w2 w
0 0

64
Problema 10

Halle la representacion de la integral de Fourier de la funcion f (x) =


xe|x| si x (, ) y estudie su convergencia en R.

Solucion
Se tiene que f (x) es una funcion impar. Examinemos, si se cumplen las
condiciones de existencia de integral de Fourier.
En primer lugar

Z Z
|x|
xe dx = 2 xex dx
0
Z

= 2 xex |
0 + ex dx
0
=21=2

la integral es convergente
Ademas, f es continua y diferenciable x.
Los coeficientes de Fourier de f son:

A(w) = 0 ya que f es una funcion impar


Z
4w
B(w) = ue|u| sin(wu)du =
(1 + w2 )2

Entonces, para todo x la integral de Fourier converge a:

Z
x 4 w
xe = sin(wx)dw
(1 + w2 )2
0

65
Problema 11

Sea f lafuncion pulso rectangular unitario de perodo 2 definida por


1
2
si < x <
f (x) =
0 si 1 x < o < x 1
a) Representar graficamente f (x)
b) Obtener la serie de Fourier de f (x) .
c) Si an () es el coeficiente n-esimo de la serie anterior, calcular los
lmites:
lm ( lm+ (an ()) , lm+ ( lm (an ()))
n 0 0 n

Solucion
b) Como f es una funcion par de perodo 2 ,entonces :

Z1 Z
1 1
a0 = f (x) dx = dx =
2 2
0 0
Z1 Z
1 1 sen(n)
an = 2 f (x) cos(nx)dx = 2 cos (nx) dx = = an ()
2 n
0 0

1 sin(n)
donde en este caso definimos an () =
n
bn = 0 n

Luego, se tiene que:



1 1 X sen(n)
f (x) + cos (nx) , x [1, 1]
2 n=1 n

c) En primer lugar calculemos:

1 sen(n)
lm+ ( lm (an ())) = lm+ ( lm ) = lm+ (0) = 0
0 n 0 n n 0
n

En segundo lugar
 
1 sen(n)
lm ( lm (ak ()) = lm ( lm+ ) = lm (1) = 1
n 0+ n 0 n n

66
Problema 12

Dada la funcion f (x) = xex con x > 0,


a) Verifique que considerando las extensiones par e impar de la funcion
f:

Z 
1 w2
Z   
2w
cos wx dw = senwx dw
0 (1 + w2 )2 0 (1 + w2 )2

b) Estudiar la convergencia de la IF para deducir que:

Z 
w2
Z   
1
dw = dw
0 (1 + w2 )2 0 (1 + w2 )2

Solucion
Consideremos para f (x) = xex con x > 0 su extension par

xex si x > 0

fp (x) = =
xex si x < 0
Z Z
1
fp (x) A (w) cos wxdw con A (w) = 2 xex cos wx dx

0 0

Ahora, consideremos la extension impar de f

xex si x > 0

fi (x) = =
xex si x < 0
Z Z
1
fi (x) B (w) sin wxdw con B (w) = 2 xex senwx dx

0 0

Podemos calcular los coeficientes A (w) y B (w) integrando por partes:

67
Z Z
x
A (w) = 2 xe cos wx dx = 2 ex (x cos wx)dx =
0
0

xex ( cos wx + wsenwx) ex ((1 w2 ) cos wx 2wsenwx)

A (w) = 2
(1 + w2 ) (1 + w2 )2 0
1 w2
 
A(w) = 2
(1 + w2 )2

Z Z
x
B (w) = 2 xe sin wx dx = 2 ex (x sin wx)dx =
0
0

xex ( sin wx w cos wx) ex ((1 w2 ) sin wx + 2w cos wx)

B (w) = 2
(1 + w2 ) (1 + w2 )2 0
 
2w
B(w) = 2
(1 + w2 )2
Construyendo las respectivas integrales de Fourier y aplicando el teo-
rema de la convergencia , puesto que f es una funcion seccionalmente
suave x > 0 ,se tiene que :

Z 
1 w2

x 2
xe = cos wxdw
(1 + w2 )2
0
Z  
x 2 2w
xe = senwxdw
(1 + w2 )2
0

Por lo tanto, las extensiones son iguales:

Z  Z 
1 w2
 
2w
cos wx dw = sin wx dw
(1 + w2 )2 (1 + w2 )2
0 0

b) En x = 0 se tiene un punto en que estas extensiones son continuas,


luego ambas integrales convergen a f (0) = 0

Z Z Z
1 w2 1 w2
dw = 0 = dw = dw
(1 + w2 )2 (1 + w2 )2 (1 + w2 )2
0 0 0

68
Problema 13
Z
Si f (x) es una funcion par ,con integral de Fourier f (x) = 1 A (w) cos(wx)dw,
0
demuestre que:
Z
a) xf (x) = 1 A (w) cos(wx)dw, donde A (w) = dA(w)
dw
0
Z
2 A(w)
b) x2 f (x) = 1 A (w) cos(wx)dw, donde A (w) = d dw 2

0
Solucion
Z
a) Se tiene que xf (x) = A (w) sin(wx)dw, es una funcion impar,
1

0
Z

entonces A (w) = 2 v f (v) sin(wv)dv (1).
0
Z Z
1
Como f (x) = A (w) cos(wx)dw con A (w) = 2 f (v) cos(wv)dv.
0 0
Z
dA(w)
Entonces, derivando el coeficiente queda dw
= 2 vf (v) sin(wv)dv (2)
0

Por lo tanto, comparando (1) y (2) se = A (w) tiene dA(w)


dw
Z
b) Como x2 f (x) = 1 A (w) cos(wx)dw, es una funcion par,
0
Z

entonces A (w) = 2

v 2 f (v) cos(wv)dv (1)
0
Z Z
1
Como, f (x) =
A (w) cos(wx)dw con A (w) = 2 f (v) cos(wv)dv.
0 0
Z
dA(w)
Por consiguiente dw
= 2 vf (v) sin(wv)dv =
0

69
Z
d2 A(w)
dw2
= 2 v 2 f (v) cos(wv)dv (2)
0
d2 A(w)
Por lo tanto, comparando (1) y (2)se tiene dw2
= A (w) .

1.10. Ejercicios propuestos


1.- Sea f una funcion de perodo dada por

sin 2x si 0 x /2
f (x) =
0 si /2 x

a) Obtener la serie de Fourier de f (x).



1
P
b) Deducir la convergencia de la serie: 4n2 1
n=1

x ( x) si 0 < x <
2.- Sea f una funcion dada por f (x) =
x ( + x) si < x < 0
a) Represente graficamente la funcion f usando Maple
b) Obtener la serie de Fourier de f (x).

P (1)(n1)
c) Deducir la convergencia de la serie: (2n1)3
n=1
3.- Un pulso triangular simetrico de altura y ancho ajustables es
a 1 xb si 0 x b
 
descrito por: f (x) =
0 si b x
ab 2ab (1cos nb)
a) Muestre que los coefientes de Fourier son: a0 = 2
, an = (nb)2

b) Tome a = 1 y b = 2
calcule y represente las cinco primeras
sumas parciales.

70
4. Sea f una funcion dada por f (x) = 1 + |x| x [1, 1]
a) Obtener la serie de Fourier de f (x).

1
P
b) Deducir la convergencia de la serie: (2n1)2
n=1

5. Encontrar la serie de coseno de Fourier de la funcion perodica de


perodo 4, dada por f (x) = e[x] 0 x 2.

1
P
b) Deducir la convergencia de la serie: 2n1
n=1


1
P
c) Usando la identidad de Parseval deducir la convergencia de la serie: (2n1)2
n=1

6. Sea f (x) = x sin x, x


a) Obtener la serie de Fourier de f.
b) Pruebe que esta serie se puede diferenciar termino a termino.
c) Use el resultado anterior para obtener el desarrollo de Fourier.
de sin x + x cos x, x .

1
P
b) Deducir la convergencia de la serie (2n1)6
n=1

0 si x 0
7. Sea f (x) =
x si 0 < x
a) Obtener la serie de Fourier de f.
b) Pruebe que esta serie se puede integrar termino a termino.
c) Use los resultados anteriores para obtener el desarrollo en
Z x
serie trigonometrica para f (u) du

8. a) Establecer que si f (x) = x, < x < entonces



X (1)n+1
x=2 sin nx
n=1
n


1 2
P
b) Con la identidad de Parseval deducir la convergencia n2
= 6
n=1

c) Muestre que la integracion de la serie de Fourier de f (x) = x, <


x<

71
conduce a.


X (1)n+1 2
=
n=1
n2 12

d) Sea f (x) una funcion continua definida en < x < con serie de
P
Fourier a0 + (an cos nx + bn sin nx). Si g(x) = f (x ) pruebe que
n=1
la serie
de Fourier de g(x) es


X
a0 + ((1)n an cos nx + (1)n bn sin nx)
n=1

e) Aplicando los resultados de a) y d), obtener la serie de Fourier de


perodo
2 de la funcion definida por g(x) = x , 0 < x < .
9. Sea f (x) una funcion seccionalmente contnua, impar de perodo 2,
con
P
serie de Fourier bn sin(nx)
n=1
Rx
a) Verificar que g(x) = f (t)dt, xR
0

es funcion par de perodo 2



bn
P
b) Deducir que n
(1 cos(nx)) es la serie de Fourier de g(x) y
n=1

P bn 1
R Rx
que n
=
( f (t)dt)
n=1 0 0

10. Sea f (x) , x R funcion impar con integral de Fourier


R
Ii = 1 B (w) sin wx dw. Pruebe que la integral de Fourier de
0
R
g (x) = f (x) sin x es: Ip = A (w) cos(wx) dw, con
0
1


[B (w + 1) B (w 1)] w>1
A(w) = 1

[B (w + 1) + B (w 1)] 0 w < 1

72

1 x2 si |x| 1
11. Sea f (x) = , obtener la integral de Fourier
0 si |x| > 1
y estudie su convergencia en x0 = 0.

cos x si |x|
12. a) Obtener la integral de Fourier de f (x) =
0 si |x| >
b) Estudiar la convergencia de la IF en x0 = 0 , x1 = .

Z
2 w
13. Establecer la igualdad 2
sin wdw = ex si x > 0 y de
1+w
0
R w2
esto deducir el valor al cual converge (1+w2 )2
dw.
0

14. Obtener la Integral de Fourier de f (x) = e|x| , x R.


R cos(wx)
Del resultado, deducir el valor de (1+w2 )
dw.
0
15. Aplicando la la representacion de la integral de Fourier
demostrar que:
R cos(w/2) cos wx

2
cos x si |x| < 2
a) dw =
0 1 w2 0 si |x| > 2
R 1 cos w

2
si 0 < x <
b) sin(wx)dw =
0 w 0 si x>
16. Si f (x)es una funcion par con integral de Fourier
R
f (x) = A (w) cos(wx)dw, demuestre que:
0

1
R w

f (ax) = a
A a
cos(wx)dw, a > 0
0
17. Pruebe que la integral de Fourier de f puede escribirse como
1 R sin(w(t x))
lm f (t) dt
w tx

73
1.10.1. Respuestas

1) a)

1 1 2X 1
f (x) + sin 2x 2
cos 4nx
2 n=1 4n 1



b) Estudie la convergencia en f 2


X 1 1
=
n=1
4n2 1 2


x ( x) si 0 < x <
2) a) f (x) =
x ( + x) si < x < 0

b)


8X 1
f (x) sin(2n 1)x
n=1 (2n 1)3

f ( + ) + f ( )
c) Como f (x) es discontinua en x = la serie converge a 2
luego


X (1)(n1) 3
=
n=1
(2n 1)2 32


3) Si a = 1 y b = 2
el grafico de la funcion es
4)
a)

74

3 4 X 1
f (x) 2 cos(2n 1)x
2 n=1 (2n 1)2

b) Como f (x) es continua x la serie converge a f (0), luego


X 1 2
=
n=1
(2n 1)2 8

c) Aplicando la identidad de Parseval, se tiene:



X 1 4
=
n=1
(2n 1)4 96

5. a)

1 + e1 2 1
X sin( n
2
) n
f (x) + 1e cos( x)
2 n=1
n 2

b) Como f(x) es discontinua en x0 = 2 converge a los lmites


laterales en ese punto, entonces se tiene la convergencia


X 1
=
n=1
2n 1 4

6. a)

1 X (1)n+1
x sin x = cos x + 2 21
cos (nx) , - x
2 n=1
n

0
b) f y f son seccionalmente suave en x y f () = f () ,
luego se satisfacen las condiciones del teorema de diferenciacion.
c)Tenemos que


1 X (1)n
x cos x + senx = sin x + 2 n sin (nx) , x
2 n=1
n2 1

7. a)

75

1 X (1)n 1 (1)n+1

f (x) + cos (nx) + sin (nx) , x
4 n=1
n2 n

b) f es seccionalmente continua en x ,entonces se


satisfacen las condiciones del teorema de integracion. Luego,
su serie puede integrarse termino a termino
Z x 
0 x 0
c) Tenemos que f (u) du = x2
2
0<x
Esta funcion esta representada por la serie obtenida al integrar
la serie de Fourier anterior


1 2 X (1)n 1 sin (nx) (1)n+1 ( cos nx + (1)n )

1
x + + +
4 4 n=1
n2 n n n

8)e)


X sin(nx)
x = 2
n=1
n

11)a) Z  
1 4 sin w 4 cos w
cos wx dw
0 w3 w2

b) La funcion es continua en x0 = 0 luego la IF converge a f (0)


Z  
4 sin w 4 cos w
dw =
0 w3 w2

12)
a) Comof (x) es una funcion par se tiene que:
Z
2 2 w sin w
A(w) = cos v cos(wv) dv =
1 w2
0
B (w) = 0

76
Por lo tanto,
Z
2 w sin w
IF = cos wxdw.
1 w2
0

b) En x0 = 0 hay un punto de continuidad de f (x), entonces

Z
2 w sin w
dw = f (0) = 1
1 w2
0

y x1 = es un punto de discontinuidad de f (x), entonces:


Z
2 w sin 2w f ( + ) + f ( ) 0+1
2
dw = =
1w 2 2
0

13) Considere una extension impar de f (x), entonces


2w
A (w) = 0 y B (w) = 1+w2

2
R w
La Integral de Fourier de f (x)es 1+w2
sin wx dw.
0
R w2
Usando la identidad de Parseval (1+w2 )2
dw = 4
0

2
R cos(wx)
14) Como f es par tiene Integral de Fourier f (x) = (1+w2 )
dw.
0

Al estudiar la continuidad en x0 = 1 se obtiene la convergencia


R cos(w)
(1+w2 )
dw = 2e
0

1.11. Auto evaluaciones


En el aprendizaje de Calculo Avanzado como parte de la matematica
se requiere el dominio de dos tipos basicos de conocimientos:
a) el conocimiento conceptual y
b) el conocimiento procedimental.
El primero esta vinculado al razonamiento y reflexion, se caracteriza
por ser un conocimiento teorico, producido por la actividad cognitiva,

77
que permite establecer relaciones entre sus componentes; su caracter es
declarativo y se asocia con el saber que y el saber por que.
El segundo esta vinculado a la accion y se relaciona con metodos y las
estrategias para representar conceptos y transformar estas representa-
ciones; usando habilidades y destrezas para analizar comprensivamente,
elaborar, comparar, resolver algoritmos, evaluar y argumentar. De este
modo el conocimiento procedimental permite depurar y refinar la con-
struccion del conocimiento conceptual, como asimismo, usar de manera
eficaz y flexible, en un contexto aplicado, los conceptos, proposiciones,
teoras y modelos matematicos. En consecuencia, esta asociado con el
saber hacer.

Evaluacion formativa
Al termino de cada unidad , se incluyen una serie de evaluaciones for-
mativas que deben realizar los propios alumnos en situaciones de auto-
evaluacion de acuerdo a la dinamica de trabajo individual, por equipos
o grupal si se utiliza en el trabajo en el aula o en trabajo personal
realizado en casa.
Por consiguiente, esta modalidad de evaluacion tiene por funcion gener-
ar evidencias de los resultados de aprendizaje logrados por los propios
estudiantes para retroalimentar su avances de aprendizaje. Asimismo,
debe proporcionar informacion a los estudiantes sobre sus aciertos, difi-
cultades, falencias y omisiones en el proceso de aprendizaje del Calcu-
lo Avanzado de modo de introducir cambios y ajustes en los meto-
dos de estudios. De este modo, las autoevaluaciones abarcan tanto los
conocimientos conceptuales como procedimentalales luego son una in-
stancia para que los estudiantes perfeccionen su capacidad para: re-
conocer los conceptos, principios, reglas y propiedades que se sustenta
el Calculo Avanzado; identificar y aplicar metodos matematicos en la
resolucion de problemas; analizar y evaluar informacion matematica
proveniente de otras ciencias ; y analizar y evaluar las soluciones de un
problema para fundamentar su pertinencia.

Estrategia de aplicacion
Como se ha senalado el objetivo fundamental de estos instrumentos
de autoevaluacion es que el alumno verifique, constate y descubra las
falencias que aun persisten en el ambito cognitivo y procedimental de

78
su aprendizaje del tema objeto del estudio, ya sea que este sea el total
o una parcialidad de un tema. Se pretende que con esta constatacion el
alumno se motive para persistir en su estudio, si aun no ha logrado el
nivel necesario que asegure su exito en las evaluaciones regulares que se
haran del tema. Para que este ejercicio tenga el exito propuesto, se debe
trabajar el tiempo estipulado, y con las condiciones medioambientales
necesarias de privacidad y silencio , que aseguren la efectividad de la
actividad, o que reproduzcan condiciones requeridas de una prueba o
examen..

Series e Integrales de Fourier

Autoevaluacion No1
El estudiante:

1) Representara una funcion f (x) definida en el intervalo [0, L] por una


serie de Fourier coseno o seno, aplicando extensiones pares o impares
segun sea el caso, y evaluar si la serie converge o no en un punto x0
dado del dominio.
2) Obtendra la suma de ciertas series infinitas empleando el teorema
de la convergencia de las series de Fourier.
3) Verificara si una serie de Fourier se puede diferenciar termino a termi-
no y aplicar el teorema de la diferenciacion para obtener la derivada de
la serie de fourier

4) Representara una funcion f (x) no perodica por una Integral de


Fourier y emplear el teorema de la convergencia para probar si la Inte-
gral Fourier converge o no en un punto x0 del dominio.
5) Probara el teorema de Parseval para una funcion f que se representa
por una integral de Fourier y deducira la convergencia de ciertas sumas
de series.
6) Representara una funcion periodica f(x) de perodo 2L por una serie
de Fourier, y aplicara el teorema de la convergencia para determinar si
la serie converge o no en un punto x0 del dominio.
Tiempo : 1 hora 20 minutos
Problema 1

79
a) Desarrollar en serie de Fourier de cosenos la funcion
x
f (x) = sin
2

definida en el intervalo [0, ].


b) Deducir la convergencia de la serie

X 1 1
=
n=1
4n2 1 2

b) Pruebe que la serie de f se puede diferenciar termino a termino


x y
utilice este hecho para obtener el desarrollo de Fourier de cos .
2
Problema 2
R
a) Probar que si f (x)R= 0 (A (w) cos (wx)+B (w) sin (wx))dw, con co-
R
eficientes A (w) = 1 f (u) cos (wu) du y B (w) = 1 f (u) sin (wu) du.
R R
Entonces 1 [f (x)]2 dx = 0 (A2 (w) + B 2 (w))dw

, |x| a R 2
b) Si f (x) = ,pruebe que 0 sinw2wa dw = a
0, |x| > a 2

Problema 3

0 si 5 < x < 0
Sea f (x) = tal que f (x + 10) = f (x)
3 si 0 < x < 5
a) Obtener la serie de Fourier de f (x)
b) Defina f (x) en los puntos 5, 0 y 5 de manera que la serie de Fourier
obtenida anteriormente converja a f (x) x con |x| 5.

Pauta de Correccion
Problema 1
Para obtener el desarrollo en serie de Fourier de cosenos de la funcion
dada, construimos una extension par de la funcion f tal que
sin x ,
 
0x
fp (x) = 2x 
sin , x < 0
2
Como esta funcion es par, de perodo 2 ,tenemos que

80
bn = 0
1
R 1
R x
 1
 x

a0 =
f (x)dx =
sin 2
dx =
2 cos 2 0
0 0
2
1 cos 2 = 2
 
a0 =
R R
an = 2 f (x) cos(nx)dx = 2

sin x2 cos(nx)dx
0 0
R 
an = 1 1 1
  
sin 2
+ n x sin 2
n x dx
0
"
1
 1
 #
cos
+ n 2
x cos 2
+ n x
an = 1 1
 1

2
+n 2
+n
0
 
2 2 4
an = 1 + =
1 + 2n 1 2n (1 4n2 )
Por lo tanto, la serie de fp en [0, ] ,es:


2 1X 4
+ cos (nx)
n=1 (1 4n2 )

La funcion fp seccionalmente continua, de perodo 2, aplicando el


teorema de convergencia de Fourier , tenemos que:


2 1X 4
f (x) = + cos (nx) , x (0, 2)
n=1 (1 4n2 )

En particular para x = 0,


2 1X 4
+ = 0
n=1 (1 4n2 )

X 4 1
=
n=1
(4n2 1) 2

x

c) Sea f (x) = sin , para 0 x .
2
i) Como f (x) = sin x2 , es continua en (0, ) .


81
ii) f 0 (x) = 12 cos x

, es continua en (0, ) .
2
00
iii) Existe f (x) = 14 sin 12 , y tambien es continua en (0, ) .


Ademas f (0) = (sin (0)) = (sin ()) = sin 2 = f ()


Luego, se satisfacen las hipotesis del teorema de diferenciacion de la
serie de Fourier, entonces para < x < ,derivando la serie de Fourier
de f , obtenemos:

x
2 1X 4
sin = + cos (nx)
2 n=1 (1 4n2 )

1 x 1X 4n
cos = 0 sin (nx)
2 2 n=1 (1 4n2 )
x
1X 8n
cos = sin (nx)
2 n=1 (4n2 1)

Problema 2
a) Mutiplicando la integral de Fourier por f y realizando el producto
interno en (, ) , se tiene

Z Z Z
2
[f (x)] dx = [(A (w) f (x) cos (wx) + B (w) f (x) sin (wx))dw]dx
Z
Z Z 0
Z
2
[f (x)] dx = (A (w) f (x) cos (wx) dx + B (w) f (x) sin (wx) dx)dw
0

Z Z
2
[f (x)] dx = (A2 (w) + B 2 (w))dw =
Z
Z 0
1
[f (x)]2 dx = (A2 (w) + B 2 (w))dw
0


, |x| a
b) Como f (x) = , es funcion par, entonces la integral
0, |x| > a
de Fourier de f es
Z
A (w) cos (wx) dw
0

82
con coeficiente
Z Z a
2 2
A (w) = f (u) cos (wu) du = cos (wu) du

0 a 0
sin wu sin wa
= 2 =2
w 0 w
2
4 sin wa
A2 (w) =
w2

Por otra parte:


Z Z a
1 2 1
[f (x)] dx = 2 dx = 2a
a

Como la funcion f es par, se tiene


1
Z Z
2
[f (x)] dx = A2 (w) dw
0

Entonces, al reemplazar los terminos


Z
sin2 wa
2a = 4 dw
0 w2
Z
sin2 wa a
2
dw =
0 w 2

Problema 3
La serie de Fourier buscada es de la forma:
P nx nx
f (x) a0 + (an cos + bn sin )
n=1 5 5
con coeficientes
 5
1 R5 3x 3
a0 = 0
3dx = =
10 10 0 2
1 0
h R nx R5 nx i
an = 0 cos dx + 3 cos dx
5 5 5 0
5
 5
3 5 nx
an = sin =0
5 n 5 0
1 hR 0 nx R5 nx i
bn = 0 sin dx + 0 3 sin dx
5 5 5 5
83
 5
3 5 nx
bn = cos
5 n 5 0
3
bn = [1 cos n]
n
Sustituyendo los coeficientes en la serie obtenemos
3 3P [1 cos n] nx
+ sin( )
2 n=1 n 5
b) Como f (x) es una funcion seccionalmente continua entonces en los

f (x+
o ) + f (xo )
puntos de discotinuidad converge a . Luego en los pun-
2
3
tos 5, 0 y 5 converge a .
2
Por tanto la funcion debe asumir estos valores en los puntos menciona-
dos para que se cumpla la covergencia propuesta.

Autoevaluacion No 2

Tiempo : 2 horas
Problema 1
Dada la funcion f (x) = x [2x] , 0 < x < 1
a) Obtener la serie de Fourier de cosenos.
1
b) Establecer la convergencia de la serie en x0 = 0 y x1 = .
2

Problema 2
Deducir la igualdad de la integral de Fourier
Z  
2 cos w sin w
x= + sin wxdw
w w2

para 0 < x < .


Problema 3
Sea f (x) continua por tramos, impar de perodo 2 con serie de Fourier
P
bn sin (nx) .
n=1
Rx
a) Verificar que g (x) = 0 f (t) dt , x R es funcion par de perodo
2.

84
b b
P n P n
b) Deducir cos (nx) es la serie de Fourier de g (x) y que
n=1 n n=1 n

b 1 R Rx
P n 
= f (t) dt dx
n=1 n 0 0

Pauta de Correccion
Problema 1
a) Consideremos una extension par de f (x) de perodo P = 2, es decir
f (x + 2) = f (x)
1
Como f (x) = x [2x] , 0 < x < 1, se tiene que 0 < x < , [2x] = 0 y
2
1
< x < 1, [2x] = 1.
2
x 1
si 0 < x <

As f (x) = 2 , luego los coeficientes de Fourier
1
x 1 si
<x<1
2
son
 2 1/2  1
R 1/2 R1 x (x 1)2
a0 = 0 xdx + 1/2 (x 1) dx = + =0
2 0 2 1/2
R 1/2 R1
an = 2 0 x cos nxdx + 2 1/2 (x 1) cos nxdx
 1/2  1  1
sin nx cos nx sin nx cos nx sin nx
an = 2 x + +2 x + 2
n (n)2 0 n (n)2 1/2 n 1/2
n

2 2 sin 2
an = 2 [cos n 1] +
(n) n
2 n 2 (1)n1
an = [(1) 1] +
(n)2 (2n 1)
bn = 0
De este modo la serie de Fourier queda

" #
X 2 n 2 (1)n1
[(1) 1] + cos nx
n=1
(n)2 (2n 1)

b) En x0 = 0 hay un punto de continuidad de f, entonces la serie


converge a

85
" #

P 2 n 2 (1)n1
f (0) = 0 = [(1) 1] +
n=1 (n)2 (2n 1)
1
Para x1 = se tiene un punto de discontinuidad de f , entonces la serie
2
converge a
 
+
f 21 + f 12

1 1

+
= 2 2
= 0.
2 2

Problema 2

x si < |x| <
Consideremos la funcion f (x) = , la cual es
0 si R |x| >

impar, entonces la integral de Fourier de f es 0 B (w) sin (wx) dx, con
coeficiente:
2 R 2 R
B (w) = 0
f (x) sin (wx) dx = x sin (wx) dx
0
 
2 cos wx sin (wx)
B (w) = x +
w w2 0
 
2 cos w sin (w)
B (w) = +
w w2
Por tanto la integral de Fourier , queda
 
2 R cos w sin (w)
+ sin (wx) dx
0 w w2
Como f es continua en (0, ) , la integral de Fourier converge a
 
2 R cos w sin (w)
f (x) = x = + sin wxdw
0 w w2
Problema 3
a) En primer lugar, verifiquemos que g (x) es una funcion par. En efecto
sea

R x Rx
g (x) = 0
f (t) dt = 0
f (u) (du), si cambiamos la variable u =
t
Pero la funcion del integrando f (u) es una impar es decir f (u) =
f (u) u
Rx
entonces g (x) = 0 f (u) (du) = g (x)

86
En segundo lugar, estudiemos el perodo de la funcion
Z x+2
g (x + 2) = f (t) dt
0
Z x Z x+2
= f (t) dt + f (t) dt
0 x
Z x Z
= f (t) dt + f (t) dt
0
Z x
= f (t) dt = g (x)
0

Por tanto g (x) tiene perodo 2

b) Integrando la serie de Fourier de f , se tiene

Z x Z
xX
g (x) = f (t) dt = bn sin (nx) dt
0 0
"
#n=1
x
X cos (nx)
= bn
n=1
n
0

X bn X bn
= cos (nx)
n=1
n n=1
n

Como g (x) es par su serie de Fourier es de la forma



P
g(x) = a0 + an cos nx donde x y coeficientes
n=1

Z
1
Z
1
a0 = g (x) dx = g (x) dx
2 0

1
Z Z x  X bn
= f (t) dt dx =
0 0 n=1
n

Autovaluacion No 3

Tiempo : 1 hora 20 minutos

87
Problema 1
Sea f (x) = cos bx, x , con f (x + 2) = f (x) , b constante
no entera.
a) Obtener la serie de Fourier de f.
b) A partir de la convergencia de la serie, deducir la igualdad
 
1 1 1
= 2b + ...
senb 2b2 b2 12 b2 22

Problema 2
 
|x|
1 si |x| < a

Sea la funcion f (x) = a
0 si |x| > a

a) Obtener la integral de Fourier de f.


2 R 1 cos w
b) Deducir que 1 x = cos wxdw, si 0 x 1; y
0 w2
R 1 cos w
0
cos wx dw = 0, si x > 1.
w2

Problema 3
a) Utilice la integral de Fourier para obtener la convergencia
Z
sin w
dw =
0 w 2
.
R 1 sin wx
b) Probar que la funcion definida por g (x) = x
0
dw, x > 0
w

g (0) = es continua en cero.
2

Pauta de Autocorreccion
Problema 1
a) Como f (x) = cos bx, en x es funcion par, entonces la
serie de Fourier de f es cosenoidal,


P
es decir f (x) a0 + an cos nx con coeficientes
n=1

88
 
1 R 1 sin b
a0 = 0
cos bxdx = sin b =
b 0 b
 
2 R 2 sin (b n) x sin (b + n) x
an = cos bx cos nxdx = +
0 2(b n) 2 (b + n) 0
 
2 sin (b n) sin (b + n)
an = +
2(b n) 2 (b + n)
2b (1)n sin b
an =
(b2 n2 )
Por tanto la serie de fourier queda


sin b 2b sin b P (1)n
f (x) + 2 2
cos nx
b n=1 (b n )

b) En x0 = 0 hay un punto de continuidad de f entonces por el teorema


de convergencia de la serie, esta converge a
sin b 2b sin b P (1)n
f (0) = 1 = + 2 2
b n=1 (b n )
Entonces
1
P (1)n
= + 2b 2 2

sin b b n=1 (b n )

(1)n


1 P
= 2b +
sin b 2b2 n=1 (b2 n2 )
 
1 1 1
= 2b + ...
sin b 2b2 b2 12 b2 22

Problema 2
a) Como:
   
|x| |x|
f (x) = 1 = 1 = f (x) x [a, a]
a a
la funcion f es par, entonces la integral de Fourier de f es
R
0
A (w) cos (wx) dx, con coeficiente:
2 hR a  x R i
A (w) = 1 cos (wx) dx + 0 cos (wx) dx
0 a a

2 Ra  x 
A (w) = 0
1 cos (wx) dx
a
 
2 1 cos aw
A (w) =
aw2

89
Por tanto la integral de Fourier queda
2 1 cos aw
Z  
cos (wx) dx,
0 aw2

b) Como la funcion f es continua para x [0, 1],aplicando el teorema


de convergencia con a = 1, se deduce que la integral de Fourier converge
a
2 R 1 cos w
f (x) = 1 x = cos wxdw, y del mismo modo
0 w2
2 R 1 cos w
f (x) = 0 = cos wxdw, x > 1
0 w2

Problema 3

1 si |x| 1
a)Sea la funcion f (x) = , como la funcion f es par,
0 si |x| > 1
entonces la integral de Fourier de f es
R
0
A (w) cos (wx) dx, con coeficiente:
2 hR 1 R i
A (w) = 1 cos (wx) dx + 0 cos (wx) dx
0 1

2 R1
A (w) = cos (wx) dx
0
 1
2 sin wx 2 sin w
A (w) = =
w 0 w
Por tanto, la integral de Fourier queda
2 sin w
Z  
f (x) = cos (wx) dx
0 w

Como la funcion es continua en x = 0,entonces la integral converge a

Z
2 sin w
f (0) = 1 = dx =
0 w
Z
sin w
dx =
0 w 2

b) Sea la integral

1
sin wx
Z
x
g (x) = dw, x > 0 g (0) =
0 w 2

90
w
aplicando el cambio de variables u = x
= xdu = dw, se tiene

Z 1 Z 1
x2 sin u x2 sin u
g (x) = xdu = du
0 xu 0 u
Calculemos ahora el lmite de esta funcion cuando x 0

Z 1
sin u x2
limg (x) = lm du
x0 x0 0 u
Z
sin u
= du = = g (0)
0 u 2

Lo que implica que g(x) es continua en cero.

91
Captulo 2

Funciones Vectoriales de una


variable real

2.1. Introduccion


La recta de R3 que pasa por el punto P 0 = (x0 , y0n, z0 ) y es paralela aoun


vector
a = (a1, a2, a3 ) se define como el conjunto P 0 + t a | t R .En
esta definicion de recta a cada numero real t corresponde el punto

P 0 + t
a de R3 es decir a cada valor t de R le asocia el punto
(x0 + ta1, y0 + ta2, z0 + ta3 ) de R3 . Tal correspondencia o asociacion
genera lo que llamaremos una funcion vectorial de una variable real


que en este caso es de R en R3 . Si denotamos por f a tal funcion
entonces su regla de correspondencia es


f (t) = (x0 + ta1, y0 + ta2, z0 + ta3 )



El dominio de f es el conjunto de todos los numeros reales y el rango



de f es la recta que pasa por el punto P o y es paralela al vector

a . Este es un ejemplo del tipo de funciones que estudiaremos en este


modulo; para tales funciones consideraremos los conceptos de lmite,
continuidad, derivada e integral. Desde el punto de vista conceptual no
hallaremos ideas nuevas y en la mayor parte de los casos las tecnicas
usadas son las mismas desarrolladas en el calculo de funciones real de
una variable real.

92
2.2. Funciones Vectoriales

Definicion 2.2.1. Una funcion vectorial de una variable real es una


funcion cuyo dominio es un conjunto de numeros reales y el rango es
un conjunto de vectores o puntos de Rn



Notacion f : D R Rn tal que t D, f (t) = (f1 (t), f2 (t), . . . , fn (t)),
donde fk : D R R para cada k = 1, 2, .., n es una funcion real de


variable real. Cada fk es la k-esima componente del vector f (t).


Si la funcion f describe el movimiento de una partcula, el vector

f (t) = (f1 (t), f2 (t), . . . , fn (t)) senala la posicion en el instante t, es


decir en estos casos t representa la variable tiempo.
Ejemplo 1



Sea f : I R R3 tal que f (t) = (cos t, sin t, t), I = [0, 2]


Hacer un esquema del rango de f
Solucion:


Pongamos f (t) = (x(t), y(t), z(t)) donde x = cos t, y = sin t, z = t.
En este caso para cualquier valor de t se cumple x2 + y 2 = 1 que


es la proyeccion en el plano XY de cualquier punto f (t) de la curva
que esta sobre el manto de un cilindro de radio unitario x2 + y 2 = 1,


y z = t senala la distancia de f (t) al plano XY.


El rango de f es entonces una curva que partiendo de (1, 0, 0)
describe un arco completo de una helicoidal en el manto del cilindro
x2 + y 2 = 1 de R3 .

93
Ejemplo 2




Sea f : I R R3 tal que f (t) = (t, t, t), describa el rango de f .
Solucion:


Las imagenes f (t) = (t, t, t) las podemos escribir vectorialmente de


la forma f (t) = (0, 0, 0) + t(1, 1, 1) lo que nos permite reconocer que
se trata de una recta que pasa por el origen (0, 0, 0) en la direccion del
vector
v = (1, 1, 1).

Ejemplo 3



Sea f : I R R3 tal que f = (t, t, 2t2 ), I = [3, 3] describa el


rango de f.
Solucion:


Ponemos f (t) = t(1, 1, 0) + t2 (0, 0, 2), de esta expresion se puede afir-


mar que f (t) es la suma de un vector a lo largo de la recta y = x
en el plano XY y un vector perpendicular al plano XY. Quiere decir


entonces que el rango de f se encuentra en el plano que contiene los
vectores (1, 1, 0), (0, 0, 2) perpendicular al plano XY.
Si se considera
en un punto
(t, t, 0) en el plano XY y u distancia al
origen, u = t + t = 2t, resulta que z = 2t2 = u2 . Por lo tanto, el
2 2


rango de f es una porcion de la parabola z = u2 que esta en el plano
y = x perpendicular al plano XY y que contiene al eje z.

94
2.3. Lmite de una funcion vectorial.

Previamente aclaremos o recordemos algunos conceptos en cuanto a la


metrica que usaremos.

Definicion 2.3.1. (distancia)




Si

a y b son una par de elementos (puntos) de Rn dados por



, a3 , . . . , a n ), b = (b1 , b2 , b3 , . . . , bn ) la distancia desde
a = (a1 , a2
a




hasta b es b a se define por

" n # 21



b
X
a = (bi ai )2

i=1

que en R2 y R3 viene a corresponder a lo que definimos como distancias


entre dos puntos.


Definicion 2.3.2. Se dice que el vector l = (l1 , l2 , l3 , . . . , ln ) es el


lmite de la funcion vectorial f : I R Rn en t0 I , si para cada
 > 0 existe un numero > 0 tal que siempre que t esta en el dominio



de f y 0 < |t t0 | < entonces f (t) l < 

2.3.1. Teorema del lmite




Teorema 2.3.1. Sea f : I R Rn funcion vectorial. Entonces

95



lm f (t) = l = (l1 , l2 , l3 , . . . , ln ) lm fk (t) = lk , k = 1, 2, ..., n
tt0 tt0



donde f (t) = (f1 (t), f2 (t), . . . , fn (t))

Demostracion.-
i) () Suponemos que lm fk (t) = lk , k = 1, 2, ..., n
tt0

Sea  > 0 dado




f (t) l = k(f1 (t) l1 , f2 (t) l2 , ..., fn (t) ln )k

= k(f1 (t) l1 , 0, 0, ..., 0) + (0, f2 (t) l2 , 0, ..., 0) + (0, 0, 0, ..., fn (t) ln )k


k(f1 (t) l, 0, ..., 0)k + k(0, f2 (t) l2 , 0, ..,0)k + ... + k(0, ..., 0, fn (t) ln )k
|f1 (t) l| + |f2 (t) l2 | + ... + |fn (t) ln |


Si lm fk (t) = lk =Si n
> 0 existe k > 0 tal que 0 < |t t0 | <
tt0
k = |fk (t) lk | < n , k = 1, 2, ..., n
tomando = mn {k } , k = 1, 2, ..., n se tiene que 0 < |t t0 | < =
|fk (t) lk | < n para todo k = 1, 2, .., n.

De la desigualdad anterior, mayorando por .
n
n

  X  
f (t) l < + ... + = = n=

n n i=1
n n

t (t0 , t0 + ), lo que prueba esta parte del teorema.





ii) () Suponemos que lm f (t) = l
tt0

Sea  > 0 dado





lm f (t) = l = (l1 , l2 , l3 , . . . , ln ) y f (t) = (f1 (t), f2 (t), . . . , fn (t))
tt0

implica que existe > 0 tal que

96




f (t) l <  todo t en el dominio de f tal que 0 < |t t0 | <

pero |fk (t) lk | < kf (t) lk todo k, entonces |fk (t) lk | <  todo t

en el dominio de f tal que 0 < |t t0 | < .
Por lo tanto
lm fk (t) = lk todo k = 1, 2, ..., n.
tt0

Se ha establecido entonces que:





lim f (t) = l = (l1 , l2 , l3 , ...., ln ) lim fk (t) = lk , k = 1, 2, 3, ...., n.
tto tto

Ejemplo 4




Si f (t) = (cos t, sin t) , calcule lm f (t)
t 2

Solucion:
 


lm f (t) = lm cos t, lm sin t = (0, 1)
t 2 t 2 t 2

Ejemplo 5

La trayectoria de una partcula en el espacio R3 esta dada por la


funcion vectorial
c (t) = (cos 2t, sin t, 2t ). Calcule: lim

c (t), limc (t)
t 2 t
y lim c (t) .
t 2

Solucion.-
lm (cos 2t, sin t, 2t

) = (lm cos 2t, lm sin t, lm 2t

) =(1, 1, 1)
t 2 t 2 t 2 t 2

lmt (cos 2t, sin t, 2t



) = (lm cos 2t, lm sin t, lm 2t

) = (1, 0, 2)
t t t

lm (cos 2t, sin t, 2t



) = ( lm cos 2t, lm sin t, lm 2t

) = (1, 1, 1)
t 2 t 2 t 2 t 2

97
2.3.2. Operaciones con funciones vectoriales


Definicion 2.3.3. Sean f , g : I R Rn funciones vectoriales,


entonces, para cada t I se define f +
g, f
g, f g, f g

de la forma siguiente:



a) ( f + g )(t) = f (t) +
g (t) = ((f1 + g1 )(t), (f2 + g2 )(t), ..., (fn + gn )(t))



b) ( f g )(t) = f (t)
g (t) = ((f1 g1 )(t), (f2 g2 )(t), ..., (fn gn )(t))



c) ( f g )(t) = f (t) g (t) = f1 (t)g1 (t) + f2 (t)g2 (t) + ... + fn (t)gn (t)
Xn
= fi (t) gi (t)
i=1




d) (f
g )(t) = f (t) g (t)
= ((f2 g3 )(t) (f3 g2 )(t)), (f3 g1 )(t) (f1 g3 )(t), (f1 g2 )(t) (f2 g1 )(t)).

en este caso para n = 3.


Definicion 2.3.4. Producto por escalar


Si : I R y f : I Rn es funcion vectorial, definimos:




f : I Rn tal que ( f )(t) = (t) f (t) = ((t)f1 (t), (t)f2 (t), ..., (t)fn (t))

Estas definiciones nos llevan al siguiente teorema

2.3.3. Teoremas del algebra de lmites



Teorema 2.3.2. Sean f , g : I R Rn funciones vectoriales,



to I , si lm f (t) =

a y lm g (t) = b ; a , b Rn ,entonces:
tto tto
h



a) lm f + g (t) = lim f (t) + lm
g (t) =

i
a + b
tto tto tto
h



b) lm f g (t) = lm f (t) lm
g (t) =

i
a b
tto tto tto
h



c) lm f g (t) = lm f (t) lm

g (t) =
i
a b
tto tto tto
h



d) lm f g (t) = lm f (t) lm
g (t) =

i
a b , con n = 3 en
tto tto tto
este caso.

98
Demostracion:
Es consecuencia directa de aplicacion del Teorema del Lmite y la
definicion
de las operaciones. Se deja como ejercicio al lector.

2.3.4. Teorema: producto de funcion escalar por


vectorial


Teorema 2.3.3. Sean : I R R y f : I R Rn , to I tal
que:


lm (t) = ; y lm f (t) =

a , R y
a Rn , entonces:
tto tto
h i

lim f (t) = lim (t)lim f (t) =
a
tto tto tto

Demostracion: Se deja como ejercicio al lector.

2.4. Continuidad



Definicion 2.4.1. Sea f : I Rn y to I , diremos que f es con-
tinua en to si para cada > 0,existe > 0 tal que t I : |t to | <



,entonces f (t) f (to ) <




Note que: si f = (f1 , f2 , f3 , ..., fn ) tal que f : I Rn y fi : I R,
i = 1, 2, 3, ..., n.


Entonces f es continua en to , ssi, fi es contnua en to para todo
i = 1, 2, 3, ..., n.
n  12
P 2
En efecto, vea que |fi (t) fi (to )| ((fi (t) fi (to ))
i=1


= si f es continua, entonces fi es continua. Reciprocamente co-


n
P
mo f (t) f (to ) |fi (t) fi (to | podemos inferir que si fi es

i=1


continua, i = 1, 2, 3, ..., n,entonces f es continua.
Por lo tanto, decimos que una funcion vectorial es continua ,si y solo
si, lo son cada una de sus funciones componentes

99
Observacion: Los teoremas de continuidad que obviaremos en este
caso y que son una replica de los teoremas de lmites se pueden probar
facilmente.

2.5. La Derivada


Definicion 2.5.1. Sea f : I R Rn funcion vectorial y to I. Se




define la derivada de f en to , denotada ddtf (t0 ) o f(to ) por el lmite:




f(to ) = lm f (to +h)
h
f (to )
, cuando este lmite existe.
h0

Geometricamente c 0 (t ) es un vector tangente a la curva C descrita


o
por la trayectoria
c (t) en el punto c (t ) orientado.
0

De la definicion del lmite se deduce que:





Teorema 2.5.1. Sea f : I R Rn , to I y f = (f1 , f2 , f3 , ..., fn )


entonces f es derivable en to si y solo si cada fi es derivable en to .

Demostracion


f (to + h) f (to )
f (to ) = lm
h0 h

100
1
= lim [f1 (to + h) f1 (to ), f2 (to + h) f2 (to ), ..., fn (to + h) fn (to )]
h0 h
h i
f1 (to +h)f1 (to ) f2 (to +h)f2 (to ) fn (to +h)fn (to )
= lim h
, lm h
, ..., lm h
h0 h0 h0
 0
= f1 (to ), f20 (to ), ..., fn0 (to )




Por lo tanto f 0 (to ) existe si y solo si fk0 (to ) existe para k = 1, 2, 3, ..., n
Definicion Si suponemos que c (t) describe una trayectoria seguida por

una partcula podemos definir al vector c(t) como el vector velocidad


en la trayectoria
c en el punto

c (t). Asimismo , definiremos la rapidez

en ese punto como c (t)

Ejemplo 6
Sea
c (t) = (r cos(t), r sin(t)), trayectoria cuyo camino corresponde a
una circunferencia de radio r. Muestre que c (t) y
c 0 (t) son ortogonales

Muestre que
c (t) y
c 0 (t) son ortogonales

Solucion.
Tenemos c (t) es derivable lo que implica c 0 (t) = (r sin t, r cos t)
Por consiguiente

c (t)
c 0 (t) = (r cos(t), r sin(t)) (r sin t, r cos t)

= r (sin t) r cos (t) r (cos t) r sin (t) = 0

c (t) y
c 0 (t) son ortogonales t.

101
2.6. Regularidad de una curva
Una diferencia significativa de la derivada de funciones vectoriales re-
specto de la suavidad o regularidad de la curva en el aspecto geometrico
es que en este caso la derivabilidad no detecta picos en la curva. Para
describir este hecho consideramos el siguiente ejemplo:

Ejemplo 7



Sea f : R R2 definida por f (t) = (t3 , t2 |t|). Su grafico como
muestra la figura es similar al grafico de y = |x| en R2 y sabemos que
la funcion f (x) = |x| no es derivable en x = 0.

Muestre que la funcion dada es derivable para t = 0.


Solucion

f (t) = (t3 , t2 |t|) x(t) = t3 , y(t) = t2 |t|

x(t) = t3 x0(t) = 3t2 , t,


y(t) = t2 |t| y0(t) = 3t2 , t > 03t2 , t < 0

y(h) y(0) h2 |h|


Ademas si t = 0 y0(0) = lim = lim = limh |h| =
h0 h h0 h h0
0


f(0) = (0, 0)
Lo que prueba que esta funcion es derivable en todo R y que hay puntos
en los cuales la derivada es cero, esto geometricamente significa que
la curva no es suave en ese punto, cambia rapidamente de direccion,
presenta un peak.

Con el objeto de advertir este comportamiento geometrico en una curva


y su relacion con la derivada demos la siguiente definicion.

102
2.6.1. Camino regular


Definicion 2.6.1. Sea f : I R Rn una funcion de clase C 1 (I).Se




dice que f (t) describe un camino regular si f 0 (t) 6= 0 t I.

Consideremos el siguiente ejemplo para aclarar aun mas este concepto

Ejemplo 8
Sea la curva C es descrita por
c (t) = (cos3 t, sin3 t), 0 t 2, que

corresponde a un Astroide

Muestre que no es una curva regular en 0 t 2,

Solucion
Esta trayectoria regular no es regular, pues en t = 0, 2 , , y 3 2
la
derivada se anula como podemos verificar en el siguiente calculo.

c(t) = (3 cos2 t( sin t); 3 sin2 t cos3 t) = (3 cos2 t sin t; 3 sin2 t cos3 t)

c(0) = (0, 0) ;
c( ) = (0, 0) ,
c() = (0, 0) , y
c( 3 ) = (0, 0)
2 2

2.6.2. Propiedades de la Derivada




Teorema 2.6.1. Sean f , g : I R Rn funciones derivables y
: I R R una funcion derivable, entonces:



i) (k f )0 (t) = k f(t)

103

0

ii) ( f + g ) (t) = f 0 (t) +
g 0 (t)

0

iii) ( f g ) (t) = f 0 (t)
g 0 (t)




iv) ( f )0 (t) = 0 (t) f (t) + (t) f 0 (t)

0


v) ( f g ) (t) = f(t)
g (t)+ f (t) g(t) Producto Punto o Producto
interno



vi) ( f )(t) = f((t))(t)

0


vii) ( f g ) (t) = f(t)
g (t) + f (t)
g 0 (t) para n = 3
Demostracion: i), ii), iii) se dejan al lector




( f )0 (t) = ((t) f (t))= g ((f1 )(t), (f2 )(t), (f3 )(t), ..., (fn )(t))
= (0 (t)f1 (t) + (t)f01 (t), 0 (t)f2 (t) + (t)f20 (t), ..., 0 (t)fn (t) + (t)fn0 (t))
= (0 (t)f1 (t), 0 (t)f2 (t), ..., 0 (t)fn (t) + (t)f10 (t), (t)f20 (t), ..., (t)fn0 (t))
= 0 (t)(f1 (t), f2 (t), ..., fn (t)) + (t)(f10 (t), f20 (t), ..., fn0 (t))



= 0 (t) f (t) + (t) f 0 (t)


0



0
v) ( f g ) (t) = f (t) g (t)
" n # n
d X X d
= fi (t)gi (t) = (fk (t)gk (t))
dt k=1 k=1
dt
n
X
= [fk0 (t)gk (t) + fk (t)gk0 (t)]
k=1
n
X n
X
= fk0 (t)gk (t) + fk (t)gk0 (t)
k=1 k=1



= f 0 (t)

g (t) + f (t)

g 0 (t)

v), vi) se dejan al lector.

2.7. Parametrizacion

Si c (t) define una trayectoria donde t es el parametro, podemos mod-


ificar la expresion que define c (t) por
c (s) de tal modo de tener el
mismo conjunto de imagenes, esto lo llamaremos reparametrizacion.

104



Definicion 2.7.1. Sean f : I R Rn tal que f = (f1 , f2 , f3 , ..., fn )
define un camino regular en el espacio Rn y : I1 I una fun-
cion de clase C 1 (I) biyectiva y tal que 0 (s) 6= 0, s I1 , entonces




f o : I1 Rn tal que f o (s) = ( f )(s) se llama reparametrizacion


de la trayectoria) f .

Observacion:
De esta definicion se tiene que:




0

1) f o (s) = ( f )(s) f (s) = f((s)) (s)

0


si (s) = t f o (s) = f(t) (s) = (s) f(t)




f 0o (s) = (s) f 0 (t)


0
como (s) es un escalar, esto significa que f o (s) es (s) veces la


velocidad que llevara una partcula parametrizada por f (t).

2) 0 (s) 6= 0 (s) > 0, s I1 v (s) < 0, s I,



0

de la expresion f o (s) = (s) f 0 (t) se puede inferir que:



a.- Si (s) > 0 , s I1 fo (s) conserva la orientacion de f (t)



b.- Si (s) < 0 , s I1 fo (s) invierte la orientacion de f




Si fo (s) es una reparametrizacion de f (t) y del hecho que fo 0 (s) =




(s) f0 (t) en cada punto fo (s) = f (t) si (s) = t, se deduce el

105
siguiente teorema.


Teorema 2.7.1. Sea f : I R R2 ( o R3 ) un camino regular



y fo = f : J R R2 ( o R3 ) una reparametrizacion de el (
donde tiene las condiciones pedidas en la definicion). Entonces la



recta tangente a la curva C (traza de f ) en f (to ) con t0 I , es


la misma que la recta tangente a C en fo (s0 ) si t0 = (s0 ).




Demostracion.- t0 = (s0 ) P 0 = f (to ) = f ((s0 ) ) =




0
fo (s0 ) ademas fo(s) = (s) f(s) implica que los vectores f (to )

0
y fo (s0 ) son paralelos,entonces las rectas tangentes a la curva en

f (to ) coinciden.

2.7.1. Ejemplos de reparametrizaciones




Sea f : [a, b] R3 ,una trayectoria regular . Entonces:



1) La trayectoria fo : [a, b] R3 tal que t 7 f (a + b t) es la


reparametrizacion de f que corresponde a la aplicacion : [a, b] 7


[a, b] dada por t 7 a + b t, llamamos a f o trayectoria opuesta a

f.


2) La trayectoria
g : [0, 1] R3 tal que t 7 f (a + (b a)t) es una


reparametrizacion de f que corresponde a la aplicacion : [0, 1] 7


[a, b] dada por t 7 a + (b a)t, y que conserva la trayectoria de f .

Ejemplo 9



Sea f : [5, 10] R3 definida por t 7 f (t) = (t, t2 , t3 ) . Reparametrizar


como trayectoria opuesta a f .
Solucion


Aplicando el apartado 1) tenemos: fo : [5, 10] R3 tal que t 7

f (5 t) = ((5 t), (5 t)2 , (5 t)3 ) es la reparametrizacion opuesta




a f que corresponde a la aplicacion : [5, 10] 7 [5, 10] dada por
t 7 5 t.
Ejercicio


Sea f : [a, b] R3 , una trayectoria regular y k una contante positiva
. Sea la aplicacion : 0, ba

k

7 [a, b] dada por (t) 7 kt + a.

106


Muestre que la trayectoria reparametizada f : 0, ba
 
k
R3


mantiene la trayectoria de f pero la recorre k veces mas rapido.

2.8. Longitud de Arco




Sea C una curva descrita por f (t) = (f1 (t), f2 (t), f3 (t), ..., fn (t)), defini-
da en un intervalo I = [a, b] en R.Sea P una particion de [a, b] y Lp
la longitud de la poligonal originada por P.

n



X
Lp = f (ti ) f (ti1 )

i=1

Para cada particion P se tiene una correspondiente Lp .



Definicion 2.8.1. La curva C descrita por f (t) = (f1 (t), f2 (t), f3 (t), ..., fn (t))
definida en [a, b] se dice que es rectificable si

{Lp /P es particion de [a, b]}

tiene una cota superior. Si C es rectificable entonces la longitud ` de C


es el supremo del conjunto de los Lp , es decir

` = sup {Lp /P es particion de [a, b]}




Definicion 2.8.2. Sea f : I R Rn una trayectoria regular de



clase C 1 . La longitud de f entre t = a, t = b denotada por `( f ) o
simplemente ` se define por:

Z b


`= f(t) dt

a

107
Ejemplo 10.

Si
c : [0, 2] R2 tal que
c (t) = (r cos t, r sin t). Es una trayectoria
regular? Cual es la longitud de la Curva asociada?
Solucion.
Como c(t) = (r sin t, r cos t) 6= (0, 0) t [0, 2] la trayectoria es
regular, luego

k
c(t)k = p(r cos t)2 + (r sin t)2 ) = r

Z 2 Z 2
`= k
c(t)k dt = rdt = 2r
0 0

Ejemplo 11.
Cual es la longitud del astroide dado por la ecuacion:
c (t) = (a cos3 t, a sin3 t) con
t [0, 2]?
Solucion:

Verifiquemos primeramente si el astroide es una curva regular:

c(t) = (3a cos2 t sin t, 3a cos t sin2 t)

Podemos inferir que el astroide no es una curva regular, pues



c(t) = (3a cos2 t sin t, 3a cos t sin2 t) = (0, 0) para t = 0, , , 3 , 2
2 2
pero dado que es simetrica respecto de ambos ejes podemos calcular su
longitud en el segmento del dominio 0, 2 y luego multiplicamos por


108
cuatro.

Z2 Z2 p
`

= k c(t)k dt = 9a2 cos4 t sin2 t + 9a2 sin4 t cos2 tdt
4
0 0

2 
sin2 t 2
Z 
3a
= 3a sin t cos tdt = 3a =
2 0 2
0
= ` = 6a

2.8.1. La Longitud de Arco como Parametro




Teorema 2.8.1. Sea f : I R Rn funcion de clase C 1 tal que



f(t) 6= 0 t I . Entonces la longitud s definida por
Zt

0
s(t) = f (u) du, con t I , de una curva puede introducirse como

0
un parametro de la curva y :

d

f
ds = 1

Demostracion:

Zt

ds


s(t) = f 0 (u) du s(t) = = f 0 (t) 0

dt
0

Claramente s = s(t) es monotona, estrictamente creciente y contnua,


por lo tanto s(t) tiene funcion inversa, que denominaremos por t(s) tal


que t(s) existe y es positiva para todo s con 0 s l( f ).

f (t(s)) = (f1 (t(s)), f2 (t(s)), ..., fn (t(s))

Derivando usando la regla de la cadena tenemos:

109


df
= (f1(t(s)) t(s), f2(t(s)) t(s), ..., fn(t(s)) t(s))
ds



df
d f dt dt
= = ds
dt ds dt

De donde se obtiene que:







d

f
f(t) f(t)
= = =1
ds ks(t)k s(t)

Observacion:



1. Del teorema anterior se desprende, la rapidez con que recorre f
es constante e igual a 1 si esta parametrizada con el parametro
longitud de arco.
2. Si
r (s) = (x(s), y(s), z(s)) describe una curva de R3 , y s es
parametro longitud de arco, entonces

d
r
Tb = es vector tangente unitario, ya que: Tb = db
r
=1

ds ds

Propiedades 2.8.1. La longitud de un arco es independiente de la


parametrizacion

Demostracion:



Sean f : [a, b] Rn un camino regular y f : [c, d] Rn una


reparametrizacion de f ; por lo cual existe de [c, d] en [a, b] de C1
que es sobreyectiva y (s) 6= 0 s [c, d]

Entonces:

110
 Zd Zd



s fo = f
(u) du = f
((u))
(u) du


c c
Zd


= f
((u)) (u)du si (u) > 0


c

 Zb



t = (u) s fo = f(t) dt = s( f )

a



s fo = s( f )

De la misma forma si (u) < 0, la parametrizacion invierte el


sentido y (c) = b, (d) = a

 Zd Zd




s f = f((u))(u) du = f((u)) ((u)) du

c c
Zd Zd




= f((u)) (u)du = f(t)) dt =

c c



s fo = s( f )

2.8.2. Parametrizacion por Longitud de Arco




Definicion 2.8.3. Sea f : I R Rn , funcion de clase C 1 y tal


que f(t) 6= 0 en I = [a, b] , se define s por

Zt

0
s = f (u) du = (t)

De la definicion de s se pueden hacer las siguientes precisiones:

Zt


ds

1. s = (t) = f(u) du = (t) = f(t)

dt
a

111
ds
2. Si t (a, b] dt
> 0 s = (t) es funcion estrictamente
creciente.
3. Por (2) s = (t) es biyectiva y por lo tanto invertible, sea t =
1 (s).



Por tanto, la parametrizacion de f en terminos s es:



f (s) = f (1 (s))

Ejemplo 12
Considere la trayectoria:
r (t) = (a cos t, asent) ,y reparametrice en
funcion de la longitud de arco s.
Solucion
La ecuacion cartesiana de la curva C es x2 + y 2 = a2 ,que corresponde
a una circunsferencia centrada en el origen de radio a.
Verifiquemos si es una curva regular, su derivada es:


r(t) = (a sin t, a cos t) 6= (0, 0) t


p
k r(t)k = a2 sin2 t + a2 cos2 t
k

r(t)k = a

Determinemos su longitud de arco,

Zt Zt
s = k

r(u)k du = adu = [au]t0 = at
0 0
s
s = at o t=
a

Por lo tanto,

r (s) = (a cos as , a sin as ) es una parametrizacion por lon-
gitud de arco de la circunferencia de radio a.

Observacion: No siempre es sencillo parametrizar por longitud de


arco, a modo de ejemplo veamos el siguiente caso.
Ejemplo 13

112
Considere la curva C curva descrita por la trayectoria r (t) = (t, t2 +1),
t [0, 3] , y parametrice en funcion de la longitud de arco s.
Solucion.
La curva corresponde a una parabola que tiene por ecuacion cartesiana:
y = x2 + 1, x [0, 3]

Determinemos la longitud de la curva:




r(t) = (1, 2t) kr(t)k = 1 + 4t2
Zt "
1 + 4u2 + 2u u 1 + 4u2 t
#
ln
s = 1 + 4u2 du = +
4 2
0 0
1 t
s = ln 1 + 4t2 + 2t + 1 + 4t2

4 2

En este caso no es posible obtener t = 1 (s) es decir despejar t en


funcion de s a partir de

1 2
t
s = (t) = ln 1 + 4t + 2t + 1 + 4t2

4 2

Luego no es posible hacer explicita la funcion t = g 1 (s).

2.9. Trayectorias y curvas


La aplicacion c : [a, b] R3 (o R2 ) continua , definida de un inter-
valo I al espacio R3 o al plano R2 , la llamaremos trayectoria. La imagen
C en R3 (o R2 ) de la trayectoria corresponde a lo que es una curva.

Definicion 2.9.1. Se llama curva de la trayectoria c : I R


R3 (R2 ) dada por

c (t) = (x(t), y(t), z(t)) al conjunto de imagenes de

c ;es decir

C=

c (t) R3 / t I = (x(t), y(t), z(t)) R3 : t I


113
Ejemplo 1: Sea c : R R3 una funcion definida por
c (t) =
(x0, y0, z0 )+t(v1, v2, v3 ) es una recta L en el espacio que pasa por (x0, y0, z0 )
y tiene la direccion v = (v1, v2, v3 ). A partir de esta funcion se deducen
las ecuaciones: x(t) = x0 + tv1, y(t) = y0 + tv2, z(t) = z0 + tv3 , que
se conocen como ecuaciones parametricas de una recta en el espa-
cio.Luego, la recta es la imagen de la trayectoria: c (t) = (x0, y0, z0 ) +
t(v1, v2, v3 ), tR
Generalmente, usamos t como variable independiente y c (t) senala la
posicion de una partcula en el espacio y t en este caso es la variable
tiempo.

Ejemplo 2: Sea c : [0, 2] R2 una funcion definida por



c (t) =
(cos t, sin t) ,cual es la curva asociada a esta trayectoria?
Solucion
A partir de las funciones parametricas tenemos , x (t) = cos t y (t) = sin t
que cumplen con
(x (t))2 + (y (t))2 = (cos t)2 + (sin t)2 = 1 t [0, 2]
Por lo tanto, es una trayectoria cuya imagen corresponde a una cir-
cunsferencia centrada en el origen y radio unitario.
Observe que c (t) = (cos 3t, sin 3t) con t 0, 2 describe la misma
 
3
curva. En este caso tenemos que trayectorias parametrizadas en forma
diferente describen una misma curva. La variable t se designa usual-
mente con el nombre de parametro. De ahora en adelante diremos que

c (t) = (x(t), y(t), z(t)) es una representacion parametrica de la curva


C, donde t es el parametro.
Definicion 2.9.2. Sea c (t):[a, b] R3 continua, una trayectoria que
describe la curva C.diremos que C en una curva cerrada si y solo si

c (a) =

c (b).

114
Una curva cerrada simple, o curva de Jordan es una curva cerrada que
tiene la propiedad :si c (t1 ) =
c (t2 ) = (t1 = t2 ) (t1 = a y
t2 = b).Como vemos esto ocurrre si la funcion c es inyectiva en I es



decir t1 , t2 I, t1 6= t2 = c (t1 ) 6= c (t2 )

Ejercicios 3
A partir de la representacion parametrica dada, describa las curvas y
encuentre las ecuaciones cartesianas de las mismas, si ello es posible:
1)
c (t) = (t, t) t0
2)
2
c (t) = (4 t , t) t [2, 3]
3)
c (t) = (cos t, sin t, 2t) 0 t
4)
c (t) = (a cos t, b sin t, 1) 0 t 2

2.10. Vectores Unitarios


2.10.1. Vector Tangente unitario
Sea
c (t) : [a, b] R3 una trayectoria y C la curva descrita por
c (t),




supongamos que c tiene derivada de tercer orden y que c(t) 6= 0
t [a, b] . Definimos el Vector Tangente unitario en un punto c (t) de
la trayectoria, como sigue:

c(t)
T(t)
b =

c(t)

En el caso que el parametro sea el parametro longitud de arco, entonces: T(s)


b =

c(s)

Propiedad
Sea
c (s) : [0, L] R3 una trayectoria parametrizada por longitud de
arco. Pruebe que los vectores T b y TT
b son ortogonales.
En efecto:
2
b T d bc
Como: T b =
T =1
b = ds
(TT) =0T
b T
b+ T b =0
b T
b T
2T b= 0

b T
T b= 0 ; es decir que T
b es ortogonal a T
b

115
2.10.2. Vector Normal

Tb 0 (s)
En todos los puntos donde T b 6= 0 definimos : N =
b ,Vector
0
T (s)
b

normal principal a la curva C en el punto


c (s).

2.10.3. Vector Binormal


Hay un tercer vector unitario que es perpendicular tanto a T b como a
N
b . Se define por el producto cruz de estos vectores, y es denominado
vector Binormal, denotado por: B b=T b N b
n o
El conjunto de vectores T, N, B forma un sistema de vectores
b b b
unitarios, ortogonales entre s, orientados positivamente en este orden
en cada punto
c (s) de la curva, es decir se cumple que:
b N
T b = 0, b B
N b = 0, b T
B b =0
b T
T b = 1, b N
N b = 1, b B
B b =1
b N
T b = B,b N b B b = T,
b b T
B b =N b
n o
En la medida que vara el conjunto de vectores Tb, N b, B
b ,este se de-
splaza a lo largo de la curva y se llama triedro movil.

Ejemplo 4
Considere la helice circular definida por:

c (t) = (3 cos t, 3 sin t, 4t), t >
0 . Hallar los vectores T , N , y B.
b b b

116
Solucion.

Sea
c (t) = (3 sin t, 3 cos t, 4) =



c (t) = 5

Zt
s
s= kc(u)k du = 5t t =
5
0
s s s


c (s) = (3 cos , 3 sin , 4 )
5 5 5

Por lo tanto: Tb =

c 0 (s) = ( 53 sin 5s , 3
5
cos 5s , 4
5
)

Del resultado anterior podemos inferir que Tb = 1

Ahora determinemos el vector normal a partir de su definicion: N


b =
Tb(s)

T (s)
b

b(s) = ( 3 cos s , 3 sin s , 0)


T 25 5 25 5
q q
3 s 2 3 s 2 3 2 3
  
T(s) = 25 cos 5 + 25 sin 5 = =
b
25 25

117
3 s 3 s
b = ( 25 cos 5 , 25 sin 5 , 0) = ( cos s , sin s , 0)
N 3 5 5
25

Del resultado anterior podemos inferir que: N =1
b

A partir de los vectores Tb y N b se tiene B b = Tb N b



bi j
b k
b

B 3
b = sin s 3 s 4 = 4 sin s i 4 cos s j + 3 j y podemos com-
5 5 5
cos 5 5 5 5 5 5 5
cos s sin s 0
5
5
probar que: B = 1
b

De los calculos anteriores podemos resumir que tenemos:


En cada punto
c (s) de la curva
Tb = ( 35 sin 5s , 53 cos 5s , 45 ), N
b = ( cos s , sin s , 0), B
5 5
b = ( 4 sin s , 4 cos s + 3 , 0)
5 5 5 5 5
s
como t = podemos reparametrizar la trayectoria en funcion de t,
5
como sigue
Tb(t) = ( 35 sin t, 35 cos t, 45 ), N b = ( 4 sin t, 4 cos t + 3 , 0)
b (t) = ( cos t, sin t, 0), B(t)
5 5 5

Como ejercicio dejamos que verifique que: Tb N


b =B b B
b N b Tb = N
b = Tb B b

2.11. Curvatura



Sea f : I Rn funcion vectorial dos veces diferenciable, parametriza-


da por el parametro longitud de arco. Al numero k(s) = f (s) se




le llama curvatura de f en el punto f (s).
Intuitivamente, de la definicion se infiere que la curvatura es una medida
de cuanto se doblauna curva, como una medida del alejamiento de la
curva de la recta tangente.
Ejemplo 5
Calcular la curvatura de la helice
c (t) = (3 cos t, 3 sin t, 4t), t 0

Solucion.

118
Sabemos que al parametrizar en funcion del arco se tiene
c (s) = (3 cos s , 3 sin s , 4 s )

5 5 5

3 s 3 s 3
k(s) = k c (s)k = ( 25 cos 5 , 25 sin 5 , 0) = 25



En
este caso, la curvatura es constante y como k(s) = k c (s)k =
T(s) ,significa que el vector tangente unitario T b (s) tiene la misma
b
rapidez de variacion de su direccion, en todos los puntos.

Ejemplo 6
Mostrar que la curvatura de una recta es cero.
Solucion

Sea P0 = (xo , yo , zo ) punto de la recta y



v = (a, b, c) su vector direccion,


entonces podemos escribir c (t) = (xo + ta, yo + bt, zo + ct)

c(t) = (a, b, c) = s = R a2 + b2 + c2 dt = ta2 + b2 + c2


t

0
s
s = t a2 + b 2 + c 2 o t=
a2 + b2 + c 2

Reparametrizando en funcion del arco se tiene

c (s) = (x + sa sb sc
o , yo + , zo + )
2 2
a +b +c 2 2 2
a +b +c 2 a + b2 + c 2
2

Derivando la expresion anterior:

c 0 (s) = 1
(a, b, c)
c (s) = (0, 0, 0)
2 2
a +b +c 2

k(s) = 0

Por lo tanto, la curvatura en cualquier punto es cero.

Crculo, circunferencia de curvatura

119
Si c (s) es un punto de la curva C y k la curvatura. La circunferencia
que es tangente la curva C en el punto c (s) de radio R = 1 se llama
k
circunferencia de curvatura y R radio de curvatura.
El centro de esta circunferencia se encuentra en la direccion del vector

c (s).

2.11.1. Calculo de curvatura usando parametro t


cualquiera en R3


Teorema 2.11.1. Sea f : I R R3 funcion vectorial, al menos


dos veces diferenciable ,tal que: f(t) 6= 0 t I. Entonces:



f
(t) f (t)


k(t) =
3

f(t)

Demostracion:
Sea s parametro longitud de arco tal que t = (s)

b (s) = f (t)
1

f(s) = T
f(s) =
f (t)
f (t) f (t)


00
Debemos calcular f (s). Usando la regla de la cadena se tiene:

120

d 1
dt dt 1 1
f (s) = f (t) con = =

dt f (t) ds ds ds
f (t)

dt


2

Por otra parte, como f (t) = f (t) f (t)

Se tiene:


d 





2 f (t) f (t) = f (t) f (t) + f (t) f (t)

dt




d  f (t) f (t)
f (t) =


dt f (t)



Volviendo a la derivada de f(s) :





d  
f (t) f (t) f (t) f (t)


dt
1
f (s) = 2
kf (t)k ds
dt






f (t) f (t)
f (t) f (t) f (t)

f (t)

1
f (s) =

2
f (t)

f (t)




2 

f (t) f (t) f (t) f (t) f (t)

f (s) =

4
f (t)



2

Como f (t) = f (t) f (t)





2 

 2
2 f (t) f (t) f (t) f (t) f (t)

f (s) =

4
f (t)



2



2 

2

2 


4 
f (t) f (t) 2 f (t) f (t) f (t) + f (t) f (t) f (t)

2

f (s) = 8
kf (t)k

121

2
2 
2
2 f(t) f (t) f (t) f (t)



f (s) =

6
f (t)



2
f (t) f (t)

2
f (s) =


6

f (t)
Por lo tanto:



f (t) f (t)


k(t) =
3

f (t)
Caso particular es el de curvas en el plano.
Ejemplo 7
Sea
r (t) = (x(t), y(t)) una trayectoria de R2 , en este caso

|x(t)y(t) x(t)y(t)|
k(t) =
[(x(t))2 + (y(t))2 ]3/2

Solucion
Se deja al alumno, como indicacion se sugiere poner



r (t) f (t) = (x(t), y(t), 0)

y aplicar la formula del teorema precedente

Ejemplo 8 Calcule la curvatura de la espiral de Arqumides = a.

Solucion.-
x = cos
= r() = (a cos , asen)
y = sin

se considera que x() = a cos y y() = asen, derivando estas


funciones se tiene

x() = a cos asen y y() = asen + a cos


x() = 2asen a cos y y() = 2a cos asen

122
Figura 2.1: Espiral de Arqumides

efectuando los productos y simplificando se tiene


x() y() x()y() = 2a2 + a2 2 = a2 (2 + 2 )

por otro lado tambien


(x(t))2 + (y(t))2 = a2 + a2 2 = (1 + 2 )a2

reemplazando en la formula se tiene


|a2 (2 + 2 )| 2 + 2
k() = y simplificando k() =
[(1 + 2 )a2 ]3/2 a(1 + 2 )3/2

para cualquier positivo.


2 + 2
Observese que lm = 0, esto significa que la curvatura
a(1 + 2 )3/2
de la espiral de Arqumides muy lejos del origen tiende a ser casi una
recta.

2.12. Planos por un punto de la curva


Sea
c : I IR R3 un camino regular dos veces diferenciable


y P0 = c (t0 ) (donde t puede ser parametro longitud de arco).



Si T , N y B son los vectores tangente, normal y binormal de la
curva en

c (t0 ) = (x0 , y0 , z0 ).Podemos definir los siguientes planos por


P0 = c (t0 )

123
2.12.1. Plano Osculador



Plano determinado por T y N en el P0 cuya ecuacion es:



(x x0 , y y0 , z z0 ) B = 0

2.12.2. Plano Normal




Plano determinado por N y B en el punto P0 cuya ecuacion es:


(x x0 , y y0 , z z0 ) T = 0

2.12.3. Plano Rectificante






Plano determinado por B y T en el punto P0 cuya ecuacion es:



(x x0 , y y0 , z z0 ) N = 0



Con estos mismos vectores en el punto P0 se definen las rectas:

124
2.12.4. Recta Tangente
Cuya ecuacion vectorial es:





P (t) = P0 + t T

Es decir (x, y, z) = (x0 , y0 , z0 ) + t (T1 , T2 , T2 ) , t IR

2.12.5. Recta Normal


Cuya ecuacion vectorial es:





P (t) = P 0 + t N

Es decir (x, y, z) = (x0 , y0 , z0 ) + t (N1 , N2 , N2 ) , t IR

2.12.6. Recta Binormal


Cuya ecuacion vectorial es:





P (t) = P 0 + t B

Es decir (x, y, z) = (x0 , y0 , z0 ) + t (B1 , B2 , B2 ) , t IR

Ejemplo 9

Considere el camino regular definido por


c (t) = (t3 , t2 , t). Obtener:

a) las ecuaciones de los planos Osculador, Normal, Rectificante y


b) las rectas tangente, normal y binormal a esta curva en el punto
(1, 1, 1).

Solucion
Previo a responder este problema observemos que para determinar los
planos

125
y rectas pedidas no es obligatorio trabajar con los vectores unitarios
por lo cual util observar que:

c(t) es vector paralelo a


c(t) =
T (t) , es decir:

T

c(t)

c (t) es vector paralelo a B (t), es decir : c (t) =
c(t)
B
(
c(t) c(t) es vector paralelo a
c (t))
N (t) es decir :

(
c(t) c(t) =
c (t))
N

De este modo, en nuestro problema particular.

c(t) = (3t2 , 2t, 1) y


c(t) = (6t, 2, 0) Lo que implica al evualar en t =
1 queda:

c(1) = (3, 2, 1)

i j k




y c(1) = (6, 2, 0) por consiguiente c(1) c (1) = 3 2

1

=

6 2 0
2i + 6j 6k = 2(i 3j + 3k)

i j k
(
c(1)
c (1)
c(1) = 2 6 6 = 18i 16j 22k = 2(9i

3 2 1
8j 11k)

As estamos en condiciones de calcular en el punto (1, 1, 1):


Plano Rectificante
(x 1, y 1, z 1) (9, 8, 11) = 0
9x 8y 11z + 10 = 0 ecuacion del plano
Plano Osculador
(x 1, y 1, z 1) (1, 3, 3) = 0
x 3y + 2z 1 = 0 ecuacion del plano
Plano Normal

(x 1, y 1, z 1) (3, 2, 1) = 0

126
3 x + 2y + z 6 = 0 ecuacion del plano

x1 y1 z1
Recta tangente: = =
3 2 1
x1 y1 z1
Recta Normal: = =
9 8 11

x1 y1 z1
Recta binormal: = =
1 3 3

2.13. Torsion
Si una partcula se mueve siguiendo un camino C, el plano osculador
en un punto P de la curva es un buen referente para observar el giro
o torsimiento de la curva, que no es otra cosa que la medida del ale-
jamiento de la curva del plano osculador en una vecindad del punto
P.
El comportamiento de la derivada del vector Binormal B(s)
b respecto


del parametro longitud de arco da la razon de cambio del vector B
respecto del plano osculador
Por otro lado se puede obsevar que:
B
b (s)B
b (s) = 1 = B(s)
b B
b (s)+Bb (s)B(s)
b = 0 = B(s)
b B
b (s) =
0 por lo que afirmamos que B(s)
b es perpendicular a B
b (s) .

b (s) Tb (s) = 0 = B
Ademas como B b (s) T(s)
b + B(s)
b Tb (s) = 0
T(s) = N b (s) T(s)
b (s) por lo que B = 0 = ddsB Tb = 0 por lo
b b b

que se deduce ddsB es perpendicular a Tb. Estas dos ultimas conclusiones


b

senalan que ddsB es un vector perpendicular a B b y a Tb, esto significa


b

entonces que ddsB es paralelo a N b ,es decir normal al plano rectificante


b

en el punto P. Este razonamiento nos permite formular la siguiente


definicion.
Sea c : I IR R3 un camino regular tres veces diferenciable


parametrizado por longitud de arco y tal que f(s) 6= 0 s I ( es
decir con curvatura no nula). Al numero real (s) tal que

127
dB
b
= (s)N
b (s)
ds

se llama Torsion de la curva en el punto c (s). Entonces, se tiene que


dB b
b
(s) = N (s)
ds
Observaciones:
dB
b
1) El signo menos tiene el proposito de que si (s) > 0,entonces
ds
tiene la direccion de Nb (s). As cuando P se mueve sobre la curva
en una direccion positiva Tb, B(s)
b gira alrededor de Tb en el mismo
sentido que un tirabuzon diestro que avanza en la direccion de Tb, como
se muestra en la figura adjunta.


2) La k (s)k = ddsB es una medida de la rapidez con que la curva se
b
despega del plano osculador
3) La primera consecuencia importante, de lo anterior, es que si B(s)
b
no vara ( B
b constante) significa que la curva se mantiene en el plano
dB
b
osculador y = 0.
ds
Afirmamos, entonces que:
c (s) describe una curva plana si y
solo si su torsion es cero

2.13.1. Calculo de la torsion usando parametro t


cualquiera (en R3 )


Teorema 2.13.1. Sea f : I R R3 funcion vectorial al menos



tres veces diferenciable tal que: f ( t) 6= 0 y f 00 ( t) 6= 0 t I.
Entonces:

128
h
0
i
f (t) f 00 (t) f 000 (t)
(t) =
0
00 2
f (t) f (t)

La demostracion de esta formula la dejamos como interesante aunque


largo ejercicio para el lector.
Ejemplo 10
La curva C resulta de la interseccion de las superficies z = 2x2 y y
z = x + y. Verifique usando la formula que (1) = 0.
Solucion.-
Sea z = 2x2 y y z = x + y = x + y = 2x2 y de aqu despejando
x
y se tiene y = 2
2x 1
Haciendo x = t se tiene una parametrizacion para la curva interseccion
de las superficies:
2t3
 

t
c (t) = t, 2 ,
2t 1 2t2 1
Las derivadas son:
2 2 2
 
c(t) = 1, (2t + 1) , 2t (2t 3)

(2t2 1)2 (2t2 1)2
4t(2t2 + 3) 4t(2t2 + 3)
 

c (t) = 0, ,
(2t2 1)3 (2t2 1)3
12(4t4 + 12t2 + 1) 12(4t4 + 12t2 + 1)
 

c (t) = 0, ,
(2t2 1)4 (2t2 1)4
En P = (1, 1, 2), t = 1 y evaluando las derivadas en t = 1 se tiene:

c (1) = (1, 3, 2) ,

c (1) = (0, 20, 20) ,

c (1) = (0, 204, 204)


1 3 2




c (t) c (t) c (t) = 0 20
20 = 0. = (1) = 0.

0 204 204
Comentario.- El resultado no poda ser otro ya que la curva C esta en
un plano, el plano z = x + y.

129
2.14. Formulas de Frenet

Fueron obtenidas por el matematico Frances Jean Frederic Frenet en


1847 en su tesis doctoral, hoy se les conoce como las formulas de Frenet.
dTb
1) = kNb
ds
dNb
2) = k Tb + B
b
ds

dB
b
3) = N
b
ds

Demostracion.-
1) Igualdad establecida en la fundamentacion de la definicion de la
torsion
2) De las respectivas definiciones se tiene:
dT b
=

c(s) y k(s) =
ds

N = b , T(t)
b b c (s)
ddsT

dT
dT
b
b
N = ds
= = ddsT N
b b b
dTb
ds ds

dT
=
c(s) y k(s) =
b
=

T(t)
b c (s) = c (s) N
b
ds
= k(s) N
b
dT
b
= k N.
c
ds
2) A partir de Nb =Bb Tb y diferenciando se tiene:

dN dB dT
= Tb + B
b b ) Tb + B
= ( N b (k N
b)
b b b
ds ds ds
b Tb) + k(B
= (N b N b k Tb.
b) = B

dN
b
b k Tb
= B
ds
130
Finalmente, las formulas de Frenet se pueden resumir en una repre-
sentacion matricial, donde la matriz de transformacion es antisimetri-
ca.
b
Tb0 (s) 0 k 0 T
b0
N (s) = k 0 N
b
0
B (s)
b 0 0 B
b

2.15. Aplicaciones de Funciones Vectori-


ales y Curvas
Cuando una partcula se mueve en el espacio y describe una curva en
tres dimensiones, es conveniente fijar a la partcula un sistema de coor-
denadas rectangular, de tal manera que una coordenada sea tangente
a la trayectoria, otra sea normal a ella y una tercera que sea ortogonal
a las dos primeras. Es decir , las coordenadas tangencial, normal y bi-
normal constituyenn un sistema
o de referencia cuya base esta constituida
por los vectores Tb, N b, B
b .

Las ecuaciones de la velocidad y la aceleracion pueden obtenerse como


sigue:

v =

r 0 = d r ds = s0 Tb
ds dt

1
a =

v0=
(s0 )2 b
00 00
r 00 = s Tb + s0 Tb0 = s Tb +
N donde =
k
Puede concluirse que:

1. la velocidad es tangente a la trayectoria y su magnitud es s0

131
2. la aceleracion tiene dos componentes.
a) Una es la componente tangencial , de magnitud (s00 ),debida
al cambio de magnitud de la velocidad.
b) La otra es la componente normal, de magnitud (s0 )2 /, hacia
el centro de curvatura. Esta componente se debe al cambio de
direccion de la velocidad.

Tanto la velocidad como la aceleracion siempre estan sobre el plano


osculador.

2.15.1. Problemas
Problema 1 Una partcula se mueve a lo largo de la curva: y = 2xx2
con una componente horizontal de la velocidad de 4m/s, es decir vx =
4m/s. Hallar las componentes tangencial y normal de la aceleracion en
el punto (1, 1), si en el instante t = 0, x(0) = 0
Solucion:
La funcion de posicion de la partcula es:


r (t) = (x(t), 2x(t) x2 (t)), t I

y derivando obtenemos su velocidad



v (t) = (x0(t), 2x0(t) 2x(t)x0(t)).
Como la componente horizontal de la velocidad es 4m/s entonces
x0(t) = 4 , integrando x(t) = 4t+C = x(0) = 0+C = 0 = C = 0
Luego r (t) = (4t, 8t 16t2 ) =

v (t) =

r 0 (t) = (4, 8 32t) =

a =
r 00 (t) = (0, 32)



r 0 (1/4)
Como r 0 (1/4) = (4, 0), entonces T (1/4) = = (1, 0) y
k
r 0 (1/4)k

N (1/4) = (0, 1) .
Luego, las componentes tangencial y normal de la aceleracion se
obtienen proyectando el vector aceleracion en esas direcciones.


a =
T
a (1/4) T (1/4) = (0, 32) (1, 0) = 0 y


aN =

a (1/4) N (1/4) = (0, 32) (0, 1) = 32

132
Problema 2
a) Una partcula se mueve en el espacio de tal forma que en el in-
stante t = t0 ,el vector velocidad es (1, 1, 1) ,y el vector aceleracion es
(2, 1, 0) .Hallar las componentes tangencial y normal de la aceleracion
en el instante t = t0 .
b) Si la partcula describe una curva en el espacio de tal manera que
los vectores velocidad y aceleracion tienen siempre magnitud constante,
pruebe que la curvatura es constante en cada punto de la curva.

Solucion:

a) Puesto que , se conoce la velocidad tenemos:




v (t0 ) = k
v (t0 )k Tb = (1, 1, 1) = k
v (t0 )k = 3

a (t0 ) = aT (t0 )Tb + aN (t0 )N b =


v (t0 )
a (t0 ) = aT (t0 ) k
v (t0 )k Tb Tb + aN (t0 ) k
v (t0 )k Tb N
b



v (t0 ) a (t0 ) (1, 1, 1) (2, 1, 0)
aT (t0 ) =
=
k v (t0 )k 3

Por otra parte:

v (t0 )

a (t0 ) = aT (t0 ) k
v (t0 )k Tb Tb + aN (t0 ) k
v (t0 )k Tb N
b

k
v (t0 ) a (t0 )k k(1, 1, 1) (2, 1, 0)k 3 2
aN (t0 ) = = =
k v (t0 )k 3 2

k

v (t)
a (t)k
b) La curvatura esta dada por: k (t) =
3
k v (t)k

Supongamos que kv (t)k = c1 yk


a (t)k = c2
Dado que k

v (t)k = c ,al derivar esta expresion , se tiene
1

v (t)

a (t) =
0,
entonces

v (t) es ortogonal con

a (t).
As
k

v (t)
a (t)k k

v (t)k k
a (t)k k
a (t)k c2
k (t) =
3 =
3 = 2 = 2
k v (t)k k v (t)k k v (t)k c1

Problema 3

133
La aceleracion de una partcula en funcion del tiempo viene dada por

a (t) = (2t, 3t2 , 4t3 ) . Si en el instante t = 0 esta en el origen de coor-


denadas con velocidad inicial
v (0) = (1, 0, 1).

a) Hallar la velocidad y la posicion en cualquier instante de tiempo.


b) Hallar el valor de t en el que la partcula pasa por el plano xy.

Solucion:

a) Sea
r (t) la posicion de la partcula en el instante t. Entonces


r (t) =
0
v (t) y r (t) =
00
a (t).

Por lo tanto:
Zt Zt Zt
d

2u, 3u2 , 4u3 du

v = a (u)du =
0 0 0
t
= (u , u , u ) 0 = (t2 , t3 , t4 )
2 3 4

Entonces:


v (t)

v (0) = (t2 , t3 , t4 ) =

v (t) = (t2 + 1, t3 , t4 1)

Ahora:
Zt Zt Zt
d

r (t) =

v (u)du = (u2 + 1, u3 , u4 1) du
0 0 0
3 4

t t t5
r t)

r (0) = ( + t, , t) =
3 4 5

t3 t4 t5
r t) = ( + t, , t)
3 4 5

t5
b) La partcula pasa por el plano xy cuando z = 0 = t =
5
0 = t = 0 y t = 5.

Problema 4
2 2

1 d y/dx
Usando la ecuacion = .
(1 + (dy/dx)2 )3/2

134
a) Hallar el radio de la curvatura de la curva representada por y =
x2 + 2x en el punto x = 2.
x2
b) Hallar los radios de la curvatura de la curva representada por +
16
y2
= 1 en los puntos x = 0 y x = 4 respectivamente.
9
Solucion:
a) La curva representada por y = x2 + 2x corresponde a una parabola
que se abre hacia arriba cuyo vertice esta en el punto (1, 1).

Usando la ecuacion calculemos el radio de curvatura: y = x2 + 2x =

dy d y 2
dx
= 2x + 2 = dx 2 = 2

1 2
= .As, el radio de curvatura en x = 0 es:
(1 + 4(x + 1)2 )3/2

1 2
=
(37)3/2
x2 y2
b) La curva representada por + = 1 corresponde a una elipse
16 9
con semiejes mayor a = 4 y menor b = 3 respectivamente.
r dy 3 v x
x2 = 16
Derivando la ecuacion y = 3 1 se tiene dx u
u x2 =
16 t
1
16
d2 y 3 1
= 16 3/2
dx2 x2

1
16

135


1 3 1
Por lo tanto , el radio de curvatura queda =

16 7x2 3/2

(1 )
256

1 3 16
As, el radio de curvatura en x = 0 es: = = =
16 3

Problema 5

El vector posicion de una partcula movil a lo largo de una curva,


esta dado por:
r = atbi + b cos tb
j + b sin tk
b en donde a y b son
1 2

d r

constante usando la ecuacion: = 2 donde es el radio de
ds
curvatura. Hallar la curvatura de la curva.

Solucion:

Parametricemos , el vector posicion en funcion de la longitud de arco


s.

136
Zt Zt t
s = k

r 0 ( )k d = k(a, b sin t, b cos t)k d = a2 + b 2

0
0 0
s
= t=
a2 + b 2
 

s s s
r (s) = a , b cos , b sin
a2 + b 2 a2 + b 2 a2 + b 2
 

a b s b s
r 0 (s) = , sin , cos
a2 + b2 a2 + b2 a2 + b 2 a2 + b 2 a2 + b 2
 

b s b s
r 00 (s) = 0, 2 2
sin , 2 2
cos
a +b a +b a +b
2 2 a + b2
2

Por consiguiente, el radio de curvatura es:

1 d2

r b a2 + b 2
= 2 = 2 = = = constante
ds a + b2 b

Problema 6
En un cierto instante, las magnitudes de la velocidad y la aceleracion
de un proyectil son 20 m/s y 80 m/s respectivamente, y el angulo de
inclinacion entre ellas es de 30o .Determinar el radio de curvatura de la
trayectoria en ese instante

Solucion:
Sean Tb y N b los vectores unitarios en las direcciones tangencial y
normal respectivamente. Como la velocidad es tangente a la trayectoria
tenemos que

v = k
v k Tb = 20 (m/s) Tb
Las componentes tangencial y normal de la aceleracion son:

137

aT = a cos = 80 cos 30o = 40 3 m/s2


aN = asen = 80 sen30o = 40 m/s2






a = 40 3 m/s2 Tb + 40 m/s2 N
 
b

El radio de curvatura puede calcularse a partir de:

k
k

2 2
vk vk 400
aN = = = = = 10m
aN 40
Problema 7
La aceleracion de una partcula, esta dada por:

a = 2tbi + 3t2b
j + 2b
k en

2
donde a esta m/s y t esta en s. Si en el instante t = 0, v = 0.

A partir de la componente normal de la aceleracion hallar el radio de
curvatura de la trayectoria de la partcula en el instante t = 1 s.
Solucion:
En primera instancia determinemos la velocidad de la partcula
Zt Zt Zt 
d



v = a (u)du = 2bi + 3 2b
j + 2b
k d = t2bi + t3b
j + 2tb
k
0 0 0

v (t)

v (0) = t2bi + t3b k =
j + 2tb
v (t) = t2bi + t3b
j + 2tb
k

Ahora evaluemos la velocidad y la aceleracion respectivamente, en el


instante t = 1s :

v (1) = bi + b
j + 2b
k

a (1) = 2bi + 3b
j + 2b
k
As la componente normal de la aceleracion en el instante t = 1 es
k
k
v (1)
k

2 3
v (1)k a (1)k v (1)k
aN (1) = = = = =
k

v (1)k kv (1)
a (1)k
3
6
k(4, 2, 1)k
3 3
6 6
Por lo tanto el radio de curvatura es: = = m
k(4, 2, 1)k 21
Problema 8

138
El vector posicion de una partcula se describe por la funcion vectorial

r (t) = a cos(wt)bi + a sin (wt) bj + bt2 b


k ,donde a y b son constantes
positivas.
Expresar la velocidad y la aceleracion en sus componentes tangencial
y normal.
Solucion:
Calculemos la velocidad y la aceleracion en coordenadas rectangulares.

v =
r 0 (t) = awsen(wt)bi + aw cos (wt) bj + 2btb
k

a =

r 00 (t) = aw2 cos(wt)bi aw2 sin (wt) b
j + 2bb
k
A continuacion , podemos calcular la rapidez y la magnitud de la acel-
eracion

k
vk= v v = a2 w2 + 4b2 t2

k
ak= a a = a2 w4 + 4b2
Ahora, estamos en condiciones de expresar la velocidad y la aceleracion
en sus componentes tangencial y normal.


v = kv k Tb = a2 w2 + 4b2 t2 Tb


v (t)
a (t) 4b2 t
aT (t) = =
k
v (t0 )k a2 w2 + 4b2 t2

k
v (t) a (t)k a4 w6 + 4a2 b2 w4 t2 + 4a2 b2 w2
aN (t) = =
k v (t0 )k a2 w2 + 4b2 t2
Por tanto, la aceleracion es:
2

4b t a4 w6 + 4a2 b2 w4 t2 + 4a2 b2 w2 b
a = Tb + N
a2 w2 + 4b2 t2 a2 w2 + 4b2 t2

Problema 9
La posicion de una partcula sobre la periferia de una rueda de 8 cm
de diametro se especifica como s(t) = t3 4t2 + 8t en donde s es la
longitud del arco, medida en cm, a lo largo de la periferia circular, a
partir de un origen conveniente, y t es el tiempo, en segundos.
Hallar la magnitud de la aceleracion k
a ken el instante en que la mag-
nitud de la aceleracion tangencial es de 4cm/s2 .
Solucion:
A partir de la ecuacion dada s(t) = t3 4t2 + 8t, obtenemos
s0 (t) = k

v k = 3t2 8t + 8

139
00
s (t) = aN (t) = 6t 8
El radio de curvatura en este caso es el radio de la rueda. As que:
= 28 = 4cm.
La aceleracion es

2

00 (s0 (t))2 b (3t2 8t + 8) b
a (t) = s (t)Tb + N = (6t 8)Tb + N
4

En el instante que aT = 4, 6t 8 = 4, de donde t = 2s. Luego, en el


instante t = 2s.

a (2) = 4Tb + 4Nb


Por lo tanto, la magnitud de la aceleracion es:

ak=
k
a a = 4 2cm/s2

2.16. Ejercicios resueltos


Ejercicio 1
Un par de trayectorias de [0, ) en R3 se definen por
c (t) = (cos t, sin t, bt)

y
r (t) = (1, 0, t). Responda las siguientes preguntas:
a) Se intersectan las curvas generadas por
c (t) y

r (t)?
b) Si estas trayectorias representan el desplazamiento de un par de
partculas.
En que puntos ,si los hay, estas partculas se encuentran?

Solucion:
a)
c (t) es la ecuacion de la helice ascendente sobre el manto del cilindro

x2 + y 2 = 1 y cada vuelta demora 2 unidades de tiempo. Asimismo,

r (t) = (1, 0, t) es una recta vertical paralela al eje axial del cilindro ,
que
esta sobre el manto de x2 + y 2 = 1 y pasa por (1, 0, 0).
Igualando las primeras componentes cost = 1 ,obtenemos que las curvas
se
intersectan para t = 0, 2, 4, . . .

140
b) Igualando las terceras componentes bt = t = Si b = 1,entonces las
partculas se encuentran en los puntos (1, 0, 0), (1, 0, 2), ..., (1, 0, 2n)
con n Z+
0.

Ejercicio 2
La curva C es definida a partir de la trayectoria
c (t) = (2 cos(t), 2 sin(t), t)

con 0 t 2 . Describa la representacion grafica de C y pruebe que


si se usa como parametro la longitud de arco s , el vector tangente a
la
curva es un vector unitario.
Solucion:
Por la continuidad de las funciones x(t) = 2cos(t), y(t) = 2sin(t) y
z(t) = t
podemos inferir que C parte del punto c (0) = (x(0), y(0), z(0)) =
(2, 0, 0)
y termina en
c (2) = (x(2), y(2), z(2)) = (2, 0, 2); ademas que
la
curva asciende a traves del manto del cilindro x2 + y 2 = 4 porque
[x(t)]2 + [y(t)]2 = [2 cos(t)]2 + [2 sin(t)]2 = 4 como se ilustra en la figura

El vector posicion de esta curva es c (t) = (2cos(t), 2sin(t), t). El vector

tangente es
c 0 (t) = (2 sin(t), 2 cos(t), 1) (a) y la longitud del vector

tangente es
kc 0 (t)k = p[2 sin(t)]2 + [2 cos(t)]2 + 1 = 5 (b)

141
La longitud total de esta curva es
Z 2

0
Z 2
Longitud = k c (t)k dt = 5dt = 2 5
0 0

Rt
Definimos s(t) = 0
kc0 (u)k du para t [0, 2] = s(t) es la longitud
de
curva C desde (2, 0, 0) hasta (x(t), y(t), z(t)).
Claramente s(t) es continua y estrictamente creciente en [0, 2] la ecuacion
s = s(t) puede resolverse para t como una funcion de s, es decir t =
t(s) (c)
En este caso t = s as es que
5
     

c (s) =
c (t(s)) = s s s
2 cos , 2 sin ,
5 5 5

es vector posicion en terminos de s, derivando


     

c 0 (s) = c 0 (t(s)) = 2 sin s 1
, 2 cos
s 1 1
,
5 5 5 5 5
     
2 s s 1
= sin , cos , (2.16.1)
5 5 5 2

Calculando su modulo
s  2   2
c 0 (s)k = 2
k sin
s
+ cos
s
+
1
5 5 5 4
r
2 1
= 1+ =1
5 4

Por lo tanto,
c 0 (s) es vector unitario.

Especificaciones:
a) Si c (t) describe la trayectoria de una partcula en el espacio, el
vector

c 0 (t) = (2 sin(t), 2 cos(t), 1) es la velocidad con que se desplaza la


partcula
por la curva C en el punto c (t), en el instante t.

142
c 0 (t)k = 5 es la rapidez con que se desplaza la partcula t, lo
b)k
que
significa que la partcula se mueve con rapidez constante t.
c) Asimismo, la longitud del arco es
Z t
s(t) = k
c 0 (t)k du
Z0 t
= 5du = 5t
0

s
s= 5t = t =
5

En general y en teora la ecuacion s = s(t) siempre se puede resolver


para t
en terminos de s, es decir tener t = t(s). En la practica existen casos
en
los que por razones algebraicas no se puede tener t = t(s) Conoces
algun
caso?

Ejercicio 3
Una partcula se mueve en el espacio con vector posicion
 3



2 2

r (t) = t A + t2 B + 2 t AB
3



donde A y B son dos vectores unitarios fijos que forman angulo
de 3
radianes.
Calcular la velocidad de la partcula en el instante t y determinar en
cuanto
tiempo recorre una distancia de 12 unidades de longitud de arco desde
la
posicion en t = 0.
Solucion:

143
La velocidad es el vector

v (t) =

r 0 (t) donde
  12



2 2

r 0 (t) = A + 2t B + 3 t AB
3 3

Por lo tanto
 1



2 2

v (t) = A + 2t B + 2 t AB
3

Para la segunda parte del problema usaremos


Z t q
s(t) =
0

k r (t)k du con k r (t)k =
0
r 0 (t)

r 0 (t)
0

Calculando

r 0 (t)

r 0 (t) =
  12 !   12 !


2



2


= A + 2t B + 2 t AB A + 2t B + 2 t AB
3 3
= 1 + 4t + 4t2 (a)

por lo tanto

r 0 (t)

r 0 (t) = (1 + 2t)2
q


= k r (t)k = (1 + 2t)2 = 1 + 2t
0

as que
Z t
s(t) = (1 + 2u) du
0
t
= u + u2 0 = t + t2


De acuerdo a lo que se pide en el problema s(t) = 12 = t2 + t 12 = 0


Las soluciones de esta ecuacion son t1 = 3 y t2 = 4 por la naturaleza
del
problema debe ser t 0 luego desconsideramos t2 = 4. En conse-
cuencia, 3
unidades de tiempo son necesarios

144
Especificaciones:


 21
 
 21


2 2


A + 2t B + 2 3
t A B A + 2t B + 2 3
t A B =






1
A A + 2t B A + 2 23 t 2 A B A + 2t A B + 4t2 B B

a) 2
 12
2
 12



+2(2t) 3
t A B B + 2 3
t A A B
1



 
 

+2(2t) 32 t 2 B A B + 4 32 t A B A B

 

Como:




A A = 1, B B = 1, A B = cos 3 = 12



A A B = 0, B A B = 0, y

    2
2
A B A B = A B = 23 = 3
4

Se concluye que:


8 3
r 0 (t)

r 0 (t) = 1 + t + 0 + t + 4t2 + 0 + 0 + 0 + +
3 4
= 1 + 4t + 4t2 = (1 + 2t)2

Ejercicio 4


Sea F : I Rn una funcion velocidad dos veces diferenciable.
a) Pruebe que la aceleracion se puede escribir como combinacion lineal
de los vectores unitarios Tb y N
b , especficamente se pide establecer que:
 2
d2 s b
 

ds
a (t) = 2
T + k(t) N
b
dt dt
= aT Tb + aN N
b

b) Aplicando a) calcule las componentes tangencial y normal del vector


aceleracion correspondiente a

r (t) = (t, cos2 t, sin2 t).
Solucion:


a) Sea

g una reparametrizacion de F por longitud de arco entonces

F (t) =

g (s) si y solo si t = t(s) s = s(t).
Aplicando la regla de la cadena, queda


ds
F (t) =

g (s(t)) = F 0 (t) =

g 0 (s) s0 (t) = Tb
dt
145
derivando nuevamente respecto de t


00
F (t) =

g 00 (s) [s0 (t)] +

2
g 0 (s) s00 (t)

Por definicion k = k(s) = k



g 00 (s)k es la, curvatura de C en g(s).
por otro lado

dTb
b = ds =
N
g 00 (s) = k

g 00 (s)k N
b
dTb
ds

Relacionando las igualdades anteriores


00   ds 2 d2 s




00
a (t) = F (t) = k g (s)k N b + Tb 2
dt dt
2
d2 s b
 
ds
= 2
T + k
g 00 (s)k Nb
dt dt
 2
d2 s b ds
= 2
T +k Nb
dt dt

lo que significa que


2
d2 s


ds
a (t) = 2 Tb + k N
b
dt dt

b) En la aplicacion de a) en
r (t) = t, cos2 t, sin2 t


tenemos r 0 (t) = (1, sin(2t), sin(2t)) .


Usando la expresion anterior, se tiene

4 sin(2t) cos(2t)
q
s (t) = k
0
r 0 (t)k = 1 + 2 sin2 (2t) = s00 (t) = p
1 + 2 sin2 (2t)

Luego, la componente tangencial aT es

2 sin(2t)
aT = s00 = p
1 + 2 sin2 (2t)

146
Ademas,
r 00 (t) = (0, 2 cos(2t), 2 cos(2t)) y haciendo el producto cruz
entre

r 0 (t) y

r 00 (t) se tiene


r 0 (t)

r 00 (t) = 2 cos(2t)b
j 2 cos(2t)b
k (e)

k

r 0 (t)
p p
r 00 (t)k = 4 cos2 (2t) + 4 cos2 (2t) = 8 cos2 (2t)

= 2 2 |cos(2t)|

Como la curvatura k en funcion de t es

k

r 0 (t)
r 00 (t)k
k=
k 3
r 0 (t)k
reemplazando
2 2 |cos(2t)|
k= 3
1 + 2 sin2 (2t) 2

de  2
ds 2 2 |cos(2t)| 2

aN = k(t) = 3 1 + 2 sin (2t)
dt 1 + 2 sin2 (2t) 2


Por lo tanto, la componente normal de la aceleracion en este caso es



2 2 |cos(2t)|
aN = p
1 + 2 sin2 (2t)

Ejercicio 5
Sea la trayectoria regular

r : R R3 , definida por:


4au2 2au(1 u2 ) a(1 u2 )
r (u) = ( , , ), a > 0
(1 + u2 )2 (1 + u2 )2 1 + u2

a) Pruebe que la funcion : R ], [ tal que t = (u) =


2 arctan u

147
define la reparametrizacion de

r,



r : ], [ R tal que r(t) = (a sin2 t, a sin(t) cos(t), a cos t)

b) Verifique que

r (t) esta contenida en una superficie esferica.
c) Probar que los planos normales a la curva descrita por r(t) pasan
por el
centro de la esfera.

Solucion

De (u) = 2 arctan u se tiene que:


0 (u) = 2
1+u2
> 0 u R, por lo tanto, es estrictamente creciente en R
y como es continua, es biyectiva, y por tanto, invertible. Ademas
t
1 (t) = tan( )
2

1 : ], [ R es de clase C 1 en ], [ .
Por lo anterior, se puede definir r(t) :


t
r (t) = (

r 1 )(t) =

r (1 (t)) =

r (tan( ))
2

4a tan2 ( 2t ) 2a tan( 2t )(1 tan2 ( 2t ) a(1 tan2 ( 2t ))


=

r (t) = ( , ,
(1 + tan2 ( 2t ))2 (1 + tan2 ( 2t ))2 (1 + tan2 ( 2t ))

Sustituyendo
q
1cos t 2 cos t
1cos t
4a( 1+cos ) 2a ( ) 2 cos t
a( 1+cos )


r (t) = ( t
,
1+cos t 1+cos t
, t
)
2 2 2
( 1+cos t
)2 ( 1+cos t
)2 ( 1+cos t
)

= (a(1 cos2 t), a cos t 1 cos2 t, a cos t)
Por lo tanto, la reparametrizacion la podemos definir por:



r (t) = (a sin2 t, a sin(t) cos(t), a cos t)

148
b) Consideremos ahora

x(t) = a sin2 t, y(t) = a sin t cos t, z(t) = a cos t =

x2 + y 2 + z 2 = a2 sin4 t + a2 sin2 t cos2 t + a2 cos2 t

= a2 sin2 t(sin2 t + cos2 t) + a2 cos2 t

= a2 (sin2 t + cos2 t) = a2

la curva esta en la esfera x2 + y 2 + z 2 = a2

c) Se

r (t0 ) un punto cualquiera de la curva.

La ecuacion del plano normal es:

N : ((x, y, z) r(t0 )) r0 (t0 ) = 0

y (0, 0, 0) N si solo si

r (t0 )

r 0 (t0 ) = 0

r (t0 )

r 0 (t0 ) = (a sin2 t0 , a sin(t0 ) cos(t0 ), a cos t0 )(2a sin t0 cos t0 , a(cos2 t0 sin2 t0 ), a sin

= 2a2 sin3 t0 cos t0 + a2 (sin t0 cos3 t0 sin3 t0 cos t0 ) a2 sin t0 cos t0


= a2 sin3 t0 cos t0 + a2 sin t0 cos3 t0 a2 sin t0 cos t0
= a2 sin t0 cos t0 a2 sin t0 cos t0
=0

(0, 0, 0) N

Ejercicio 6
Sea

r : R+ R3 , definida por:

r = (t2 , 23 t3 , t),la trayectoria regular
que
describe una partcula que se mueve a lo largo de una curva C. Para el
instante t = 1, determine:

149
a) Su velocidad ,rapidez y aceleracion.
b) Los versores tangente, normal y binormal a trayectoria en ese in-
stante
c) La curvatura y torsion de la curva en ese punto.
d) Las componentes tangencial y normal de la aceleracion en el punto.
e) La hodografa del movimiento o trayectoria que se determina en el
espacio
de velocidad.

Solucion:
a) A partir de la definicion tenemos que la velocidad ,rapidez y acel-
eracion
en funcion del tiempo estan dadas por:

v (t) =
r 0 (t) = (2t, 2t2 , 1) =
v (1) =

r 0 (1) = (2, 2, 1)

|
v (t)| =
r 0

r 0 = (4t2 + 4t4 + 1) = |
p
v (1)| = 3

a (t) =
r (t) = (2, 4t, 0) =
00
a (1) = (2, 4, 0)

b) Los versores se pueden calcular usando las identidades




r 0 (t)

r 0 (1) (2,2,1)
Tb (t) = = Tb (1) = =
|

r 0 (t)| |

r 0 (1)| 3


00 00
r 0 (t)

r (t)

r 0 (1)

r (1) (4,2,4)
B
b (t) =


r (t) r 00 (t)
= B
b (1) =


r (1) r 00 (1)
=
| 0
| | 0
| 6

b (t) Tb (t) = N b (1) Tb (1) = (1,2.2)


N
b (t) = B b (1) = B
3
c) La curvatura y torsion en el punto se pueden determinar usando
las identidades:
00 00


r 0 (t)

r (t) r 0 (1)

r (1)

6 2
(t) = 3 = (1) = 3 = = >0
|

r 0 (t)| |

r 0 (1)| 33 9

00 000 00 000


r 0 (t)
r (t)
r (t)

r 0 (1)
r (1)
r (1) 8 2
(t) =

00 2 = (1) =

00 2 = 36
= 9
| 0
r (t) r (t) | | 0
r (1) r (1) |
d) Derivando la velocidad tenemos


a (t) =

r (t) = |
00
v (t)| Tb (t) + (t) |
0 2 b
v (t)| N (t) ,

entonces las componentes tangencial y normal de la aceleracion son:

150
aT = |
aT = |

0 0
v (t)| = 4t 2+ 8t3) v (1)| = 4 + 8 = 12 3
=4
(4t +4t4 +1) = (4+4+1)

a = (1) |
2 2
a = (t) | v (t)|
N N v (1)| = 2 (3)2 = 2
9
e) A partir del vector velocidad

v (t) =

r 0 (t) = (2t, 2t2 , 1) podemos
inferir
sus componentes

x0 (t) = 2t
2
y 0 (t) = 2t2 = y 0 (x) = x2
z 0 (t) = 1 z0 = 1

Por lo tanto, la hodografa es una parabola en el plano z 0 = 1 del espacio


de velocidades.
Ejercicio 7
Sea

r : R+ R3 , definida por:

r = (a cos t, asent, (t)), trayectoria
regular que describe una partcula que se mueve a lo largo de una
curva C. Cual debe ser la funcion (t) para que la trayectoria este
contenida en un plano para todo t?.

Solucion:
La trayectoria de la partcula es plana si y solo s la torsion es nula
para
todo t. Es decir:
00 000


r 0 (t)
r (t) 000
= 0 t =

r 0 (t)

r (t)

r (t) 00
(t) =

00 2 r (t) = 0 t,
| 0
|
r (t) r (t)

luego derivemos y calculemos el producto mixto




r 0 = (asent, a cos t, 0 (t))



r 00 = (a cos t, asent, 00 (t)) =


r 000 = (asent, a cos t, 000 (t))

000


r 0 (t)

r (t)
00
r (t) = a2 ( 000 (t) + 0 (t)) = 0 =

( 000 (t) + 0 (t)) = 0

La ecuacion diferencial homogenea de tercer orden , tiene solucion de


la
forma (t) = et ; entonces

0 (t) = et = 00 (t) = 2 et = 000 (t) = 3 et

151
reemplazando terminos en la ecuacion anterior: (3 +)et = 0, et 6= 0
t conduce a la ecuacion caracterstica (3 + ) = 0, luego
(t) = C1 + C2 cos t + C3 sent, donde C1 , C2 , C3 son constantes reales.

Por lo tanto, si la trayectoria esta dada por




r (t) = (a cos t, asent, C1 + C2 cos t + C3 sent)

la partcula siempre se movera sobre un mismo plano.

Ejercicio 8

a) Sea r : I R R3 una trayectoria regular de modo que

r 0 (t) 6= 0 t. supongamos que hay un t0 I para el que la distancia


del
origen al punto
r (t ) R3 alcanza un valor mnimo. Pruebe que en
0
ese
punto

r 0 (t0 ) es perpendicular a

r (t0 ) .
b) Usando lo anterior , hallar los puntos en que la recta dada por

r (t) = (t + 1, 3t 2, 2t 1) esta mas cerca del origen.

Solucion:
a) En efecto consideremos la funcion distancia d(t)= |

r (t)|de un punto
cualquiera al origen y supongamos que hay un t0 I, tal que se
alcanza un mnimo, entonces se tiene que


r (t0 )

r 0 (t0 )
d0 (t0 ) = |
0
r (t0 )| = = 0,
|
r (t )| 0

de donde se tiene que



r (t0 )

r 0 (t0 ) = 0 , por lo tanto,

r (t0 ) y

r 0 (t0 )
son ortogonales en ese punto.
b) Basta encontrar un t0 I para el que la distancia del punto

r (t0 )
R3
alcanza su valor mnimo que satisface
r (t )
0

r 0 (t ) = 0
0


r (t0 )

r 0 (t0 ) = (t0 + 1, 3t0 2, 2t0 1) (1, 3, 2) = 0 =
1
(t0 + 1) + 3(3t0 2) + 2(2t0 1) = 0 = t0 =
2
152
Por lo tanto, el punto buscado es
1 3
, 12 , 0
 
r 2
= 2
.
Ejercicio 9
Se llama evoluta de una curva
r : I R+ R2 a la curva que
describen
los centros de curvatura de

r . Determine la evoluta de la curva

2
r (t) = (t, t ).
Solucion:
La curva que describe el centro de curvatura esta dada por la ecuacion:

c (t) =
1 b
r (t)+ (t) N (t) , determinemos la curvatura y el versor normal.


r 0 (t) = (1, 2t, 0) =
00
r (t) = (0, 2, 0) =

r 0 (t)

r (t) = (0, 0, 2) =
00 r 0 (t)

r (t)
00 r 0 (t) = (4t, 2, 0)

Ademas:
0
r (t)

r (t) = 2 y
0
r (t)

r (t)

00
00
1/2
r 0 (t) = 2 (1 + 4t2 )
00


r 0 (t)

r (t)

2
(t) = 3 =
|

r 0 (t)| (1 +4t2 )3/2


00
r 0 (t)

r (t)

r 0 (t) (4t,2)
N
b (t) =

00
= , reemplazando terminos en la
| r (t) r (t) r 0 (t)
0
| 2(1 +4t2 )1/2
ecuacion tenemos
3/2
c (t) = (t, t2 )+ (1
+4t2 ) (4t,2)
= 4t3 , 3t2 + 1

,que corresponde
2 2(1 +4t2 )1/2 2
a
la ecuacion parametrica de la evoluta.

1.10 Ejercicio 10

Sea
r : I R R3 una trayectoria regular, dada por
r = (x(t), y(t), z(t)),
t I. Pruebe que la ecuacion del plano osculador en

r (t0 ), t0 I,es:

x x (t0 ) y y (t0 ) z z (t0 )

x0 (t0 ) y 0 (t0 ) z 0 (t0 ) = 0

x00 (t0 ) y 00 (t0 ) z 00 (t0 )
Solucion:



La ecuacion del plano osculador es : ( f r (t0 )) B (t0 ) = 0, donde

bi j k
b


b



B (t0 ) = r 0 (t0 ) r 00 (t0 ) = x0 (t0 ) y 0 (t0 ) z 0 (t0 )

x00 (t0 ) y 00 (t0 ) z 00 (t0 )

153

00 00 00 00
B (t0 ) = (y 0 (t0 ) z (t0 ) y (t0 ) z 0 (t0 ) , x (t0 ) z 0 (t0 ) x0 (t0 ) z (t0 ) ,
x0 (t0 ) y 00 (t0 ) x00 (t0 ) y 0 (t0 ))

pongamos: x0 = x (t0 ) , y0 = y (t0 ) y z0 = z (t0 )




(f
r (t0 )) B (t0 ) = 0 =

00 00 00 00
(x x0 , y y0 , z z0 ) (y00 z0 y0 z00 , x0 z00 x00 z0 , x00 y000 x000 y00 ) = 0

desarrollando
00 00 00 00
(y00 z0 y0 z00 )(x x0 ) + (x0 z00 x00 z0 )(y y0 ) + (x00 y000 x000 y00 )(z z0 ) = 0

esto ultimo se puede escribir



x x (t0 ) y y (t0 ) z z (t0 )

x0 (t0 ) y 0 (t0 ) z 0 (t0 ) =0
00 00

x (t0 ) y (t0 ) z 00 (t0 )

lo que prueba la hipotesis.


Ejercicio 11
Un automovilista se desplaza por una carretera recta. En el instante
t=0
llega a una rotonda la que recorre con un trayectoria


f (t) = (acost, asent, bt(2 t)), t [0, 2]

En el instante t = 2 sale de la rotonda y vuelve a continuar por una


carretera recta.
a) Calcule la curvatura maxima de la rotonda para t [0, 2]en que pun-
to
ocurre?
b) Determine la torsion de la rotonda para t [0, 2] .Haga un grafico
de la
torsion en funcion del tiempo.

154
Solucion:
a) Calculemos la curvatura de la trayectoria mediante la identidad:

0
00
f (t) f (t)
(t) =
3
| f 0 (t)|

0

p 
f (t) = (asent, a cos t, 2b(1t)) = f 0 (t) = a2 + 4b2 (1 t)2

00
f (t) = (acost, asent, 2b) =

0

f (t) f 00 (t) = (2ab cos t + 2ab(1 t)sent, 2absent 2ab(1
t) cos t, a2 )

0
p
f (t) f 00 (t) = a a2 + 4b2 (1 + (1 t)2 ,reemplazando en la identi-

dad,
tenemos p
a a2 + 4b2 (1 + (1 t)2
(t) =
(a2 + 4b2 (1 t)2 )3/2

Sea (t) = a2 + 4b2 (1 t)2 = 0 (t) = 8b2 (1 t) = 0 = En


t=1
hay un punto crtico de (t)
Como 00 (1) = 8b2 > 0, en t = 1 hay un mnimo de (t) y un maximo
de
(t) pues son inversamente proporcionales

a2 +4b2


Luego, (t) = a2
y se alcanza en el punto f (1) = (a cos 1, asen1, b).

0
00 000

f (t) f (t) f (t)
b) Calculemos ahora la torsion; (t) =

2 t
| f 0 (t) f 00 (t)|

000



f (t) = (asent, a cos t, 0) = f 0 (t) f 00 (t) f 00 (t) = 2a2 b(1 t)
2a2 b(1t)
(t) = a2 [a2 +4b2 (1+(1t)2 ]
Se tiene que
2b 2b
(0) = [a2 +8b2 ]
, (1) = 0, (2) = [a2 +8b2]

a2 +4b2
0 (t) = 0 = 4b2 (1 t)2 = a2 + 4b2 = (1 t)2 = 4b2
a2
(1 t)2 = 1 + 4b2
> 1 = (1 t)2 > 1 lo cual es imposible porque
(1 t) 1
para 0 t 2.
Por lo tanto, no tiene punto crtico en[0, 2]

155
El grafico es del tipo
Ejercicio 12
a) Demuestre que la curva descrita por

r (t) = (t cos t, t sin t, t) se
encuentra
sobre la superficie de un cono. Dibuje la curva.
b) Si una partcula parte del origen siguiendo la trayectoria anterior-
mente
descrita, determine en que punto intersecta la esfera:

x2 + y 2 + z 2 = 2.

c) Calcule la longitud de la curva desde el origen al punto de iterseccion.

Solucion:
a) A partir de las ecuaciones parametricas tenemos:

x (t) = t cos t
y (t) = t sin t = x2 + y 2 = t2 (cos2 t + sin2 t) = x2 + y 2 = t2
z (t) = t

z 2 = x2 + y 2
corresponde a la ecuacion de un cono, cuyo sector superior ,se dibuja
en el grafico adjunto t 0
b) Calculemos para que valor del parametro t la partcula intersecta la
esfera, sustityendo las ecuaciones parametricas en la
esfera x2 + y 2 + z 2 = 2t2 = 2 = t = 1.Luego, la posicion del punto
de impacto es
r (1) = (cos 1, sin 1, 1)
c) La longitud de la curva es:
Z 1 q
l= k

r 0 (t)k du con k

r 0 (t)k =
r 0 (t)

r 0 (t)
0

156
con
r 0 (t) = (cos t t sin t, sin t + t cos t, 0)


q


0
k r (t)k = r 0 (t)

r 0 (t) = 2 + t2 =
 x2 + x2 1
Z 1 " #

l = 2 + t2 dt = ln 2 + x2 + x +
0 2
0

ln( 3 + 2) 3
l = +
2 2

Ejercicio 13
Un proyectil es lanzado desde el nivel del suelo con una velocidad inicial
de 100 m/seg con un angulo de elevacion de 30o .Determine:
a) la funcion vectorial y las ecuaciones parametricas de la trayectoria
del proyectil.
b) la altura maxima alcanzada.
c) el alcance del proyectil
d) la velocidad y rapidez en el impacto contra el suelo.
e) la curvatura en el punto de impacto.
Solucion:
a) Inicialmente tenemos t = 0 ,

r 0 = (0, 0) ,y

v 0 = (100 cos 30, 100 sin 30)
Integrando:

a (t) =

r (t) = (0, 10) =


v (t) =

r (t) = (100 cos 30, 10t + 100 sin 30)

Integrando por segunda vez se obtiene:



r (t) = ((100 cos 30) t, 5t2 +
(100 sin 30)t)

157

Evaluando las funciones trigonometricas queda:

r (t) = (50 3 t, 5t2 +
50t)
Por lo tanto, las ecuaciones parametricas de la trayectoria del proyectil
son

x(t) = 50 3 t, y(t) = 5t2 + 50t
b) Determinemos el tiempo que demora en llegar a la altura maxima
dy
dt
=0 = 10t + 50 = 0 = t = 5s
Asi la altura maxima h alcanzada por el proyectil es
h = y(5) = 125 + 250 = 125m
c) El alcance maximo se logra cuando y(t) = 0, es decir si: 5t2 +50t =
0
t(5t + 50) = 0 t = 0, t = 10.

Entonces el alcance es: x (10) = 500 3

d) La velocidad del proyectil en el impacto es:

v (10) = (50 3, 50)
00


r 0 (10)

r (10)

e) La curvatura en el punto de impacto es:K (10) = 3 =
k

r 0 (10)k
5 3
104

Ejercicio 14
Sea C una curva determinada por la interseccion de los cilindros:
x2 = 1 y, z 2 = y
a) Parametrizar C de forma

r (t) = (x (t) , y (t) , z (t)), t I.
Indicacion: x2 + z 2 = 1
b) Obtener Tb, Nb , B,
b K y en P = (0, 1, 1)
Solucion:
a) Se puede parametrizar como

r (t) = (cos t, sin2 t, sin t), t [0, 2]
,
Calculemos el valor del parametro para

r (t1 ) = (cos t1 , sin2 t1 , sin t1 ) = (0, 1, 1) = t1 = 2 .


As, r 2 = (0, 1, 1).


b)
r 0 (t) = ( sin t, sin t cos t, cos t) =
r 0 2 = (1, 0, 0) =


158
Tb 2 = (1, 0, 0)


r 00 (t) = ( cos t, 2 cos 2t, sin t);


00
r 2 = (0, 2, 1)



r 0 2
 00 
= (0, 1, 2) =
0 
r 2
00
r 2
r 2 = 5
     
= B b = 0, 1 , 2 y Nb = 0, 2 , 1
2 5 5 2 5 5

Derivando por tercera vez tenemos:

r 000 (t) = (sin t, 4 sin 2t, cos t) = r


000

= (1, 0, 0)
2

0 

00 
r 0 2

b = r 2 r
 
2 2 , 1

N = 0,
2
r 0 2 r 00 2
r 0 2
 5 5

Por otra parte


000
00



 r 0 ( 2 )
r ( 2 )




r 0 ( 2 ) 00
r ( 2 )
r ( 2 )
K 2 =
3 = 5 y 2
=

00 2 =0
k r ( 2 )k
0
k (2)
r 0
r ( 2 )k

Ejercicio 15
Dada la ecuacion parametrica de la trayectoria
r (t) = (2t3 3t2 , t
2arctan(t)), encontrar todos los valores de t para los cuales la curva
(i) Tiene tangente horizontal.
(ii) Tiene tangente vertical.
(iii) No es regular.
Solucion:
El vector tangente a la curva

r (t) = (2t3 3t2 , t 2arctan(t)) es

0 2 t2 1 y 0 (t)
r (t) = (6t 6t, 2 ) cuya pendiente es m (t) = 0 =
t +1 x (t)
y 0 (t) t+1
m (t) = 0
=
x (t) 6t (t2 + 1)
(i) Para que la tangente sea horizontal, esta tiene que existir, es decir



r 0 (t) 6= 0 , y ademas m (t) = 0. Por tanto, t = 1.
ii) Para que la tangente sea vertical, esta tiene que existir, es decir

r 0 (t) 6=

0 , y ademas m(t) = . Por tanto,t = 0.




(iii) Para que la curva sea no regular r 0 (t) = 0 = x0 (t) = y 0 (t) = 0,
es decir, t = 1.

Ejercicio 16

159
Se llama evoluta de una curva parametrizada regular r (t) ,con curvatu-
ra no nula, al lugar geometrico de los centros de curvatura. Denotamos
la evoluta de
r (t) por
c (t).

(i) Encontrar una parametrizacion de c (t).


 2

t
(ii) Hallar la evoluta de la parabola r (t) = t, .
2
 


(iii) Hallar la evoluta de la helice r (s) = 2 cos(s), 2 sen(s), 22 s ,
2 2

s IR y comprobar que es regular y que s es su parametro arco.


Solucion:
(i) Si

r (t) es punto de la curva y
c (t) su correspondiente centro de

curvatura para t entonces la condicion que define el lugar geometrico


1
se tiene la relacion
c (t)

r (t) = N (t) de donde se deduce que
k(t)
la
ecuacion de la evoluta es


c (t) =
1
r (t) + N (t)
k(t)

(ii) Calculamos en primer lugar el vector normal y la curvatura de la


parabola.

 2

t
r (t) = t, = r 0 (t) = (1, t) = |

r 0 (t)| = 1 + t2 =
2

(t, 1)
N (t) =
1 + t2

Por otra parte

|

r 0 (t)
r (t)| 1
r (t) = (0, 1) = k(t) =
3 = 3
| r 0 (t)| 1 + t2
Por lo tanto
 2 3
2
 
c (t) = t, t + 1 + t  (t, 1) = t3 , 1 + 3 t2

2 1 + t2 2
iii) Calculamos el vector tangente a la curva a partir de su definicion
   


0
r (s) = 2 cos(s), 2 sen(s), 2 s = r (s) = 2 sen(s), 2 cos(s), 22
2 2 2 2 2

160
= |

r 0 (s)| = 1
 
Por tanto T (s) = 2 sen(s), 2 cos(s), 22
b 2 2

Por otra parte,



Tb0 (s) =

r 00 (s) = 22 cos(s), 22 sen(s), 0 = |


r 00 (s)| = 2
2

de donde


r 00 (s)
N (s) = = ( cos(s), sen(s), 0)
|
b
r 00 (s)|

Ademas k(s) = |
r 00 (s)| = 2
2

Finalmente
1  

c (s) =

r (s) + N (s) = 22 cos(s), 22 sen(s), 22 s , s IR
k(s)
 
lo que implica
c 0 (s) =
2
2
sen(s), 2
2
cos(s), 2
2
s , y por tanto que

|
c 0 (s)| = 1 para cada s IR y en definitiva que
c es regular y
esta
parametrizada por arco s.
Ejercicio 17
Encontrar una parametrizacion de las siguientes curvas.
(i) y = x2 + 3x
(ii) x2 + y 2 + 2y = 0
y2
(iii) x2 =1
4
(iv) x2 + 3y 2 = 1
Calcular en cada caso los vectores tangente unitario Tb, normal N
b , la
curvatura (t) y la torsion (t)
Solucion:

(i) Como se trata de una parabola podemos parametrizarla tomando


x = t = y = t2 + 3t
As
r (t) = (t, t2 + 3t), y calculamos el vector tangente,

r 0 (t) =
(1, 2t + 3)
= k
r 0 (t)k = 4t2 + 12t + 10

161

r 0 (t) (1, 2t + 3)
y Tb (t) =
0
=
k r (t)k 4t2 + 12t + 10

Para calcular el vector normal en IR2 tenemos en cuenta que es un


vector
ortogonal al tangente y unitario.
b (t) = (2t 3, 1)
N
4t2 + 12t + 10
|

r (t)
r (t)|
La curvatura de una curva plana es k(t) =
3 , con-
0
| r (t)|
siderando
que IR2 es subespacio de IR3
En nuestro caso
r 0 (t) = (1, 2t + 3, 0) =
r 00 (t) = (0, 2, 0)
2
Por tanto, k(t) =  3 , la curvatura de la parabola vara
4t2 + 12t + 10
en
funcion de t.
Finalmente, la torsion de la curva plana es


r 0 (t)
r (t)

r 000 (t)
(t) =
3 = 0, pues

r 000 (t) = (0, 0, 0)
0
| r (t)|
lo que significa que la curva siempre esta en el mismo plano.
(ii) La ecuacion dada representa una circunferencia de centro el (0, 1)
y
radio r = 1, ya que x2 + y 2 + 2y = x2 + (y + 1)2 1.
Por tanto, una parametrizacion es

r (t) = (cos(t), 1 + sen(t))
Calculamos el vector tangente.

r 0 (t) = (sen(t), cos(t)) = k


r 0 (t)k = 1 =


r 0 (t)
T (t) =
b
= (sen(t), cos(t))
k r 0 (t)k
Para calcular el vector normal tenemos en cuenta que es un vector
ortogonal
al tangente y unitario.
Nb (t) = (cos(t), sen(t))

162
|

r 0 (t)
r (t)|
La curvatura de una curva plana es k(t) =
3 , con-
| r 0 (t)|
siderando
que IR2 es subespacio de IR3
En nuestro caso

r 0 (t) = (sen(t), cos(t), 0) =

r 00 (t) = ( cos(t), sent(t), 0)
= |
r (t)
r (t)| = 1
Por tanto, k(t) = 1 la curvatura de una circunsferencia es constante
Finalmente, la torsion de la curva plana es


r 0 (t)
r (t)

r 000 (t)
(t) =
3 = 0,pues

r 0 (t)

r (t)

r 000 (t) =
0
| r (t)|
(0, 0, 0).
(iii) La ecuacion dada representa una hiperbola de semieje a = 1 y b
= 2.
Por tanto, una parametrizacion es
r (t) = (cosh(t); 2senh(t))
Calculamos el vector tangente.
r 0 (t) = (senh(t), 2 cos h(t))

= k p
r 0 (t)k = senh2 (t) + 4 cos h2 (t)


r 0 (t) (senh(t), 2 cos h(t))
y Tb (t) =
0
=p
k r (t)k senh2 (t) + 4 cos h2 (t)
Para calcular el vector normal tenemos en cuenta que es un vector
ortogonal
al tangente y unitario.

Nb (t) = p(2 cos h(t), senh(t))


senh2 (t) + 4 cos h2 (t)
|

r 0 (t)
r (t)|
La curvatura de una curva plana es k(t) =
3 , con-
0
| r (t)|
siderando que IR2 es subespacio de IR3
En nuestro caso :r 0 (t) = (senh(t), 2 cos h(t), 0) =

r 00 (t) =
(cosh(t); 2senh(t), 0)
= |r 0 (t)

r (t)| = 2
2
Por tanto k(t) = hp i3 , la curvatura de la hiperbo-
2 2
senh (t) + 4 cos h (t)
la

163
vara en funcion de t.
Finalmente, la torsion de la curva plana es


r 0 (t)
r (t)

r 000 (t)
(t) =
3 = 0, pues

r 0 (t)

r (t)

r 000 (t) =
0
| r (t)|
(0, 0, 0)
(iv) La ecuacion dada representa una elipse de centro el (0, 0)y semiejes
a=1
1
y b=
3
1
Entonces, una parametrizacion es r (t) = (cos(t), sen(t))
3
Calculamos el vector tangente.

1
3p
r 0 (t) = (sen(t), cos(t)) = k r 0 (t)k = 2sen2 (t) + 1 =
3 3


r 0 (t) 3 1
T (t) =
b
0
=p (sen(t), cos(t))
k r (t)k 2sen2 (t) + 1 3
Para calcular el vector normal tenemos en cuenta que es un vector
ortogonal
al tangente y unitario.

3 1
Nb (t) = p ( cos(t), sen(t))
2sen2 (t) + 1 3
|

r 0 (t)
r (t)|
La curvatura de una curva plana es k(t) =
3 , con-
| r 0 (t)|
siderando
que IR2 es subespacio de IR3 .
En nuestro caso

1 1
r 0 (t) = (sen(t), cos(t), 0) = r 00 (t) = ( cos(t), sent(t), 0)
3 3
1
= |
r (t)

r (t)| =
3
3
Por tanto, k(t) = hp i3 la curvatura de la elipse vara en
2
2sen (t) + 1
funcion de t.

164
Finalmente, la torsion de la curva plana es


r 0 (t)

r (t)

r 000 (t)
(t) = =0
|
3
r 0 (t)|

pues

r 0 (t)

r (t)

r 000 (t) = (0, 0, 0)
Ejercicio 18
Sean
r : I IR3 una curva regular y consideremos c la evoluta de

r


. Demostrar que c es regular si y solo si la torsion y la derivada de la
curvatura de
r no se anulan simultaneamente en ningun punto y en
este caso demostrar que el parametro arco de c , s satisface que

1
q
(k 0 (t))2 + (k (t))2 ( (t))2 |

0 2
(s (t)) = 2 r 0 (t)| t I
k (t)

Solucion:
La evoluta de esta definida como la curva

c (t) =
1
1
r (t) + N (t) =

r (t) + (t) N (t), donde (t) = es el
k(t) k(t)
radio
de curvatura de
r.
Por lo tanto, si s denota el parametro arco de r tenemos que s (t) =
|
r 0 (t)|

c 0 (t) =



r 0 (t) + (t) N (t) + (t) N 0 (t) =

0 0
r 0 (t) + (t) N (t) +


(t) N 0 (s) s0 (t)
Por Frenet sabemos que:

0 b (s) = 0
N (s) = k (s) Tb (s) (s) B N (s) = (s) Tb (s)
(s) B b (s)

c 0 (t) = |
0
h i
r 0 (t)| Tb (t)+ (t) N (t)+ (t) s0 (t) Tb (s) (s) (s) B
b (s)

= c 0 (t) = 0 (t)

N (t) (t) (t) |

r 0 (t)| B
b (t) .
c 0 (t)| = (0 (t))2 + 2 (t) 2 (t) |
As |
p
r 0 (t)|)2 puesto que el Triedro
de
Frenet es ortonormal. Sustituyendo y 0 por su expresion en funcion
de la curvatura, resulta que
s
(k 0 (t))2 2 (t) 1
q

0
| c (t)| = + 2 0 2
| r (t)| = 2 (k 0 (t))2 + k 2 (t) 2 (t) |

r 0 (t)|
2
(k (t)) 4 k (t) k (t)

165


para cada t I y por tanto, como (t) y |

r 0 (t)| 6= 0 c es regular
en t I
si y solo si k 0 (t) = (t) = 0.
La expresion anterior muestra tambien que cuando
c es regular, la
derivada
de su parametro arco coincide con el termino de la derecha de la igual-
dad
Ejercicio 19
Consideremos la curva C dada por

r (t) = (et , e2t , t), t R.
Hallar su curvatura y su torsion en el punto (1, 1, 0). Es cierto que la
curva
tiene torsion negativa en todos sus puntos?
Solucion:
Calculamos primero las derivadas:


r 0 (t) = (et , 2e2t , 1) k

r 0 (t)k = 1 + e2t + 4e4t

r 00 = (et , 4e2t , 0)


r 0 r 00 = (4e2t , et , 2e3t )
0
r
00
r = e2t + 16e4t + 4e6t

000
r = (et , 8e2t , 0)
Y podemos obtener la curvatura y la torsion en cada punto:

r 0 00

r e2t + 16e4t + 4e6t
k(t) = =
k 3 3
r 0 (t)k

1 + e2t + 4e4t


r 0 r
00 r
000
4e3t
(t) = =
kr 0 2 2
r 00 k

e2t + 16e4t + 4e6t

En el punto (1, 1, 0) = (et0 , e2t


0 , t0 ) = t = 0

De donde deducimos que la curvatura y la torsion son



r 0 (0) 00

r (0) 21
k(0) = =
k
3
r 0 (0)k 63/2


r 0 (0) r (0)
00 000
r (0) 4
(0) =

2 =
0
k r (0) r (0)k 00
21
Por ultimo, la funcion (t) es claramente positiva, t IR

Ejercicio 20

166
Considere la curva C dada por

r (t) = (cosh(t), sinh(t), t), t IR
(a) Dibuje aproximadamente su traza.
(b) Si se recorre la traza partiendo desde punto (1, 0, 0), tras recorrer
una

longitud de arco 2 sobre la curva, cuales son las coordenadas del
punto del espacio en el que nos encontraremos?
Solucion:
Consideremos la proyeccion de la curva sobre el plano XY :
x2 y 2 = cos h2 (t) sinh2 (t) = 1 se trata de una rama de hiperbola,
cuyo
eje de simetria es X, toma valores positivos de X y se recorre en el
sentido
creciente de Y .
La tercera componente es simplemente z = t, para valores t > 0 la
curva
se eleva con respecto al plano z = 0 hacia, mientras que para valores
de
t < 0, baja con respecto el plano z = 0.

Determinemos el valor del parametro en el punto de partida


(1, 0, 0) = (cosh(t), sinh(t), t) = t = 0
Empezamos en s(0), la longitud de la curva, desde ese punto, se mide
la
longitud de arco.
Calculemos la rapidez, y luego la longitud del arco

167

r (t) = (cosh(t), sinh(t), t), t IR

r 0 (t) = (senh(t), cos h(t), 1), t
IR

= k p p
r 0 (t)k = cosh2 (t) + sinh2 (t) + 1 = 2cosh2 (t) = 2cosh(t)
.
As que la longitud desde s(0) hasta
Rt
s(t) = 0 k
Rt 0
r (u)k du = 2 0 cosh(u)du = 2 senh(u)|t0 = 2senh (t)
Ahora solo queda determinar el valor de t que hace que la longitud sea

exactamente 2. Esto es, resolver

2 = 2sinh(t) 1 = sinh(t) t = arcsinh(1).
De manera que estaremos en el punto de coordenadas

r (arcsinh(1)) = (cosh(arcsinh(1)), sinh(arcsinh(1)), arcsinh(1)) =


r (arcsinh(1)) = ( 2, 1, arcsinh(1))
Ejercicio 21
Consideremos la curva C dada por

r (t) = (t, t2 , t3 ) R.
Hallar su curvatura y su torsion en el punto (0, 0, 0). En que punto
tiene
la curva una torsion (en valor absoluto) maxima?
Solucion:
Calculamos primero las derivadas:

r 0 (t) 00

r (t) 4 + 36t2 + 36t4
k(t) = =
k 3 3
r 0 (t)k

1 + 4t2 + 9t4


r 0
r
00 r
000
12
(t) =
00 2
=
0
kr r k [4 + 36t2 + 36t4 ]2
De donde deducimos que la curvatura y la torsion son

r 0 (0) 00

r (0)
k(0) = =2
k
3
r 0 (0)k


r 0 (0)
r (0)
00 000
r (0)
(0) = =3
k
r 0 (0) 2
r 00 (0)k
Como el denominador que aparece en la expresion de (t) es siempre
una cantidad positiva que toma su valor mnimo en t = 0, concluimos
que el valor absoluto de la torsion alcanza su valor maximo en t = 0.

168
2.17. Ejercicios propuestos

1. Sean f~, ~g : [a, b] Rn funciones derivables, probar que:

a) (f~ + ~g )(t) = f~ (t) + ~g (t) , y constantes


b) (f~)0 (t) = 0 (t) f~ (t) + (t)f 0 (t) si = (t) es una funcion escalar.
 0
~
c) f ~g (t) = f~0 (t) ~g (t) + f~(t) ~g 0 (t)
 0
~
d) f ~g (t) = (f 0 g) (t) + (f g 0 ) (t)

2. Para f~(t) = (e2t , t2 , e2t ) calcular:

a) f~0 (t)
b) f~00 (t)

c) f~0 (t)

3. Obtener una funcion parametrica vectorial de la curva resultante


al intersectar las superficies:

x2 + y 2 + z 2 = 2
p
yz= 2 (1 + y)

169
4. Parametrizar en la forma ~r0 = ~r(t) = (x(t), y(t), z(t)), t [a, b] las curvas C
definidas por:

a) Recta por A = (1, 2, 3) y B = (2, 2, 2)


b) Recta interseccion de los planos x + y + z = 0 y y z = 0

c) Interseccion del cilindro recto x2 + y 2 = 9 y el plano z = 1

d) El cilindro x2 ax + y 2 = 0 interseccion la esfera x2 + y 2 + z 2 = a2

5. Calcular la longitud l de la curva descrita por la trayectoria ~r(t) =


(a cos3 t, a sin3 t), t [0, 2]
6. Una hormiga camina sobre la superficie S : x(u, v) = sin u cos v, y (u, v) =
sin u sin v, z (u, v) = cos u

6
u 2 , 2 v en el instante t = 0, se mueve de tal
forma
 que en cualquier tiempo t > 0 describe la trayectoria

r (t) =

3 1 3
2
sin t, 2 , 2 cos t .
Demuestre que la hormiga se mueve sobre la superficie esferica S y
determine cuando y en donde la hormiga abandona el sector descrito
de la esfera

170
7. Halle el o los valores de t para los cuales el vector tangente a la
curva descrita por r(t) = (2t2 + 1, 3t 2), t R sea paralelo al
vector ~u = (2, 1).
8. Calcule
2 2 3 3 4 4
lm( (x+t)t x , (x+t)t x , (x+t)t x )
t0

9. Discuta la continuidad de la funcion


(t, sint t ) si t 6= 0
f (t) = {
(0, 1) si t = 0




10. Demuestre que: si f (t) es constante, entonces f y f 0 son or-
togonales.
11. Encuentre un vector tangente y la recta tangente a la helice conica
de representacion parametrica


t
f (t) = (t cos t, t sin t, )
2
en los puntos (0, 0, 0) y (0, 2 , 14 )


12. Sea la helice descrita por f (t) = (cos t, sin t, t) t [0, 2 ]. Pruebe


que en ningun punto de esta curva f 0 (t) es paralelo a la cuerda



de f (0) a f ( 2 )
13. Una partcula se mueve en el plano a lo largo de la espiral = e
con una rapidez constante de 5 pies por segundo.

171
a) Cuales son la velocidad y la aceleracion de la partcula cuando
= 4 ?
b) Cuanto tarda la partcula en ir desde el punto correspondiente a
= 0 hasta el punto correspondiente a = ?
c) Si = 0 cuando t = 0, proporcione ecuaciones parametricas para la
trayectoria de la partcula.

14. Determine los puntos en que la recta tangente a la curva de R3


dada por

~r(t) = 6t + 2t3 + (6t 2t3 ) + 3t2 k




es paralela al plano x + 3y + 2z = 5


15. Demuestre que el camino descrito por f : [2, 2] R2 definida


por f (t) = (t2 1, t3 + 2t2 t 2) no es simple. Es cerrado?Es
cerrado simple?
16. Determine las ecuaciones de la recta tangente y del plano nor-


mal a la curva descrita por f : I R3 definida por f (t) =


(et sint, et cost, 5t) en el punto P = f (0)



17. Sea f : [0, 5] R3 la trayectoria f (t) = (sin t, cos t, t). Obtenga



una reparametrizacion f c de f que conserve su orientacion y que


recorra la trayectoria de f en la quinta parte del tiempo en el que


lo hace f .
18. Un objeto ubicado inicialmente en el origen de coordenadas se
desplaza de acuerdo al movimiento

~r(t) = (t2 , t cos t, t sin t)

a) Determine el punto en que el objeto impacta a la esfera x2 +y 2 +z 2 =


2
b) Calcular la longitud recorrida por el objeto hasta el instante del
impacto.
c) Calcular el angulo entre la trayectoria y la esfera en el instante del
impacto.

172
19. El movimiento de una partcula se realiza en el crculo unitario del
plano XY de acuerdo con la formula (x, y, z) = (cos t2 , sin t2 , 0) ,
t 0.

a) Como funciones de t. Cual es el vector velocidad y la rapidez de la


partcula?
b) En que punto del crculo se debe liberar la partcula para que
golpee un blanco localizado en (2, 0, 0)?
c) En que tiempo t se debe liberar? (usar t 0 mas pregunta b))
d) Cual es su velocidad y rapidez en el instante en que se libera?
e) En que instante se golpea el blanco?

20. Si f : I IR es una funcion diferenciable, demostrar que la


grafica de f , es decir la curva y = f (x), es una curva regular y
calcular sus vectores tangente , normal, su curvatura, su radio de
curvatura, y su centro de curvatura.
21. Sea C una curva de IR3 dada por la ecuacion r (t) = (4 cos(t), 4sen (t) , 4 cos t(t)), t
[0, 2]

a) Verificar que es una elipse.


b) Determinar los vectores Tb, Nb, B
b en cualquier punto C
c) Calcular la curvatura y torsion de C en todo punto.
d) Determinar en que puntos de C la curvtura k es maxima y en cuales
es mnima

2.17.1. Respuestas
2.

a) f~0 (t) = 2 (e2t , t, e2t )


b) f~00 (t) = 2(2e2t , 1, 2e2t )

c) f~0 (t) = 2 e4t + e4t + t2


3. ~r(t) = (cos t, 1 + sin t, 2 sin t), 0 t 2
4. a) ~r(t) = (1 + t, 2, 3 + 5t) tR

173
b) ~r(t) = (2t, t, t) tR
c) ~r(t) = (3 cos t, 3 sin t, 1) t [0, 2]

d) ~r(t) = ( a2 + a2 cos t, a2 sin t, 2a sin 4t ) t [0, 2]

3 1
5. 6a, 6 t = 2 , (x, y, z) = ( , , 0)
2 2
7. t = 32
8. (2x, 3x2 , 4x3 )
9. Es continua


10. f f 0 = 0
11. 1, 0, 21

,
1
r (t) = (t, 0, 2 t)
1
(0, 2 , 4 ),

r (t) = ( 2 t, 2 + t, 14 + t 2
1
)
13.

a) ~v = (0, 5), ~a = ( 252 e 4 , 0)

b) 15 2(e 1)


c) f (t) = [ 52 t + 1](cos ln( 52 t + 1), sin ln( 52 t + 1))

14. ~r(2) = (28, 4, 12)





15. No es simple f (1) = (0, 0) = f (1)
16. ~r(t) = (t, 1 + t, 5t), x + y + 5z = 1 plano normal
17. f : [0, 1] R3 , f (s) = (sin(5s), cos(5s), 5s)
18.

a) (1, cos1, sin1)


b) 1,57

19.

a) ~v = (2t sin t2 , 2t cos t2 , 0); s = 2t



3
b) ( 21 , 2
, 0)
q
5
c) 3
q q
d) v = 2 5
3
( 23 , 21 , 0); s = 2 5
3

3 3+10
e) 2 15

174
20. La grafica de la funcion es la curva
r : I IR2 determinada por

r (x) = (x, f (x))

Como
r 0 (x) = (x, f 0 (x)), resulta que |
p
r 0 (x)| = 1 + (f 0 (x))2 >
0 x I y por tanto la curva es regular
(1, f 0 (x))
(f 0 (x) , 1)
Tb (x) = p y N (x) = p
1 + (f 0 (x))2 1 + (f 0 (x))2
p
|f (x)| ( 1 + (f 0 (x))2 )3
k(x) = p y R(x) =
( 1 + (f 0 (x))2 )3 |f (x)|

0 2
c (x) = (x, f (x)) + (1 + (f (x) ) (f 0 (x) , 1)

|f (x)|

21.

A partir de las ecuaciones parametricas, se tiene que:


x (t) = 4 cos(t)
y (t) = 4sen (t) = x2 + y 2 = 16
z (t) = 4 cos(t) z=x
(4sen (t) , 4 cos (t) , 4sen (t)) b (16, 0, 16)
Tb (t) = p , B (t) =
16 + 16sen2 (t) 16 2

N b (t) Tb (t) = (cos


b (t) = B (t) , 2sen (t) , cos (t))
p
2 1 + sen2 (t)

16 2
k(t) = hp i3 , (t) = 0
43 1 + sen2 (t)

La curvatura es maxima si t = 0, = P0 = (4, 0, 4)


3
La curvatura es mnima si t = , = P1 = (0, 4, 0)
2 2

2.18. Auto Evaluaciones


Autoevaluacion No 1
El estudiante:
1) Calculara ecuacion cartesiana de la trayectoria, curvatura y torsion
en un punto de la curva, dada la ecuacion vectorial de una curva regular,

175
dos veces diferenciable, con ecuaciones parametricas en funcion de un
parametro cualquiera t,
2) Determinara la ecuacion de la recta tangente y del plano osculador


a la curva descrita por la trayectoria f : I R R3 en un punto
de la curva.
3) Resolvera problemas de dinamica de partculas querequiera calcular
longitud de una curva, curvatura maxima , torsion de la curva, ecua-
ciones de rectas ,y planos principales en un punto dado de la curva.

Tiempo : 2 horas

Tiempo : 2 horas

Pregunta 1
Sea la helice

r (t) = (a cos t, a sin t, bt) , b 6= 0, determine:
1 2
a) los valores de a y b si su curvatura es (t) = y su torsion (t) = .
5 5
b) La ecuacion del cilndro sobre el cual se encuentra la helice.

Pregunta 2
t3
   
2 8
Para la curva C dada por:

r (t) = , 2t, en P0 = , 4, 1
3 t 3
Obtener las ecuaciones vectoriales y cartesianadas de:
a) Recta tangente.
b) Planos osculador.

Pregunta 3
En el instante t= 0 , una casa es lanzada al espacio por causa de un
tornado siguiendo la trayectoria


r (t) = eat cos t, eat sin t, beat b


con a > 0, b > 0, t 0.


a) Calcular la distancia que recorre la casa hasta el instante t = .
b) En el instante t1 con t1 [0, ] , en el cual la trayectoria tiene
curvatura maxima, un residente que dorma tranquilo es lanzado

176
hacia el exterior.
i) que trayectoria sigue el residente?
ii) en que punto se encuentra la casa en t = ?
iii) Obtener la ecuacion de plano osculador a la trayectoria de la casa
en t = .

Pauta de Autocorreccion

Pregunta 1
En primer lugar determinemos la curvatura la torsion en funcion del
tiempo


r 0 (t) = (a sin t, a cos t, b) t IR = k

r 0 (t)k = a2 + b2

00
r (t) = (a cos t, a sin t, 0)

r 000 (t) = (a sin t, a cos t, 0)

r0 (t)
r 00 (t) = (ab sin t, ab cos t, a2 ) = k r 0 (t)

r 00 (t)k =
a a2 + b 2

r 0 (t)

r 00 (t)

r 000 (t) = a2 b sin2 t, a2 b cos2 t, 0


Entonces, se tiene


r 0 (t) 00

r (t) a
(t) = = ,y
k 3/2 a2 + b 2
r 0 (t)k


r 0 (t)
r (t)
00
r 000 (t) b
(t) = =
k 3/2 a + b2
2
r 0 (t)k
Luego, a partir de los valores dados de curvatura y torsion, obtenemos
a 1 b 2
= , = = a = 1, b = 2
a2 + b 2 5 a2 + b 2 5

Por tanto la ecuacion de la helice es




r (t) = (cos t, sin t, 2t)

b) A partir de la ecuaciones parametricas de la helice , se tiene


x = cos t, y = sin t, z = 2t 0 = x2 + y 2 = 1, z 0, corresponde a
un

177
cilindro de radio r = 1, cuyo eje axial coincide con el eje z.

Pregunta 2
a) La ecuacion de la recta tangente al punto P0 se define por

f () =

r (t0 ) + r 0 (t0 ) , con IR
 3   

t0 2 8
siendo r (t0 ) = , 2t0 , = , 4, 1 = 2t0 = 4 t0 = 2
3 t0 3
Calculemos ahora la direccion de la recta tangente
   

0 2 2
0 1
r (t) = t , 2, 2 = r (t0 ) = 4, 2,
t 2
Sustituyendo terminos en la ecuacion de la recta tangente, obtenemos
   
8 1
(x, y, z) = , 4, 1 + 4, 2, , IR
3 2

Eliminando el parametro , tenemos la ecuacion cartesiana


8
x
3 = y4 = z1
4 2 12

b) La ecuacion del plano osculador al punto P0 se define por





f (t) r (t0 ) B (t0 ) = 0, siendo B (t0 ) =


r 0 (t0 )

r 00 (t0 ) el
vector
normal al plano osculador en el punto P0 .
   

0 2 2
00 4
Como r (t) = t , 2, 2 = r (t) = 2t, 0, 3
t t
 

00 1
r (t0 ) = 4, 0,
2
.
De manera que

i j k

B (t0 ) =

r 0 (t0 )

r 00 (t0 ) = 4 2 21 = (1, 4, 8)
4 0 1
2
Por tanto la ecuacion de plano, queda

178
  
8
(x, y, z) , 4, 1 (1, 4, 8) = 0
3
3x 12y 24z = 64

Pregunta 3
a) Derivando la ecuacion de la trayectoria se tiene



r 0 (t) = aeat cos t eat sin t, aeat sin t + eat cos t, abeat


= eat (a cos t sin t, a sin t + cos t, ab)

Calculando la norma de este vector obtenemos



k

r 0 (t)k = eat a2 + 1 + a2 b2

Entonces, la longitud de la curva es


Z
 a 


0 2 2 2
e 1
l( r ) = k r (t)k dt = a + 1 + a b
0 a


r 0 (t) 00

r (t)
b) La curvatura esta dada por la expresion (t) =
k 3/2
r 0 (t)k
Derivando por segunda vez,

00
r (t) = aeat (a cos t sin t, a sin t + cos t, ab)
+eat (a sin t cos t, a cos t sin t, 0)

=

r 0 (t)
00
r (t) = e2at (ab(sin t a cos t), ab(a sin t + cos t, a2 + 1)

=
0
r (t)
00

r (t) = e2at a2 + 1 a2 + 1 + a2 b2
Luego, obtenemos

a2 + 1
(t) =
eat (a2 + 1 + a2 b2 )

Por tanto, (t) es maximo para t1 = 0, cuando se tiene el valor mnimo


del
denominador

179
i) El residente sale despedido por la recta tangente a la trayectoria del
tornado, en el instante t1 = 0, luego la ecuacion de la recta sera


R (0) =

r (0) +

r 0 (0) = (1, 0, 0) + (a, 1, ab)

ii) En el instante t = la casa se encuentra en el punto




r () = (ea , 0, bea b)

iii) El plano osculador a la trayectoria de la casa en t1 = es





(R
r ()) B () = 0

donde el vector binormal se determina por




B () =

r 0 ()
00
r () = e2a a2 b, ab, a2 + 1


Por tanto, la ecuacion que produce es

(x ea , y, z (bea b)) e2a a2 b, ab, a2 + 1 = 0




Autoevaluacion No 2

Tiempo : 2 horas
Pregunta 1
Una partcula se desplaza a partir del instante t = 0 siguiendo la trayec-
toria:


r (t) = (t, ln (sec t + tagt) , ln sec t) , 0 t <
2
Determinar la velocidad y rapidez en el instante que ha recorido una

2
distancia igual .
2
Pregunta 2
Dada la curva C definida por las superficies x2 +y 2 +z 2 = 11, x2 +y 2 = 2,

z > 0, determinar en el punto 0, 2, 3 ;
a) la ecuacion del plano osculador
b) los versores tangente, normal y binormal

180
Pregunta 3
Para la curva C dada por

r =

r (s) ,donde s es el parametro longitud
de
arco, deducir las formulas:
000
a)
r 0 (s)

r (s) = 2 (s)
b)
r 0 (s)

r (s)
00 000
r (t) = 2

c) Usar las formulas anteriores para calcular , de la curva C


 

4 3
r (s) = cos s, 1 sin s, cos s
5 5

d) Identificar la curva C

Pauta de Autocorreccion

Pregunta 1
Derivando la ecuacion de la trayectoria:



r (t) = (t, ln (sec t + tagt) , ln sec t) , 0 t <
2

obtenemos la velocidad



r 0 (t) = (1, sec t, tagt) , 0 t <
2
En consecuencia la rapidez de la partcula, queda


p
r 0 (t) = 1 + sec2 t + tag 2 = 2 sec t

Entonces, la distancia recorrida en funcion del tiempo, es



Z t
 
2 1 + sin t
s (t) = 2 sec udu = ln
0 2 1 sin t

A continuacion determinemos el tiempo t0 , que corresponde a la dis-


tancia

181

2
:
2
 
2 1 + sin t0 2
ln =
2 1 sin t0 2
 
1 + sin t0
ln = 1
1 sin t0
 
e1
t0 = arcsin 0, 48
e+1

Por lo tanto la velocidad en ese tiempo es




v (t0 ) =

r 0 (t0 ) = (1, sec (0,48) , tag (0, 48)) = (1; 1, 127; 0, 521)

y la rapidez
q
k

r 0 (t0 )k = 1 + (1, 127)2 + (0, 521)2

Pregunta 2
a) Determinemos la ecuacion de la curva C, sustituyendo la segunda
ecuacion
en la primera, obtenemos 2 + z 2 = 11 = z 2 = 9, z = 3. Como
z > 0, la curva
dada es la circunsferencia x2 + y 2 = 2, z = 3.

La ecuacion parametrica de la curva es 
r (t) = 2 cos t, 2 sin t, 3 ,
y el

valor del parametro en el punto es 
r (t0 ) = 2 cos t0 , 2 sin t0 , 3 =

0, 2, 3

= 2 cos t0 = 0 = t0 = 2 .
Calculemos ahora la ecuacion del plano osculador, derivando, obten-
emos


r 0 (t) = 2 sin t, 2 cos t, 0 = r 0 2 = 2, 0, 0
  


r (t) = 2 cos t, 2 sin t, 0 =
00
r 00 2 = 0, 2, 0
 

El vector normal al plano en el punto dado es



i j k


r 0 2
r 00 2 = 2
 
0
0 = (0, 0, 2)
0 2 0

182
La ecuacion del plano osculador viene dada por
h
i  
f r 2 N 2 = 0 (x.y.z) 0, 2, 3 (0, 0, 2) =


z=3
b) El versor tangente, en el punto es


r 0 2

0, 2, 0
t=  = = (0, 1, 0) .


b
r0
2 2
El versor binormal viene dado por


r 0

r 00

(0, 0, 2)
bb = 2 2 
0
00
= = (0, 0, 1) .
r 2 r 2 2
Finalmente el versor normal es
b = bb b
n t = (1, 0, 0) .
Pregunta 3
a) A partir de
r =

r (s) tenemos que

0
r (s) = Tb (s)
derivando por segunda vez y aplicando la identidad de Frenet, queda
00

0
r (s) = Tb (s) = Nb (s) =
000

0
r (s) = N b 0 (s)
b (s) + N
Utilizando nuevamente la identidades de Frenet , obtenemos
000

 
0
r (s) = N (s) + B T =
b b b
000

0
r (s) = N b 2 Tb
b (s) + B
Calculando el producto interno y considerando la ortonormalidad de la
base
n o
Tb, N
b, B
b , produce

000


r (s)

0
 0 
r (s) = Tb (s) N b 2 Tb = 2
b (s) + B

00 000

r (s)

r (s)

0
 
b (s) 0 N
r (s) = Tb (s) N b 2 Tb
b (s) + B

= 2

b) A partir de
4
cos s, 1 sin s, 53 cos s

r (s) = 5
=

183

0
r (s) = 45 sin s, cos s, 35 sin s =


00
r (s) = 45 cos s, sin s, 53 cos s =


000
r (s) = 4 sin s, cos s, 3 sin s

5 5
Realizando el producto interno
000

r (s)
0
r (s) = 1 = 2 = = 1
00 000

r (s)
0
r (s)

r (s) = 0
c) Desde = 1 podemos inferir que la curva es una circunferencia y
por
= 0 concluimos, que la curva es plana.
d) Eliminando el parametro s en las ecuaciones parametricas
 
4 3
(x, y, z) = cos s, 1 sin s, cos s
5 5

Produce
x2 + (y 1)2 + z 2 = 1, 3x + 4z = 0

Autoevaluacion No 3

Pregunta 1
 
1 1
Dada la ecuacion parametrica de la curva

r (t) = t, + 1, t ,
t t
en el
punto (1, 2, 0) , determine:
a) la curvatura de la curva
b) La torsion de la curva

Pregunta 2
Sea r = r () la ecuacion de una curva C en coordenadas polares, con
a b angulo polar y P () punto de C. Demuestre que
Rbp
a) La longitud de C desde P (a) hasta P (b) es L = a r2 + (r0 )2 d
0 2
2(r ) rr00 + r2

b) La curvatura de C en P () es = .
[r2 + (r0 )2 ]3/2

184
c) Usando lo anterior,calcular la longitud L y la curvatura de la
espiral
logartmica r () = e , donde es constante.
Pregunta 3
Un punto se mueve sobre la parabola y = x2 . Si en el instante t = ,
P
0 00
se encuentra en (0, 0) y s () = a ,s () = b ; donde la funcion s = s (t)
es la longitud de arco de la curva y a , b constantes. Determine la
velocidad y la aceleracion de P en el instante t = .

Pauta de Autocorreccion

Pregunta 1
El valor del parametro en el punto viene dado por
 

1 1
r (t0 ) = t0 , + 1, t0 = (1, 2, 0) = t0 = 1
t0 t0
Derivando sucesivamente
 

1 1
r (t) = 1, 2 , 2 1 =
0 r 0 (1) = (1, 1, 2)
t t
 

2 2
r (t) = 0, 3 , 3 =
00
r 00 (1) = (0, 2, 2)
t t
 

6 6
r 000 (t) = 0, 4 , 4 = r 000 (1) = (0, 6, 6)
t t
La curvatura se determina mediante la identidad

r 0 (1) 00

r (1) k(2, 2, 2)k 2
(1) = = =
k 3/2 6
r 0 (1)k 63

Mientras que la torsion esta dada por la identidad



r 0 (1)
r (1)
00
r 000 (1) k(2, 2, 2) (0, 6, 6)k
(1) =

2 = =0
k r 0 (1) r 00 (1)k 63
Observese que la torsion es cero en todos los puntos ya que la curva
esta
contenida siempre en el mismo plano.
Pregunta 2

185
a) En este caso podemos parametrizar la ecuacion de la curva C de la
forma

r =r () = (r () cos , r () sin )
Derivando esta expresion respecto de obtenemos

r =
0 0 0 0
r () = (r () cos r () sin , r () sin + r () cos )
De modo que
1/2 q 0
0
 0
0
r () = r () r ()
= (r cos r sin )2 + (r0 sin + r cos )2
=
0 p
r () = r2 + (r0 )2 .
Entonces la longitud del arco entre = a y = b para la curva
r ()
es
R b
0
Rbp
L= a
r () d = a r2 + (r0 )2 d.


r 0 () 00

r ()
b) Deduciremos la curvatura usando la formula () =
k 3/2
r 0 ()k
Tenemos que

0 0
r = r () (cos , sin ) + r () ( sin , cos )
Derivando por segunda vez obtenemos

00 00 0
r = r () (cos , sin ) + 2r () ( sin , cos ) r () (cos , sin )
Determinemos el producto vectorial considerando R2 un subespacio
vectorial
de R3

r
0 00
h 0 i
r = r (cos , sin , 0) + r( sin , cos , 0)
h 00 0
i
r (cos , sin ) + 2r ( sin , cos ) r(cos , sin )
 0 00

= 0, 0, 2(r )2 rr + r2

ya que se satisfacen que


(cos , sin , 0) (cos , sin , 0) = (0, 0, 0) y
(cos , sin , 0) ( sin , cos , 0) = (0, 0, 1)
De esta forma


r
0 00
0 00

r = 2(r )2 rr + r2

186
Por lo tanto


r 0 () 00
0 2
2(r ) rr00 + r2

r ()
() = =
k 3/2
r 0 ()k [r2 + (r0 )2 ]3/2
0 00
c) Sea r () = e = r () = e = r () = 2 e .
Aplicando directamente las formulas obtenidas en a) y b), obtenemos
la longitud de la curva y su curvatura
Z bp Z bp
2 0 2
L = r + (r ) d = e2 + 2 e2 d
a a
1 + 2 b
e ea

=

0 2
2(r ) rr00 + r2

(1 + 2 ) e2
() = =
[r2 + (r0 )2 ]3/2 (1 + 2 ) 1 + 2 e3
1
=
1 + 2 e

Pregunta 3
Dada la ecuacion cartesiana de la curva y = x2
Sea

r (t) = (x (t) , y (t)) la funcion vectorial que describe el movimiento
del
punto tal que y (t) = [x (t)]2 , entonces al derivar esta expresion tenemos
0 0 00 0 00
y (t) = 2x (t) x (t) = y (t) = 2((x (t))2 + x (t) x (t)).
Sabemos que en el instante t = 0,

r () = (x () , y ()) = (0, 0) =
x () = 0, y () = 0
0 00 0 2
Al evaluar las derivadas de y produce: y () = 0, y () = 2 x ()
Por otra parte , sabemos que:
0

0
q 0 0
s () = r () = (x ())2 + (y 0 ())2 = x () = a


 0 00 0 00 
00

0
0 x (t) x (t) + y (t) y (t)
s () = r (t) () = () =
k
r 0 (t)k
0 00
00 x () x () 00
s () = = b = x () = b
a
187
Por lo tanto, la velocidad y la aceleracion en el punto P son

0
 0 
r () = x () , y 0 () = (a, 0)
00


 00 
r () = x () , y 00 () = b, 2a2


188
Captulo 3

Funciones de varias variables

3.1. Introduccion
El concepto de funcion, ya establecido, en cursos precedentes puede ser
extendido a situaciones del tipo


f : A IRm B IRn , m, n IN

en el sentido que son correspondencias que asignan a cada vector


x





A un unico vector y B anotado y = f ( x ) . Consideradas las
componentes de
x y de y la igualdad anterior, tambien, se anota de
la forma


(y1 , y2 , . . . , yn ) = f (x1 , x2 , . . . , xm );
estas funciones se denominan funciones vectoriales de variable vectorial
, o bien, n funciones dependientes de m variables independientes. Sus
respectivos dominio de definicion A y recorrido B se denominan campos
vectoriales.
En este curso se consideran funciones del tipo


f : A IRm B IR

esto es con m > 1 y n = 1 , que por las caracterstica de los espacios


pasan a llamarse, funciones reales de variable vectorial ( de m variables
independientes). En la unidad anterior fueron consideradas funciones
vectoriales


f : A IR IRn

189
es decir, con m = 1 y n > 1. Estas funciones se denotan como


y = f (x1 , x2 , . . . , xm )

alternativamente
y = f ( x ) conx = (x , x , . . . , x ) y se estudiaran
1 2 m
sus propiedades en cuanto a variaciones, graficas, aplicaciones, etc, a
partir de los conceptos de lmite, continuidad, y derivada. En particular,
para los casos usuales m = 2 y m = 3, se denotan z = f (x, y), (x, y)
A IR2 ; w = f (x, y, z)(x, y, z) A IR3 , respectivamente.

Ejemplos

Caso 1)
a) La correspondencia z = f (x, y) = xy , A = IR2 , B = IR , restringi-
dos x e y a valores positivos f (x, y) = xy corresponde al area del
rectangulo de lados x e y (comparado con f (r) = r2 , esta representa
el area del crculo de radio r y es funcion del tipo f : A IR B
IR, o sea con m = 1 y n = 1.
b) La correspondencia w = f (x, y, z) = xyz , tiene A = IR3 , B = IR
, y con x, y, z positivos representa el volumen V del prisma recto de
aristas x, y, z.
Caso 2)
a) La funcion f : A IR2 B IR dada por
p
z = f (x, y) = x2 + y 2 , A = IR2 , B = IR,

representa la distancia del punto P = (x, y) al origen O = (0, 0) de


IR2 .
b) La funcion f : A IR3 B IR dada por
p
w = f (x, y, z) = x2 + y 2 + z 2 , A = IR3 , B = IR,

representa la distancia del punto P = (x, y, z) al origen O = (0, 0, 0)


de IR3 .
Caso 3)

1
La funcion z = f (x, y) = p , en que f : A IR2 B IR
+ x2 y2
tiene dominio A = IR2 ,y recorrido B = IR, representa el potencial

190
electrostatico en cada punto P = (x, y) del plano debido a una carga
unitaria colocada en el origen O = (0, 0) de IR2 .
Nota: En estos ejemplos de funciones con m = 2, o m = 3 se puede
considerar los conceptos de lneas de nivel y superficies de nivel como

S = {(x, y) A/ f (x, y) = c}

para c constante dada; y

S = {(x, y, z) A/ f (x, y, z) = c}

para c constante dada.


n p o
2 2 2
a) En el caso 2) se tiene que S = (x, y) IR / x + y = c con

c 0 son circunsferencias de radio c (o el origen si c = 0).
( )
1
b) En el caso 3) S = (x, y) IR2 {(0, 0)} / p = c, c > 0
x2 + y 2
1
las lneas que verifican x2 + y 2 = 2 se llaman lneas equipotenciales
c
alrededor de la carga, describen circunsferencias centradas el (0, 0) de
1
radio .
c
Unido a lo anterior, en topografa las curvas determinadas por ecuacion
f (x, y) = c se llaman tambien contornosp de nivel de una superficie.
Por ejemplo, la funcion z = f (x, y) = 25 x2 y 2 que tiene

A = (x, y) IR2 / x2 + y 2 25 , B = [0, 5]




representa es
puna montana. ferica de radio basal 5 y altura 5; con c = 3
se tiene 25 x2 y 2 = 3, de lo cual se tiene el contorno de nivel
x2 + y 2 = 16.
Caso 4)
La presion P ejercida por un gas ideal encerrado en un cilindro piston es
T
dada por la funcion P (T, V ) = k donde k cte, T es la temperatura
V
y V el volumen del cilindro. En este caso las lneas de nivel reciben el
nombre de lneas isobaricas P = c, o isotermas cuando T = c con T
funcion temperatura en cada punto (x, y, z) .
Caso 5)

191
a) El area S de la superficie del cuerpo humano es una funcion del
peso w y la altura h dada por S (w, h) = 0, 091w0,425 h0,725 (deduccion
emprica), donde w esta en libras y h en pulgadas y S en pie2 .
b) Tambien S es dada por S (h, t) = 2ht donde h es la altura en cm
y t es la longitud de la circunferencia maxima del muslo en cm. Por
ejemplo, una nina de altura h = 156 cm y 50 cm de de circunsferencia
maxima del muslo tienen una superficie corporal de 15,600 cm2 .

Caso 6)
La funcion f (x, y) = C xa y 1a con C y a constantes, donde x son las
unidades de trabajo e y las unidades de capital representa un modelo
de unidades de produccion (Modelo de Cobb-Douglas), que se utiliza
en economa y en la evaluacion de proyectos.

Caso 7)
a) La potencia electrica para un voltage E y resistencia R es la funcion
E2
de dos variables P (E, R) = .
R
b) La resistencia total al conectar en paralelo dos resistencias R1 , R2
1 1 1
es regido por la ecuacion = + , de donde se deduce R =
R R1 R2
R (R1 , R2 ) .
Caso 8)
a) La aceleracion centrpeta de una partcula que se mueve en la cir-
v2
cunferencia de radio r siendo v la rapidez es dada por a (r, v) = .
r
b) El desplazamiento vertical de una cuerda larga sujeta en el origen,
que cae bajo la accion de su propio peso es dada por

g (2axt x2 ) , si 0 x at

u (x, t) = 2a2 g
2 t2 , si x > at, a cte
2a
Caso 9)
La concentracion molecular C (x, t) de un lquido es dada por
x2
C (x, t) = t1/2 e k t

y esta funcion verifica la ecuacion de difusion

192
k 2C C
= .
4 x2 t

Caso 10)
Cuando una chimenea de h metros de altura emite humo que contiene
contaminante oxido ntrico la concentracion C (x, z) a x km de distancia
y z metros de altura es dada por
 
a b(zh)2

b(zh)2
C (x, z) = 2 e x 2
+e x 2
x
C C
Calcular e interpretar los valores de , ,en el punto P = (2, 5)
x t
para a = 200, b = 0, 02; h = 10 m.
Caso 11)
Un ejemplo de una funcion que depende de 4 variables es la que es-
tablece la ley de Poiseuille en la cual la intensidad del flujo de un fluido
viscoso (como la sangre) traves de un conducto (como una arteria), es
R4
Q (R, L, p1 , p2 ) = k (p1 p2 )
L
con k cte, R radio de conducto, L su longitud y p1 , p2 presiones en los
extremos del conducto.

3.2. Funciones Escalares de Variable Vec-


torial
3.2.1. Conceptos Topologicos
Nuestro espacio universo sera Rn pensado preferentemente con n = 2
o n = 3.
Si

x Rn entonces cada vector
x = (x , x , . . . , x ) esta conformado
1 2 n
por una n- upla ordenada de numeros reales.
La metrica que se usara, es la metrica usual, es decir si

x ,

y Rn tal
que:

x = (x1 , x2 , . . . , xn )

y = (y1 , y2 , . . . , yn )

193
entonces

n
! 21
k

x

X
yk= (xi yi )2
i=1

A kk se le llama norma euclidea y permite calcular la distancia entre


los puntos x e y.
Si y = 0 entonces
n
! 12
k

X
xk= x2i
i=1

Vecindad

Sea x0 Rn , una vecindad de x0 es:

V (x0 ) = {x Rn | kx x0 k < }

En R2 esta vecindad es un disco centrado en P0 . En R3 es una esfera


centrada en P0 y de radio
Caso especial es el de la vecindad despuntada

V (x0 ) = V (x0 ) {x0 }

Punto Interior

Sea S Rn , x S es un punto interior de S si existe > 0 tal que:

V (x) S

Conjunto Abierto

Sea A Rn , diremos que A es un conjunto abierto si y solo si todos


sus puntos son interiores.

194
Ejemplo: Cuales de los siguientes conjuntos son abiertos?

A = {(x, y) R2 | x2 + y 2 1} No es conjunto abierto


A = {(x, y) R2 | |x| < 1, |y| < 1} Es conjunto abierto
A = {(x, y) R2 | 0 x < 1} No es conjunto abierto

Conjunto Cerrado

Sea B Rn , diremos que B es un conjunto cerrado si su complemento


Rn B es abierto.

Punto Frontera

Sea A Rn , x0 es un punto frontera de A si y solo si > 0 : V (x0 )


A 6= y V (x0 ) (Rn A) 6=

Interior de un Conjunto

Sea A Rn ,el conjunto de todos los puntos interiores de A se llama el


o
interior de A y se denota A o sea:
o
A = {x | x es punto interior de A}

Frontera de un Conjunto

Sea A Rn , la frontera de A es el conjunto de todos los puntos frontera


de A y se denota F r(A).

Proposicion 1 Si S es abierto, no contiene ninguno de sus puntos


fronteras.
Dem. Directamente de la definicion de conjunto abierto.

195
Proposicion 2 Si S es cerrado, contiene a todos sus puntos fronteras.

Segmento Lineal

Si P1 , P2 Rn , el segmento lineal que une P1 con P2 es P = (1 t)P1 +


P2 , 0 t 1. P1 se llama punto inicial y P2 punto terminal.

Linea Poligonal

Una poligonal esta formada por un numero finito de segmentos lineales


unidos sucesivamente por sus extremos.

Punto Aislado

Sea A Rn , x0 es punto aislado de A si y solo si x0 A y > 0 :

V (x0 ) A =

Punto de Acumulacion

Sea A Rn , x Rn , x es punto de acumulacion de A si toda


vecindad despuntada de x contiene puntos de A, es decir:
x es punto de acumulacion de A V (x0 ) A 6=

Region

Sea R Rn ,diremos que el conjunto R es una region si es un conjunto


abierto y cualquier par de puntos de ella pueden unirse mediante una
lnea poligonal, contenida en R.

196
Region Cerrada

Es una region unida con su frontera.

Teorema 3.2.1. Sea C Rn y C es conjunto cerrado, entonces C


contiene en todos sus puntos de acumulacion.

3.2.2. Aspectos Geometrico de las Funciones Es-


calares
Sea f : D Rn R esta funcion a cada x D, x = (x1 , x2 , . . . , xn )
le asigna una imagen b R, b un escalar.
b = f (x1 , x2 , . . . , xn )
a este tipo de funciones se les llama usualmente campo escalar.
Con el objeto de resaltar sus aspectos geometricos, especialmente para
funciones de dominio en R2 o R3 analizaremos conceptos como, grafi-
cas, curvas de nivel, superficies de nivel, trazos, etc. de estas funciones.
Sea f : D Rn R si x D = x = (x1 , x2 , . . . , xn ), D es el
dominio de la funcion f.

f (D) = Im(f ) = {z R | z = f (x1 , x2 , . . . , xn )}

3.2.3. Grafica de una Funcion


Sea f : D Rn R , definimos la grafica de f como el subconjunto
de Rn+1 formado por todos los puntos (x1 , x2 , . . . , xn , f (x1 , x2 , . . . , xn ))
para cada (x1 , x2 , . . . , xn ) Rn . Simbolicamente

Gf = (x1 , x2 , . . . , xn , f (x1 , x2 , . . . , xn )) Rn+1 | (x1 , x2 , . . . , xn ) D




As, si n = 2 y f es tal que z = f (x, y) entonces su grafica sera:

Gf = (x, y, z) R3 | z = f (x, y)


197
3.2.4. Curvas y Superficies de Nivel

Sea f : D Rn R y C un escalar. Entonces el conjunto de puntos de


nivel de valor C ,se define como el conjunto de puntos (x1 , x2 , . . . , xn )
D para los cuales f (x1 , x2 , . . . , xn ) = C.Simbolicamente, el conjunto
de nivel

S = {(x1 , x2 , . . . , xn ) D | f (x1 , x2 , . . . , xn ) = C}

Si n = 2 el conjunto {(x, y) | f (x, y) = C} es una curva de nivel.


Si n = 3 el conjunto {(x, y, z) | f (x, y, z) = C} es una superficie
de nivel.

Ejemplo:
1.- Trazar la curva de nivel de f (x, y) = x y, C = 0, 1, 2

2.- Trazar la curva de nivel de f (x, y) = 10 x2 4y 2 , C = 0, 1, 2


Es familia de elipses centradas para C < 10, cuyo eje mayor se
encuentra sobre el eje x

Observacion: Las curvas y las superficies de nivel permiten


dar una imagen de la grafica de la funcion.

198
3.2.5. Lmite
Sea f : D Rn R una funcion escalar y x0 un punto de acumulacion
del dominio D. Formularemos la definicion de lmite de una funcion
escalar de la siguiente forma.

Definicion 3.2.1. La funcion f tiene como lmite al numero L R


cuando x x0 , si para cada  > 0, existe (, x0 ) > 0 tal que

x D y 0 < kx x0 k < entonces |f (x) L| < 

Simbolicamente:

lm f (x) = L  > 0, > 0 :


xx0
x D y 0 < kx x0 k < = |f (x) L| < 

En el caso particular n = 2 esta definicion es

lm f (x, y) = L  > 0, > 0 : (x, y) D y


(x,y)(x0 ,y0 )

0 < k(x, y) (x0 , y0 )k < = |f (x, y) L| < 

Ejemplo

Sea f (x, y) = x2 + 2xy. Determinar si existe lm (x2 + 2xy)


(x,y)(3,1)

199
Analisis:
Si (x, y) (3, 1) significa que x esta cercano a 3 e y esta cercano a
1 por lo tanto el valor de x2 + 2xy debe estar proximo a 3 se espera
entonces que si este lmite existe debe ocurrir que:

x2 + 2xy = 3

lm
(x,y)(3,1)

Observe que:
q q
|x 3| (x 3) (x 3)2 + (y + 1)2 = k(x, y) (3, 1)k
2

q q
|y + 1| (y + 1) (x 3)2 + (y + 1)2 = k(x, y) (3, 1)k
2

por lo cual:


k(x, y) (3, 1)k < = |x 3| < y |y + 1| < x2 + 2xy 3 <

por otro lado , se tiene


2
x + 2xy 3 = (x 3)2 + 2(x 3)(y + 1) + 4(x 3) + 6(y + 1)
|x 3|2 + 2 |x 3| |y + 1| + 4 |x 3| + 6 |y + 1|

Sin perdida de generalidad se puede poner la condicion < 1 entonces


mayorando termino a termino, produce

2
x + 2xy 3 < |x 3| + 2 |y + 1| + 4 |x 3| + 6 |y + 1|
< + 2 + 4 + 6 = 13


Definiendo = min 1, 13 ,

Todo lo anterior permite afirmar que:



k(x, y) (3, 1)k < x2 + 2xy 3 < 13 x2 + 2xy 3

< 13 = 13 =
13
200
Lo cual prueba que lm (x2 + 2xy) = 3
(x,y)(3,1)

Consideremos otro ejemplo en que usamos tambien la definicion pero


otro procedimiento

Ejercicio:

Probar que

x2 y 2
 
lm =0
(x,y)(0,0) x2 + y 2

Solucion: Sea  > 0 y

2
x2 x2 + y 2 , y 2 x2 + y 2 = x2 y 2 x2 + y 2
x2 y 2
= x2 + y 2 , (x, y) 6= (0, 0)
x2 + y 2

p
Sea = , k(x, y) (0, 0)k = k(x, y)k = x2 + y 2

p p
x2 + y 2 < = x2 + y 2 < = x2 + y 2 <
2 2 2 2
xy
2 = x y <
x + y 2 x2 + y 2

Lo que prueba que

x2 y 2
 
lm =0
(x,y)(0,0) x2 + y 2

201
Los Teoremas de Lmites

Sean f : D1 Rn R , g : D2 Rn R tales que si a =


(a1 , a2 , . . . , an ) es punto de acumulacion de D1 y D2 y lm f (x) = L,
xa
lm g(x) = M, aqu x = (x1 , x2 , . . . , xn ). Entonces:
xa

a.) lm (f + g) (x) = L + M
xa

b.) lm (f g) (x) = L M
xa

c.) lm (f g) (x) = L M
xa
 
f L
d.) lm (x) = , M 6= 0
xa g M

La demostracion de estas propiedades es identica a la correspondiente


de las funciones de R en R.

Teorema 3.2.2. Sea f : D Rn R tal que lm f (x) = L, y g : I


xa
R R es una funcion continua en I. Entonces g f : D Rn R y
 
lm (g f ) (x) = g lm f (x) = g(L)
xa xa

2
Ejemplo: Evaluar lm ex+y
(x,y)(3,2)

Solucion:
Sea g(z) = ez continua en todo R, lm x + y 2 = 7, entonces
(x,y)(3,2)

2
lm ex+y = e7
(x,y)(3,2)

Trayectorias:

Sea (x, y) un punto de R2 . Una trayectoria por (x, y) es cualquier recta


o curva que contenga a (x, y)

202
Regla de las dos trayectorias Una condicion necesaria (no sufi-
ciente) para que lm f (x, y) exista y sea L, es que si los lmites
(x,y)(x0 ,y0 )
lm f (x, (x)) y lm f (x, (x)) existen , para cualquier trayectoria
xx0 xx0
y = (x) , y = (x) que pase por (x0 , y0 ),deben valer L

x2 y
Ejemplo: Sea f (x, y) = x4 +y 2
Existira lm f (x, y)?.
(x,y)(0,0)

Solucion:
Dominio de f es R2 {(0, 0)} y claramente (0, 0) es punto de acumu-
lacion
del dominio de f .
Sea T1 = {(x, y) | y = ax} familia de rectas que pasan por el origen si
el
lmite existe debera ocurrir que:

x2 y x2 (x)
lm = lm
(x,y)(0,0) x4 + y 2 x0 x4 + 2 x2

(x)
= lm 2 =0
x0 x + 2

Esto senala que si el lmite existe este debe ser cero, seguimos averiguan-
do:
Sea T2 = {(x, y) | y = x2 } parabola por el origen.

x2 y x4 1
lm 4 2
= lm 4 4
=
(x,y)(0,0) x + y x0 x + x 2

No puede ser, el lmite si existe no puede tener dos valores diferentes.


Luego, no existe
x2 y
lm
(x,y)(0,0) x4 + y 2

Lmites Iterados

Se llaman lmites iterados a los siguientes.


   
lm lm f (x, y) ; lm lm f (x, y)
xx0 yy0 yy0 xx0

203
x2 y 2
Ejemplo: Sea f (x, y) = x2 +y 2
, (x, y) 6= (0, 0) .Determine los lmites
iterados de f.
Solucion:
Tenemos que (0, 0) es punto de acumulacion del dominio de f .
Evaluemos, los lmites iterados

x2 y 2 x2
 
lm lm 2 = lm =1
x0 y0 x + y 2 x0 x2

x2 y 2 y 2
 
lm lm = lm 2 = 1
y0 x0 x2 + y 2 y0 y

Los conceptos de lmites y lmites iterados se relacionan segun los sigu-


ientes
teoremas.
Teorema 3.2.3. Si lm f (x, y) existe, y si para cada x en una
(x,y)(x0 ,y0 )
vecindad reducida de x0 , lm f (x, y) existe. Entonces
yy0
 
lm lm f (x, y) = lm f (x, y)
xx0 yy0 (x,y)(x0 ,y0 )

Este teorema nos lleva a formular otro equivalente.


Teorema 3.2.4. Si lm f (x, y) existe, y si para cada y en una
(x,y)(x0 ,y0 )
vecindad reducida de y0 , lm f (x, y) existe. Entonces
xx0
 
lm lm f (x, y) = lm f (x, y)
yy0 xx0 (x,y)(x0 ,y0 )

Ahora, se puede combinar ambos teoremas, lo que produce:


Teorema 3.2.5. Si lm f (x, y) existe, y si para cada x en una
(x,y)(x0 ,y0 )
vecindad reducida de x0 , lm f (x, y) existe, y si para cada y de una
yy0
vecindad reducida de y0 , lm f (x, y) existe. Entonces
xx0
   
lm lm f (x, y) = lm lm f (x, y) = lm f (x, y)
yy0 xx0 xx0 yy0 (x,y)(x0 ,y0 )

x2 y 2
En el ejemplo anterior podemos concluir que no existe lm 2 2 ,pues
(x,y)(0,0) +y
x

de existir su lmites iterados deben ser iguales.

204
3.2.6. Continuidad

Una funcion f es continua en a ( a es punto de acumulacion de D Rn )


si para cada > 0 existe > 0 tal que

x D : kx ak < = |f (x) f (a)| <


donde x = (x1 , x2 , ..., xn ), a = (a1 , a2 , ..., an ). O lo que es lo mismo:
Se tiene que f es continua en un punto a si:
i) f (a) existe, ii)lm f (x) existe y ; iii)lm f (x) = f (a).
xa xa

Es decir, para que una funcion de varias variables sea continua en un


punto debe estar definida all, debe tener lmite en el y el valor de la
funcion en el punto debe ser igual al valor del lmite en ese punto. En
R2 podemos enunciar esta propiedad de la siguiente forma.
Una funcion f en continua en un punto interior (x0 , y0 ) de una region
R si f (x0 , y0 ) esta definida y

lm f (x, y) = f (x0 , y0 )
(x,y)(x0 ,y0 )

f sera continua en la region R si es continua en cada punto de R. Las


funciones que no son continuas se dicen que son discontinuas.

Teoremas de Continuidad

Son similares a los teoremas para funciones de una variable. Esto sig-
nifica que, si una funcion es combinacion de otras funciones y estas
funciones a su vez son continuas entonces la funcion es continua excep-
to en aquellos puntos en los que no esta definida.


x2 y 2
si (x, y) 6= (0, 0)

Ejemplo. Sea la funcion f (x, y) = x2 + y 2 ,estudie
0 si (x, y) = (0, 0)

la continuidad de f en R2 .
Solucion.

205
Claramente si (x, y) 6= (0, 0), f es un cuociente de funciones con-
tinuas por lo que tambien es continua. Ademas, la funcion es continua
en (0, 0) pues se demostro en 1.4.2, que

x2 y 2
lm = 0 = f (0, 0),
(x,y)(0,0) x2 + y 2

Por lo tanto, esta funcion es continua en todo R2 . Continuando con


este mismo ejemplo, podemos usar coordenadas polares para mostrar
de que este ultimo lmite vale cero. Decir que (x, y) (0, 0) , en
coordenadas polares es equivalente a que r 0 (independientemente
del valor de ).
x2 y 2
Expresando la funcion f (x, y) = en coordenadas polares,
x2 + y 2
x = r cos , y = r sin , obtenemos la funcion
r2 cos 2 sin 2 2
2 cos sin
2
g(r, ) = = r
cos 2 + sin 2 cos 2 + sin 2

observese que
cos 2 sin 2
() = = cos 2 sin 2 1.
cos 2 + sin 2

esta acotada, y ademas


(r) = r2

Lo que implica que:

lmg(r, ) = lm(r)() = 0.
r0 r0

Podemos concluir que si () M esta acotada, en una vecindad con


centro en el origen, y (r) 0 cuando r 0,entonces lm(r)() =
r0
0.

Continuidad en un conjunto abierto.

Diremos una funcion es continua en un conjunto abierto U Rn ,si ella


es continua en cada punto del conjunto U .

206
3.2.7. Derivadas Parciales

Sea f : D R2 R funcion de dos variables que esta definida en una


vecindad del puntos (x0 , y0 ). La derivada parcial de f respecto de x en
(x0 , y0 ) se define por:

f f (x0 + h, y0 ) f (x0 , y0 )
(x0 , y0 ) = lm
x h0 h

Similarmente, la derivada parcial de f respecto de y en (x0 , y0 ) se define


por:
f f (x0 , y0 + h) f (x0 , y0 )
(x0 , y0 ) = lm
y h0 h
siempre que estos lmites existan. Las derivadas de orden superior son
una reiteracion de la definicion anterior, es decir, derivadas sucesivas,
as:

2f
 
f fx (x0 + h, y0 ) fx (x0 , y0 )
= = lm
x2 x x h0 h
2f
 
f fy (x0 , y0 + h) fy (x0 , y0 )
2
= = lm
y y y h0 h
2
 
f f fy (x0 + h, y0 ) fy (x0 , y0 )
= = lm
xy x y h0 h

Ejemplo:

Ejemplo:

Calcular las derivadas parciales de:

f (x, y) = x4 sin y + cos xy

Solucion:
De acuerdo con la definicion debemos calcular

(x + h)4 sin y + cos(x + h)y (x4 sin y + cos xy)


lm
h0 h

207
Desarrollando el numerador se tiene:

x4 sin y + 4x3 h sin y + 6x2 h2 sin y + 4xh3 sin y + h4 sin y


+ cos(xy) cos(hy) sin(xy) sin(hy) x4 sin y cos(xy) =
4x3 h sin y + 6x2 h2 sin y + 4xh3 sin y + h4 sin y
+ cos(xy)(cos(hy) 1) sin(xy) sin(hy) =

Calculando lmite

(x + h)4 sin y + cos(x + h)y (x4 sin y + cos xy)


lm
h0 h
(cos(hy) 1) sin(hy)
= 4x3 sin y + cos xylm sin(xy)lm
h0 h h0 h

Como
(cos(hy) 1) sin(hy)
lm =0 y lm =y
h0 h h0 h

Tenemos que este lmite y por tanto la derivada es


f
(x, y) = 4x3 sin y y sin(xy)
x

De igual forma se puede calcular por definicion.

f
(x, y) = x4 cos y x cos xy
y

Derivadas Parciales Cruzadas

Teorema 3.2.6. (Teorema de Schwarz)


2f
Sea f : D R2 R una funcion, D abierto. Si las derivadas y
xy
2f
existen y son continuas en D, entonces:
yx
2f 2f
=
xy yx

208
Demostracion queda propuesta.
Como ejemplo ilustrativo considere la funcion
(
x3 yxy 3
x2 +y 2
si (x, y) 6= (0, 0)
f (x, y) =
0 si (x, y) = (0, 0)

y verifique que:

2f 2f
(0, 0) = 1, (0, 0) = 1
xy yx

Observacion: Use la definicion y encuentre estos resultados.

3.3. Diferenciabilidad en dos variables


Sea f una funcion definida en una vecindad de (x0 , y0 ). Diremos que
f es diferenciable en (x0 , y0 ) si existen numeros A y B tales que:

f (x0 + h, y0 + k) f (x0 , y0 ) = Ah + Bk + (h, k) k(h, k)k

y si el residuo tiene la propiedad lm (h, k) = 0. Diremos que f


k(h,k)k0
es diferenciable en una region, si es diferenciable en cada punto de la
region.

Teorema 3.3.1. Si f es diferenciable en (x0 , y0 ), entonces f es con-


tinua en (x0 , y0 ).

Demostracion:
Sea (x, y) V ((x0 , y0 )) y

f (x0 +h, y0 +k)f (x0 , y0 ) = A (x x0 )+B (y y0 )+ (h, k) k(x, y) (x0 , y0 )k

Ademas sabemos que

|x x0 | k(x, y) (x0 , y0 )k , |y y0 | k(x, y) (x0 , y0 )k

entonces

209
kf (x0 + h, y0 + k) f (x0 , y0 )k |Ah + Bk + (h, k)| k(x, y) (x0 , y0 )k

Si (x, y) (x0 , y0 ) , tenemos que (h, k) 0 y k(x, y) (x0 , y0 )k


0
por lo tanto kf (x0 + h, y0 + k) f (x0 , y0 )k 0

lm f (x0 + h, y0 + k) = f (x0 , y0 )
(h,k)(0,0)

La diferenciabilidad es tambien una condicion mas fuerte que la exis-


tencia de las derivadas parciales.

Teorema 3.3.2. Si f es diferenciable en (x0 , y0 ), entonces las derivadas


parciales de primer orden existen en (x0 , y0 ) y

fx (x0 , y0 ) = A
fy (x0 , y0 ) = B

Demostracion
Sea f diferenciable en (x0 , y0 ) =


f (x0 + h, y0 + k) f (x0 , y0 ) = Ah + Bk + (h, k) h2 + k 2

dividiendo por h 6= 0 y haciendo k = 0 se tiene:

f (x0 + h, y0 ) f (x0 , y0 ) |h|


=A+
h h

f (x0 + h, y0 ) f (x0 , y0 ) |h|
=
A = | (h)| = | (h)|

h h

pero lm (h) = 0
h0

f (x0 + h, y0 ) f (x0 , y0 )
lm =A
h0 h
De manera analoga

210
f (x0 , y0 + k) f (x0 , y0 )
lm =B
k0 k
Se puede tener tambien el siguiente criterio para la diferenciabilidad de
una
funcion.

Teorema 3.3.3. Sea f : D R2 R una funcion. Si f tiene primeras


derivadas parciales continuas en una region D. Entonces f es diferen-
ciable en cada punto de D.

Nota: Todas las ideas dadas para funciones de dos variables se pueden
extender a funciones definidas en un espacio n dimensional.

Diferencial Total

En dos dimensiones:
Si f es diferenciable en (x0 , y0 ) entonces la diferencial total es:

f f
df = dx + dy en (x0 , y0 )
x y

En tres dimensiones:.
Si f es diferenciable en (x0 , y0 , z0 ) entonces la diferencial total es:

f f f
df = dx + dy + dz en (x0 , y0 , z0 )
x y z

3.3.1. Derivada Direccional


Gradiente

Sea f : D Rn R, P D abierto y f diferenciable en P. Entonces el


vector gradiente de f en P se denota f (P ) y se define por la formula:
 
f f f
f (P ) = (P ) , (P ) , ..., (P )
x1 x2 xn

211
Ejemplo: Sea f (x, y, z) = 3x3 y 2 z. Calcular f (1, 2, 3).
Solucion:
La funcion tiene derivadas parciales continuas, entonces

f (x, y, z) = 9x2 y 2 z, 6x3 yz, 3x3 y 2




Evaluando, el gradiente de la funcion f en el punto (1, 2, 3) queda

f (1, 2, 3) = (108, 36, 12) = 12 (9, 3, 1)

Propiedades
Si f y g son funciones diferenciables demuestre que:

(f g) = f g + gf
 
f gf f g
= si g no es cero
g g2
Ejemplo
Si f es diferenciable en una variable y g = f (x2 + y 2 + z 2 ) . Calcular
g g.
Solucion.
Si ponemos u = x2 + y 2 + z 2 al derivar parcialmente se tiene
g g g
= 2xf 0 (u) ; = 2yf 0 (u) ; = 2zf 0 (u).
x y z

= g = 2f 0 (u)(x, y, z) = g g = 4 [f 0 (u)] 2 [(x, y, z) (x, y, z)]

= 4 [f(u)] 2 (x2 + y 2 + z 2 )
(Definicion Derivada direccional)
Sea f : D Rn R y P0 un punto interior de D. Sea u b vector
unitario kbuk = 1. La derivada direccional de f en P0 en la direccion de
u
b se define por:

u) f (P0 )
f (P0 + hb
lm
h0 h
Si este lmite existe f tiene derivada direccional en P0 en la direccion
f
de u
b y la denotamos (P0 ) .
b
u
212
f
Teorema 3.3.4. Si f es diferenciable en P, entonces (P ) existe
b
u
para todos los vectores u
b, unitarios y
f
(P ) = f (P ) u
b
b
u

Demostracion:
Considerando la demostracion en R3 .Sean P0 = (x, y, z) y u
b = (u1, u2 , u3 )
Por definicion de diferenciabilidad se tiene:

u) f (P0 ) = f (x + hu1 , y + hu2 , z + hu3 ) f (x, y, z)


f (P0 + hb
f f f
= h u1 + h u2 + h u3 + |h|
x y z
 
f f f
= h u1 + u1 + u3 + |h|
x y z

Dividiendo por h y tomando lmite cuando h 0 se tiene 0 y

u) f (P0 )
f (P0 + hb f f f
lm = u1 + u2 + u3
h0 h x y z

f
(P0 ) = f (P0 ) u
b
b
u
f
Nota: Para cada u b vector unitario fijo, la derivada direccional de-
b
u
fine una nueva funcion, a la cual a su vez se le puede aplicar la definicion
de derivada direccional y tenemos as las derivadas direccionales de or-
den superior.

2f
 
f
2
=
bu b
u b
u

Ejemplo
Sea f (x, y) = x3 y 2 , calcular la derivada direccional de f en el punto
P0 (1, 2) en la direccion del vector

u = (4, 3).
Solucion.
Como f (x, y) = x3 y 2 en una funcion diferenciable (x, y) R2 ,entonces

213
f
(P ) = f (P ) u
b
b
u
En primer lugar calculemos el vector gradiente
f (P0 ) = (3x2 y 2 , 2x3 y)
En segundo lugar calculemos el vector unitario

u (4, 3)
u
b= =
kuk 5
Se deduce que
f  (4, 3)
(x, y) = 3x2 y 2 , 2x3 y
b
u 5
Por lo tanto, evaluando en P0 (1, 2) queda
f  (4, 3)
(1, 2) = 3 (1)2 22 , 2 (1)3 2
b
u 5
96 12 108
= + =
5 5 5
Teorema 3.3.5. Supongamos que f tiene segundas derivadas parciales
continuas en una vecindad de un punto P. Entonces:
2f
2
u )2 f
(P ) = (b
bu
Demostracion:

f
= f1 u1 + f2 u2 + f3 u3
b u
2f f1 f2 f3
2
= u1 + u2 + u3
bu b
u b
u bu
2
f
= u1 (f11 u1 + f12 u2 + f13 u3 ) + u2 (f21 u1 + f22 u2 + f23 u3 )
bu2
+u3 (f31 u1 + f32 u2 + f33 u3 )

Como las derivadas cruzadas son iguales se tiene

2f
= u21 f11 + 2u1 u2 f12 + 2u1 u3 f13 + u22 f22 + 2u2 u3 f23 + u23 f33
bu2
 2

= u1 + u2 + u3 f = (bu )2 f
x y z

214
f
Consecuencias de (P ) = f (P ) u
b
b
u
f
1.) Si f (P ) = 0 = (P ) = 0 bu

u
2.) La direccion de maximo crecimiento de la derivada direccional viene
dada
f
por (P ) = kf (P )k en la direccion del vector gradiente.
bu
3.) La direccion de mayor decrecimiento (o mnimo crecimiento) viene
dada en
f
la direccion f (P ) y el valor mnimo es (P ) = kf (P )k .
bu

Demostracion:
Basta considerar

f
(P ) = f (P ) u
b = kf (P )k cos
b
u

3.3.2. Plano tangente y recta normal

Sea z = f (x, y) la ecuacion de una superficie cualquiera. En lo que sigue


conviene para mayor comprension del razonamiento usar la expresion
F (x, y, z) = 0 para denotar la superficie de nivel de la funcion

F (x, y, z) = f (x, y) z = 0 o
F (x, y, z) = z f (x, y) = 0

Sea entonces la superficie F (x, y, z) = 0 y P0 = (x0 , y0 , z0 ) un punto


de ella. Sea
r (t) = (x(t), y(t), z(t)) la ecuacion parametrica de una
curva C en dicha superficie y que pasa por el punto (x0 , y0 , z0 ).

Tomando la diferencial de F (x, y, z) = 0 tenemos

F F F
dx + dy + dz = 0
x y z

En P0 = (x0 , y0 , z0 ) de la curva donde



r (t0 ) = P0 se tiene

215
F (P0 ) F (P0 ) F (P0 )
x(t0 )dt + y(t0 )dt + z(t0 )dt = 0 =
x y z
F (P0 ) F (P0 ) F (P0 )
x(t0 ) + y(t0 ) + z(t0 ) = 0
x y z

Es decir
 
F (P0 ) F (P0 ) F (P0 )
, , (x(t0 ), y(t0 ), z(t0 )) = 0
x y z
5F (P ) 0
r(t ) = 0
0

Esto nos dice que 5F (P0 ) es perpendicular a r(t0 ). Esto ocurre para
toda curva C que pase por P0 y como
r(t0 ) es vector tangente a
C se tiene que 5F (P0 ) es perpendicular a toda recta tangente a
la superficie en P0 . Por lo tanto, 5F (P0 ) es un vector normal a la
superficie en P0 .

Plano Tangente

La expresion de la forma:

F (P0 ) F (P0 ) F (P0 )


(x x0 ) + (y y0 ) + (z z0 ) = 0
x y z
es la ecuacion del plano tangente a la superficie F (x, y, z) = 0 en el
punto P0 = (x0 , y0 , z0 ).
Si la superficie es z = f (x, y) = F (x, y, z) = z f (x, y) = 0 la
ecuacion del plano tangente se puede escribir, tambien como:

z z0 = fx (x0 , y0 )(x x0 ) + fy (x0 , y0 )(y y0 )

Recta Normal

Sean P0 (x0 , y0 , z0 ) un punto dado y el vector director de esta recta


5F (P0 )
La ecuacion vectorial es:

216
F (P0 ) F (P0 ) F (P0 )
(x, y, z) = (x0 , y0 , z0 ) + t( , , ), tR
x y z
Ecuacion parametrica:

F (P0 )
x = x0 + t
x
F (P0 )
y = y0 + t , tR
y
F (P0 )
x = z0 + t
z

Eliminando el parametro t obtenemos la ecuacion cartesiana:

x x0 y y0 z z0
= =
Fx (P0 ) Fy (P0 ) Fz (P0 )

Ejemplo: Sea z = ex (cos y + 1) en el punto (0, 2 , 1) ,calcular las

ecuaciones del plano tangente y la recta normal a la superficie.

Solucion.
La componentes del vector gradiente son

fx (x, y) = ex (cos y + 1), fy (x, y) = ex (seny)



fx (0, ) = 1, fy (0, ) = 1
2 2

Reemplanzando terminos en la ecuacion del plano tenemos:



z1 = 1 (x 0) + (1)(y )
2

= z 1 = x y +
2

Finalmente, la ecuacion del plano tangente queda

217

xyz+ +1=0
2

y

(x, y, z) = (0, , 1) + t(1, 1, 1)
2

la ecuacion de la recta normal en su forma vectorial.

3.3.3. Funcion Compuesta. La Regla de la Cadena.

En este modulo abordaremos en forma basica la siguiente problematica.


Si u es una funcion diferenciable de variables x, y, z, , y a su vez
estas ultimas son funciones de otras variables nuevas t y/o s podemos
encontrar la primera derivada parcial de u con respecto a las nuevas
variables t y/o s expresada en terminos de las derivadas parciales de
las funciones dadas?. As, por ejemplo, si un fenomeno fsico esta ocur-
riendo digamos en una region cilndrica, resultara mejor expresar las
cantidades que interesen en terminos de coordenadas cilindricas y no
en cartesianas.

Caso particular:

Sea f : D R2 R tal que u = f (x, y), tiene dos variables


independientes. Supongamos que cada una de estas variables es diferen-
ciable de una simple variable independiente t. Si x = x(t) y y = y(t),
la derivada de la funcion compuesta (regla de la cadena) con respecto
a t es

du u dx u dy
= +
dt x dt y dt
du
= u

r 0 (t)
dt

218
Caso particular de dos variable independientes simples.

Sea f : D R2 R tal que u = f (x, y), tiene dos variables


independientes. Estas variables a su vez son funciones diferenciables
de dos variables simples independientes t y s. La expresion de las
derivadas de la funcion compuesta (o regla de la cadena) es:

u u dx u dy
= +
t x dt y dt

u u dx u dy
= +
s x ds y ds
Las expresiones anteriores las podemos expresar matricialmente como:

    dx dy
u u u u dt dt
, = , dx dy
t s x y
ds ds

dz x
Ejemplo 1: Hallar , si z = , donde x = et y y = ln t.
dt y
Solucion.
Aplicando la derivacion compuesta, tenemos
 
dz u dx u dy dz 1 t x 1
= + = = e + 2
dt x dt y dt dt y y t
et et 1
 
dz 1 1
= 2
= et
dt ln t (ln t) t ln t t(ln t)2

u u
Ejemplo 2: Hallar y si u = f (x, y), donde x = t2 s2 ,
t s
y = ets

Solucion.

219
En este caso

u dx dy u
= fx (x, y) +fy (x, y) = = fx (x, y)(2t)+fy (x, y)(s ets )
t dt dt t

Similarmente

u dx dy u
= fx (x, y) +fy (x, y) = = fx (x, y)(2s)+fy (x, y)(t ets )
s ds ds s

De forma mas general si

f : D Rn R tal que z = f (y1, y2, y3, , yn, ), es funcion de n


variables independientes. Supongamos tambien que cada una de estas
variables independientes es funcion diferenciable de otras m variables
simples independientes x1 , x2 , x3 , , xm . La expresion de la derivada
de la funcion compuesta (regla de la cadena) es similar, pudiendose
escribir cada una de estas derivadas por

n
z X z yi
= , j = 1, 2, ..., m
xj i=1
y i x j

Esta ecuacion se puede escribir utilizando matrices, como

y1 y1

...
    x1 xm
z z z z .. ..
,..., = ,..., . .
x1 xm y1 yn
yn yn

x1 xm

 
yi
La matriz n m se denomina matriz jacobiana
xj i=1...n, j=1...m
de la transformacion yi = yi (x1, x2, x3, , xm ).

220
Ejemplo 3: Sea f (x, y, z) = x + x2 y + zey , en donde x, y, y z ,
estan relacionadas con u y v a traves de la transformacion

x = uv, y = u2 v 2 , z = u sin v

f f 2f
Calcule , , y , en el punto (u, v) = (2, 2).
u v vu

Solucion.
La matriz jacobiana para esta transformacion es

xu xv v u
yu yv = 2u 2v
zu zv senv u cos v

xu xv 2 2
yu yv (2, 2) = 4 4
zu zv sin 2 2 cos 2

Para (u, v) = (2, 2), los valores de x, y y z son 4, 0 , y 2sen2


respectivamente.Entonces la matriz reglon (fx fy fz ) es

(fx fy fz ) = 1 + 2xy x2 + zey ey




evaluando en el punto (4, 0, 2sen2) queda

1 + 2xy x2 + zey ey (4, 0, 2sen2) = (1



16 + 2sen2 1)

De donde

xu xv
(fu fv ) = (fx fy fz ) yu yv
zu zv
= (fx xu + fy yu + fz zu fx xv + fy yv + fz zv )

Evaluando

(fu fv ) (2, 2) = (66 + 9 sin 2 62 8 sin 2 + 2 cos 2)

Por lo tanto
f f
(2, 2) = 66 + 9 sin 2 y (2, 2) = 62 8 sin 2 + 2 cos 2
u v
221
Ahora bien, como

fu = fx xu + fy yu + fz zu
= (1 + 2xy )v + (x2 + zey )2u + ey sin v

Se tiene
2f
= fuv = 2(xyv + yxv )v + 1 + 2xy + 2(2xxv + zey yv + ey zv )u
vu
+ yv ey senv + ey cos v

Evaluando en el punto (2, 2) se tiene

2f
(2, 2) = 1 36sen2 + 9 cos 2
vu

3.3.4. Funcion Implcita


Cuando definimos funciones en forma implcita decimos, por ejemplo,
sea y = (x) una funcion diferenciable definida implcitamente por
medio de la ecuacion F (x, y) = 0, o bien, sea z = f (x, y), definida
implcitamente por la ecuacion F (x, y, z) = 0.
El primer caso lo podemos ejemplificar con una funcion y = (x) defini-
da por la ecuacion de la hiperbola x2 4xy 3y 2 = 9.
Muchas veces las ecuaciones que relacionan a las variables pueden ser
tan complejas y no lineales, que no es posible esperar encontrar rela-
ciones sencillas, si las hay, explcitas que expresen a una variable en
terminos de las otras.
En terminos generales, lo que se pide es que esta relacion entre las
variables exista localmente, es decir en alguna vecindad de un punto
donde las ecuaciones se satisfacen. La mayora de las veces no se esperan
resultados globales, es decir, no todos los puntos que satisfacen las
relaciones implcitas satisfacen tambien las explcitas.
Con el objeto de ilustrar estas ideas analicemos el siguiente ejemplo.

Ejemplo: Pruebe que la ecuacion z 5 + z + xy = 0 define una funcion


z = f (x, y) para todos los valores x e y.
Solucion.

222
Se debe establecer que para cada x e y se puede resolver en forma
unica para z la ecuacion de quinto grado. Ya que esto no es obvio,
razonamos utilizando la grafica de

u = z5 + z + a

grafica pendiente

Como u(z) = 5z 4 + 1, a = u es estrictamente creciente, entonces

para cada a existen z1 y z2 tal que u(z1 ) < 0 < u(z2 ) por lo cual
existe un unico z tal que u(z) = 0 para cada a .

Si se identifica a con la cantidad xy se ha establecido el hecho


planteado.

Luego, z = f (x, y) se define implcitamente por la ecuacion z 5 + z +


xy = 0

Derivacion implcita

Sea F : U R2 R definida en un conjunto abierto U . Sea P0 =


(x0 , y0 ) U un punto tal que:
i) F (x0 , y0 ) = 0
F F
ii) , son continuas en alguna vecindad V (P0 ) y
x y
F
iii) (x0 , y0 ) 6= 0.
z
Entonces existe una vecindad (x0 , x0 + ) de x0 , (y0 a, y0 + a)
vecindad de y0 , y una funcion unica f (implcita) de clase C 1 tal
que:
a) f (x0 ) = y0 y f (x) (y0 a, y0 + a) x (x0 , x0 + )

b) F (x, f (x)) = 0 x (x0 , x0 + )


c) x, V (x0 ) tiene derivadas que pueden calcularse como:

223
dy Fx (x, y)
=
dx Fy (x, y)

En el caso de funciones de dos variables definida implcitamente, el


siguiente teorema afirma la existencia de funciones implcitas bajo cier-
tas circunstancias y da formulas para obtener las derivadas parciales
de estas funciones.

Teorema 3.3.6. (Teorema de la funcio implcita)


Sea F : U R3 R definida en un conjunto abierto U . Sea P0 =
(x0 , y0 , z0 ) U un punto tal que:
i) F (x0 , y0 , z0 ) = 0
F F F
ii) , , son continuas en alguna vecindad V (P0 ) y
x y z
F
iii) (x0 , y0 , z0 ) 6= 0.
z
Entonces existe una vecindad V = V (x0 , y0 ), y una vecindad
(z0 a, z0 + a) de z0 y una funcion unica f (implcita) de clase C 1
sobre V tal que:
a) f (x0 , y0 ) = z0 y f (x, y) (z0 a, z0 + a) (x, y) V.

b) F (x, y, f (x, y)) = 0, (x, y) V.


c) (x, y) V = V (x0 , y0 ) tiene derivadas que pueden calcularse como:

F
f (x, y) (x, y, f (x, y))
= x
x F
(x, y, f (x, y))
z

F
(x, y, f (x, y))
f (x, y) y
=
y F
(x, y, f (x, y))
z

Este teorema se puede generalizar a mas variables, de tal modo que


si se dan las condiciones exigidas por el teorema y u = f (x, y, z, )

224
esta definida implcitamente por F (x, y, z , , u) = 0 y Fu (x, y, z ,
, u) 6= 0, entonces:

F F
f f y
= x , =
x F y F
z z

,etc.

z z
Ejemplo. Calcular y si z se define implcitamente en la
x y
ecuacion

x2 y 8xyz = yz + z 3

Solucion.
Tenemos que:

F (x, y, z) = x2 y + 8xyz + yz + z 3

Derivando parcialmente con respecto a x , y ,z.

Fx (x, y, z) = 8yz 2xy,


Fy (x, y, z) = 8xz + z x2 ,
Fz (x, y, z) = 8xy + y + 3z 2

Claramente estas derivadas son continuas en R3 , son funciones polinomicas,


Por consiguiente, F pertenece a C1 .
Para todo (x, y, z) en que Fz (x, y, z) 6= 0 se tiene.

z 8yz 2xy
=
x 8xy + y + 3z 2
z 8xz + z x2
=
y 8xy + y + 3z 2

225
3.3.5. Jacobiano

Si f1 , f2 , f3, . . . , fn son funciones diferenciables de Rn en R y si (x1 ,


x2 , x3 , . . . , xn ) D su dominio tal que

f1 = f1 (x1 , x2 , x3 . . . , xn )
f2 = f2 (x1 , x2 , x3 . . . , xn )
..
.
fn = fn (x1 , x2 , x3 . . . , xn )

El Jacobiano de las funciones f1 , f2 , f3, . . . , fn , respecto de las


variables x1 , x2 , x3 . . . , xn se define por el determinante de las primeras
(f1 , f2 , f3, . . . , fn )
derivadas parciales y se denota ,
(x1 , x2 , x3 . . . , xn )
f1 f1 f1


x 1 x2
. . . xn
f2 f2 f2
(f1 , f2 , f3, . . . , fn ) x1 x2 . . . xn
= ..
(x1 , x2 , x3 . . . , xn )

f f .


fn
n n
. . .
x1 x2 xn

Dos funciones definidas implcitamente

Teorema 3.3.7. Sean F , G: U R4 R definidas en un conjunto


abierto F (x, y, u, v) y G(x, y, u, v). Sea P0 = (x0 , y0 , u0 , v0 ) U un
punto tal que:
i) F (x0 , y0 , u0 , v0 ) = 0 y G(x0 , y0 , u0 , v0 ) = 0
ii) F y G tienen derivadas parciales continuas en alguna vecindad
V (P0 ) y
(F, G)
iii) (x0 , y0 , u0 , v0 ) 6= 0.
(u, v)
Entonces las ecuaciones F (x, y, u, v) = 0 , G(x, y, u, v) = 0 definen
funciones implcitas u = u(x, y), v = v(x, y) en alguna vecindad V
(x0 , y0 ), las cuales tienen primeras derivadas parciales continuas con
respecto a cada una de las variables, tal que :
a) u0 = u(x0 , y0 ), v0 = v(x0 , y0 ).
b) F (x, y, u (x, y) , v (x, y)) = 0 , G(x, y, u (x, y) , v (x, y)) = 0, V (x0 , y0 )

226
c) Las derivadas parciales de u y v en V (P0 ) estan dadas por

(F, G) (F, G)
u (x, v) u (y, v)
= , =
x (F, G) y (F, G)
(u, v) (u, v)
(F, G) (F, G)
v (u, x) v (u, y)
= , =
x (F, G) y (F, G)
(u, v) (u, v)

u v
Ejemplo Calcule las derivadas parciales y donde u y v
x x
estan definidas por las ecuaciones implcitas

x2 + 2uv = 1, x3 u3 + v 3 = 1

Solucion.
En este caso F (x, u, v) = x2 + 2uv 1 = 0 y G(x, u, v) = x3 u3 +
v3 1 = 0

(F, G) 2v 2u
= = 6(u3 + v 3 )
(u, v) 3u2 3v 2

(F, G)
6= 0 para todo (x, u, v) donde u 6= 0 y v 6= 0.
(u, v)

(F, G) 2x 2u
= 2 = 6x(v 2 xu)
(x, v) 3x 3v 2

(F, G) 2v 2x 2
= 2 2 = 6x(xv + u )
(u, x) 3u 3x

Entonces, derivando u y v con respecto a x, obtenemos

u 6x(v 2 xu) x(xu v 2 )


= =
x 6(u3 + v 3 ) u3 + v 3
y

227
v 6x(xv + u2 ) x(xv + u2 )
= =
x 6(u3 + v 3 ) u3 + v 3

3.3.6. Maximos y Mnimos


Maximos y Mnimos (extremos locales o relativos) para funciones de
dos o mas variables.
Sea f : U R2 R, U conjunto abierto.
i) f tiene un maximo local en x0 U si f (x0 ) f (x) x B (x0 , y0 )
ii) f tiene un mnimo local en x0 U si f (x0 ) f (x) x B (x0 , y0 )

Ejemplo 1: La funcion f (x, y) = 4 x2 y 2 tiene un maximo local


en (0, 0), pues f = f (0,0) f (x, y) = x2 + y 2 0 ya que x2 0,
y 2 0 (x, y) B (0, 0).

Ejemplo 2: La funcion z = f (x, y) = x2 + y 2


Tiene un mnimo local en (0, 0) : f (0, 0) x2 + y 2 = f (x, y)
f (0, 0) = 0 es mnimo absoluto.

p
Ejemplo 3: La funcion z = f (x, y) = 1 3
x2 + y 2
f (0, 0) = 1 es mnimo local y absoluto.
Observacion: Si las desigualdades son validas en todo U se tendra ex-
tremo absolutos.

Punto Crtico: (Condicion necesaria)

Definicion. Un punto del dominio de f es un punto crtico si todas


las derivadas parciales de f son cero en el punto o f no es diferenciable
en el punto.

Caso particular:
Si f : R2 R, (x0 , y0 ) es punto crtico de f si y solo si :
i)
f (x0 , y0 ) f (x0 , y0 )
= 0, =0
x y

228
o
ii)
f (x0 , y0 ) f (x0 , y0 )
@ , y/o @
x y

f f
Ejemplo : Sea z = f (x, y) = x2 y 2 = = 2x = 2y
x y
= (0, 0) punto crtico.

Punto silla:

Definicion : Sea punto (x0 , y0 ) Dom(f ) . Si cualquier B (x0 , y0 ) con-


tiene puntos (x, y) B (x0 , y0 ) tales que f (x, y) f (x0 , y0 ) > 0 y
puntos (x, y) B (x0 , y0 ) tales que f (x, y) f (x0 , y0 ) < 0 se llama
punto ensilladura.
Nota. Un punto crtico en que f no es maximo ni mnimo se llama
punto silla.

Ejemplo: En la funcion f (x, y) = x2 y 2 , (0, 0) es punto ensilladura,


pues f = f (0, 0) f (x, y) = x2 + y 2 no se puede decir nada aun del
signo, pero para f = f (0,0) f (x, 0) = x2 0 y f = f (0, 0)
f (0, y) = y 2 0, permite concluir que (0, 0) es punto ensilladura.
Observacion: Las definiciones anteriores son extensibles a funciones
de mas variables.

Teorema 3.3.8. Sea f una funcion continua de dos variables definida


en una region cerrada y acotada R del plano XY. Entonces:
a.) Al menos hay un punto (x0 , y0 ) R en que f alcanza su valor
mnimo.
b.) Al menos hay un punto (x0 , y0 ) R en que f alcanza su valor
maximo.

Observacion: Este Teorema es extensible a mas variables.


Definicion: Sea f : U Rn R funcion definida en un conjunto
abierto U Rn y sea x U . Suponga que todas las derivadas parciales
de segundo orden existen en x. A la matriz cuadrada de orden n.

229
2f
A = (aij )i,j=1,...,n donde aij = (x)
xi xj
x = (x1 , x2 , ..., xn )

se llama matriz Hessiana (o simplemente Hessiano) de la funcion f en


x y se denota H(x).

Caso particular: Para f (x, y)


!
2f 2f
x2 yx
H(x, y) = 2f 2f
xy y 2

Caso particular: Para f (x, y, z)

2f 2f 2f

x2 yx zx
2f 2f 2f
H(x, y, z) =

xy y 2 zy
2f 2f 2f
xz yz z 2

Definicion. Sea la matriz


a11 a12 . . . a1n
.
a21 . .

An =

.. ..
. .
an1 ann

Consideremos las submatrices Ak de An (k = 1, 2, . . . , n) definidas de


las siguientes maneras:

  a11 a12 a13
a11 a12
A1 = (a11 ) , A2 = , A3 = a21 a22 a23 , ,
a21 a22
a31 a32 a33

a11 a12 . . . a1n
.
a21 . .

An = .

.. . ..


an1 ann

230
Teorema 3.3.9. (Consideraciones suficientes para la existencia de ex-
tremos locales)
Sea f : U Rn R una funcion definida en un conjunto abierto U de


Rn y x un punto crtico de f (es decir f (x) = 0 ) y supongamos que
las derivadas parciales de segundo orden son continuas en una vecindad
abierta de x. Entonces:
a.) Si todas las submatrices Ak de la matriz Hessiana H(x) definidas
de la forma anterior tienen determinante positivo, f tiene un mnimo
local en x.
b.) Si todas las submatrices Ak de la matriz Hessiana H(x), definidas de
la forma anterior, tienen determinantes de signo alternado comenzando
por A1 < 0, f tiene un maximo local en x.

Ejemplo 1: Sea f : R2 R definida por f (x, y) = x2 + 3y 2 2x


12y + 13. Determine los valores extremos de f .
Solucion
Aplicando la condicion necesaria de punto crtico, obtenemos

f (x, y) = (2x2, 6y 12) = (0, 0) = (1, 2) es el unico punto crtico.


Calculemos la matriz Hessiana
 
2 0
H(x, y) =
0 6
Examinemos los determinantes de las submatrices
A1 = (2) = det A1 = 2 > 0
 
2 0
A2 = = det A2 = 12 > 0
0 6
f tiene mnimo local en (1, 2).

Ejemplo 2: Sea f : R2 R definida por f (x, y) = (x2 + 3y 2 ) e1(x ).


2 +y 2

Cuales son los puntos crticos de f ?


Solucion.
De la condicion de punto crtico tenemos:
= 2xe1(x +y ) 2x (x2 + 3y 2 ) e1(x +y ) = 0 =
f 2 2 2 2

231
= e1(x ) (2x 2x3 6xy 2 ) = 0
f 2 +y 2

= 6ye1(x ) 2y (x2 + 3y 2 ) e1(x2 +y2 ) = 0 =


f 2 +y 2

= e1(x ) (6y 6y 3 2x2 y) = 0


f 2 +y 2

De donde obtenemos, el sistema de ecuaciones


 
x x3 3xy 2 = 0 x (1 x2 3y 2 ) = 0
= =
3y 3y 3 x2 y = 0 y (3 3y 2 x2 ) = 0
Puntos Crticos: (0, 0), (0, 1), (0, 1), (1, 0), (1, 0), (1, 0)

Determine en que puntos hay maximos, mnimos o puntos sillas.

y
Ejemplo 3: Sea f : R4 {0, 0, 0, 0} R y f (x, y, z, u) = x + +
x
z u 1
+ +
y z u
Determine los valores extremos de f.
Solucion
Determinemos los puntos crticos de la funcion f

f y
= 1 =0
x x2
f 1 z
= 2 =0
y x y
f 1 u
= 2 =0
z y z
f 1 1
= 2 =0
y z u

= (1, 1, 1, 1) Punto Crtico.


La matriz Hessiana esta definida como

2f 2f 2f 2f
x2 yx zx ux
2f 2f 2f 2f
y 2
xy zy uy

H(x, y, z, u) =
2f 2f 2f 2f


xz yz z 2 uz
2f 2f 2f 2f
xu yu zu u2

232
2y
x12

x3
0 0
x12 2z
y3
y12 0
H(x, y, z, u) =
0 y12 2u
z3
z12
0 0 z12 2
u3


2 1 0 0
1 2 1 0
H(1, 1, 1, 1) =
0 1 2 1
0 0 1 2

41 = 2, 42 = 3, 43 = 2, 44 = 5

Hay un mnimo local en (1, 1, 1, 1).

Teorema 3.3.10. (Condiciones suficientes para la existencia de ex-


tremos locales en dos variables.)
Sea f : U R2 R una funcion de dos variables que tiene derivadas
parciales de segundo
orden continuas en el conjunto abierto U. Sea
fxx fxy
det(H (x, y)) = (x, y) (x, y) U. Sea (x0 , y0 ) U tal
fyx fyy
que f (x0 , y0 ) = 0. Entonces
i) Si det(H (x, y)) > 0 y fxx (x0 , y0 ) < 0, f tiene una maximo local en
(x0 , y0 ) .
ii) Si det(H (x, y)) > 0 y fxx (x0 , y0 ) > 0, f tiene un punto silla en
(x0 , y0 ) .
iii) Si det(H (x, y)) = 0 no podemos concluir nada.

3.3.7. Extremos Restringidos


Pensemos, que se tiene una funcion f (x, y) sujeta a cierta condicion
g(x, y) = 0. Se quiere maximizar o minimizar f (x, y) con la condicion
que (x, y) satisfaga la ecuacion g(x, y) = 0.
En este caso podemos elaborar una respuesta aplicando la condicion
necesaria de valor extremo para f (x, y)

df f f dy
= + =0
dx x y dx

233
dy
Aplicando el teorema de la funcion implcita podemos calcular a
dx
partir de la restriccion:
g g dy
g(x, y(x)) = 0 = + =0
x y dx

g
dy
= = x
g
dx y

Sustituyendo en la primera ecuacion tenemos:


!
g
df f f g
= + x
g
=0 , 6= 0
dx x y y
y

f g f g
= + =0
x y y x

(f, g)
= =0
(x, y)
Lo que implica que los puntos crticos deben cumplir dos condiciones:

g(x, y) = 0.
(f, g)
= 0
(x, y)

d2 f
Ademas, la decision de valor extremo resulta del signo de en cada
dx2
punto crtico, ya que f (x, y) = f (x, y(x)) = f (x).

Ejemplo 1. Obtenga las dimensiones de un rectangulo de permetro


dado que determinan la mayor y la menor area de este.
Solucion.
Si x e y denotan las longitudes de los lados del rectangulo, el problema
consistira en buscar los extremos de la funcion

f (x, y) = xy, x, y > 0

234
sujeta a la restriccion g(x, y) = x+y L2 = 0 , donde L es el permetro
dado.
Entonces aplicando las condiciones necesaria para obtener los puntos
crticos tenemos:
L
g(x, y) = x + y = 0.
 2
(f, g) y x
= = y = x
(x, y) 1 1

Sustituyendo el resultado de la segunda ecuacion en la primera, se tiene:

L
x=y=
4
Luego hemos encontrado que la funcion f tiene un unico punto crtico
en
 
L L
P = , .
4 4
Ademas, determinemos la naturaleza del punto usando el criterio de la
segunda derivada

df dy
f (x, y (x)) = xy = =y+x
dx dx
L dy
g(x, y) = x + y = 0 = = 1
2 dx
df
=yx
dx
d2 f dy
2
= 1 = 2 < 0
dx dx
2
Por tanto, en el punto ( L4 , L4 , L16 ) hay un maximo relativo sobre la curva
de
L
interseccion de las dos superficies z = xy y x + y = 2

La situacion se puede generalizar para funciones de mas variables o mas


condiciones.

235
Por otra parte, para resolver este mismo tipo de problemas de maximo
y
mnimo podemos utilizar otro metodo que se fundamenta en el siguiente
teorema.
Teorema 3.3.11. (Multiplicadores de Lagrange)
Sean f : U Rn R, g : U Rn R funciones de C 1 . Sean x0 U
tal que g(x0 ) = 0 y S = {x U : g(x) = 0} supongamos ademas que
g(x0 ) 6= 0. Si f |S (f restringida a S) tiene un maximo o mnimo
local en S, en x0 ,entonces existe R tal que:
f (x0 ) + g(x0 ) = 0

Nota: Lo anterior significa que x0 es un punto crtico de f |S .


Metodo: Construir F (x, y, ) = f + g y determinar puntos crticos de
F

Ejemplo 1. Utilice el procedimiento que se origina a partir de este


teorema para obtener el maximo local del ejemplo anterior.
Solucion.
En este caso
f (x, y) = xy, x, y > 0
sujeta a la condicion:
L
g(x, y) = x + y =0
2
.
Sea F (x, y, ) = xy + (x + y L2 ), entonces
Fx = y + = 0
Fy = x + = 0
L
F = x + y = 0
2
de aqu x = , y = y sustituyendo en la tercera ecuacion:
L
2
= 0 = = L4 .
L L
El punto crtico es (x0 , y0 ) = ( , ).
4 4
2
El punto ( L4 , L4 , L16 ) es el maximo relativo sobre la curva de interseccion
L
de las dos superficies z = xy y x + y = 2

236
x2 y 2
Ejemplo 2. De todos los rectangulos inscritos en la elipse 2 + 2 =
a b
1,
a > 0 y b > 0 con lados paralelos al los ejes, determine el de mayor
area.
Solucion.
En este caso
f (x, y) = 4xy, x, y > 0

x2 y 2
y g(x, y) = + 2 1=0
a2 b
 2
y2

x
Sea F (x, y, ) = 4xy + + 2 1 = 0 , entonces
a2 b

x 2a2 y
Fx = 4y + 2 = 0 = =
a2 x
y 2b2 x
Fy = 4x + 2 2 = 0 = =
b y
2 2
x y
2
= 2
a b

Reemplazando esta ultima expresion en la ecuacion

x2 y 2
F = 2 + 2 1 = 0 =
a b
 
a b
obtenemos a un unico punto crtico: ,
2 2
Ademas, determinemos la naturaleza del punto usando el criterio de la
segunda derivada

df dy
f (x, y (x)) = 4xy = = 4y + 4x
dx dx
d2 f dy d2 y
= = 4 + 4x 2
dx2 dx dx
2 2
x y dy b2 x
g(x, y) = + 1 = 0 = =
a2 b2 dx a2 y
d2 y b2 1 b2 x 2
 
= = +
dx2 a2 y a2 y 3

237
 
a b
Evaluando segunda derivada en el punto P , , pro-
2 2
duce

d2 f b2 x b4 x 3 d2 f
 
b
2
= 8 2
4 4 3
= 2
= 12 < 0
dx ay a y dx a
 
a b
Por tanto, el punto , hay maximo relativo cuyo valor
2 2
 
a b
es f , = 2ab
2 2

Ejemplo 3: Obtenga los extremos posibles de f (x, y) = x2 + 24xy +


8y 2 con la restriccion x2 + y 2 25 = 0.
Solucion:
Sea F (x, y, ) = x2 + 24xy + 8y 2 + (x2 + y 2 25) .

Fx = 2x + 24y + 2x = 0 = (1 + )x + 12y = 0 (1)


Fy = 24x + 16y + 2y = 0 = 12x + (8 + )y = 0 (2)
F = x2 + y 2 25 = 0 (3)

Multiplicando la ecuacion (1) por 12 y (3) por(1 + ) produce.

12(1 + )x + 144y = 0
12(1 + )x + (8 + )(1 + )y = 0

Restando ambas ecuaciones

= 2 + 9 136 = 0 = 1 = 8; 2 = 17

18x + 24y = 0
Si = 8 = = 3x + 4y = 0 = y = 43 x
24x + 32y = 0
2
x2 + 34 x = 25 = x = 4
Lo que da los puntos crticos (4, 3)

238
Si = 17 en forma similar se obtiene que (3, 4) son puntos
crticos.
Localizacion del maximo y el mnimo absoluto de f sobre un
dominio compacto.
Para determinar maximos y mnimos absolutos e una region cerrada
y acotada encerrada por una curva suave, se debe:
Determinar todos los puntos crticos en el interior de la region.
Usar Lagrange para determinar puntos crticos en la frontera.
Evaluar f en los puntos, crticos.
Comparar todos estos valores y seleccionar el maximo y el mni-
mo.

Criterio de la Segunda Derivada para Extremos Restringidos.

Sea f (x, y) funcion a maximizar o minimizar y g(x, y) = 0 la condicion:


S = {(x, y) R2 | g(x, y) = 0} .Sea F (x, y, ) = f (x, y) + g(x, y)
Sea (x0 , y0 , ) punto crtico de F

f g
|
x (x0 ,y0 )
+ x |(x0 ,y0 ) = 0
f g
|
y (x0 ,y0 )
+ y |(x0 ,y0 ) = 0
g (x0 , y0 ) = 0

Se desea utilizar el criterio de la segunda derivada para identificar ex-


tremos locales de la funcion f (x, y) bajo la condicion g (x, y) = 0 lo
que conduce encontrar los maximos y mnimos de funciones de una
variable f (x, y(x)) .
De la condicion, calculemos y = y(x) y reemplacemos en f |S .
Aqu podemos considerar f como funcion de una variable, es decir
f (x, y) = f (x, y(x)) en S.
2f
Nos proponemos calcular x2

df f f dy
= +
dx x y dx

239
 2
d2 f 2f 2 f dy 2 f dy 2 f dy f d2 y
= + + + +
dx2 x2 yx dx xy dx y 2 dx y dx2
 2
2f 2 f dy 2 f dy f d2 y
= + 2 + +
x2 yx dx y 2 dx y dx2

g g dy
g(x, y(x)) = 0 = + =0
x y dx

g
dy
= = x
g
dx y

Derivando nuevamente g respecto de x

2
2g 2 g dy 2 g dy 2 g g d2 y

dy
+ + + + =0
x2 yx dx xy dx y 2 dx y dx2

dy
Sustituyendo se tiene:
dx

! !2
2 2 g 2 g
g d2 y
 
g g x g x g
2
+2 g
+ 2 g
+ =0 /
x yx y
y y
y dx2 y

2 2
2 g g 2 g g 2 g d2 y
 
g 1 g
2 +  2 + =0
x2 y yx x y 2 x g y dx2
y

"  2 #
d2 y 1 2 g g 2 g g 2 g g 1
= 2 =  2 2 +2 g
dx g x y yx x y 2 x y
y

d2 f
Reemplazando en dx2

240
! !2
g g
d2 y 2f 2f 2
f
= 2 x
g
+ 2 x g
dx2 x2 yx y
y y
"  2 #
f 1 2 g g 2 g g 2 g g 1
+  2 2 +2 g
y g x y yx x y 2 x y
y

f g f g
y0 = y(x0 ), |x0 = |x0 ; |x0 = |x
x x y y 0

Luego

f f
|
x x0 y
|x0
(x0 , y0 ) , g
= , g
=
|
x x0 y
|x0

"  2  2
d2 f 1 2f g 2 f g g 2 f g
2
=  2 2
2 + 2
dx g x y yx x y y x
y
 2  2 #
2g g 2 g g g 2g g
+ 2 2 + 2
x y yx x y y x
"
2
2f 2g 2f 2g
    
1 g
=  2 + 2 2 +
g x2 x y yx yx
y
  2 #
2f 2g

g g g
+ +
x y y 2 y 2 x

Podemos escribir entonces:

"  2  2 #
d2 f 1 2F g 2 F g g 2 F g
2
=  2 2
2 + 2

dx g x y yx x y y x
y

En el parentesis, cuadrado se tiene

241

g g g g
g x y g y x

= 2F 2F +

2F 2F
x yx y2 y yx

x2

g

g
g g

0 0
x y x y
g 2 F 2 F g 2F 2F
= x x2 yx = x

x2 yx
g 2 F 2 F g 2F 2F

y yx y2 y yx y 2

De lo anterior se plantea la siguiente definicion


Si f y g son funciones como las definidas antes y F = f +g. Llamamos
hessiano limitado de la funcion F a
g g

0
x y
g 2 F 2 F
HF = x 2 en (x0 , y0 )
g x2 F yx

2F
y yx y2

De tal modo que

d2 f HF (x0 , y0 )
2
=  2
dx g
y
(x ,
0 0y )

Teorema en extremos restringidos

Consideremos nuevamente

d2 f HF (x0 , y0 )
2
=  2
dx g
y
(x 0 , y0 )

2
De esta expresion, dado que el signo de la segunda derivada ddxf2 depende
 2
g
solo del determinante Hessiano limitado, ya que y (x0 , y0 ) > 0,
inferimos el siguiente criterio de la segunda derivada para extremos
restringidos.
d2 f
a) Si HF (x0 , y0 ) > 0, entonces dx2
< 0 y la funcion tiene un maximo
local condicionado en (x0 , y0 ).

242
d2 f
b) Si HF (x0 , y0 ) < 0, entonces dx2
> 0 y la funcion tiene un mnimo
local condicionado en (x0 , y0 ) .
c) Si HF (x0 , y0 ) = 0 no hay informacion del punto(x0 , y0 ).

Ejemplo: Hallar los extremos de f (x, y) = (x y)2 sujeta a la


restriccion x2 + y 2 1 = 0
Solucion:
Sea F (x, y, ) = (x y)2 + (x2 + y 2 1)

Fx = 2 (x y) + 2x = 0
= 2 (x + y) = 0
Fy = 2 (x y) + 2y = 0
F = x2 + y 2 = 1

a) Si escogemos = 0 , y (x + y) 6= 0 obtenemos los puntos crticos


de f libre de restriccion.

b) Si 6= 0 = y = x. Reemplazando en la tercera ecuacion , se


obtienen dos puntos crticos de la funcion f sujetos a la restriccion.
   
1 1 1 1
P1 2 , 2 y P 2 2 , 2

Determinemos el Hessiano lmitado para decidir la naturaleza de estos


puntos crticos


0 2x 2y

HF (x, y) = 2x 2 + 2 2
2y 2 2 + 2
= 8 x2 + 2xy + y 2 8 x2 + y 2
 

Sustituyendo y = x en la primera ecuacion se tiene = 2 6=


0,entonces el Hessiano se reduce a

HF (x, y) = 8 x2 + 2xy + y 2 + 16 x2 + y 2
 

= 8 x2 2xy + y 2


Evaluando en el primer punto crticos produce

243
 
1 1
HF , = 16 > 0
2 2

d2 f  1 1
 
1 1

entonces , 2 < 0, luego en el punto
, 2 la funcion

dx2 2 2
f tiene un maximo local condicionado.
Por simetra, el segundo punto tambien corresponde a un maximo local
condicionado de f.
El criterio de la segunda derivada para extremos restringidos se puede
generalizar para funciones de mas de dos variables.
En el caso de una funcion de tres variables x, y, z y sujeta a la sola
restriccion g(x, y, z) = 0, formamos el Hessiano correspondiente a

F (x, y, z, ) = f (x, y, z) + g (x, y, z)

de la siguiente forma

g g g

0
x y z
g 2F 2F 2F
x2
HF = x yx xz

g 2F 2F 2F
y 2
y yx yz

2F 2F 2F
g

z xz yz z 2

y sea
0 g g
x y
g 2F 2F
A3 = x

x2 yx
2F 2F
g
y 2
y yx

El criterio de la segunda derivada en este caso expresa:


Sea (x0 , y0 , z0 ) punto crtico.
a) Si HF < 0 y A3 > 0,entonces la funcion f tendra un maximo
condicionado en (x0 , y0 , z0 ).
b) Si HF < 0 y A3 < 0,entonces la funcion f tendra un mnimo
condicionado en (x0 , y0 , z0 ).
c) Si HF > 0,entonces la funcion f no tiene extremos condicionado
en (x0 , y0 , z0 ).
d.) Si HF = 0, no hay informacion acerca del punto (x0 , y0, z0 ).

244
Ejemplo Hallar los extremos de la funcion f (x, y, z) = x2 + y 2 + z 2
sujeta a la restricion g (x, y, z) = z 2 + 2x y 2 1 = 0.
Solucion:
Sea la funcion F = x2 + y 2 + z 2 + (z 2 + 2x y 2 1)

Fx = 2x + 2 = 0 = 2 (x + ) = 0 = x =
Fy = 2y 2y = 0 = 2y(1 ) = 0 = y = 0
Fz = 2z + 2z = 0 = 2z(1 + ) = 0 = z = 0
F = z 2 + 2x y 2 1 = 0

Reemplazando en F : 02 +2()02 1 = 0 = = 21 , y obtenemos


el punto crtico 12 , 0, 0


0 2 2y 2z

2 2 0 0
HF =
2y 0 2 2 0

2z 0 0 2 + 2
= 4 4 4x2 2y 2 2y (2 2) + 2z 2 (2 2) (2z)


 
1 1
HF , 0, 0, = 16 < 0
2 2

A3 = 8y 2 4 (2 2) = 0 12 < 0
1

Hay un mnimo condicionado de f en el punto 2
, 0, 0 .
Observese que las otras opciones no producen puntos crticos en IR3

3.4. Problemas Resueltos

3.4.1. Continuidad y diferenciabilidad


Problema

Dada la funcion f : R2 R definida como


( xy
arctg 2 , si (x, y) 6= (0, 0)
f (x, y) = x + y2 .
0 , si (x, y) = (0, 0)

245
a) Verificar si f es continua en IR2
f f
b) Calcular si existen las derivadas parciales , en IR2
x y
Solucion.
a) Tenemos que f (x, y) es continua (x, y) 6= (0, 0) puesto que es
xy
composicion de dos funciones continuas, como son arctg y 2 .
x + y2
Para estudiar la continuidad en el punto (0, 0) tenemos que calcular
lm f (x, y) lo que haremos a traves de la trayectoria y = mx,
(x,y)(0,0)

mx2 m
entonces lm f (x, mx) = lmarctg 2 2
= lmarctg
(x,y)(0,0) x0 x +y x0 2 + m2
que depende de la pendiente m, por lo que este lmite no existe.
Por lo tanto f no es continua en el punto(0, 0)

b) Para (x, y) 6= (0, 0) la funcion admite derivadas parciales, que son:

f y 2 x2
(x, y) = y 2 ,
x (x + y 2 )2 + x2 y 2
f x2 y 2
(x, y) = x 2
y (x + y 2 )2 + x2 y 2

Para (x, y) = (0, 0) ,se tiene

f f (h, 0) f (0, 0)
(0, 0) = lm
x h0 h
arctg0 0
= lm = lm0 = 0
h0 h h0

Analogamente, derivando con respecto a y

f f (0, h) f (0, 0)
(0, 0) = lm
y h0 h
arctg0 0
= lm = lm0 = 0
h0 h h0

Por lo tanto, existen las derivadas parciales en (x, y) = (0, 0) .

246
Problema

Dada la funcion f : IR2 IR definida como


2
x seny 2
, si (x, y) 6= (0, 0)
f (x, y) = x2 + y 2 , probar que es
0 , si (x, y) = (0, 0)

diferenciable en el punto P0 = (0, 0) .Es continua la funcion


en ese punto?
Solucion.
Tenemos que utilizar la definicion y ver si el siguiente lmite es cero:
|f df |
L= lm
(h,k)(0,0) h2 + k 2

h2 senk 2
donde f = f (h, k) f (0, 0) = ,y
h2 + k 2
f f
df = (0, 0) h + (0, 0) k
x y
En este caso
f f (h, 0) f (0, 0)
(0, 0) = lm
x h0 h
2
h 0
2
0
= lm h =0
h0 h
Asimismo
f f (0, k) f (0, 0)
(0, 0) = lm
y h0 k
0 senk 2
2
0
= lm k =0
h0 k
h2 senk 2
Luego, df = 0, entonces L = lm
(h,k)(0,,0) (h2 + k 2 ) h2 + k 2

h2 senk 2 h2 k 2
g (h, k) =
(h2 + k 2 )3/2 (h2 + k 2 )3/2
(h2 + k 2 )(h2 + k 2 ) p 2
3/2
= (h + k 2 ) <
2
(h + k ) 2

247
Si = . As L = 0 y f es diferenciable en P0 = (0, 0) .
De lo anterior se deduce que f es es continua en (0, 0) ya que es
diferenciable en dicho punto.

3.4.2. Regla de la cadena


Problema 1

Sea la ecuacion zxx + 2zxy + zy = 0 , donde

u+v uv u2 v 2
x(u, v) = , y(u, v) = , z(u, v) = w (u, v)
2 2 4

Muestre que al cambiar las variables independientes (x, y) por (u, v)


y la
funcion z por w la ecuacion se reduce a 2 4wuu = 0.
Solucion.
En primer lugar, calculamos la aplicacion inversa

u(x, y) = x + y, v (x, y) = x y.

Derivando parcialmente estas ultimas expresiones se tiene:


ux = 1, uy = 1; vx = 1, vy = 1
Usando estos resultados y la regla de la cadena, obtenemos

zx = zu ux + zv vx = zu + zv
zy = zu uy + zv vy = zu zv
Reiterando la derivacion parcial y usando la regla de la cadena por
segunda
vez
zxx = (zx )u ux + (zx )v vx = zuu + zvu + zuv + zvv
zxy = (zx )u uy + (zx )v vy = zuu + zvu (zuv + zvv )
zyy = (zx )u uy + (zv )v vy = zuu zvu (zuv zvv )
Suponiendo que z es una funcion continua con primeras derivadas par-
ciales

248
continuas, entonces
zxx + 2zxy + zy = 4zuu = 0
2u 1
Finalmente, zu = wu = zuu = wuu
4 2
Por tanto, se tiene
1
wuu = 0
2

Problema 2

Una funcion z = z (x, y) se dice que es armonica si tiene derivadas


parciales de segundo orden continuas y ademas zxx + zyy = 0.
x y
Sean u = 2 2
, v= 2 . Pruebe que:
x +y x + y2
i) u y v son armonicas
ii) (ux )2 = (vy )2
iii) (uy )2 = (vx )2
iv) ux vx = uy vy
b)Si f (x, y) es unafuncion armonica, entonces la funcion w (x, y) =
x y
f , 2
x + y x + y2
2 2

es tambien armonica
Solucion.
x y 2 x2 2xy
a) i) u = 2 2
= u x = 2 2 2
, uy = 2
x +y (x + y ) (x + y 2 )2
y 2xy x2 y 2
v= = vx = , vy =
x2 + y 2 (x2 + y 2 )2 (x2 + y 2 )2

Derivando parcialmente por segunda vez se tiene

2x(x2 + y 2 )2 (y 2 x2 )2(x2 + y 2 )2x


uxx =
(x2 + y 2 )4
2x3 6xy 2
=
(x2 + y 2 )3

249
2x(x2 + y 2 )2 (2xy)2(x2 + y 2 )2y
uyy =
(x2 + y 2 )4
2x3 + 6xy 2
=
(x2 + y 2 )3

Lo anterior implica que


uxx + uyy = 0

Analogamente para ,
vxx + vyy = 0

Ademas
2 2
y 2 x2 x2 y 2
 
2
ii) (ux ) = = = (vy )2
(x2 + y 2 )2 (x2 + y 2 )2
 2  2
2 2xy 2xy
iii)(uy ) = 2 2 2
= 2 2 2
= (vx )2
(x + y ) (x + y )
 2 2
2xy(y 2 x2 )
   
y x 2xy
iv) ux vx = 2 = = uy vy
(x2 + y 2 )2 (x + y 2 )2 (x2 + y 2 )2
b) Aplicando derivacion compuesta tenemos:

f f
wx = ux + vx ,
u v
f f
wy = uy + vy
u v

Derivando parcialmente por segunda vez, queda

2f 2f
 
f
wxx = 2
ux + vx ux + uxx +
u vu u
 2
2f

f f
+ ux + 2 vx vx + vxx
uv v v
 2
2f

f f
wyy = 2
uy + vy uy + uyy +
u vu u
 2
2f

f f
+ uy + 2 vy vy + vyy
uv v v

Finalmente

250
2f 2 2f f
wxx = 2
(u x ) + vx ux + uxx +
u vu u
2f 2f f
+ ux vx + 2 (vx )2 + vxx
uv v v
2f 2 2f f
wyy = 2
(u y ) + uy vy + uyy +
u vu u
2f 2f f
+ uy vy + 2 (vy )2 + vyy
uv v v

Sumando terminos y utilizando las igualdades establecidas en a) se


tiene

2f 2f
 
f
wxx + wyy = 2
+ 2 (ux )2 + (uxx + uyy )+
u v u
 2
2f

f f
+ 2
+ 2 (vx )2 + (vxx + vyy ) = 0
u v v

3.4.3. Derivacion Implcita


Problema 1

a) Sea f : R R2 una funcion tal que grad f (1, 1) = (2, 4) y g :


R3 R2 una funcion tal que sus funciones coordenadas gi : R3
R, i = 1, 2 tienen los siguientes gradientes grad g1 (1, 1, 1) = (2, 3, 1),
(f g)
grad g2 (1, 1, 1) = (5, 4, 2). Si g(1, 1, 1) = (1, 1). Obtener (1, 1, 1).
x

b) Utilizando el teorema de la funcion implcita determine si es posible


3
escribir y en terminos de x para la funcion F (x, y) = x4 exy 1 = 0
en una vecindad del punto (1, 1), y ademas encuentre su derivada.

Solucion
Sean las coordenadas cartesianas designadas en R3 por (x, y, z) y en R2
por (u, v), y tomando u = u(x, y, z), v = v(x, y, z) tenemos que:

f g f u f v
(1, 1, 1) = (g(1, 1, 1)) (1, 1, 1) + (g(1, 1, 1)) (1, 1, 1)
x u x v x
f u f v
= (1, 1) (1, 1, 1) + (1, 1) (1, 1, 1)
u x v x
251
Notemos que
 
f f
grad f (1, 1) = (1, 1), (1, 1) = (2, 4),
u v
   
g1 g1 g1 u u u
grad g1 = , , = , , = (2, 3, 1),
x y z x y z
   
g2 g2 21 v v v
grad g2 = , , = , , = (5, 4, 2),
x y z x y z

As
f g
(1, 1, 1) = 2 2 + 4 5 = 16.
x
Primeramente note que (1, 1) F 1 (0, 0) y ademas

F 3 F 3
= 4x3 y 3 exy 1 , = 3xy 2 exy 1
x y

las cuales son continuas en R2 en particular para alguna vecindad


V ((1, 1), ) de (1, 1) donde F
y
(1, 1) = 3 6= 0 por lo tanto podemos
ocupar el teorema de la funcion implcita, y definir f : V ((1, 1), ) R
con y = f (x) y 1 = f (1) cuya derivada es
3
0 4x3 y 3 exy 1
y = .
3xy 2 exy3 1

Problema 2

a) Determine las derivadas parciales u/x, u/y, v/x, v/y, donde


u, v son funciones definidas implicitamente por el sistema

F (x, y, u, v) = xeu+v + uv 1 = 0,
G(x, y, u, v) = yeuv 2uv 1 = 0.

alrededor del punto p = (1, 1, 0, 0).

2 2 2z
b) Sea la funcion z = f (u + v , u/v) obtener .
u2

252
Solucion.
a) Verificando las hipotesis del Teorema de la funcion implcita podemos
concluir que:

u (yeuv + 2u)eu+v
= u+v uv
x 2x ye + yeuv u 2vxeu+v + 2xeu+v u + vyeuv

v eu+v (yeuv 2v)


= u+v uv
x 2x ye + yeuv u 2vxeu+v + 2xeu+v u + vyeuv

u e( u v)(xeu+v + u)
= u+v uv
y 2x ye + yeuv u 2vxeu+v + 2xeu+v u + vyeuv

v (xeu+v + v)euv
= u+v uv
y 2x ye + yeuv u 2vxeu+v + 2xeu+v u + vyeuv
b) Definiendo x = x(u, v) = u2 + v 2 , y = y(u, v) = u/v tenemos que
z = f (x, y), entonces
z z x z y z z 1
= + = 2u + ,
u x u y u x y v
luego
2z
 2
2 z y 1 2 z y 2 z x
  
z z x
= 2 + 2u + + +
u2 x x2 u yx u v y 2 u xy u
 2
2z 1 1 2z 2z 1
  
z x
= 2 + 2u 2u + + 2u +
x x2 yx v v y 2 xy v
Utilizando finalmente el teorema de Schwarz tenemos que
2z z 2
2 z 1 2 z 4u 2 z
= 2 + 4u + + .
u2 x x2 v y 2 v xy

Problema 3

a) Si u = f (x, y, z) define una funcion diferenciable, y z se define im-


plicitamente como una funcion de x e y por la ecuacion g(x, y, z) =
0 con los atributos pedido en el teorema de la funcion implcita.
Pruebe que u tiene primeras derivadas parciales de x e y dadas
por:
(f,g) (f,g)
u (x,z) u (y,z)
= y ; = y
x z
y z

253
b) Si u = x2 y + z 2 , y z = g(x, y) se define implcitamente por la
ecuacion

x2 y 3z + 8yz 3 = 0

Calcule:
u u
(1, 0, 0) y (1, 0, 0)
x y
Solucion.

a) Utilizando la regla de la cadena tenemos

u f f z
= +
x x z x

por otra parte si g(x, y, z) = 0 define implcitamente a z = z(x, y)


entonces
g
z
= x
g
x z

reemplazando en la ecuacion anterior


g
u f f x
= + ( g )
x x z z
f g f g (f,g)
x z
z x (x,z)
= g
= g
z z

Similarmente
g
u f f z z y
= + y = g
y y z y y z

De lo anterior se deduce
g
u f f y
= + ( g )
y y z z
f g f g (f,g)
y z
z y (y,z)
= g
= g
z z

254
b) En este caso u = f (x, y, z) = x2 y + z 2 y z = z(x, y) se define
implcitamente

por g(x, y, z) = x2 y 3z + 8yz 3 = 0 ,luego tenemos

g g g
= 2xy , = x2 + 8z 3 , = 3 + 24yz 2
x y z

derivadas que son todas continuas por lo que se afirma que g es de C1

Ademas g(1, 0, 0) = 0 y gz
(1, 0, 0) = 3 6= 0
Entonces por el teorema de la funcion implcita se tiene que existe
V = V (1, 0) y una vecindad (a, a) de z = 0 y una funcion z(x, y)
de
C1 sobre V tal que

z(1, 0) = 0 y z(1, 0)(a, a)

Calculemos el Jacobiano

(f, g) 2xy 2z
=

2
(x, z) 2xy 3 + 24yz
= 2xy(3 + 24yz 2 ) 2xy2z
= 2xy(3 + 24yz 2 2z)

Por otra parte


g
= 3 + 24yz 2
z
Entonces

u 2xy(3 + 24yz 2 2z)


=
x 3 + 24yz 2
u 0
(1, 0, 0) = =0
x 3
Tambien:

(f, g) x2 2z
= 2 3

2
(y, z) x + 8z 3 + 24yz
= 3x2 + 24x2 yz 2 2x2 z 16z 3
= x2 (24yz 2 2z 3) 16z 3

255
Finalmente

u x2 (24yz 2 2z 3) 16z 3
=
y 3 + 24yz 2
u 3
(1, 0, 0) = =1
y 3

3.4.4. Plano Tangente a una Superficie

Problema 1

Probar que los planos tangentes a la superficie S: xyz = a, a > 0


constante, en cualquier punto de S forma con los planos coordenados
un tetraedro de volumen constante.
Solucion.
Sea la superficie S descrita por la funcion implcita F (x, y, z) = xyz
a = 0.
los vectores normales a la superficie S satisfacen F (x, y, z) = (yz, xz, xy)



Si P 0 = (x0 , y0 , z0 ) S,entonces N (P0 ) = F (x0 , y0 , z0 ) = (y0 z0 , x0 z0 , x0 y0 )


La ecuacion del plano tangente al punto P 0 S, esta definida por


(
r P ) N (P ) = 0,luego y z (x x )+x z (y y )+x y (z z ) =
0 0 0 0 0 0 0 0 0 0 0
0.
Entonces, las trazas de este plano con los ejes coordenados son
3x0 y0 z0
i) Si x = , y = 0, z = 0 = = = 3x0
y0 z0
3x0 y0 z0
ii) Si x = 0, y = , z = 0 = = = 3y0
x0 z0
3x0 y0 z0
iii) Si x = 0, y = 0, z = = = = 3z0
x0 y 0
El volumen del tetraedro es:
(3x0 ) (3y0 ) (3z0 ) 9
V = = = a, constante.
6 6 2

256
Problema 2

a) Probar que S1 dada por F (x, y, z) = 0 y S2 dada por G(x, y, z) = 0


son ortogonales en sus puntos de interseccion s y solo si
Fx Gx + Fy Gy + Fz Gz = 0.

b) Probar que las superficies S1 : x2 + y 2 + z 2 2x + 4y 8z = 0 y


S2 : 3x2 y 2 + 2z 2 6x 4y 16z + 31 = 0 son ortogonales.
Solucion.


a) La normal a S1 dada por F (x, y, z) = 0 es N 1 = F (x, y, z) =
(Fx , Fy , Fz ).
Del mismo modo, la normal a S2 dada por G(x, y, z) = 0 es



N 2 = G(x, y, z) = (Gx , Gy , Gz )

Por definicion, ambas superficies son ortogonales si F G = 0


(Fx , Fy , Fz ) (Gx , Gy , Gz ) = 0 =
Fx Gx + Fy Gy + Fz Gz = 0

b) Sea S1 : x2 + y 2 + z 2 2x + 4y 8z = 0 =

(x 1)2 + (y + 2)2 + (z 4)2 21 = 0

y S2 : 3x2 y 2 + 2z 2 6x 4y 16z + 31 = 0 =

3(x 1)2 (y + 2)2 + 2(z 4)2 = 0

Entonces S1 tiene normal




N 1 = F (x, y, z) = (2x 2, 2y + 4, 2z 8)

Asimismo , S2 tiene normal




N 2 = G(x, y, z) = (6x 6, 2y 4, 4z 16)

= F G = (2x 2, 2y + 4, 2z 8) (6x 6, 2y 4, 4z 16)


= (2x 2) (6x 6) + (2y + 4) (2y 4) + (2z 8) (4z 16)
= 4 [3(x 1)2 (y + 2)2 + 2(z 4)2 ] = 4 0 = 0
Por lo tanto, la superficie S1 es ortogonal a S2 .

257
3.4.5. Derivadas Direccionales

Problema 1

2xy 2
, si (x, y) 6= (0, 0)
Sea f (x, y) : x2 + y 4 . Determine, si existe, la
0 , si (x, y) = (0, 0)

derivada direccional de f en P0 = (0, 0).


Solucion.
Sea eb = (e1 , e2 ) un versor en una direccion cualquiera de IR2 tal que
f ((0, 0) + (e1 , e2 )) f (0, 0) f (e1 , e2 ) f (0, 0)
Dbe f (0, 0) = lm = lm
0 0
2 (e1 ) (e2 )2 23 e1 e22
2 4
(e1 ) + (e2 ) 2 (e21 + 2 e42 )
Dbe f (0, 0) = lm = lm
0 0
Por lo tanto
2e2
Dbe f (0, 0) = lm 2 ,este lmite existe si y solo si e1 6= 0
0 e1

Problema 2

x (x + y)
, si (x, y) 6= (0, 0)
Sea f (x, y) = x2 + y 2 . Determine, si existe,
0 , si (x, y) = (0, 0)
la
derivada direccional de f en P0 = (0, 0).
Solucion.
Sea eb = (e1 , e2 ) un versor en una direccion cualquiera de IR2 tal que
f ((0, 0) + (e1 , e2 )) f (0, 0) f (e1 , e2 ) f (0, 0)
Dbe f (0, 0) = lm = lm
0 0
e1 (e1 + e2 )
Dbe f (0, 0) = lm
0 (e2 2
1 + e2 )

Este lmite existe si y solo si e1 (e1 + e2 ) = 0 = e1 = 0 o (e1 + e2 ) = 0


i) Si eb = (0, e2 ) = 0, Dbe f (0, 0) = 0
ii) Si eb = (e1 , e1 ) = 0, Dbe f (0, 0) = 0.

258
Problema 3

Hallar la derivada direccional de f (x, y, z) = x2 yz 3 en el punto P0 =


(1, 1, 1) en la direccion de la tangente a la trayectoria :
r (t) =
t
(e , 1 + 2sen (t) , t cos (t)) .
Solucion.
Como f (x, y, z) = x2 yz 3 es una funcion diferenciable en R3 ,entonces
t, donde f (P ) = (2xyz 3 , x2 z 3 , 3x2 yz 2 )
Dbt f (P0 ) = f (P0 ) b
El punto P0 que corresponde a t = t0 es:

r (t0 ) = (et0 , 1 + 2sen (t0 ) , t0 cos (t0 )) = (1, 1, 1) = et0 = 1


As, t0 = ln (1) = 0
El vector tangente a la curva es:

r (t) = (et , 2 cos (t) , 1 + sen (t)) , entonces


0 0
r (0) = (1, 2, 1) y
el vector tangente unitario en esta direccion queda

0
r (0) (1,2,1)
t=
b
0 =
k r (0)k 6

Por tanto, la derivada direccional es


(1,2,1)
Dbt f (P0 ) = (2, 1, 3) = 3 >0
6 6
El valor positivo, significa que f aumenta en esta direccion.

Problema 4

Calcular
la  direccional de f (x, y, z) = xy +xz yz en el punto
derivada
P0 = 2, 2, 0 en direccion de la tangente a la curva determinada
por las superficies

x2 + y 2 + z 2 = 4, x + y + z = 0

Solucion.
Como f (x, y, z) = xy + xz yz es una funcion diferenciable en
R3 ,entonces
Dbt f (P0 ) = f (P0 ) b
t, donde f (P ) = (y + z, x z, x y) =
 
f (P0 ) = 2, 2, 2 2

259

0
r (P0 )
Aun falta calcular el vector b
t=
, que es tangente a la curva
r 0 (P )
k 0 k

determinada por las superficies dadas.


Sea C dada por

r (x) = (x, y (x) , z (x)) = 0
r 0 (x) = 1, y (x) , z 0 (x) ,

0
donde y (x) , z 0 (x) se calculan implcitamente a partir del sistema
de ecuaciones por derivacion con respecto a x, en el entendido
que y = y (x) , z = z (x) . En efecto:
2x + 2yy 0 + 2zz 0 = 0 21 yy 0 + zz 0 = x
0 0 0 0
1+y +z =0 y + z = 1
Resolviendo el ultimo sistema obtenemos

x z y x

0
1 1 x + z 0 1 1 xy
y (x) = = , z (x) = =
y z
yz y
z
yz
1 1 1 1
Evaluando

0  2 0  2 2
y 2 = =1yy 2 = = 2
2 2
Luego

0
r (P0 ) (1, 1, 2)
= r
0
2 = (1, 1, 2) = t = =
b 0
k r (P0 )k 6
Por lo tanto, se tiene que

(1, 1, 2) 4
Dbt f (P0 ) = ( 2, 2, 2 2) =
6 6

3.4.6. Valores extremos


Problema 1

Sea f : U R2 R definida en el abierto U , dada por f (x, y) =


x4 2px2 y 2 + 3 donde p es una constante entera. Calcular los valores
extremos de la funcion
Solucion
Apliquemos la condicion necesaria de punto crtico, calculando las derivadas
parciales e igualandolas a cero, luego
fx = 4x3 4px = 4x(x2 p); fy = 2y.

260
Tenemos que fy = 2y = 0 = y = 0, pero en la derivada parcial
respecto de x hay que analizar el signo de p.

i) Si p > 0, f x = 4x(x2 p) = 0 = x = 0, p y los puntos
crticos son

P1 = (0, 0), P2 = ( p, 0) y P3 = ( p, 0).
ii) Si p = 0, f x = 4x3 = 0 = x = 0, y el unico punto crtico es
P1 = (0, 0).
iii) Si p < 0, f x = 4x(x2 p) = 0 = x = 0, y el unico punto
crtico es
P1 = (0, 0)
Estudiemos el Hessiano para ver si los puntos obtenidos son maximos,
mnimos o puntos silla.
fxx = 12x2 4p, fxy = 0, fyx = 0, fyy = 2
Por tanto el determinante de la matriz Hessiana es

12x2 4p 0
H (x, y) =
0 2

Estudiemos los puntos crticos segun los valores del parametro p


1) Si p > 0: En este caso hay 3 puntos crticos.

4p 0
H (0, 0) = = 8p > 0 y fxx = 4p < 0
0 2

Entonces, el punto P1 = (0, 0) es un Maximo Relativo.



8p 0
= 16p < 0
H ( p, 0) =
0 2


Por lo tanto, los puntos P2 = ( p, 0) y P3 = ( p, 0) son Puntos Silla.
2)
Si p < 0: En este caso en P1 = (0, 0) hay un punto crtico. H (0, 0) =
4p 0
0 2 = 8p < 0, luego en P1 = (0, 0) hay un Punto Silla.

3) Si p = 0: En este caso hay un unico punto crtico en el origen, pero


al calcular el determinante del Hessiano, obtenemos que H(0, 0) = 0,
luego no se tiene informacion.

261
Sin embargo, podemos comprobar la naturaleza del punto aplicando la
definicion de extremo local en las cercanas del origen, siguiendo dos
trayectorias distintas, para la funcion: f (x, y) = x4 y 2 + 3. Entonces
f (x, 0) f (0, 0) = x4 > 0 para todo (x, 0) V (0, 0)
f (0, y) f (0, 0) = y 2 < 0 para todo (0, y) V (0, 0).
Por lo tanto, P1 = (0, 0) es un Punto de Silla

Problema 2

Encuentre los valores extremos de la f (x, y) = x2 + y 2 21 x4 .


Solucion.
Derivando parcialmente con respecto a x e y tenemos:
fx (x, y) = 2x 2x3
fy (x, y) = 2y
Esta claro que fx y fy son funciones continuas en R2
Aplicando la condicion necesaria para los puntos crticos de f
tenemos: 2x 2x3 = 0; 2y = 0.
Al resolver el sistema obtenemos tres puntos crticos.
P0 = (0, 0) , P1 = (1, 0) , P2 = (1, 0)
Determinemos el Hessiano H (x, y) :

fxx (x, y) fxy (x, y) 2 6x2 0
H (x, y) =
= = 4 12x2
fyx (x, y) fyy (x, y) 0 2
Evaluemos el Hessiano H (x, y) en cada uno de los puntos:
i) Para P0 = (0, 0) = H (0, 0) = 4 > 0 y fxx (0, 0) = 2 > 0
Entonces en P0 hay un mnimo relativo f (0, 0) = 0.
ii) Para P1 = (1, 0) = H (1, 0) = 8 < 0 .
Entonces, en P1 hay punto silla de f.
iii) Para P2 = (1, 0) = H (1, 0) = 8 < 0 .
As, en P2 tambien hay un punto silla de f.

262
Problema 3

Encuentre los valores extremos de la f (x, y) = x2 + y 2 xy + x + y


en el dominio D = {(x, y) R2 / x 0, y 0, x + y 3} .
Solucion.
En primer lugar, determinemos los valores extremos en el conjunto
abierto: D = {(x, y) R2 / x < 0, y < 0, x + y > 3} .
Derivando parcialmente con respecto a x e y tenemos:
fx (x, y) = 2x y + 1
fy (x, y) = 2y x + 1
Observe que fx y fy son continuas en R2
Aplicando la condicion necesaria para los puntos crticos de f
tenemos el sistema:
2x y = 1; x + 2y = 1.
Al resolver este sistema obtenemos un unico punto crtico
P0 = (1, 1) D
Determinemos el Hessiano H (x, y) :

fxx (x, y) fxy (x, y) 2 1
H (x, y) =
= = 3 (x, y) D :
fyx (x, y) fyy (x, y) 1 2
As, P0 = (1, 1) = H (1, 1) = 3 > 0 y fxx (1, 1) = 3 > 0
Se concluye, que en P0 hay un mnimo relativo f (1, 1) = 1.
En segundo lugar, estudiemos la condicion que se presenta en la
frontera de D.
a) Si y = 0, f (x, 0) = x2 + x con x [3, 0]
Determinemos los puntos crticos en este borde
0
f (x) = 2x + 1 = 0 = x = 21
Luego, hay un punto critco en P1 = 21 , 0 D.

00
Como f (x) = 2 > 0, x [3, 0] , entonces en P1 = 12 , 0 hay


un mnimo f 21 , 0 = 41 .


b) Si x = 0, f (0, y) = y 2 + y con y [3, 0]


Determinemos los puntos crticos en este borde

263
0
f (y) = 2y + 1 = 0 = y = 12
Luego, se tiene un punto critco en P2 = 0, 21 D.

00
Como f (y) = 2 > 0, y [3, 0] ,entonces en P2 = 0, 12 hay


un mnimo f 0, 21 = 41 .


c) Si x+ y = 3, f (x, x 3) = 3x2 + 9x + 6 con x [3, 0]


Determinemos los puntos crticos en este borde
0
f (x) = 6x + 9 = 0 = x = 32 = y = 23
Luego, hay un punto critco en P2 = 32 , 32 D.

00
Como f (x, x 3) = 6 > 0, x [3, 0] ,entonces P2 = 32 , 23


en hay un mnimo f 23 , 23 = 43 .


3.4.7. Multimplicadores de Lagrange para extremos


restringidos
Problema 1

Obtener los extremos de la funcion f (x, y) = x3 + 2xy + y 2 que se


encuentren en la recta x + y = 0.
Solucion
En primer lugar consideremos la funcion de Lagrange y apliquemos la
condicion necesaria de punto crtico:
F (x, y, ) = x3 + 2xy + y 2 + (x + y)
Fx = 3x2 + 2y + = 0 (1)
F y = 2x + 2y + = 0 (2)
Fz = x + y = 0 (3)
De la (3) ecuacion obtenemos: y = x que sustituida en la segunda:
2x 2x + = 0 = = 0
Sustituido este valor en la ecuacion (1), queda: 3x2 2x = 0, x = 0,
x = 2/3
 
2 2
Entonces los puntos crticos del lagrangiano son. P1 (0, 0), P2 ,
3 3
.
Clasificaremos los puntos con el determinante del Hessiano Limitado:

264

0 1 1

H (x, y) = 1 6x 2

1 2 2

evaluando en los puntos crticos obtenidos anteriormente.



0 1 1
d2 f
Consideremos H (0, 0) = 1 0 2 = 2 > 0, entonces < 0 y en
1 2 2 dx2
P1 (0, 0) hay un maximo local condicionado de f, cuyo valor es f (0, 0) =
0.

 0 1 1
d2 f

2 2
Por otra parte H , = 1 4 2 = 2 > 0, entonces <0
3 3 1 2 2 dx2
 
2 2
y en P2 , hay un mnimo local condicionado de f, cuyo valor
 3 3
2 2 4
es f , = .
3 3 27

Problema 2
x2 y2
En que puntos de la elipse 2 + 2 = 1, la tangente a este lugar
a b
geometrico forma con los ejes coordenados un triangulo de area mnima.
Solucion.
Sea la ecuacion de la tangente a la elipse en el punto (x0 , y0 ) .
x0 x y 0 y
+ 2 =1
a2 b
1
Sea f (x, y) = xT yT , el area que forma la recta tangente con los ejes
2
coordenados, donde xT yT se determinan a partir de la ecuacion de
la tangente.
a2 b2
Si yT = 0 = xT = xT = 0 = yT =
x0 y0
1 a2 b 2
As f (x, y) = es la funcion a estudiar, que verifica la condicion:
2 x0 y 0
x20 y02
+ 2 1 = 0.
a2 b
Consideremos la funcion :

265
1 a2 b 2 x20 y02
 
L (x, y, ) = + + 2 1
2 x0 y 0 a2 b

1 a2 b 2 x0
Lx (x, y, ) = 2
+ 2 2 = 0 1/y0
2 x0 y 0 a
1 a2 b 2 y0
Ly (x, y, ) = 2
+ 2 2 = 0 /x0
2 x0 y 0 b
x20 y02
L (x, y, ) = 2 + 2 1 =0
a b
Multiplicando las ecuaciones anteriores por los coeficientes que se
indican, tenemos
1 a2 b 2 2 x0
1,0) 2 2 + =0 1/y0
2 x0 y0 a2 y0
1 a2 b 2 2 y0
2,0) 2 2 + =0 1/x0
2 x0 y0 b2 x 0
x2 y 2
3,0) 20 + 20 1 = 0
a b

Restando 2,0 1,0 se tiene


2 x0 2 y0 a2 2
= 2 = x20 = y
2
a y0 b x0 b2 0
Sustituyendo este resultado en 3.0) se tiene un unico punto crtico de
f en
 
a b
P0 = ,
2 2
Mediante el criterio de la segunda derivada se puede determinar la
naturaleza del punto crtico
0
1 a2 b 2 a2 b 2
 
0 y0 x0 y0 (x)
f (x, y(x)) = = f (x) =
2 x0 y0 2 (x0 y0 )2
donde a partir de la condicion obtenemos
00 
b2 x 0 b 2 y 0 x0 y 0

2x0 2y0 0 0 0
+ 2 y0 (x) = 0 = y0 (x) = 2 = y0 (x) = 2
a2 b a y0 a y02
As
0 00 0
a2 b 2 (x0 y0 )2 (2y0 + x0 y0 (x)) 2(x0 y0 )(y0 + x0 y0 (x))2
 
00
f (x) =
2 (x0 y0 )4
Produce

266
 
00 a b
f , >0
2 2
 
a b
Por lo tanto, en el punto P0 = , un mnimo de f
2 2

Problema 3

Se desea construir una tolva para un silo, que tenga una capacidad
de 100 m3 y forma de cono circular recto de 2m de radio, coronado
por un cilindro circular recto, empleando un mnimo de material para
la superficie. Calcular las alturas x del cilindro e y del cono para tal
objeto.
Solucion:
p
Sea la funcion superficie definida por f (x, y) = 2 4 + y 2 +4x
Con la condicion que el volumen sea g (x, y) = 43 y + 4x 100 = 0
Entonces formemos la funcion:
p 4

L (x, y, ) = f (x, y) = 2 4 + y 2 +4x + 3
y + 4x 100
Lx (x, y, ) = 4 + 4 = 0 = = 1
4y 4 y 1
Ly (x, y, ) = p + = 0 = p =
2 4+y 2 3 2 4+y 2 3
L (x, y, ) = 43 y + 4x 100 = 0

4
9y 2 = 4 (4 + y 2 ) = 5y 2 = 16
= y =
5
Sustituyendo en la restriccion se tiene

16 100
4x = 100
3 5
34 5
= x =
4
En consecuencia, se tiene un unico punto crtico en
 
100 4 4
P0 = ,
4 5 5
La condicion de mnimo de f se estable mediante la segunda derivada
p
2 0 4yy 0
f (x, y(x)) = 2 4 + y + 4x = f (x) = p + 4
2 4 + y2
4
3
y + 4x 100 = 0 = y 0 (x) = 3

Por lo tanto, sustituyendo y 0 (x) , y derivando por segunda vez

267
!
0 6y 00 (3(4 + y 2 ) y 2
f (x) = p + 4 = f (x) = 6
4 + y2 (4 + y 2 )3/2
00
f (P0 ) > 0 = Valor mnimo
As el valor mnimo de la funcion es:
   
100 4 4 q
16 100 4 20
f , = 2 4 + 5 +4 = 100
4 5 5 4 5 3 5

Problema 4

Determine la distancia mnima y maxima del origen a la curva de in-


7
terseccion del paraboloide z = x2 y 2 y el plano x + y + z = 2.
4
Solucion:
En este caso es conveniente los valores extremos del cuadrado de la
distancia con respecto al origen en vez de la distancia misma. Por lo
tanto, se deben hallar los valores extremos de la funcion:

f (x, y, z) = x2 + y 2 + z 2

sujeta a las restriciones

7
g (x, y, z) = z + x2 + y 2 = 0
4
h (x, y, z) = x + y + z 2 = 0

Para aplicar el metodo de los multiplicadores de Lagrange se define


 
2 2 2 7 2 2
F (x, y, z, 1 , 2 ) = x +y +z +1 z + x + y +2 (x + y + z 2)
4
Fx = 2 (1 + 1 ) x + 2 = 0 (1,0)
Fy = 2 (1 + 1 ) y + 22 = 0 (2,0)
Fz = 2z + 1 + 2 = 0 (3,0)
7
F 1 = z + x2 + y 2 = 0 (4,0)
4
F 2 = x + y + z 2 = 0 (5,0)
1,0) 2,0) : 2 (1 + 1 ) (x y) = 0 = 1 = 1 o y = x
1
Si 1 = 1 ,entonces de 1) 2 = 0 y de 3) z =
2
268
1
Si z = , entonces de 4) y 5) se obtiene: 2x2 3x + 1 = 0
2
1
Resolviendo la ecuacion anterior, sus soluciones son: x1 = 1; x2 =
2
 
1 1 1
x1 = 1 = y1 = = 1, , es punto crtico.
2 2 2
 
1 1 1
x2 = = y2 = 1 = , 1, es punto crtico.
2 2 2
1
Por otra parte:4) 5) = x2 + y 2 x y + =0
4

1 2 2
Si y = x = 2x2 2x + = 0, resolviendo la ecuacion x =
4 4
!
2+ 2 42 2 2+ 2 2+ 2 42 2
y=x= = z = = , ,
4 4 4 4 4
es punto crtico de f.
!
2 2 4+2 2 2 2 2 2 4+2 2
y=x= = z = = , ,
4 4 4 4 4
es punto crtico de f.
As
!
2 2 2 2 42 2 1 
fmax , , = 9+2 2
4 4 4 4
 
1 1 3
fmn , 1, =
2 2 2

Como la curva interseccion del paraboloide y el plano es una curva


cerrada, la distancia mnima y la distancia maxima al origen son re-

r
3 1
q
spectivamente y 9 + 2 2 .No necesitamos mas pruebas por
2 2
las caracteristicas geometricas del problema.

Problema 5

Demuestre que las distancias maxima y mnima desde el origen a la


x2 y 2 z 2
curva de interseccion definida por + + = 1, z = x + y.
4 5 25
Solucion:

269
Debenos encontrar los valores extremos de la funcion

f (x, y, z) = x2 + y 2 + z 2

sujeta a las restriciones

x2 y 2 z 2
g (x, y, z) = + + 1=0
4 5 25
h (x, y, z) = x + y z = 0

Para aplicar el metodo de los multiplicadores de Lagrange se define


 2
y2 z2

2 2 2 x
F (x, y, z, 1 , 2 ) = x +y +z +1 + + 1 +2 (x + y z)
4 5 25
Aplicando la condicion necesaria de punto crtico
 
1
Fx = 2 1 + x + 2 = 0 (1,0)
 4
1
Fy = 2 1 + y + 2 = 0 (2,0)
5
1
Fz = 2(1 + + 2 = 0 (3,0)
5
x2 y 2 z 2
F 1 = + + 1=0 (4,0)
4 5 25
F 2 = x + y z = 0 (5,0)
Despejando de estas ecuaciones x, y, z se tiene
22 52 252
x= ; y= ; z= ; 6,0)
1 + 4 21 + 10 21 + 50
Al dividir 5,0 por 2 6= 0 (lo cual esta justificado porque de otro
modo de 1,0, 2,0 y 3,0, se tendra x = y = z = 0).
2 5 25
+ + = 0. Multiplicando por 2 (1 + 4) (21 + 10) (21 + 50)
1 + 4 21 + 10 21 + 50
y simplificando da
1721 + 2451 + 750 = 0 = (1 + 10)(171 + 75) = 0
75
de donde: 1 = 10, 1 =
17
2 2 52
Caso i) Si 1 = 10, entonces de 6,0 : x = ; y = ; z = .
3 2 6
22 22 522 180
Sutituyendo en 4.0 da: + + 1 = 0 = 22 = =
36 20 66 19
270
r
5
2 = 6
19
Por lo tanto, se tienen dos puntos crticos.
r r r ! r r r !
5 5 5 5 5 5
P1 = 2 ,3 ,5 y P2 = 2 .3 , 5
19 19 19 19 19 19
r r r !
5 5 5
Evaluando en la funcion se tiene f 2 , 3 , 5 = 10
19 19 19
75 342 172
Caso ii) Si 1 = , entonces de 6,0 : x = ; y = ;z =
17 7 4
172
.
28
2 2 52 (140)2
Sutituyendo en 4.0 da: 2 + 2 + 2 1 = 0 = 22 = =
36 20 66 (17)2 (646)

2 = 14017 646
Por lo tanto, se tienen otros dos puntos crticos mas.
   
40 35 5 40 35 5
P1 = , y P2 = , ,
646 646 646 646 646 646
 
40 35 5 75
Evaluando en la funcion se tiene f , , =
646 646 646 17
75
Asi el valor maximo buscado es 10 y el valor mnimo es
17

Problema 6

Se desea construir un silo, que tenga una capacidad de V0 con forma de


cilindro circular recto de altura h y radio basal r . Calcular la altura
h del cilindro y radio basal r de manera que la superficie total sea
mnima.
Solucion:
Sea la funcion superficie definida por f (r, h) = 2r2 +2rh
Con la condicion que el volumen sea g (x, y) = r2 h V0 = 0
Entonces formemos la funcion:
L (r, h, ) = 2r2 + 2rh + (r2 h V0 )

271
Lr (r, h, ) = 4r + 2h + 2rh = 0 1,0)
Lh (r, h, ) = 2h + r2 = 0 2,0)
L (r, h, ) = r2 h V0 = 0 3,0)

2
De 2,0) se tiene: = y sustituyendo este valor en 1,0) obtenemos
r
h = 2r
r r
3 V0 V0
Si h = 2r, entonces de 3.0) r = ;h = 2 3
2 2
En consecuencia, se tiene un unico punto crtico en
r r !
3 V0 V0
P0 = ,2 3
2 2
La condicion de mnimo de f se establece mediante la segunda derivada
0
f (r, h(r)) = 6r2 = f (r) = 12r
00
= f (r) = 12 > 0

Por lo tanto, se tiene un valor mnimo de f si h = 2r


As el valor mnimo de la superficie es:
r r !  2/3
3 V 0 3 V0 V0
f ,2 = 6
2 2 2

Problema 7:

Determinar los extremos absolutos de la funcion f (x, y) = y 3 + x2 y +


2x2 + 2y 2 4y 8 en el conjunto D = {(x, y) IR2 /x2 + y 2 1} .
Solucion:
En primer lugar estudiemos los puntos del interior de D, para ver si
existen maximos o mnimos locales.
La condicion necesaria, de los puntos interiores candidatos a extremos,
es
f
= 2x(y + 2) = 0
f (x, y) = (0, 0) = f x
= 3y 2 + x2 + 4y 4 = 0
y
i) La primera ecuacion implica que x = 0 o y = 2. Si y = 2, la
segunda ecuacion implica que x = 0, luego se tiene un punto crtico en
P0 = (0, 2) , sin embargo , P0
/ D.

272
ii) Si x = 0, la segunda ecuacion es 3y 2 + 4y 4 = 0 = y = 2,
2
y= .
3
 
2 4
Las coordenadas del punto P1 = 0, verifican < 1,entonces P1
3 9
8 8 8 256
D. Ademas f (P1 ) = + 8= = 9, 48.
27 9 3 27
En segundo lugar, estudiemos los puntos de la frontera de D usando
la funcion f (x, y) = y 3 + x2 y + 2x2 + 2y 2 4y 8 bajo la restricion
g (x, y) = x2 + y 2 1 = 0.
Usemos el metodo de los multiplicadores de Lagrange.
Sea L (x, y, ) = y 3 + x2 y + 2x2 + 2y 2 4y 8 + (x2 + y 2 1),
y obtenemos:
L
= 2x(y + 2) + 2x = 0 (1,0)
x
L
= 3y 2 + x2 + 4y 4 + 2y = 0 (2,0)
y
L
= x2 + y 2 1 = 0 (3,0)

De la ecuacion 1,0 se tiene que x = 0 o (y + 2) + = 0
a) Si x = 0,en 3.0 se tiene y 2 1 = 0 = y = 1.Luego se tienen
otros dos puntos crticos
P2 = (0, 1) y P2 = (0, 1) que satisfacen las ecuaciones 1,0 y 3,0.
Para comprobar que tambien satisfacen la ecuacion 2,0 , sustituyamos
el ella
3
P1 = (0, 1) = = IR
2
5
P2 = (0, 1) = = IR.
2
Si evaluamos las funcion en los puntos encontrados obtenemos:
f (P2 ) = 1 + 2 4 8 = 9
f (P3 ) = 1 + 2 + 4 8 = 3
b) Si (y + 2) + = 0 = (y + 2), en 2,0 se tiene 3y 2 + x2 +
4y 4 + 2y(y + 2) = 0
= x2 + y 2 = 4, resultado que contradice la ecuacion 3,0, x2 + y 2 = 1.
Luego, esta condicion no produce un punto crtico.

273
Por lo tanto, comparando los valores de la funcion en los tres puntos
encontrados, podemos inferir que el maximo absoluto se alcanza en P3
y que el mnimo absoluto se alcanza en P1

Problema 8:

Determine las dimensiones de una caja rectangular, sin tapa


superior,que ha de tener un volumen dado V0 ,de manera que
su superficie sea mnima.
Solucion:

Sea la funcion superficie definida por f (x, y, z) = xy + 2xz + 2yz


Con la condicion que el volumen sea g (x, y, z) = xyz V0 = 0
Entonces formemos la funcion:
L (x, y, z, ) = f (x, y) = xy + 2xz + 2yz + (xyz V0 )

Lx (x, y, z, ) = y + 2z + yz = 0 (1,0)
Ly (x, y, z, ) = x + 2z + xz = 0 (2,0)
Lz (x, y, z, ) = 2x + 2y + xy = 0 (3,0)
L (x, y, z, ) = xyz V0 = 0 (4,0)

2z
Despejando y y x de 1,0 y 2,0 se tiene 5.0) y = x = 1+z , sutituyendo
en 3,0) produce:
8z 4z 2 1
1+z + (1+z)2
= 0 = z =
6,0)
Reemplazando en 6,0) en 5,0) se tiene
y = x = 4 . Sustituyendo en 4.0
 1/3
= V0 = = V160
4 4
  1

Por lo tanto, se tiene un unico punto crtico de f en


 1/3 1/3 V 1/3 
P0 = 4 V160 , 4 V160 , 160 .
Examinemos la naturaleza del punto crtico usando el Hessiano limita-
do:
g g g

0
x y z
g 2 L 2 L 2 L
x x2 xy xz
H (x, y, z) = g 2 L 2 L 2 L =
y xy y2 yz
g 2 L 2 L 2 L
2

z xz yz z

274


0 yz xz xy
yz 0 1 + z 2 + y
H (x, y, z) =
xz 1 + z 0 2 + x
xy 2 + y 2 + x 0
 1/3 1/3 V0 1/3 
Evaluemos el Hessiano H 4 V160 , 4 V160 , 16
2/3 2/3 2/3
0 4 V160 4 V160 16 V160


V
2/3
4 0
0 2 6
H= 16
V0
2/3
4 16 2 0 6

V
2/3
16 16 0
6 6 0
Usando propiedades de determinantes, obtenemos que el valor de Hes-
siano es:


0 1 1 4

V0 4/3
 1 0 2 6
H = 16 16

1 2 0 6

4 6 6 0
4
H = 2048,0 (0,062 5V0 ) 3 < 0

0 1 1

Ademas: 3 = 1 0 2 = 4 > 0
1 2 0
Entonces
 lafuncion f tendra un maximo condicionado en el punto
1/3 1/3 1/3
P0 = 4 V160 , 4 V160 , V160

3.4.8. Aplicacion al calculo de errores


Problema 1:

El periodo T de un pendulo simple depende de la longitud l y de la


aceleracion de gravedad g del lugar y esta dado por:
r
l
T = 2 . Hallar a a) el error absoluto y b) el error relativo , al
g
calcular T con l = 0, 6 m y g = 10m/s2 si los valores verdaderos eran
l = 58, 5cm y g = 9, 8m/s2 .
Solucion:
r
l
a) Sea T = 2 . el perodo de un pendulo simple.
g

275
El error absoluto de T es T, que en este caso es aproximadamente
dT. as se tiene:
r
T T l
El error absoluto de T = dT = dl + dg = dl dg
l g lg g3
Error de l = l = dl = (0, 6 0, 585) m = 0, 015m

Error de g = g = dg = (10 9, 8) m/s2 = 0, 2m/s2


r
0, 6
El error absoluto de T = dT = (0, 015) (0, 2)
0, 6x10 1000
 r 
dT 1 l
b) El error relativo de T = = r dl dg
T l lg g3
2
g

 
1 1
El error relativo de T = dl dg .
2l 2g
 r 
dT 1 l
El error relativo de T = = r dl dg
T l lg g3
2
g

3.5. Ejercicios Propuestos

3.5.1. Lmites
Problema 1

Determinar si existen los siguientes limites, y si existen, calcular su


valor

xseny
a) lm ln ysenx
(x,y)(0,0)

x2 y 2
b) lm
(x,y)(0,0) x2 + y 2

Solucion:
xseny
a) lm ln ysenx = 0, b) El lmite de la funcion f no existe
(x,y)0

276
Problema 2

En los siguientes casos, usar coordenadas polares para concluir que el


lmite de f (x, y) cuando (x, y) tiende a (0, 0) existe y vale cero.
y3
a) lm
(x,y)(0,0) x2 + y 2

x3 y 4
b) lm
(x,y)(0,0) x4 + y 4

3.5.2. Diferenciabilidad, continuidad


Problema 1

1
xysen si y 6= 0

Dada la funcion f (x, y) = y , probar que es
0 (x, 0)
diferenciable en el punto P0 = (0, 0) ,es continua en este punto?
Solucion:
La funcion f es diferenciable en P0 = (0, 0) , implica que es continua
en (0, 0)

Problema 2

p |x| y

si (x, y) 6= (0, 0)
Dada la funcion f (x, y) = x2 + y 2 .
0 si (x, y) = (0, 0)

a) Estudiar su continuidad en IR2


b) Estudiar la existencia de derivadas parciales en IR2 y determinarlas,
si es que existen.
c) Estudiar su diferenciabilidad en IR2

Solucion:
a) La funcion f es en continua en IR2
b) Existen derivadas parciales en
f f
IR2 {(o, a) /a 6= 0} ,No existe (0, a) , (0, a) = 0,
x y

277
f f
(0, 0) = 0, (0, 0) = 0
x y
c) La funcion f no es diferenciable en P0 = (0, 0) ,puesto que
|f df |
lm 6= 0
(h,k)(0,0) h2 + k 2

Problema 3
p
Verificar que f (x, y) = |xy| es continua y no diferenciable en (0, 0) .

Problema 4
( 4 4
arctg xx2 +y
+y
2 si (x, y) 6= (0, 0)
Dada la funcion f (x, y) =
a si (x, y) = (0, 0)
a) Determinar el valor de a para que la funcion sea continua en el
origen.
b) Para este valor de a calcular fx (0, 0), fy (0, 0)
c) Hallar la derivada direccional Dub f (1, 0), siendo u
b un vector unitario
o
que la forma un angulo de 60 con la parte positiva del eje OX.
Solucion:
a) f(x,y) es continua en (0,0) si a= f(0,0)
b) fx (0,0)=0 , fy (0,0)=0
1
c) Dub f (1, 0) = 2

3.5.3. Derivadas parciales


Problema 1

x2 y 2
xy si (x, y) 6= (0, 0)

Sea f (x, y) = x2 + y 2
0 si (x, y) = (0, 0)

f f
a) Deducir que (0, y) = y y; (x, 0) = x x
x y
2f 2f
b) Verificar que (0, 0) 6= (0, 0)
yx xy

278
Problema 2

x + y si x = 0 o y = 0
Dada la funcion f (x, y) =
1 si x =6 0, y 6= 0
f f
a) Verifique que (0, 0) = 1 ; (0, 0) = 1
x y
b) Muestre que la funcion f no es continua en (0, 0)

Problema 3
xy
Sea u = , muestre que u satisface la ecuacion:
x+y
2u 2u 2
2 u
x2 + 2xy + y =0
x2 yx y 2

Problema 4
1
Sea u = p , probar que u satisface la ecuacion:
x2 + y2 + z2
2u 2u 2u
+ + =0
x2 y 2 z 2

Problema 5

Enuncie las condiciones del teorema de Schwarz de igualdad de las


derivadas cruzadas.

Problema 6

Justifique si es cierto que una funcion f : IR3 IR diferenciable en


IR3 tiene derivadas parciales.
Solucion
Si una funcion f es diferenciable en

x o y la diferencial es Df (

x o) ,


entonces para todo vector unitario vb 6= 0 existe D f ( x ) que verifica
v
b o

Dvbf (

x o ) = Df (

x o ) vb. En particular, esto sucede para los vectores
(1, 0, 0) , (0, 1, 0) , (0, 0, 1) , por lo que existen las derivadas parciales

279
Problema 7

Suponga que z es funcion de las variables x e y , que satisface la ecuacion


que se da en cada caso . Encontrar las derivadas parciales de primer
z z
orden y
x y
a) x3 + y 3 + z 3 + senxz + cos yz = 15
b) ez + x2 ln z + y = 0.
Solucion:
z (3x2 + z cos xz) z (3y 2 zsenxz)
a) = 2 , = 2
x 3z + x cos xz ysenyz y 3z + x cos xz ysenyz
z 2xz ln z z z
b) = z 2
, = z
x ze + x y ze + x2

Problema 8

Suponga que existen funciones u y v , que satisfacen las siguientes


ecuaciones.

u cos v = x + 1
u sin y = x + y

u u v v
Calcular , , ,
x y x y
Solucion:
u u v cos v senv v cos v
= cos v + senv , = senv, = , =
x y x u y u

3.5.4. Derivadas Direccionales


Problema 1

Calcule la derivada direccional de la funcion dada en la direccion del


vector indicado.
a) f (x, y, z) = 2x2 y 2 z 2 , en (1, 2, 2) hacia (4, 5, 0) .
b) f (x, y, z) = 2x2 8xy + z 2 , en el punto (4, 4, 1) en la direccion de
la

280
normal exterior a la superficie x2 + y 2 + z = 17
Solucion:
8 446
a) Dvbf (1, 2, 2) = , b) Dvbf (4, 4, 1) =
22 129

Problema 2

Determine la ecuacion del plano tangente al paraboloide x2 +y 2 +z1 =


1 1
 
0 y que pasa por los puntos 1, 0, 2 y 0, 1, 2 .
Solucion:
3
La ecuacion de plano tangente es: x + y + z =
2

Problema 3

Sea S una superficie dada por la ecuacion x3 + y 3 + z 3 a3 = 0, con a


constante, y un plano tangente en P0 = (xo , yo, zo ) S que intersec-
ta los ejes coordenados X,Y,Z en , , respectivamente. Probar que
3/2 + 3/2 + 3/2 = a3/2 es constante.

Problema 4

Determinar el plano tangente a la superficie S


x + y + z = a, (a > 0)

en el punto P0 = (xo , yo, zo ) S y demostrar que este plano corta los


ejes
coordenados en segmentos cuya suma de longitudes es constante.
Solucion:
x xo y yo z zo
+ + =0
xo yo zo

l = a( xo + yo + zo ) = a, (a > 0)

281
Problema 5

Muestre que si el punto P0 = (xo , yo, zo ) pertenece al elipsoide de


ecuacion:
x2 y2 z2
: 2 + 2 + 2 = 1 con ( < c2 < b2 < a2 )
a b c
y al manto del hiperboloide de ecuacion:
x2 y2 z2
: 2 + 2 + 2 = 1 con (c2 < < b2 < a2 )
a b c
Entonces las superficies y se cortan ortogonalmente en P0 = (xo , yo, zo ) .

Regla de la cadena

Problema 1

Sean f : R2 R y g : R R, con f (x, g (x)) = x sin (g (x)) .


0
Sabiendo que g (1) = 0 y que f (1, 0) = 1, calcular g 0 (1) .
Solucion:
El valor de la derivada es g 0 (1) = 1

Problema 2

Sea f : D IR2 IR una funcion diferenciable homogenea de


grado p, es
decir que verifica la condicion f (tx, ty) = tp f (x, y) t > 0, (x, y)
IR2 .
f f
a) Demuestre la ecuacion x +y = pf.
x y
b) Pruebe que la siguientes funciones son homogeneas f (x, y) = xy,
f (x, y) = x2 + 3xy + y 2 y luego verifique la ecuacion anterior.

Problema 3

Sea g(t) = f (x (t) , y (t)) de clase C2 , deduzca que:

2 2
g 00 (t) = fxx (x, y) (x0 ) +2fxy (x, y) x0 y 0 +fyy (x, y) (y 0 ) +fx (x, y) x00 +fy (x, y) y 00

282
Problema 4

Sea f = f (x, y) de clase C2 con fx (0, 1) = 2, fy (0, 1) = 1; fxx (0, 1) = 0,


fxy (0, 1) = 1, fyy (0, 1) = 1.Si h(t) = f (t2 , 1 + t3 ) .
Calcule la derivada compuesta (f 0
r ) (t) y evalue h(0) = 4
Solucion:
00
h (t) = fxx (x, y) (x0 )2 + 2fxy (x, y) x0 y 0 + fyy (x, y) (y 0 )2 + fx (x, y) x00 +
fy (x, y) y 00
h00 (0) = 4

Derivacion implcita
Sea z = f (x, y) definida por z = u + vdonde u = u(x, y) y v = v(x, y)
son funciones
definidas de manera implcita por las ecuaciones
F = u + eu+v x = 0
G = v + euv y = 0
Si u = v = 0 entonces x = y = 1: Calcular zx (1, 1).
Solucion:
zx (1, 1) = 1

Problema 5
 
x
Verifique que la funcion f (x, y) = g satisface la ecuacion:
y2
f f
2x +y =0
x y

Problema 6
 xy 
Sea g , x2 + y 2 = 0 una ecuacion que define a z como una funcion
z
de x e
y. Verifique que si gx ; gy y gz existen y son continuas en toda la region
en la que
gz 6= 0; entonces

283
z(x2 y 2 )
yzx xzy =
xy

Problema 7

Si F (xz, yz) = 0 define a z como funcion implcita de x e y y ademas


cumple con las
condiciones del teorema de la funcion implcita en cada punto de una
region R; entoncesverifique que, en R; se satisface la ecuacion

xzx + yzy = z

3.5.5. Puntos crticos maximos y mnimos


Problema 1

Sea f : U R2 R definida en el abierto U , dada por:


f (x, y) = x4 + y 4 + 4axy + 8a4 , (a R). Calcular los valores extremos
de la funcion.
Solucion:
Si a = 0, entonces, el unico punto crtico es P0 = (0, 0) (mnimo) .
Si a > 0, tenemos
tres puntos
crtico es P0 = (0, 0) (punto silla),
P1 = ( a, a), P2 = ( a, a) (mnimos).
Si a < 0, tenemos
tres puntos
crtico
es P0 = (0, 0) (punto silla),
P1 = ( a, a), P2 = ( a, a) (mnimos).

Problema 2

Hallar los extremos absolutos de la funcion f (x, y, z) = x + y + z en el


conjunto D = ((x, y, z) IR3 /x2 + y 2 z 1) .
Solucion:
 
1 1 1 1
El mnimo de f se alcanza en , , y vale , y el maximo se
2 2 2 2

 
1 1 1
alcanza en , , y vale 1 + 2
2 2 2

284
Problema 3

Sea la funcion f (x, y) = xy (y 2 x2 )


a) Determine y clasifique los puntos crticos de f
b) Detemine los maximos y los mnimos de f en la region
0 x 1; 0 y 1.
Solucion:
a) En P0 = (0, 0) hay un punto silla. En P1 = ( 13 , 1) hay maximo de
f ,
y en P2 = (1, 13 , ) hay un mnimo de f
2 2
b) El maximo es f ( 13 , 1) = y el mnimo f (1, 13 ) =
3 3 3 3

Problema 4

Calcular la distancia mnima del punto (0, b) a la parabola x2 4y = 0.


Solucion:
d=2

Problema 5

Calcular el paraleleppedo de volumen maximo que tiene una diagonal


de longitud 1.
Solucion:
 
1 1 1 1
V , , =
3 3 3 3 3

Problema 6

Hallar el punto de la elipse 3x2 2xy+3y 2 = 8 para la cual la respectiva


recta tangente esta a la menor distancia del origen.
Solucion:
Se tienen cuatro puntos crticos P0 = (1, 1) , P1 = (1, 1) ,
 
P2 = 2, 2 , P3 = 2, 2 .

285
Los puntos donde esta la menor distancia son:
 
d 2, 2 = d 2, 2 = 2
Los puntos donde esta la mayor distancia :

d (1, 1) = d (1, 1) = 2

Problema 7

Un canaleta cuya seccion transversal tiene forma de trapecio, con angu-


los iguales en la base, se fabrica doblando bandas iguales a lo largo de
ambos lados de una larga plancha de metal,de 12 pulgadas de ancho.
Encuentre los angulos de la base y el ancho de los lados que producen
la maxima capacidad de la canaleta.
2
Solucion: angulos , ancho de lados 4 pulgadas
3

Problema 8

Pruebe que para angulos x, y, z cuya suma es x + y + z = 2 ,


se cumple la desigualdad
1
sin x sin y sin z
8

3.6. Aplicaciones Derivada Direccional


Ejemplo 1

Suponga que la temperatura en grados Celsuis en el punto (x, y) cerca


de un aeropuerto esta dada por
1
f (x, y) = [7400 4x 9y 0, 03xy] ( con las distancias x e y
180
medidas en kilometros).
a) Suponga que un avion despega del aeropuerto en la ubicacion P (200, 200)
y se dirige al noreste en la direccion especificada por el vector v =
(3, 4) .Cual es la tasa de cambio inicial de la temperatura en la direc-
cion dada?.

286
b) Si el avion sale del aeropuerto en P y vuela en la direccion

v con
ds
una rapidez v = = 5 km/min. Cual es la tasa de cambio inicial de
dt
la temperatura en la direccion dada por unidad de tiempo?
Solucion:
a) Como f es una funcion diferenciable en IR2 , la derivada direccional
de la funcion f en el punto P en la direccion

v esta dada por:

Dub f (P ) = f (P ) u
b



u (3, 4)

3 4

Determinemos el versor ub= = = ,
k
uk 32 + 42 5 5
 
1 1
Ahora calculemos el gradiente f (P ) = (4 0, 03y) , (9 0, 03x)
180 180
La derivada direccional en cualquier punto es
   
1 1 3 4
Dvbf (P ) = (4 0, 03y) , (9 0, 03x) ,
180 180 5 5

Evaluemos la derivada direccional el punto P (200, 200), se encuentra


que:
    
3 10 4 15 18
Dvbf (200, 200) = + =
5 180 5 180 180
o
C
Dvbf (200, 200) = 0, 1
km

Lo que significa que se observara una disminucion de 0, 1o C de temper-


atura en esa direccion por cada km de viaje.
b) La tasa de variacion de la temperatura con respecto al tiempo
esta dada por
o
 
df df ds
C
= = Dvbf (P ) k v k = 0, 1 (5km/min)
dt ds dt km
df km
= 0, 5
dt mn

287
Ejemplo 2

Suponga ahora que consideramos que la temperatura en grados Celsuis


en el punto (x, y, z) cerca de un aeropuerto esta dada por
1
f (x, y, z) = [7400 4x 9y 0, 03xy] 2z , donde f es la tem-
180
peratura por kilometro de altitud.
a) Si un halcon esta inmovil en el aire, en el punto P (200, 200, 5) y
sobre el aeropuerto desciende en forma subita con una rapidez de 3
km/min en la direccion dada por el vector (3,4,-12) . Cual es la tasa
de cambio instantanea que experimenta el ave?
b) En que direccion debe descender el halcon que esta en el punto
P(200,200) a una altitud de 5 km, a fin de calentarse lo mas rapido?
c) Que tan rapido subira su temperatura conforme el ave baje a una
rapidez de 3 km/min?
d) Cual sera la direccion de la brujula y el angulo de descenso conforme
vuele en esa direccion particular?
Solucion:
a) La derivada direccional de la funcion f en el punto P en la direccion

v esta dada por:


Dub f (P ) = f (P ) u b

u (3, 4, 12)

3 4 12

Determinemos el versor ub= =q = , ,
kuk 2 2 2 5 5 13
3 + 4 + (12)
El vector gradiente de temperatura es
 
1 1
f (P ) = (4 0, 03y) , (9 0, 03x) , 2
180 180

El gradiente en la posicion inicial es


 
10 15
f (200, 200, 5) = , , 2
180 180

Por tanto , la tasa de cambio inicial de la temperatura del ave respecto


de la distancia en la direccion dada es

df
= Dub f (P ) = f (P ) u
b
ds
288
Evaluemos la derivada direccional la posicion inicial P (200, 200, 5),

      
3 10 4 15 12 47
Dvbf (200, 200, 5) = + 2 =
13 180 13 180 13 26

ds
Como la rapidez del ave es =3 km/min, entonces la tasa de cambio
dt
por unidad de tiempo que experimenta el halcon es
 o  o
df df ds
47 C C
= = Dvbf (P ) k v k = (3km/min) 5, 4
dt ds dt 26 km mn

c) El valor maximo esta dado por

df
= [Dvbf (P )]max = kf (P )k
ds s 2  2
10 15
= + + (2)2
180 180
= 2,0025

Y se alcanza en en la direccion dada por el gradiente

f (P ) (10, 15, 360)


n
b= =
kf (P )k 360, 45

La rapidez del halcon es 3 , por lo que la tasa temporal de cambio de


la temperatura en esa direccion experimentada por el halcon es
o o
 
df df ds
C C
= = Dvbf (P ) k v k = 2, 0025 (3km/min) 6, 0075
dt ds dt km mn

3.7. Aplicaciones de Maximos y Mnimos

3.7.1. Aplicacion al campo de la mecanica




Sea F : D R3 un campo de Fuerza definido en cierto dominio D de
R3 . Consideremos ahora una partcula de masa m que se mueve a lo
largo de una trayectoria

r (t) bajo la accion de este campo de fuerza.

289
La ecuacion de movimiento de la partcula esta dada por la segunda
ley de Newton:


F (r (t)) = m
r (t) ()



Si el campo vectorial F es conservativo, esto es, F = V , entonces:

1
m k

r 0 (t)k + V (

r (t)) = c
2
donde c es una constante. El primer termino se llama energa cinetica
y el segundo corresponde a la funcion potencial V.
Si diferenciamos la expresion anterior usando la regla de la cadena:
 
d 1


m k r (t)k + V ( r (t)) = m r 0 (t)

r (t)+V ( r (t))

0 00 0
r (t) =
dt 2
0

m r (t) + V ( r (t))
 0
r (t) = 0 m
r (t) + V (
00 00
r (t)) = 0


Por tanto, m r (t) = V (r (t)) = F (
00
r (t)) lo que demuestra la
ecuacion de Newton () .
Un punto
r o D se llama posicion de equilibrio si la fuerza en ese


punto es cero: F (r ) = 0.Un punto
o
r que sea de equilibrio se llama
o
estable si para todo > 0 y > 0, podemos escoger numeros o > 0
y o > 0 tales que un punto situado en cualquier lugar a una distancia
menor que o de r o ,despues de recibir inicialmente energa cinetica en
una cantidad menor que o , permanecera para siempre a una distancia
de
r o menor que y poseera energa cinetica menor que .

As, si tenemos una posicion de equilibrio, la estabilidad en


r o sig-


nifica que una partcula que se mueve lentamente cerca de r o siempre
permanecera cerca de
r o y se mantendra moviendose lentamente.
Ahora, si tenemos un punto de equilibrio inestable r o , entonces



r (t) = r o resuelve la ecuacion de Newton F ( r (t)) = m



r (t) ,


pero las soluciones cercanas pueden alejarse de r conforme trascurra
o
el tiempo.

Proposicion 3.7.1. i) Los puntos crticos de un potencial son posi-


ciones de equilibrio.
ii) En un campo conservativo, un punto r o en el cual el potencial
alcance un mnimo local estricto, es una posicion de equilibrio estable.

290
Demostracion

1) La primera afirmacion es bastante obvia debido a la definicion de




campo conservativo: F = V, los puntos de equilibrios
r o son
exactamente los puntos crticos de V, en los cuales V (

r o) = 0
2) Para probar la afirmacion ii), haremos uso de la ley de conservacion
de energa . Tenemos

1 1
m k

r 0 (t)k + V (

r (t)) = m kr 0 (0)k + V (

r (0))
2 2
Escojamos un pequena vecindad de r , y asumamos que la partcula
o
tienen poca energa cinetica. Conforme t crece, la partcula se aleja
de
r o sobre una trayectoria
r (t) y V (
r (t)) crece pues V (
r (0)) es
un mnimo estricto , de modo que la energa cinetica debe decrecer .
Si la energa cinetica inicial es suficientemente pequena, entonces, para
que la partcula escape de la vecindad de r o , fuera de la cual V ha
crecido en una cantidad definida, la energia cinetica tendria que volverse
negativa, lo cual es imposible. Asi, la particula no puede escapar de la
vecindad.
Sea una partcula en un campo de potencial V restringido a manten-
erse sobre la superficie de nivel S dada por la ecuacion (x, y, z) = 0


con 6= 0. Si en la ecuacion de Newton F ( r (t)) = m r (t) () ,



reemplazamos F con la componente de F paralela a S, aseguramos
que la partcula permanecera en S.

Proposicion 3.7.2. i) Si en un punto P sobre la superficie S el poten-


cial V |S tiene un valor extremo, entonces el punto P es una posicion
de equilibrio sobre la superficie.
ii) Si un punto P S es un mnimo local estricto del potencial V|S ,
entonces el punto P es una posicion de equilibrio estable.

Ejemplo Sea el campo gravitacional cerca de la superficie de la tier-




ra; esto es, sea F = (0, 0, mg) donde g es la aceleracion de gravedad
. Determine la funcion potencial gravitacional y cuales son las posi-
ciones de equilibrio, si una partcula con masa m esta restrigida a la
esfera g (x, y, z) = x2 + y 2 + z 2 r2 = 0, (r > 0)?Cuales son estables?.
Solucion:

291
V
Tenemos que Fz = = mg = V (x, y, z) = mgz
z
Usando el metodo de los multiplicadores de Lagrange podemos localizar
los extremos posibles, tenemos que:
L (x, y, z, ) =
V (x, y, z) + g (x, y, z)
mgz + x2 + y 2 + z 2 r2

L (x, y, z, ) =
Lx (x, y, z, ) 2x = 0 = 6= 0 y x = 0
= (1,0)
Ly (x, y, z, ) 2y = 0 = 6= 0 y y = 0
= (2,0)
mg
Lz (x, y, z, ) = mg + 2z = 0 = = y z 6= 0 (3,0)
2z
L (x, y, z, ) = x2 + y 2 + z 2 r2 = 0 ( 4,0)

Reemplazando (1,0), (2,0), (3,0) en ( 4,0)

mg
z 2 r2 = 0 = z = r y =
2r

Luego, se deduce que los puntos P1 = (0, 0, r) y P2 = (0, 0, r) son


posiciones de equilibrio.

3.7.2. Aplicaciones a la geometra


Ejemplo 1 Determine la distancia mnima desde el origen (0,0,0) a
y2 z2
la superficie S del elipsoide x2 + + = 1.
4 9
Solucion:
Sabemos que la distancia pentre un punto P y el origen esta dada por
la funcion d(x, y, z) = x2 + y 2 + z 2 . Sin embargo, por razones de
simplicidad en los calculos , en lugar de la funcion anterior vamos a
considerar la funcion f (x, y, z) = x2 + y 2 + z 2 en atencion a que f
tendra un mnimo en un punto si y solo si d lo tiene.
Se trata de obtener los extremos condicionados de la funcion distancia
y2 z2
f (x, y, z) = x2 + y 2 + z 2 sujeta a la condicion x2 + + = 1.
4 9
Formemos la funcion auxiliar de Lagrange

2 2 2 2 y2 z2
F (x, y, z, ) = (x + y + z ) + (x + + 1)
4 9
292
y consideremos entonces el sistema.
Fx (x, y, z, ) = 2x + 2x = 0
1
Fy (x, y, z, ) = 2y + y = 0
2
2
Fz (x, y, z, ) = 2z + z = 0
9
2
y z2
F (x, y, z, ) = x2 + + 1=0
4 9
De las tres primeras ecuaciones obtenemos
2x(1 + ) = 0 = x = 0 = 1
1
y(2 + ) = 0 = y = 0 = 4
2
1
2z(1 + ) = 0 = z = 0 = 9
9

Sustituyendo y = z = 0 en la cuarta ecuacion produce


x2 1 = 0 = x = 1
.
Reemplazando x = z = 0 en la cuarta ecuacion produce
y2
1 = 0 = y = 2
4
Sustituyendo x = y = 0 en la cuarta ecuacion produce
z2
1 = 0 = z = 3
9
Luego , se obtienen seis puntos crticos
P0 = (1, 0, 0) , P1 = (1, 0, 0) , P2 = (0, 2, 0) , P3 = (0, 2, 0)
P4 = (0, 0, 3) , P5 = (0, 0, 3) .
Evaluando la funcion en los puntos encontrados debera haber un maxi-
mo y
un mnimo
f (1, 0, 0) = 1, f (0, 2, 0) = 4, f (0, 0, 3) = 9
Se tiene que el mnimo de f se encuentra en los puntos (1, 0, 0) y es
igual 1. y el maximo esta localizado en los puntos (0, 0, 3) y vale 9.

293
Ejemplo 2 Determine la distancia mnima y maxima del origen a la
7
curva de interseccion del paraboloide x2 + y 2 + z = 0 y el plano
4
x + y + z 2 = 0.
Solucion:
Igual que el ejemplo anterior resulta mas conveniente hallar los ex-
tremos del cuadrado de la distancia
p respecto del origen en vez de la
funcion distancia d (x, y, z) = x + y 2 + z 2
2

Por consiguiente, se trata de obtener los extremos condicionados de


la funcion distancia f (x, y, z) = x2 + y 2 + z 2 sujeta a las condiciones
7
g(x, y, z) = x2 + y 2 + z = 0 y h(x, y, z) = x + y + z 2 = 0
4

Formemos la funcion auxiliar de Lagrange


 
2 2 2 2 2 7
F (x, y, z, 1 , 2 ) = (x +y +z )+1 x + y + z +2 (x + y + z 2)
4

y consideremos entonces el sistema.

Fx (x, y, z, 1 , 2 ) = 2x(1 + 1 ) + 22 = 0 (1)


Fy (x, y, z, 1 , 2 ) = 2y(1 + 1 ) + 22 = 0 (2)
Fz (x, y, z, 1 , 2 ) = 2z + 1 + 2 = 0 (3)
7
F1 (x, y, z, 1 , 2 ) = x2 + y 2 + z = 0 (4)
4
F2 (x, y, z, 1 , 2 ) = x + y + z 2 = 0 (5)

De las dos primeras ecuaciones se obtiene


1 = 1 o y = x
Consideremos primero el caso 1 = 1.A partir de (1) se obtiene 2 =
1
0 . Sustituyendo estos valores en (3) z = .
2
Reemplazando z en (4) y (5), produce
5
x2 + y 2
= 0
4
3
x+y = 0
2

Resolviendo el sistema se obtienen los puntos crticos

294
   
1 1 1 1
P0 = 1, , , P1 = , 1,
2 2 2 2
Al evaluar ambos puntos en la funcion distancia, obtenemos
r r
1 1 3
d (x, y, z) = 1 + + =
4 4 2
Consideremos ahora y = x , a partir de las ecuaciones (4) y (5) se
obtiene
7
2x2 + z = 0
4
2x + z 2 = 0

Resolviendo el sistema se obtienen los puntos crticos


!
1 2 1 2 2
P3 = , ,1 ,
2 4 2 4 2
Al evaluar todos estos puntos en la funcion distancia, obtenemos
!
1 2 1 2 2 1p
d , ,1 = 92 2
2 4 2 4 2 2
Como la curva de interseccion del paraboloide y el plano es cerrada, las
distancias maxima y minima absoluta del la curva al origen son:


r
1p 3
dmax = 9 + 2 2, dmn =
2 2

Ejemplo 3 Encuentre el volumen maximo de una caja rectangular


x2 y 2 z 2
inscrita en el elipsoide 2 + 2 + 2 = 1 con sus caras paralelas a los
a b c
planos coordenados.
Solucion:
Sea P(x,x,z) el vertice de la caja que esta en el primer octante donde
x > 0, y > 0, z > 0. Por la simetria del problema se desea maximizar
la funcion volumen f (x, y, z) = 8xyz sujeta a la condicion g(x, y, z) =
x2 y 2 z 2
+ 2 + 2 1=0
a2 b c
Formemos la funcion auxiliar de Lagrange
 2
y2 z2

x
F (x, y, z, ) = 8xyz + + 2 + 2 1
a2 b c

295
y consideremos entonces el sistema.
2
Fx (x, y, z, ) = 8yz + x = 0 (1)
a2
2
Fy (x, y, z, ) = 8xz + 2 y = 0 (2)
b
2
Fz (x, y, z, ) = 8xy + 2 z = 0 (3)
c
2 2
x y z2
F (x, y, z, ) = 2 + 2 + 2 1 = 0 (4)
a b c
Multiplicando las ecuaciones (1), (2)y (3) por x, y, x respectivamente,
produce

2 2
8xyz + x = 0 (1,1)
a2
2
8xyz + 2 y 2 = 0 (2,2)
b
2
8xyz + 2 z 2 = 0 (3,3)
c
Entonces , obtenemos
2 2 2 2 2 2
x = 2 y = 2 z = 8xyz
a2 b c
Para obtener el volumen maximo se requiere que x, y, x 6= 0 y 6= 0
Concluimos entonces que
x2 y2 z2
= =
a2 b2 c2
Sustituyendo esta expresion en la ecuacion (4), obtenemos

x2 a
3 1 = 0 = x =
a2 3
As, sucesivamente se tiene un unico punto crtico
 
a b c
P0 = , ,
3 3 3
Por lo tanto , la caja tiene un volumen maximo dado por
 
a b c 8
fmax , , = abc
3 3 3 3 3

296
3.7.3. Aplicacion al campo de la economa

Supongase que la produccion de cierto producto de una compana man-


ufacturera es una cantidad Q , donde Q es una funcion de f (K, L) donde
K es la cantidad de capital (o inversion) y L es la cantidad de trabajo
realizado. Si el precio del trabajo es p, el precio del capital es q y la
compania no puede gastar mas de B dolares, como podemos hallar la
cantidad de capital y de trabajo que maximice la produccion Q?
Solucion:
Se esperara que si se incrementa la cantidad de capital o de trabajo,
entonces la produccion debera incrementarse; esto es:

Q Q
0 y 0
K L

Tambien se esperaria que conforme se anada trabajo a una cantidad


dada de capital, obtendremos menos productos adicionales por nuestro
esfuerzo, esto es:
2Q
<0
K 2

De manera analoga,
2Q
<0
L2

Con estas hipotesis sobre Q , es razonable esperar que las curvas de


nivel de la produccion- llamadas isocuantas- Q(K, L) = c, se vean
como las esbozadas en la figura, con c1 < c2 < c3.

pendiente imagen

Podemos interpretar la convexidad de las isocuantas como sigue: si nos


movemos hacia la derecha a lo largo de una isocuanta dada , se em-
plea mas capital para reemplazar una unidad de trabajo y producir
la misma cantidad. La restriccion de presupuesto significa que debe-
mos mantenernos dentro del triangulo acotado por los ejes y la recta
pL + qK = B. Geometricamente, es claro que producimos mas al gas-
tar nuestro dinero de tal manera que seleccionemos la isocuanta que
solamente toca, pero no cruza, la recta presupuesto.

297
Como el punto maximo esta en la frontera de nuestro dominio, aplicare-
mos el metodo de los multiplicadores de Lagrange para hallar el maxi-
mo. Para maximizar Q = f (K, L) sujeto a la restriccion pL + qK = B,
buscamos los puntos crticos de la funcion auxiliar

H (K, L, ) = f (K, L) + (pL + qK B)

As queremos:

HK (K, L, ) = fK (K, L) + q = 0 = fK (K, L) = QK (K, L) = q


HL (K, L, ) = fL (K, L) + p = 0 = fL (K, L) = QL (K, L) = p
H (K, L, ) = pL + qK B = 0 = pL + qK B = 0

Con estas ecuaciones podemos encontrar los puntos crticos de la fun-


cion Q. Luego, usando derivacion implicita podemos determinar el pun-
to donde se maximiza la produccion.
En el ejemplo anterior , representa algo interesante. Mas adelante
vamos a interpretar .
Sean k = qK y l = pL, de modo que k es el valor en dolares del capital
empleado, y l es el valor en dolares del trabajo empleado, entonces las
ecuaciones se convierten en:

H (k, l, ) = f (k, l) + (l + k B)

Hk (k, l, ) = fk (k, l) + = 0 = fk (k, l) = Qk (k, l) =


Hl (k, l, ) = fl (k, l) + = 0 = fl (k, l) = Ql (k, l) =
H (k, l, ) = l + k B = 0 = l + k B = 0

Comparando con las primeras dos ecuaciones del caso anterior se tiene
que:

Q 1 Q 1
= = = =
k q K l p K
As , en el punto optimo de produccion , el cambio marginal en la pro-
duccion por dolar de inversion de capital adicional, es igual al cambio
marginal de la produccion por dolar de trabajo adicional, y es este

298
valor comun. En el punto optimo, el intercambio de un dolar de capi-
tal por un dolar de trabajo no cambia la produccion. Fuera del punto
optimo, la produccion marginal es distinta, y un intercambio, o el otro,
incrementan la produccion.

Ejemplo Suponga que la produccion total de una compania esta da-


da por la funcion P = P (K, L) = K 1/4 L1/2 donde K denota el numero
de unidades de capital usado y L es el numero de unidades de trabajo
usado.
a) Suponga que cada unidad de capital K cuesta 1 millon de dolares
y cada unidad de trabajo L cuesta 7 millones de dolares.Considere el
problema de maximizar la produccion total P si el presupuesto total
es de 10 millones de dolares.
b) Suponga que el capital K aumenta en una razon de 4 % por ano, y
el trabajo L aumenta en una razon de 5 % por ano. Determine la razon
de crecimiento de la produccion P por ano.
Solucion:
Tenemos que extremar P = P (K, L) = K 1/4 L1/2 bajo la condicion
K + 7L 10 = 0
Sea la funcion auxiliar de Lagrange

F (K, L.) = K 1/4 L1/2 + (K + 7L 10)

y consideremos entonces el sistema.


1 3/4 1/2
FK (K, L.) = K L +=0 (1)
4
1 1/4 1/2
FL (K, L.) = K L + 7 = 0 (2)
2
F (K, L.) = K + 7L 10 = 0 (3)

Multiplicando (1,0) por 4K y (2,0) por 2L, produce:

K 1/4 L1/2 + 4K = 0 (1,1)


K 1/4 L1/2 + 14L = 0 (2,1)

Restando las ultimas ecuaciones tenemos:

299
4K 14L = 0

Entonces

2(2K 7L) = 0 = = 0o (2K 7L) = 0

Si = 0 nos quedamos sin restricion, luego

7
6= 0y(2K 7L) = 0 = K = L
2

Reemplazando el resultado anterior en (3,0) ,queda


7 20 10
L + 7L = 10 = L = entonces K =
2 21 3
 
10 20
Asi, se tienen un unico punto crtico en P0 = , .
3 21
   1/4  1/2
10 20 10 20
Por tanto, la produccion maximizada sera P , =
3 21 3 21
ii) Supongamos que K y L aumentan en una razon de 4 % y 5 %,
respectivamente, por ano
dK
= 0, 04K (1,0)
dt
dL
= 0, 05L (2,0)
dt

Usando la regla de la cadena sobre la funcion P (K, L) tenemos:

dP P K P L
= +
dt K t L t
dP 1 3/4 1/2 K 1 1/4 1/2 L
= K L + K L (3,0)
dt 4 t 2 t

Sustituyendo (1,0) y (2,0) en (3,0)


dP 1 3/4 1/2 1
= K L (0, 04K) + K 1/4 L1/2 (0, 05L)
dt 4  2 
dP 1 1
= K 1/4 L1/2 0, 04 + 0, 05
dt 4 2

300
Reemplazado en la expresion anterior P = K 1/4 L1/2
dP
Finamente, se tiene: = 0, 035P,lo que significa que la produccion
dt
aumenta 3,5 % por ano.

Ejemplo Una compania planea gastar 10.000 dolares en publicidad.


Cuesta 3.000 dolares un minuto de publicidad en la television y 1.000
dolares un minuto de publicidad en la radio. Si la empresa compra
x minutos de comerciales en television e y minutos de comerciales en
la radio, su ingreso, en miles de dolares, esta dado por f (x, y) =
2x2 y 2 + xy + 8x + 3y. Como puede la empresa maximizar su
ingreso invirtiendo en la publicidad?
Solucion:
Se desea maximizar la funcion f (x, y) = 2x2 y 2 + xy + 8x + 3y
bajo la restriccion g(x, y) = 3x + y 10 = 0.
Consideremos la funcion auxiliar

L (x, y, ) = 2x2 y 2 + xy + 8x + 3y + (3x + y 10)

Primero,apliquemos la condicion necesaria de punto crtico L (x, y, ) =


0.

Lx (x, y, ) = 4x + y + 8 3 = 0 (1)
Ly (x, y, ) = 2y + x + 3 = 0 (2)
L (x, y, ) = 3x + y 10 = 0 (3)

De la ecuaciones (1) y (2)se obtiene

y = 3 + 4x 8 (1,1) , x = + 2y 3 (2,1)

Reemplazando (2,1) en (1,1) queda:

y = 3 + 4 ( + 2y 3) 8 = y = 7 + 8y 80

Luego, se tiene
20
y= (1,2)
7

Asi, (1,2) en (2,1) produce:

301
19
x= (2,2)
7

Sustituyendo (1,2) y (2,2) en (3,0) , obtenemos:


1
4 1 = 0 = = .
4
 
73 69
Entonces (1,2) y (2,2) nos da un unico punto crtico P0 = ,
28 28
.
Determinemos la naturaleza de este punto critico usando el criterios de
la segunda derivada.
El hessiano para f (x, y) es
   
73 69 fxx fxy 4 1 73 69
H , = = =7>0 y fxx ,
28 29 fxy fyy 1 2 28 28
= 4 < 0
 
73 69
Por lo tanto, la funcion es maxima en el punto P0 = , .
28 28
73
As , la empresa tendria que comprar minutos de comerciales en
28
69
television y minutos de comerciales en radio.
28

3.7.4. Problemas Propuestos de Aplicaciones


1.- Sea una partcula que se mueve en un campo de potencial en R2
dado por V (x, y) = 3x2 + 2xy + 2x + y 2 + y + 4. Hallar los puntos de
equilibrio estable si los hay.
 
1 1
Solucion: Hay un unico punto de equilibrio estable en , .
4 4
2.- Sea una partcula moviendose en un campo de potencial en R2 dado
por V (x, y) = x2 + 4xy y 2 8x 6y. Hallar todos los puntos de
equilibrio.Cuales, si los hay, son estable?
Solucion : Hay un unico punto de equilibrio inestable en (2, 1) .
3.- Sea una partcula restringida a moverse sobre la esfera x2 +y 2 +z 2 =
1, sujeta a fuerzas gravitacionales, asim como al potencial adicional
V (x, y, z) = x + y.Hallar los puntos de equilibrio estable, si los hay.

302
1/2
Solucion : Hay un unico punto de equilibrio estable en (2 + m2 g 2 ) (1, 1, mg) .

4.- Usando la informacion anterior , encuentre el punto optimo para la


funcion de produccion Q (K, L) = AK L1 , donde A y son con-
stantes positivas y 0 < < 1, que se usa para modelar la econonoma
nacional. Q es, entonces, la produccion agregada de la economa para
una entrada de capital y trabajo dada .
KQ pL
Solucion: En el optimo: =
1

5.- Una compania usa aluminio, hierro y magnesio para producir acc-
cesorios de automoviles. La cantidad de accesorios que puede producir
usando x toneladas de aluminio, y toneladas de hierro y z toneladas de
magnesio es Q (x, y, z) = xyz. El costo de la materia prima es: aluminio
6 dolares por tonelada; hierro 4 dolares por tonelada ; y magnesio 8
dolares por tonelada. Cuantas toneladas de aluminio, hierro y magne-
sio deberan usarse para manufacturar 1000 accesorios al menor costo
posible?
20
3

Solucion: x = 3 , y = 10 3 3, z = 5 3 3
3

6.- Una Pyme cuenta con 8.000 dolares para importar dos tipos de
bebidas energeticas . Si x son la unidades de bebidas energeticas que se
12x
importaran desde Holanda, y se estima que venderan unidades de
x+6
esta bebidas a un precio de 200 dolares cada una. Si y son la unidades
de bebidas energeticas que se importaran desde Alemania, estimandose
24y
que venderan unidades, a un precio de 200 dolares cada una.
y+3
Si el costo por unidad vendida de cada bebida es de 50 dolares.
a) Determine cuantas unidades de cada bebida energetica deben im-
portar para maximizar su utilidad.
b) Determine la utilidad maxima.

Solucion:La funcion utilidad esta dada por la diferencia entre el precio


de venta de las bebidas y el costo de importacion
a) 78, 5 bebidas Holandesas y 81, 5 bebidas Alemanas.
b) Utilidad U (78, 5; 81, 5) = 5144, 7 dolares.

303
7.- La funcion de produccion de una compana es Q (x, y) = xy. El
costo de produccion es C (x, y) = 2x+3y.Si esta compana puede gastar
C (x, y) = 10, cual es la maxima cantidad que puede producir?

 
5 5 25
Solucion: P , =
2 3 6

3.8. Auto evaluaciones


Autoevaluacion No 1
El estudiante:
1) Determinara si una funcion f : D R2 R definida en el conjun-
to abierto D, es o no diferenciable en un punto del dominio, empleando la
definicion de diferenciabilidad.
2) Calculara la derivada direccional maxima de una funcion f diferenciable
en un punto evaluando el gradiente y el vector unitario necesario.
3) Utilizara el metodo de los multiplicadores de Lagrange para localizar
los posibles valores extremos de una funcion f : RN R, definida en
un conjunto abierto , sujeta a una o dos condiciones gm (x) = 0 , m = 1,2 y
m<N

Tiempo: 2 horas
Pregunta 1
Estudie la diferenciabilidad de la funcion f (x; y) = x2 + 2y 2 , en el punto
(x0 ; y0 ) = (1; 2).

Pregunta 2
Hallar los coeficientes a y b para que la derivada direccional maxima

de la funcion eax+by cos(x + y) z = 0 en el punto (0, 0) sea 3 2 en la
direccion de la bisectriz del primer cuadrante.
Pregunta 3

Calcular la mnima distancia desde el origen (0, 0) hasta la recta


2x + y = 1,usando el teorema de los Multiplicadores de Lagrange.

Pauta Autocorreccion

304
Pregunta 1
En primer lugar, sus derivadas parciales en el punto (x0 ; y0 ) = (1; 2)
existen y valen
f f
(x, y) = 2x = (1, 2) = 2
x x
f f
(x, y) = 4y = (1, 2) = 8
y y
En segundo lugar, para mostrar que f es diferenciable tenemos que
demostrar
que se cumple

f f
f (x, y) f (x0 , y0 ) + (x0 , y0 ) (x x0 ) + (x0 , y0 ) (y y0 )
x y
lm q =0
(x,y)(x0 ,y0 )
(x x0 )2 + (y y0 )2

En efecto, tenemos que mostrar que

(x2 + 2y 2 ) (9 + 2(x 1) + 8(y 2))


lm q =0
(x,y)(1,2)
(x 1)2 + (y 2)2
Para demostrar esto , traslademos el origen usando el cambio de variables
u = (x 1) y v = (y 2) .De este modo, tenemos que
((u + 1)2 + 2(v + 2)2) (9 + 2u + 8v)
lm
(u,v)(0,0) u2 + v 2
si se desarrollan los parentesis, se obtiene

u2 + 2v 2
lm
(u,v)(0,0) u2 + v 2

Si nos acercamos al origen por una recta v = ku, obtenemos


u2 + 2v 2 u2 (1 + 2k 2 )
lm = lm =0
(u,v)(0,0) u2 + v 2 u0 u 1 + k 2

Esto no prueba que el valor del lmite sea cero. Lo que nos indica es que
si
tal lmite existe, debe ser cero.
As, se requiere la aplicacion directa de la definicion de lmite para concluir
que este existe y vale cero
Para aplicar la definicion se tiene que
2
u + 2v 2 2u2 + 2v 2 2 u2 + v 2

305
Luego

2
u + 2v 2 2(u2 + v 2 )

0 = 2 u2 + v 2
u2 + v 2 u2 + v 2
vemos que
2(u2 + v 2 )

u2 + v 2 < = = u2 + v 2 <
2
u +v 2 2
Es decir, con = 2 nos queda
2
u + 2v 2


k(u, v) (0, 0)k < =
0 < 
u2 + v 2
lo que nos dice que efectivamente

u2 + 2v 2
lm =0
(u,v)(0,0) u2 + v 2

Por lo tanto la funcion f es diferenciable en el punto (1, 2) .

Pregunta 2
La funcion z = eax+by cos(x + y) es continua en todo IR2 por ser com-
posicion
de funciones continuas.

f
z0x = = aeax+by cos(x + y) eax+by sen(x + y)
x
f
z0y = = beax+by cos(x + y) eax+by sen(x + y)
y

Ademas, las derivadas parciales son continuas en todo IR2 .Por tanto la
funcion es diferenciable en todo en todo IR2 .
Esto significa que la derivada direccional en un punto P (x, y) en una
direccion u
b se puede obtener como el producto escalar del gradiente de la
funcion en el punto y el versor que senala la direccion en el punto considerado

Dub f (x, y) = f (x, y) u
b=3 2
Por otro lado, el gradiente de f nos da la direccion hacia donde la derivada
direccional es maxima, que en este caso corresponde a la bisectriz del primer
cuadrante, luego tenemos:
!
f (0, 0) 2 2
kf (0, 0)k = 3 2 y u b= = ,
kf (0, 0)k 2 2

306
Calculando el gradiente en el punto P(x, y) se tiene:

f (x, y) = (aeax+by cos(x + y) eax+by sen(x + y), beax+by cos(x + y)


eax+by sen(x + y))

Evaluando en el origen, obtenemos:

f (0, 0) = abi + bb
j
se tiene que cumplir que:

!

 
a b 2 2
a2 + b 2 = 3 2 y u
b= , = , = a = b
3 2 3 2 2 2
Por lo tanto, al sustituir el resultado en la primera ecuacion, queda:
a=b=3

Pregunta 3 p
Hay que calcular los mnimos de la funcion d (x, y) = x2 + y 2 bajo la
condicion 2x + y 1 = 0. Sin embargo, podemos simplificar
calculos si se trabaja con la funcion f (x, y) = x2 + y 2 , puesto que
los maximos y minmos de ambas funciones coinciden.
Determinemos los puntos crticos usando la funcion de Lagrange
Sea la funcion de Lagrange F (x, y, ) = x2 + y 2 + (2x + y 1) ,
aplicando la condicion necesaria de punto critico , se tiene
Fx (x, y, ) = 2x + 2 = 0 = x =

Fy (x, y, ) = 2y + = 0 = y =
2
F (x, y, ) = 2x + y 1 = 0

Sustituyendo los dos primeros resultados en la tercera ecuacion, produce


2 1 2
x = ,y = , = .
5 5 5  
2 1
Entonces, existe un unico punto crtico, que es P = , .
5 5
Determinemos si es maximo o mnimo, usando la funcion de Lagrange
2
para el valor obtenido F (x, y) = x2 + y 2 + (2x + y 1)
3
Derivando parcialmente, queda
4
Fx (x, y) = 2x + = Fxx (x, y) = 2, Fxy (x, y) = 0
3
2
Fy (x, y) = 2y + = Fyy (x, y) = 2
3
307
Luego, el determinante
de la matriz Hessiana es:
2 0
H (x, y) = = 4 6= 0. Como Fxx = 2 > 0 , se trata de un mnimo
0 2
de F .

Autoevaluacion No 2

El estudiante:
1) Determinara la continuidad de una funcion f : R2 R, dada f (x, y)
y un punto (x0 , y0 ) del dominio.
2) Evaluara las derivadas parciales, de primer y segundo orden, de una
funcion f (x, y) en un punto dado del dominio , por definicion.
3) Calculara las derivadas parciales de primer y segundo orden de una
funcion f (x, y) usando el algebra de derivadas.
4) Empleara la definicion de derivada direccional para determinar la ex-
istencia y variacion de una funcion f en un punto (x, y) de su dominio en
una direccion vb,
5) Empleara el teorema de la funcion implicita, en la vecindad de un
punto (x0 , y0 ) del dominio y calculara las derivadas de una funcion implcita
de primer y segundo orden.
6) Usara la regla de la cadena para probar que una funcion compuesta es
solucion de una ecuacion diferencial parcial.

Tiempo: 2 horas
Pregunta 1  x sin(xy)
si (x, y) 6= (0, 0)
x2 +y 2
Dada la funcion f (x, y) =
0 si (x, y) = (0, 0)
a) estudie la continuidad de f en todo IR2
f f
b) calcular y en todo IR2
x y
c) estudie la derivada direccional de f en (0, 0)
d) determine si f es diferenciable en (0, 0).

Pregunta 2
Dada F (x, y, z) = x2 + y 2 + z 2 + xy + 2z 1 = 0
a) Verificar si F (x, y, z) = 0 define en el punto P (0, 1, 0) a z como
funcion
implcita de x e y, es decir, z = f (x, y)
b) Calcular zx (0, 1) , zy (0, 1) , zxx (0, 1) y zyy (0, 1)

Pregunta 3

308
La ecuacion de onda
2u 2u
= a
t2 x2
donde a es una constante, describe el movimiento de una onda, que puede
ser una onda de sonido, una onda de luz o una onda que viaja a lo largo
de una cuerda vibrante.Si f y g son funciones de una sola variable dos
veces derivables, compruebe que la funcion u(x, t) = f (x + at) + g(x at)
satisface la ecuacion de onda.
Pauta Autocorreccion
Pregunta 1
(a) En los puntos de IR2 distintos de (0, 0) la funcion es continua por ser
cuociente de funciones continuas con denominador no nulo.
En (0, 0) estudiemos la continuidad de f :
i) Existe f (0, 0) = 0
ii)
x sin (xy)
lim f (x, y) = lim
(x,y)(0,0) (x,y)(0,0) x2 + y 2

Al usar coordenadas polares se tiene


x sin (xy) r cos sin (r2 cos sin )
lim = lm
(x,y)(0,0) x2 + y 2 r0 r2
2
= lmr cos sin = 0 = f (0, 0)
r0

Puesto que sin ' si 0,y [0, 2] , 1 < cos2 sin < 1,en el
ultimo lmite se tiene que el producto de una cantidad infinitesimal
por una funcion acotada es cero.
Por lo tanto la funcion es continua en todo IR2
b) Las derivadas parciales (x, y) 6= (0, 0) son:
f [sin (xy) + xy cos (xy)] (x2 + y 2 ) 2x [x sin (xy)]
=
x (x2 + y 2 )2
f [x cos (xy)] (x + y 2 ) 2y [x sin (xy)]
2 2
=
y (x2 + y 2 )2
Ademas, las derivadas parciales para (x, y) = (0, 0) son
h sin 0
f f (h, 0) f (0, 0) 2 0 0
(0, 0) = lm = lm h = lm = 0
x h0 h h0 h h0 h
0 sin 0
f f (0, k) f (0, 0) 2 0 0
(0, 0) = lm = lm k = lm = 0
y k0 k k0 k k0 k

309
Por lo tanto las derivadas parciales de primer orden existen en todo IR2
c) Calculamos
la derivada direccional en cualquier direccion u
b = (a, b) tal
que kb
uk = a + b = 1utilizando, la definicion
f (ta, tb) f (0, 0)
Dub f (0, 0) = lm
t0 t
ta sin(t2 ab) ta(t2 ab)
t2 0 2 0
= lm = lm t
t0 t h0 t
2
Dub f (0, 0) = a b

Por tanto, existe la derivada direccional de la funcion f en el origen


en cualquier direccion ub = (a, b).
d) Examinemos si funcion es diferenciable en el origen.
Utilizando la definicion de diferenciabilidad, se tiene:

f (h, k) f (0, 0) fx (0, 0) h fy (0, 0) k


lm =
(h,k)(0,0) k(h, k)k
h sin (hk)
00h0k h sin (hk)
lm h2 + k 2 = lm
(h,k)(0,0) h2 + k 2 (h,k)(0,0) (h2 + k 2 )3/2

Calculemos este lmite utilizando coordenadas polares


h sin (hk) r cos sin (r2 cos sin )
lm = lm
(h,k)(0,0) (h2 + k 2 )3/2 r0 r3
r cos (r2 cos sin )
= lm
r0 r3
= cos2 sin 6= 0

El lmite no es cero y por lo tanto la funcion no es diferenciable en (0, 0).

Pregunta 2
a) F (x, y, z) = 0 define a z = f (x, y) en una vecindad de P (0, 1, 0) si:
i) El punto P es un punto de la superficie, es decir, F (0, 1, 0) = 0.
En efecto, al evaluar F (0, 1, 0) = 0 + 1 + 0 + 0 + 0 1 = 0.
ii) Fx , Fy , Fz son continuas en una vecindad de P.
En efecto:
Fx (x, y, z) = 2x + y , Fy (x, y, z) = 2y + x, Fz (x, y, z) = 2z + 2
son funciones polinomicas y estas son continuas en IR2 .
iii) Ademas, se debe cumplir Fz (0, 1, 0) 6= 0
Como Fz (x, y, z) = 2z + 2 entoncesFz (0, 1, 0) = 0 + 2 = 2 6= 0.

310
El teorema de la funcion implcita garantiza una vecindad V (0, 1) en la
cual podemos definir una funcion z = f (x, y) tal que F (x, y, f (x, y)) = 0.

b) La funcion tiene derivadas continuas en V (0, 1) que pueden calcularse


por:
Fx (x, y, z) 2x + y Fx (0, 1, 0) 1
zx (x, y) = = = zx (0, 1) = =
Fz (x, y, z) 2z + 2 Fz (0, 1, 0) 2
Fy (x, y, z) 2y + x Fy (0, 1, 0)
zy (x, y) = = = zy (0, 1) = =1
Fz (x, y, z) 2z + 2 Fz (0, 1, 0)
Para calcular las derivadas de segundo orden basta derivar (1) y (2) re-
specto
a x e y respectivamente:
[(2z + 2)2 (2x + y) (2zx )] 5
zxx (x, y) = 2
= zxx (0, 1) =
(2z + 2) 4
[(2z + 2) 2 (2y + x) (2zy )]
zyy (x, y) = = zyy (0, 1) = 2
(2z + 2)2
Pregunta 3
Derivando u(x, t) con respecto a x se tiene que :
u
= f 0 (x + at) + g 0 (x at)
x
2u
= f 00 (x + at) + g 00 (x + at)
x2
Asimismo las derivadas u(x, t) con respecto a t estan dadas por :
u
= af 0 (x + at) ag 0 (x at)
t
2u
= a2 f 00 (x + at) + a2 g 0 (x at)
t2
Sustituyendo obtenemos que
2u 2 00 2 0 2 00 0
2
2 u
= a f (x + at) + a g (x at) = a (f (x + at) + g (x at)) = a
t2 x2

Autoevaluacion No 3
El estudiante:
1) Resolvera problemas de aplicacion empleando la derivada direccional
de una funcion f diferenciable en un punto (x, y) en una direccion dada por
vector unitario ub,
2) Empleara el teorema de la derivada implicita para obtener las derivadas
parciales de primer y segundo orden de una funcion implicita.
3) Hallara los puntos crticos de una funcion f : R2 R aplicando
la condicion necesaria de punto crtico.

311
4) Utilizara el criterio de la segunda derivada para determinar los ex-
tremos locales de funciones de dos variables.

Tiempo: 2 horas
Pregunta 1
El conjunto de los puntos (x, y) tal que 0 x 5 , 0 y 5 es un
cuadrado
colocado en el primer cuadrante del plano XY . Supongamos que se calien-
ta
ese cuadrado de tal manera que T (x, y) = x2 + y 2 es la temperatura en el

puntoP (x, y). En que direccion se establecera el flujo de calor en el punto


P0 (3, 4) ?.
Pregunta
 xy 2 
Sea g , x2 + y 2 = 0 una ecuacion que define a z como una funcion
z
de
x e y. Verifique que si gx ; gy y gz existen y son continuas en toda la region
en la que gz 6= 0; entonces

z (x2 y 2 )
yzx xzy =
xy

Pregunta 3
Hallar los valores extremos locales, absolutos y puntos sillas de
f (x, y) = xy(1 x2 y 2 ) en [0, 1] [0, 1].

Pauta Autocorreccion
Pregunta 1


El flujo de calor en la region esta dado por una funcion vectorial f (x, y)
y
su valor en cada punto depende de las coordenadas de este. Sabemos


que f (x, y) es perpendicular a las curvas de isonivel T (x, y) = c donde
c es constante. Por consiguiente, el gradiente de la funcion T (x, y)


verifica esta condicion. Entonces f (x, y) = T (x, y) donde es una
constante positiva ,llamada conductividad termica. Notese que el signo
negativo indica que el calor fluye desde puntos de mayor temperatura
a puntos de menor temperatura.

Como T (3, 4) = 25 el punto P esta en la isoterma T (x, y) = 25 , que es


un cuadrante de la circunferencia x2 + y 2 = 25. Sabemos que el flujo

312


de calor en P0 (3, 4) es f (3, 4) = KT (3, 4) .
Apartir de T (x, y) = 2xbi + 2yb jse tiene que T (3, 4) = 6bi + 8b
j.As, el


flujo de calor en P0 es: f (3, 4) = (6i + 8j).
b b
Como la conductividad termica es positiva se puede afirmar que el
calor fluye en P0 en la direccion del vector unitario.
 
(6bi + 8b
j) 3b 4 b
b=
u = i+ j
36 + 64 5 5

Pregunta
 xy 2  xy
Sea g , x2 + y 2 = 0 y u = , v = x2 + y 2 .
z z
Entonces h y i
gx g u + g v 2x
zx = =  z xy 
gz gu 2
z
h x i
gy gu + g v 2y
zy = =  z xy 
gz gu 2
z
Sumando terminosh y i h x i
gu + gv 2x gu + gv 2y
yzx xzy = y z x z
xy xy
gu 2 gu 2
z z
(x2 y 2 )
gu z (x2 y 2 )
= z =
xy xy
gu 2
z
Pregunta 3
Sea f (x, y) = xy(1 x2 y 2 ) en el interior del cuadrado (0, 1) (0, 1).
Obtengamos sus puntos crticos
f (x, y) = 0
fx (x, y) = y(1 x2 y 2 ) + xy(2x) = y 3x2 y y 3 = 0
fy (x, y) = x(1 x2 y 2 ) + xy(2y) = x x3 3xy 2 = 0
Factorizando las ecuaciones anteriores, se tiene
y(1 3x2 y 2 ) = 0 = y = 0 o 1 3x2 y 2 = 0
x(1 x2 3y 2 ) = 0 = x = 0 o 1 x2 3y 2 = 0
Consideremos y = 0, x = 0 = P0 (0, 0) no pertenece al interior del
cuadrado.
Tomemos ahora las expresiones
y = 0; 1 x2 3y 2 = 0 = dos puntos criticos P1 (1, 0); P2 (1, 0)
estos puntos no pertenecen al interior del cuadrado.
Resolvamos las ecuaciones

313
x = 0; 1 3x2 y 2 = 0 = P3 (0; 1); P4 (0; 1) estos puntos
tampoco pertenecen al interior del cuadrado.
Examinemos, las ecuaciones

1 x2 3y 2 = 0
= y 2 = 1 3x2 = 1 x2 3(1 3x2 ) = 0
1 3x2 y 2 = 0
1 1
4x2 = 1 = x = ; x = esta ultima coordenada no pertenece al
2 2
interior del cuadrado, luego , al sustituir en la segunda ecuacion
anterior produce:  
1 2 2 1 1 1 1
x = ; 1 x 3y = 0 = y = ; y = = dos punto P5 ;
2 2 2 2 2
y  
1 1
P6 ; / al interior del cuadrado.
2 2  
1 1
Entonces estudiemos el Hessiano en el punto P5 ;
2 2
2
fxx(x, y)= 6xy, 3 fxy1(x, y) = 1 3x  3y 2 , fyy (x, y) = 6xy
1 1 1 1 3
H , = 21 2 = 2 > 0 y f
3 xx , = < 0 =
2 2 2 2   2 2 2  
1 1 1 1 1
hay un mnimo local de f en P5 , cuyo valor es f , = .
2 2 2 2 8
Finalmente, estudiemos la funcion en la frontera del cuadrado:
Para x [0, 1]; y = 0 ; f (x, 0) = 0 = mnimo en este segmento
abierto .
Ahora, en los vertices, f (0; 0) = 0 = mnimo local y f (1, 0) = 0.
0
Para y [0, 1]; x = 1; f (1; y) = y 3 = f (y) = 3y 2 = 0
00
y = 0; f (y) = 6y; f 00 (0) = 0
no existen valores extremos en el segmento abierto .
Ahora, en los vertices f (1, 0) = 0; f (1, 1) = 1. Por lo tanto hay mnimos
locales en (1, 0),(1, 1).
Para x [0, 1]; y = 1; f (x; 1) = x3 = f 0 (x) = 3x2 = 0
x = 0; f 00 (x) = 6x; f 00 (0) = 0 entonces no existe ni maximo ni mnimo
en el segmento abierto.
En los vertices x = 0; x = 1 tenemos f (0, 1) = 0; f (1, 1) = 1 =

mnimos locales.

Para x = 0; y [0, 1]; f (0; y) = 0 = mnimo local en el segmento


abierto.

314
Captulo 4

Integracion Multiple

Falta una introduccion

4.1. Integrales dobles y triples


4.1.1. Integrales Dobles
Aspectos geometricos
Sea R un rectangulo representado por R = [a, b] [c, d] y f una funcion
continua definida sobre R, es decir

f : R R2 R

Caso de funciones no negativas:


Supongamos que f (x, y) 0 y (x, y) R tal que la grafica de z = f (x, y)
esta arriba del plano xy ,determinando una region V del espacio R3 , bajo la
superficie z = f (x, y) y sobre la region R.
Antes de dar una definicion en el lenguaje de las Sumas de Riemann,
podemos decir que; bajo las condiciones anteriores el volumen de la region
V corresponde en este caso a lo que llamaremos integral doble de f sobre
R y que denotaremos:
Z Z Z Z
f (x, y)dA o f (x, y)dxdy
R R

315
Ejemplo:

Sea f (x, y) = x2 + y 2 y R = [0, 1] [0, 2].

Figura 4.1: paraboloide

El volumen bajo el paraboloide z = x2 + y 2 sobre el rectangulo R corre-


sponde a la integral doble de f sobre R en este caso.
Z Z
(x2 + y 2 )dA
R

Debemos estar claros eso si, que el concepto de integral doble es mucho
mas que esta interpretacion geometrica

Integral doble sobre un rectangulo


Sea R = [a, b] [c, d] un rectangulo y f una funcion acotada definida
sobre R es decir existe M > 0 tal que
M f (x, y) M para (x, y) R
Observacion: Una funcion continua sobre un rectangulo cerrado siempre es
acotada.

Particion
Sean P1 = {x0 , x1 , ..., xn } particion de [a, b]
P2 = {y0 , y1 , ..., yn } particion de [c, d]
Al conjunto P = P1 P2 = {(xi , yj ) / 0 i n, 0 j n} lo llamare-
mos particion de R de valor n n.
Sea kP1 k = max {4xi = xi xi1 / i = 1, 2, ..., n}
kP2 k = max {4yi = yi yi1 / j = 1, 2, ..., n}

316
Norma de la particion.
La norma de P denotada kP k se define por

kP k = max {kP1 k , kP2 k}


kP k = max {lij : diagonal de Rij }
(no es unica forma de definir norma de P , pero esta es la que usaremos)

Sumas superiores y sumas inferiores


Sean Rij rectangulo [xi1 , xi ] [yj1 , yj ] 1 i, j n y 4ij = area del
rectangulo Rij = (xi xi1 ) (yj yj1 )
Definimos ahora, sumas inferiores y sumas superiores de Riemann de f
respecto de la particion dada, por
n
X
sP (f ) = mij (f )4ij ,
i,j=1
X n
SP (f ) = Mij (f )4ij
i,j=1

Como consecuencia de estas definiciones podemos decir de estas sumas:


i) Si P es una particion cualquiera de R

sP (f ) SP (f )

ii) Si P 0 es particion mas fina que P ( P P 0 ) entonces

sP (f ) sP 0 (f ) y SP 0 (f ) SP (f )

iii) Si P1 y P2 son dos particiones cualquiera de R

sP1 (f ) SP2 (f )

Con estas sumas formamos los respectivos conjuntos:


Conjunto de sumas inferiores.

{sP (f )/P es particion de R}


Conjunto de sumas superiores.

{SP (f )/P es particion de R}

317
Si m y M son cortas inferior y superior respectivamente de f en R en-
tonces
si A = (b a) (d c).

i) sP (f ) m A para todo P particion de R


es decir el conjunto de sumas inferiores es acotado inferiormente
ii) SP (f ) M A P particion de R, es decir, el conjunto de sumas
superiores es acotado superiormente.
Por lo tanto, haciendo uso del axioma del supremo (o del nfimo) de la
axiomatica de los numeros reales podemos definir.
Si R es un rectangulo de R2 y f una funcion acotada sobre R definimos:
a) Integral Inf erior de f sobre R por
Z Z
f dA = sup {sP (f ) : P es particion de R}
R

b) Integral Superior de f sobre R por


Z Z
f dA = nf {SP (f ) : P es particion de R}
R

Sumas e Integrales
Las definiciones de estas respectivas integrales permiten afirmar que para
toda particion P de R y toda funcion acotada definida sobre R
Z Z Z Z
sP (f ) f dA f dA SP (f )
R R

Estamos ahora en condiciones de formular la definicion de integral doble


sobre un rectangulo en base a sumas superiores e inferiores.
Una funcion f (x, y) definida y acotada sobre un rectangulo R se dice que
es Riemann integrable sobre R si
Z Z Z Z
f dA = f dA
R R

Si f es integrable
R R sobre R,R Rentonces la integral doble definida de f sobre R
se denota por R
f dA o R
f dxdy y en tal caso
Z Z Z Z Z Z
f dA = f dA = f dA
R R R

318
Nota: Alternativamente en cursos de calculo se define:
Z Z n
X
f dA = lm f (xi , yj )Aij, (xi , yj ) Rij
R kpk0
i,j=1

lo que no es contradictorio sino que complementario y resultan planteamien-


tos equivalentes. Esto es la definicion utilizando el concepto de sumas inter-
medias de Riemann.

Teorema 4.1.1. Cualquier funcion continua definida en un rectangulo cer-


rado R es integrable

Demostracion:
La demostracion de este hecho no resulta de interes en este curso a pesar
de su enorme importancia, dejemos las cosas aqu a la imaginacion del
estudiante.

Proposicion 4.1.1. (Propiedades basicas de la Integral Doble)


De la definicion se desprende que:
1) Si f (x, y) = 1 todo (x, y) R,la integral resulta el area de la region
Z Z
A = Area de R = dA
R

2) Si f es integrable en R
Z Z Z Z
cf dA = c f dA
R R

3) Si f y g son funciones integrables en R


Z Z Z Z Z Z
(f + g)dA = f dA + gdA
R R R

4) Si f y g son funciones integrables en R y f (x, y) g(x, y) para todo


(x, y) R entonces Z Z Z Z
f dA gdA
R R

5) Si f es integrable sobre R, entonces |f | es integrable sobre R y


Z Z Z Z


f dA |f | dA
R R

319
Teorema 4.1.2. (Teorema del Valor Medio para Integrales Dobles)
Si f (x, y) es continua sobre rectangulo R con area A(R), entonces existe
un punto (, ) en el interior de R tal que
Z Z
f (x, y)dA = f (, ) A(R)
R

Demostracion:
Sea

m = mn {f (x, y) : (x, y) R}
M = max {f (x, y) : (x, y) R}

y supongamos que m < M. Entonces m f (x, y) M, y si f no


es identicamente igual a m o M , entonces
Z Z
m A(R) < f (x, y)dA < M A(R)
R

El teorema del valor medio de las funciones continuas asegura que existe
un
punto (, ) en el interior R tal que
Z Z 
f (, ) = f (x, y)dA A(R)
R

Z Z
= f (, ) A(R) = f (x, y)dA
R

4.1.2. Integrales sobre conjuntos acotados de R2


En este caso extenderemos la definicion de integral doble a regiones que
no son necesariamente rectangulos, sino que regiones acotadas en general.
Supongamos que S es una region cerrada y acotada de R2 , por ejemplo un
circulo, un triangulo, un rombo etc. , cualquier region con estas caractersticas
se puede poner dentro de un rectangulo R
Sea R rectangulo que contiene a region cerrada y acotada S y f una
funcion definida y acotada en S, extendemos f a R de la siguiente forma

f (x, y), (x, y) S
fR (x, y) =
0, (x, y) R S
fR la consideraremos como la extension de f a todo R.

320
Sea S una region acotada de R2 y f una funcion definida y acotada sobre
S, si R es en rectangulo tal que R R R S y fR la extension de f a R del tipo
definido aqu, entonces si existe f dA , definimos
R R
Z Z Z Z
f dA = fR dA
S R

Importante.

Las propiedades enunciadas, de la integral doble en rectangulos siguen


siendo validas en conjuntos mas generales lo que se puede justificar por la
definicion anterior

Integrales Iteradas
Una integral de la forma
Z bZ h(x)
f (x, y)dydx
a g(x)
Rb
se llama integral iterada y se interpreta como a F (x)dx donde para cada
R h(x)
x [a, b],con F (x) = g(x) f (x, y)dy.
Si f (x, y) es funcion continua sobre {(x, y) : a x b, g(x) y h(x)}
y
G(x, y) una primitiva en la segunda variable, de f (x, y), es decir ,
G(x, y)
= f (x, y) para cada x [a, b] y todo g(x) y h(x), el
y
teorema
Fundamental del calculo permite que
Z h(x)
y=h(x)
F (x) = f (x, y)dy = G(x, y) |y=g(x) = G(x, h(x)) G(x, g(x))
g(x)

Se puede interpretar entonces la integral iterada como un proceso sucesivo


de integracion as
Z b Z h(x) Z b Z h(x) !
f (x, y)dydx = f (x, y)dy dx
a g(x) a g(x)

De manera similar se tiene


Z d Z h(y) !
Z d Z h(y)
f (x, y)dxdy = f (x, y)dx dy
c g(y) c g(y)

321
Ejemplo.
Z 1 Z x Z 1 Z x 
2 2
(x + 4xy)dydx = (x + 4xy)dy dx
0 0 0 0
Z 1  2 x
= x y + 2xy 2 0 dx
0

Z 1 Z 1
1
3 3 3 3 4 3
= (x + 2x )dx = 3x dx = x =
0 0 4 0 4

Otros
R 4ejemplos:
R 2 Interprete y evalue:
i) 0 0 x ydxdy
R2R4
ii) 0 0 x ydydx
R1Rx
iii) 0 0 sin(x2 )dydx
R2R3
iv) 0 x (x2 + y) dydx
R 5 R x2
v) 0 2x (x + y)dydx

Algunas Respuestas
R4R2 R 4 hR 2 i R 4 h 2 i2
i) 0 0 x ydxdy = 0 0 x ydx dy = 0 x2 y dy =
0
R4 h
2 32
i4
32
= 0 2 ydy = 2 3 y = 3
R1Rx 0
iii) 0 0 sin(x2 )dydx = 1cos
2h
1
R2R3 i
iv) 0 x (x + y) dydx 16 1 72
2

A continuacion examinaremos la evaluacion de la integral doble por medio


de integrales iteradas.

4.1.3. Teorema de Fubini


Sea f una funcion continua en una region R cerrada y acotada, entonces
a) Si R = {(x, y) a x b, g1 (x) y g2 (x)} y g y son funciones
continuas en [a, b] se tiene:
Z Z Z bZ g2 (x)
f dA = f (x, y)dydx
R a g1 (x)

b) Si R = {(x, y) c y d, h1 (y) x h2 (y)} y h1 y h2 son

322
Figura 4.2: Region tipo a

Figura 4.3: Region tipo b

funciones continuas en [c, d] se tiene


Z Z Z d Z h2 (y)
f dA = f (x, y)dxdy
R c h1 (y)

Ejemplo

Este ejemplo ilustra como este teorema se adapta a la situacion del prob-
lema, en este caso se pide calcular
Z Z
xydA
R

y R es la region triangular del plano con vertices en los puntos A(-6,-2),


B(-1,3) y C(9,-7).
Solucion:

323
Figura 4.4: Region triangular del plano

La region se debe subdividir en dos subregiones del tipo (a)., tal como
lo muestra la figura siguiente
Los segmentos de recta AB, BC Y AC tienen ecuaciones y = x + 4,
y = x + 2 e y = 13 x 4 respectivemente. Las regiones pueden
escribirse.

I : 6 x 1, 31 x 4 y x + 4
II : 1 x 9, 13 x 4 y x + 2
Entonces aplicando T. de Fubini a ambas regiones se tiene

Z Z Z 1 Z x+4 Z 9 Z x+2
xydA = xydydx + xydydx
R 6 31 x4 1 13 x4

1 1  2 x+4 1 9  2 x+2
Z Z
= xy 1 x4 dx + xy 1 x4 dx
2 6 3 2 1 3
Z 1 Z 9
1 1
= (4x3 + 24x2 )dx + (4x3 30x2 54x)dx
9 6 9 1
1025
=
27
Se observa que R tambien puede subdividirse en dos regiones del tipo
(b) mediante una recta paralela al eje horizontal que pase por A.
Proposicion 4.1.2. Suponga que S es una region acotada y sea C una
curva la cual divide a S en dos subregiones S1 y S2 .
Si f es continua en S , lo es tambien en S1 y S2 , y
Z Z Z Z Z Z
f (x, y)dA = f (x, y)dA + f (x, y)dA
S S1 S2

324
Figura 4.5: Region triangular del plano, tipo b

Demostracion.- Directamente de la definicion eligiendo un rectangulo sufi-


cientemente grande que contenga a S y extendiendo f de S a R,
de S1 a R, S2 a R.

RR 2
Ejemplo. Calcular S
(x + y)dA, donde S es la region limitada por
la recta y = x y la curva y = x3

Figura 4.6: Region comprendida entre las curvas y = x y y = x3

325
Solucion.- En este caso

S1 = (x, y) R2 : 1 x 0, x y x3


S2 = (x, y) R2 : 0 x 1, x3 y x


Z Z Z Z Z Z
2 2
(x + y)dA = (x + y)dA + (x2 + y)dA
S S1 S2

Z Z Z 0 Z x3  3
2 x Z 0
y
(x2 + y)dA = (x2 + y)dydx = x2 y + dx
S1 1 x 1 2 x
Z 0
x6 x2
= [(x5 + ) (x3 + )]dx
1 2 2
 6 0
x7 x4 x3

x
= +
6 14 4 6 1
(1)6 (1)7 (1)4 (1)3
= + +
6 14 4 6
1
=
84

1 x 1 x
y2
Z Z Z Z Z 
2 2 2
(x + y)dA = (x + y)dydx = x y+ dx
S2 0 x3 0 2 x3
1
x2 6
Z
x
= ) (x5 + )]dx
[(x3 +
0 2 2
 4 1
x x3 x6 x7
= ( + )( + )
4 6 6 14 0
4 3 6 7
1 1 1 1
= ( + )( + )
4 6 6 14
5
=
28

Por lo tanto Z Z
1 5 1
(x2 + y)dA = + =
S 84 28 6

326
4.1.4. Areas y Volumenes
Area
Como se dijo en la introduccion y de acuerdo a la idea geometrica si R
es una region plana entonces el area de R se calcula con la integral doble.
Z Z
A(R) = dA
R

Ejemplo. Calcule el area de la region interior a la circunferencia x2 + y 2 =


2ax arriba de la parabola ay = x2 , a > 0.

Solucion:

Figura 4.7: Region en el plano xy, acotada por la circunferencia (x a)2 + y 2 = a2


x2
y la parabola y = a2
n 2 o
Sea R = (x, y) IR2 / 0 x a, xa y 2ax x2


Z Z Z a Z 2axx2
A = dA = dydx
x2
R 0 a
Z a x2
= ( 2ax x2 )dx
0 a
a2
= (3 4)
12

Volumen
Si R es una region plana, z = f (x, y), z = g(x, y) son dos superficies
tal que f (x, y) g(x, y) (x, y) R , el volumen entre ambas superficies

327
al interior de la region se puede calcular usando la siguiente integral doble

Z Z
V = [f (x, y) g(x, y)]dA
R

Ejemplo
1) Use la integral doble para determinar el volumen del tetraedro acotado
por los planos coordenados y el plano 3x + 6y + 4z 12 = 0.
Solucion: Para determinar la region R hacemos z = 0 y encontramos
su interseccion con el plano dado xy . As z = 0 = 3x + 6y = 12 = y =
21 x + 2
La region en el plano xy esta acotada por el eje x, el eje y y la recta
y = 12 x + 2, por lo tanto

Figura 4.8: Region en el plano xy, acotada por el x = 0, y = 0 y la recta y = x2 +2

Figura 4.9: Tetraedro

R = (x, y) IR2 / 0 x 4, 0 y 12 x + 2


328
12 3x 6y
z = f (x, y) = = 3 34 x 23 y y z = g(x, y) = 0
4
Z Z
V = f (x, y)dA
R
Z Z   Z 4 Z 1 x+2  
3 3 2 3 3
= 3 x y dA = 3 x y dydx =
R 4 2 0 0 4 2
1
Z 4  x+2 4
3y 2 2
  Z  
3 3 2 3
= 3 x y dx = x x + 3 dx
0 4 4 0 0 16 2
 3 4
3x2

x
= + 3x = 4
16 4 0

Otros ejemplos: RR
1) Usando integral iterada calcule s (x2 + 2y) dA, donde S: region
comprendida entre y = x2 e y = x
2) Calcule el volumen del solido limitado por los cilindros x2 + z 2 = 16 y
y 2 + z 2 = 16 .
Este es un interesante ejercicio, puede empezar por bosquejar el
solido o una parte de el, pues se puede aprovechar su simetra.

4.1.5. Cambio de variable


Un cambio de variables adecuado puede no solo simplificar el integrando
sino tambien la region donde se evalua la integral.
Sea f una funcion continua definida sobre la region R cerrada y acotada.
Considerese la integral doble
Z Z
f (x, y)dxdy
R
Definimos T , transformacion invertible

x = x(u, v), (u, v) S


y = y(u, v), (u, v) S
(x, y)
tal que 6= 0, la que produce una correspondencia biunvoca
(u, v)
entre R y S donde R es una region en xy y S es la nueva region en plano
uv. resultado de la transformacion T .
Si P es una particion definida en R la transformacion induce a su vez una
correspondiente particion en S de tal modo que si Rij es un subrectangulo

329
generado por la particion P en R, denotaremos por Sij el correspondiente
subrectangulo en S

Si 4 Aij = Area de Rij y 4 A0ij = Area de Sij


Se tiene la siguiente razon entre las areas.

4Aij (x, y) (x, y)
4 A0ij
= 4Aij
4A0ij (u, v) (u, v)
Entonces

(x, y)
f (x, y) 4 Axy f (x, y) 4 Auv
(u, v)
De esta relacion y la definicion de integral doble se tiene el siguiente
teorema de cambio de variable.

Teorema 4.1.3. Sea R una region en el plano xy acotado por una curva
simple cerrada y suave y que S es la imagen de R bajo la transformacion T
invertible, definida:

x = x(u, v), (u, v) S


y = y(u, v), (u, v) S

donde x(u, v), y(u, v) son continuamente diferenciables en un dominio que


contiene a S en cual
(x, y)
J 6= 0
(u, v)
Si f (x, y) define una funcion continua sobre R se tiene:
Z Z Z Z
(x, y)
f (x, y)dxdy = f (x (u, v) , y (u, v)) dudv
R S (u, v)

Ejemplo

Calcular Z Z
3xydA
R
Sea R la region limitada por las rectas x 2y = 0, x 2y = 4, x + y = 4,
x + y = 1.
Solucion:
Sea u = x + y, v = x 2y
Resolviendo el sistema lineal obtenemos

330
Figura 4.10: Aplicacion definida por una ecuacion vetorial

Figura 4.11: Aplicacion definida por una transformacion lineal

%
u=x+y x = 31 (2u + v)
=
v = x 2y y = 31 (u v)

Ademas el jacobiano de la transformacion es

(x, y) x x
2 1
2 1 1
u v = 13 3 1 = =
(u, v) = y
y
u v 3
3 9 9 3

Aplicando el teorema del cambio de variable , obtenemos

331
Z Z Z Z
1 1 1
3xydA = 3( (2u + v) (u v)) dA
R 3 3 3
Z SZ
1
= (2u + v) (u v) dA
9 S
1 4 0
Z Z
= (2u + v) (u v) dvdu
9 1 4
104
=
9

Ejemplo
Calcular
Z Z p
x2 + y 2 dA
R

R region del plano xy limitada por x2 + y 2 = 4, x2 + y 2 = 9

Figura 4.12: Region del plano xy limitada por x2 + y 2 = 4, x2 + y 2 = 9

Solucion:
Sea x = r cos , y = r sin el cambio de variable a polares
entonces
(x, y) x x cos r sin

r
(r, ) = y y = sin r cos = r

r

332
Z Z p Z Z
x2 + y 2 dA = r |r| drd
R S
Z 2 Z 3
= r2 drd
Z0 2 2
19 38
= d =
0 3 3

4.2. Aplicaciones de la integral doble

4.2.1. Masa de una region plana de densidad variable.

Sea (x, y) funcion positiva y definida sobre un conjunto cerrado y aco-


tado S con area no nula, que indica la densidad en cada punto (x, y) de S.

La masa de S es la integral de la funcion densidad.


Z Z
M (S)= (x, y)dA
S

En el caso que la densidad es constante = k, la masa es el producto del


area por la densidad: M (S) = k A(S).

Ejemplo.

Encuentre la masa de un crculo de radio a si su densidad es veces la


distancia al centro.
Solucion:

Con el uso de coordenadas polares el calculo de la integral resultante es


mas sencillo.

333
Figura 4.13:

2 a
2a3
Z Z p Z Z
M (S) = x2 + y 2 dydx = 2 dd =
S 0 0 3

4.2.2. Momentos y centroide de una region plana

Para un conjunto S acotado y de area positiva , y una funcion densidad


definida en S, tenemos las siguientes definiciones.

Primer momento con respecto al eje y:


Z Z
My = (x, y)xdA
S

Primer momento con respecto al eje x:


Z Z
Mx = (x, y)ydA
S

Segundo momento con respecto al eje y:


Z Z
Iy = (x, y)x2 dA
S

334
Segundo momento con respecto al eje x:
Z Z
Ix = (x, y)y 2 dA
S

Segundo momento Polar con respecto al origen:


Z Z
I0 = (x, y)(x2 + y 2 )dA
S

Centroide:  
My M x
(x, y) = ,
M M
Cuando la funcion densidad es variable y esta asociada con la distribucion
de la masa, los segundos momentos se llaman tambien momentos de inercia
y el centroide se le llama tambien centro de masas.
En una forma mas general, el primer y segundo momento de un conjunto
S se puede definir con respecto a una linea recta cualquiera L.
Z Z
ML = (x, y)D(x, y)dA
Z ZS
IL = (x, y) [D(x, y)]2 dA
S

Siendo D(x, y) la distancia de la recta L al punto (x,y).


Ejemplo.

Una lamina triangular tiene los vertices (0, 0), (1, 0) y (1, 2), y tiene
densidad (x, y) = x2 y. Halle su centro de masa.

Solucion:
En este caso R = {(x, y) R2 / 0 x 1, 0 y 2x}

Debemos calcular M = M (S), My y Mx :


Z 1 Z 2x 1 2x
x2 y 2
Z Z Z 
2 2
M = x ydA = x ydydx = dx
S 0 0 0 2 0
1 1
2x5
Z 
4 2
= M = 2x dx = =
0 5 0 5

335
Figura 4.14:

Calculemos ahora
Z Z Z 1 Z 2x
2
My = x yxdA = x3 ydydx
S 0 0
Z 1  6 1
2x
= 2x5 dx =
0 6 0
1
= My =
3

Por ultimo

Z Z Z 1 Z 2x
2
Mx = x yydA x2 y 2 dydx
S 0 0
Z 1  2 3 2x Z 1 5
xy 8x
= dx = dx
0 3 0 0 3
 6 1
8x 4
= Mx = =
18 0 9

As tenemos que
 
My M x 5 10
(x, y) = , =( , )
M M 6 9

336
4.3. Integrales triples
4.3.1. Ideas preliminares
Los conceptos a desarrollar en lo referente a la Integral en su genesis, su
significado y su calculo en el caso de tres variables es similar al mismo tema
en funciones de una variable y de dos variables. El tratamiento hecho en el
caso de la integral doble se extiende en forma natural a las integrales triples
en sus mecanismos conceptuales y metodos de calculo involucrados solo hay
un cambio en el escenario, el espacio R3 .
Atendiendo a la declaracion anterior no haremos el detalle de la gen-
eracion del concepto porque como ya lo dijimos se trata de una generalizacion.
Para mantener el marco de referencia, pensemos en sumas superiores, sumas
inferiores, integral superior e integral inferior para funciones definidas sobre
una caja rectangular de tipo:

h (x, y, z) : {a1 x b1 , a2 y b2 , a3 z b3 }

Con este trasfondo se plantea el teorema de Fubini que permitira el calculo


de la integral triple.

4.3.2. Teorema de Fubini


Si f (x, y, z) esta definida sobre una region R = {a1 x b1 , a2 y b2 , a3 z b3 } ,
entonces:

Z Z Z Z b1 Z b2 Z b3
f (x, y, z)dv = f (x, y, z)dzdydx
R a1 a2 a3

siempre que estas integrales existan.


Observacion: Hay otras cinco formas de calcular la integral triple de-
pendiendo del orden de integracion en la integral iterada.
Ejemplo:
RR R
Calcule R
(x2 + yz) dv, donde R = {0 x 2, 1 y 2, 1 z 3}

Solucion:

337
Z Z Z Z 2 Z 2 Z 3
2
x2 + yz dzdydx
 
x + yz dv =
R 0 1 1
2 2  3
z 2
Z Z
2
= x z+y dydx
0 1 2 1
Z 2 Z 2    
2 9 2 y
= 3x + y x + dydx
0 1 2 2
Z 2Z 2
4x2 + 4y dydx

=
0 1
Z 2
 2
= 4x2 y + 2y 2 1 dx
Z0 2
12x2 + 6 dx

=
0
2
= 4x3 + 6x 0
= 44

4.3.3. Teorema de la integral triple (Para dominios


mas generales)
Si f (x, y, z) esta definida sobre un conjunto acotado R formado por todos
los puntos tales que a1 x b1 , y1 (x) y y2 (x) y z1 (x, y) z
z2 (x, y)entonces:
Z Z Z Z b1 Z y2 (x) Z z2 (x)
f (x, y, z)dv = f (x, y, z)dzdydx
R a1 y1 (x) z1 (x)

siempre que ambas integrales existan.

Importante.
? Hay otras cinco formas de calcular la integral triple dependiendo el
orden de integracion para el calculo de la integral iterada.

? Si S es un conjunto acotado el cual tiene area en el plano XY y f (x, y, z)


es una funcion definida, acotada y no negativa sobre S, y si R es el conjunto
de todos los (x, y, z) tal que (x, y) S y 0 z f (x, y), entonces la region
R tiene volumen si y solo si f (x, y) es integrable sobre S y en tal caso

338
Z Z
V (R) = f (x, y)dA
S

? Si R es una rigion de R3 que tiene volumen, entonces

ZZ Z
V (R) = f (x, y, z)dv con f (x, y, z) = 1
R

Ejemplo

Sea R la region acotada por los paraboloides z = x2 + y 2 y


2z = 12 x2 y 2 . Usando integral triple calcule el volumen de R.
La representacion grafica de estas superficies es.

Figura 4.15: Region acotada por los paraboloides z = x2 + y2 y 2z = 12 x2 y2

Por separado, tenemos

Solucion:
La curva de interseccion es el crculo x2 + y 2 = 4, z = 4
Si f (x, y, z) = 1 se tiene el volumen

339
Figura 4.16:

ZZ Z
v(R) = f (x, y, z)dv
R
12x2 y 2
Z 2 Z 4x2 Z
2
=
dzdydx
2 4x2 x2 +y 2

Z 2Z 4x2 
4 x2 + y 2

= 6 dydx
0 0
Z 2  23
= 4 4 x2 dx
0
= 12

Ejemplo

Calcular el volumen de la region del espacio limitada por las superficies


clindricas x2 + z 2 = 1, y 2 + z 2 = 1
Solucion:
La representacion grafica de estas superficies es
Utilizaremos la simetra del problema y proyectaremos la region al plano
xz ( tambien se podra proyectar al plano yz ).
La proyeccion nos da un crculo de radio 1

340
Figura 4.17: Region acotada por los cilindros x2 + z 2 = 1, y 2 + z 2 = 1

La region se puede expresar:

1 x 1

1 z2 y 1 z2

1 x2 z 1 x2

Expresando el calculo del volumen como una integral triple tenemos


Z Z Z
V = dxdydz
R

usando integrales iteradas



Z 1 Z 1x2 Z 1z 2 Z 1 Z 1x2
V =
dydzdx =
2 1 z 2 dzdx
1 1x2 1z 2 1 1x2

Si seguimos por este camino llegamos a una expresion dficil de


resolver ( intentelo), recurriremos entonces al cambio de orden de
integracion que es un recurso siempre disponible

341

Z 1 Z 1x2 Z 1 Z 1z 2
V = 2
2 1 z dzdx = 2 1 z 2 dxdz

1 1x2 1 1z 2
Z 1 3 1

z 16
= 4(1 z 2 )dxdz = (4z 4 ) =
1 3 1 3

El volumen calculado es
16
V = unidades de volumen
3

4.3.4. Cambio de variable para integrales triples


Sea T : U R3 R3 una transformacion de clase C 1 definida por:

x = x (u, v, w)
y = y (u, v, w)
z = z (u, v, w)
Recordando el jacobiano de la transformacion se tiene:
x x x

(x, y, z) u
y
v
y
w
y
J= =
(u, v, w) u
z
v
z
w
z
u v w

Como en el caso anterior de dos variables, el jacobiano mide como la curva


la transformacion distorsiona su dominio.
Formula de cambio de variable para integrales triples
Sea R una region en el espacio xyz y S una region en el espacio uvw que
corresponde a R bajo la transformacion T definida por x = x (u, v, w) , y =
y (u, v, w) y z = z (u, v, w) siempre que T sea de clase C 1 y uno a uno,
(x,y,z)
(u,v,w)
6= 0 en S. Entonces:

ZZ Z ZZ Z
f (x, y, z)dv = f (x (u, v, w) , y (u, v, w) , z (u, v, w)) |J| dudvdw
R S

Donde
(x, y, z)
J=
(u, v, w)
Los cambios mas usados en integrales triples es a coordenadas cilindricas
y coordenadas esfericas dependiendo de la naturaleza del problema.

342
Coordenadas cilndricas
El cambio de variable es:
x = r cos
y = r sin
z=z

Figura 4.18: Cambio a coordenadas cilndricas

Supongamos que: P es un punto del espacio de coordenadas xyz, P1


proyeccion de P en plano xy, r radio vector de O a P1 y el pangulo entre

eje x y OP1 , medido del lado positivo del eje x entonces r = x2 + y 2 y
= arctan xy Tenemos


cos r sin 0
(x, y, z)
J = = sin r cos 0
(r, , z) 0 0 1
= r cos2 + r sin2 = r

ZZ Z ZZ Z
f (x, y, z)dv = f (r cos , r sin , z) r drddz
R S
Ejemplo:

Use coordenadas cilndricas para calcular el volumen del solido limitado


por el paraboloide z = x2 + y 2 y el plano z = 4.
Solucion:
En el espacio xyz, la figura es al interior del elipsoide y limitado por
arriba por el plano z = 4 que es un plano paralelo al plan xy

343
Figura 4.19: Solido limitado por el paraboloide z = x2 + y 2 y el plano z = 4

Aprovechando la simetra del solido calculamos la cuarta parte de el (por


conveniencia). En esta situacion la region transformada viene descrita por:
2
0 r 2, 0 ,r < z < 4
2
La descripcion de la region en las nuevas variables es fundamental para
el acertado planteamiento de la integral doble como integral iterada, en este
caso esta circunstancia es evidente


Z Z Z 2 Z 4
V 2
= rdzdrd
4 0 0 r2

Z
2
Z 2
4r r3 drd

=
0 0
Z
2
= 4d = 2
0

V = 8

Coordenadas Esfericas
El cambio de variable es:
x = cos sin
y = sin sin
z = cos

344
Figura 4.20: Cambio a coordenadas esfericas

Supongamos que P es un punto del espacio de coordenadas xyz


P1 proyeccion de P en plano xy

magnitud del radio vector OP

el angulo entre eje x y OP1 , medido del lado positivo del eje x

angulo formado por OP y el eje Z, medido del lado positivo del eje z.


cos sin sin sin cos
(x, y, z)
J = = sin sin cos sin 0
(, , )
cos cos sin cos sin
= 2 sin

4.3.5. Formula del cambio de variable

ZZ Z ZZ Z

f ( cos sin , sin sin , cos ) 2 sin drddz

f (x, y, z)dv =
R S

Las coordenadas esfericas se usan preferentemente en el caso en que uno


o ambas superficies que acotan la region de integracion es una esfera centrada
en el origen, esto se observa en los siguientes ejemplos.
Ejemplo 1

345
Figura 4.21: Solido limitado imitada inferiormente por el semicono z 2 = x2 +
y 2 , z 0 y superiormente por la esfera x2 + y 2 + z 2 = 9.

Hallar el volumen de la region solida limitada inferiormente por el semi-


cono z 2 = x2 + y 2 , z 0 y superiormente por la esfera x2 + y 2 + z 2 = 9.
Solucion:
Haciendo la interseccion de

x2 + y 2 + z 2 = 9 y z = x2 + y 2

resulta que la interseccion de estas superficies es una circunferencia en el


plano z = 32 definida por las ecuaciones

9 3
x2 + y 2 = , z =
2 2

esto permite visualizar que (0, 32 , 32 ) es un punto de la interseccion por lo


que 0 4 , la esfera tiene radio 3 por lo cual 0 3, y 0 2.
Como se esta calculando el volumen de una region que es simetrica respecto
del eje z, la cuarta parte de la region queda descrita por


0 3, 0 ,0
2 4
346
esto implica

ZZ Z Z Z Z 3
V 2 4
= f (x, y, z)dV = 2 sin ddd
4 R 0 0 0
Z Z
V 2 4
= 9 sin dd
4 0 0
  !
9 21
Z
V 2 9
= + 9 d =
4 0 2 2 2

Por lo tanto !
21
V = 18
2
Ejemplo 2

Utilice coordinadas
p esfericas para hallar el volumen del solido que esta ar-
riba del cono z = x2 + y 2 y debajo de la esfera x2 + y 2 + z 2 = z

p
Figura 4.22: Solido que esta arriba del cono z = x2 + y 2 y debajo de la esfera
x2 + y 2 + z 2 = z

Solucion:

1 1
x2 + y 2 + z 2 = z x2 + y 2 + (z )2 =
2 4
es una esfera que pasa por el origen y tiene centro en (0, 0, 12 ).
La ecuacion de la esfera en coordinadas esfericas es = cos ,a su vez de
la ecuacion del cono se infiere que 0 4 .

347
Por lo que la region en coordenadas esfericas esta descrita por

, 0 , 0 2, 0 cos
4
El volumen de la region es
ZZ Z Z 2 Z Z cos
4
V = f (x, y, z)dV = 2 sin ddd
R 0 0 0
Z 2 Z  3  cos
4
= sin dd
0 0 3 0
Z 
cos4 4

2 4 3 2
= sin cos d = . =
3 0 3 4 0
8
Por lo tanto

V =
8

4.3.6. Masa, Momentos, y Centroide de una Region


del Espacio

Como en el caso de dos dimensiones, si (x, y, z) funcion positiva y con-


tinua, definida sobre una region compacta (conjunto cerrado y acotado) W
con volumen, que indica la densidad en cada punto (x, y, z) de W.
La masa de W es dada por la integral de la funcion densidad.
Z Z Z
M (W )= (x, y, z)dV
W

El primer momento de W se define respecto de algun plano, y el segundo


momento (o momento de inercia) con respecto a algun plano, lnea o punto.
Daremos aqui solo las formulas tpicas planos coordenados, ejes y el ori-
gen.
Primer momento con respecto al plano yz.
Z Z Z
Myz = (x, y, z)xdV
W

Segundo momento con repecto al plano yz.


Z Z Z
Iyz = (x, y, z)x2 dV
W

348
Segundo momento con repecto al eje x
Z Z Z
Ix = (x, y, z)(y 2 + z 2 )dV
W

Segundo momento polar con respecto del origen


Z Z Z
I0 = (x, y, z)(x2 + y 2 + z 2 )dV
W

 
Myz Mzx Mxy
Centroide: ( x, y, z) = M
, M , M .
Ejemplo

Determinar el centroide de la porcion de la esfera x2 + y 2 + z 2 a2 , en


el primer octante, asumiendo densidad constante.
Solucion:
El problema no pierde generalidad si suponemos que = 1, y claramente
x = y = z.
Necesitamos calcular solamente
Z Z Z
Mxy = (x, y, z)zdV
W

y usando coordenadas esfericas esta integral queda



a
a4
Z Z Z
2 2
Mxy = ( cos )2 senddd =
0 0 0 16
3
Como V = a6 , el centroide es 3
a, 83 a, 38 a

8
Ejemplo

Encontrar el momento de inercia IL de un cilindro circular recto co a radio


de la base, h altura y densidad proporcional a la distancia al eje del cilindro,
con respecto a una recta L paralela al eje del cilindro y a una distancia b
de el.

349
Solucion:

La recta L se define por: x = b, y = 0

El cilindro es descrito por: 0 r a, 0 z h

La densidad es = kr

Entonces

Z Z Z
IL = (x, y, z)((x b)2 + y 2 )dV
W
Z 2 Z a Z h
= kr((x b)2 + y 2 )rdzdrd
0 0 0
Z 2 Z a Z h
= kr2 (r2 + b2 )dzdrd + 0
0 0
 02 
3 a b2
= 2ka h 5 + 3

4.4. Ejercicios resueltos integrales triples y


dobles

4.4.1. Calculo de integrales dobles en coordenadas rectangu-


lares cartesianas

Problema 1


ZZ
Calcular x + ydxdy si D es la region acotada por las respectivas
D
rectas y = x, y = x y x = 1
Solucion:
Se tiene que la region D = {(x, y) IR2 / 0 x 1; x y x}

350
Figura 4.23: D region acotada por y = x, y = x y x = 1

1 x

ZZ Z Z
x + ydxdy = x + ydydx
D 0 x
Z 1
2 x
= (x + y)3/2 dx

3 0 x
Z 1
2
= (2x)3/2 dx
3 0
25/2 2 1
5/2
= (x)
3 5 0
8 2
=
15

Problema 2
ZZ p
Calcular x2 y 2 dxdy si D es el dominio limitado por el triangulo
D
de vertices A (0, 0) , B(1, 1), C (1, 1) . Vease figura 4.23
Solucion
Entonces se tiene que el dominio esta delimitado por las rectas y = x,
y = x y x = 1.
Luego el dominio de integracion es:

D = (x, y) IR2 / 0 x 1; x y x


.
Integrando a franjas verticales, resulta

351
ZZ p Z 1p Z x
x2 y 2 dxdy = x2 y 2 dydx
D 0 x
Z 1Z x r  y 2
= x 1 dydx
0 x x
y
Hacemos el cambio de variables = sent = dy = x cos tdt y
x
determinemos los limites. 
x
Para y = x = arcsen = arcsen (1) = .
x  2
x
Para y = x = arcsen = arcsen (1) =
x 2
Por tanto



Z 1 Z x
r  y 2 Z 1 Z
2
x 1 dydx = x2 1 sen2 tdtdx
0 x x 0 2

Z 1Z 2
= x2 cos2 tdtdx
0 2

Z 1 Z
1 + cos 2t
2
= x2 (
)dtdx
0 2 2
Z 1  
2 t sen2t 2
= x + dx
0 2 4 2

1 2
Z
= x dx
2 0
 1
x3
= =
2 3 0 6

352
Problema 3

ZZ
y 2x2 dxdy si D es la region acotada por |x| + |y| = 2

Calcular
D

Figura 4.24: D region acotada por |x| + |y| = 2

Solucion:
Se tiene que la region D = {(x, y) IR2 / |x| + |y| 2}
Si escogemos la region con una particion de tipo I, es necesario utilizar
dos integrales iterativas porque para 2 x 0 , la frontera inferior de la
region es la grafica de y = x 2, y la superior es y = x + 2;y para 0 x 2
la frontera inferior de la region es la grafica de y = x 2, y la superior es
y = x + 2
Entonces se tiene D = D1 D2 tal que D1 D2 = .
donde D1 = {(x, y) IR2 / 2 x 0, x 2 y x + 2}
D2 = {(x, y) IR2 / 0 < x 2, x 2 y x + 2}
Por otra parte la funcion del integrando f (x, y) = y 2x2 es simetrica con
respecto al eje y, es decir (x, y, z) D existe (x, y, z) tal que f (x, y) =
y 2(x)2 = f (x, y) .
Por lo tanto

353
ZZ Z 2 Z x+2
2
y 2x2 dydx
 
y 2x dxdy = 2
D 0 x2
Z 2  2
 x+2
y 2

= 2 + 2x y dx
0 2 x2
Z 1
4x3 8x2 dx

= 2
0
  2
4 8 3
= x x
3
 0
64 32
= 2 16 =
3 3

Problema 4
ZZ
x2 + y 2 dxdy si D = {(x, y) IR2 / x2 + y 2 1} .Usando

Calcular
D
coordenadas cartesianas

Figura 4.25: D region acotada por x2 + y 2 1

Solucion:
Usando coordenadas cartesianas, la region de integracion es un crculo
centrado en el origen de radio uno
Por lo tanto

D = (x, y) IR2 / 1 x 1, 1 x2 y 1 x2

354

ZZ Z 1 Z 1x2
2 2
(x2 + y 2 )dydx

x +y dxdy =
D 1 1x2
Z 1
2
3 1x
y
= (x2 y + ) dx
1 3 1x2
Z 1
1p
= 2 (x2 1 x2 + (1 x2 )3 )dx
1 3
Z 1 Z 1p
2 2
= 2 x 1 x2 dx + (1 x2 )3 dx
1 3 1

Con ayuda de una tabla de integrales obtenemos que:

1
Z 1
2
x 1
x 1 x2 dx = ( 2 2
1 x + (x 1 x + arcsenx)
1 4 8 1
1 1
= (arcsen(1) arcsen (1) = ( + ) =
8 8 2 2 8

Z 1
1
p x p 3x p 3
(1 x2 )3 dx = ( (1 x2 )3 + (1 x2 ) + arcsenx)
1 4 8 8 1
3
=
8
Por lo tanto:
ZZ
2 2 3
x2 + y 2 dxdy =

+ =
D 8 3 8 2
Notese que la solucion del problema usando coordenadas cartesianas es
bastante compleja

Problema 5

ZZ
Calcular xydxdy si D es la region acotada por y = x, y =
D
3x 18, y 0.Usando coordenadas cartesianas.
Solucion.
Si escogemos la region con una particion de tipo I, es necesario utilizar
dos integrales iterativas porque para 0 x 6 , la frontera inferior de la
region es la grafica de y = 0, y la superior es y = x;y para 6 x 9 la

355

Figura 4.26: D y = x, y = 3x 18, y 0. Region de tipo I


frontera
inferior de la region es la grafica de y = 3x 18, y la superior es
y= x
Luego tenemos que D = D1 D2 tal que D1 D2 = .

Entonces D1 = {(x, y) IR2 / 0 x 6, 0 y x}

D2 = (x, y) IR2 / 6 < x 9, 3x 18 y x
Por lo tanto

ZZ ZZ ZZ
xydxdy = xydxdy + xydxdy
D D1 D2

Z 6Z x Z 9 Z x
= xydydx +
xydydx
0 0 6 3x18
6  x 9  2  x
y2
Z  Z
y
= x dx + x
dx
0 2 0 6 2 3x18
1 6 2 1 9
Z Z
= x dx + (2x2 + 18x)dx
2 0 2 6
6  3 9
x2

1 3 x 185
= x + +9 =
6 0 3 2 6 2

Si escogemos la region con una particion de tipo II, es necesario utilizar


solo una integral iterativa porque para 0 y 3 , la frontera izquierda
de la region es la grafica de x = y 2 mentras que la frontera derecha queda
y2
determinada por la grafica x = + 6, obteniendo as la region
3
y2
 
2 2
D1 = (x, y) IR / y x + 6, 0 y 3
3

356

Figura 4.27: D y = x, y = 3x 18, y 0. Region de tipo II

la integral iterativa queda


ZZ Z 3 Z (y 2 /3)+6
xydxdy = xydxdy
D 0 y2
3 (y2 /3)+6
x2
Z 
= ydy
0 2 y2
Z " 2 2 #(y2 /3)+6
1 3 y + 18
= y4 ydy
2 0 3 2
y
Z 3
1
8y 5 + 36y 3 + 324y dy
 
=
18 0
 3
1 4 6 4 2
= y + 9y + 162y
18 3
  0
1 4 185
= 36 + 36 + 2 36 =
18 3 2

357
Problema 6

Encontrar el area de la region determinada por las desigualdades: xy 4,


y x, 27y 4x2 .

Figura 4.28: Region determinada por xy 4, y x, 27y 4x2

Solucion.
Sabemos que xy = 4 tiene por grafica una hiperbola equilatera, y = x
es la recta bisectriz del primer cuadrante y 27y = 4x2 corresponde a una
parabola. Veamos cuale son los puntos de interseccion de estas curvas con el
proprosito deconfigurar el dominio de integracion
xy = 4
= x2 = 4 = x = 2 = y = 2
y=x
)
27y = 4x 2
 x = 0 27
= 27x = 4x2 = 27 = y = 0, y =
y=x x= 4
 4
xy = 4 4
2 = x = 3, y =
27y = 4x 3
ZZ
Para calcular el area A(R) = dxdy, podemos escoger una particion
D
del
dominio de tipo I o de tipo II.
Consideremos
 dos subregiones de tipo I 
2 4
D1 = (x, y) IR / 2 x 3, y x
 x 
2 27 4 2
D2 = (x, y) IR / 3 x , x y x
4 27

358
Si proyectamos sobre eje x
ZZ ZZ ZZ
A(R) = dxdy = dxdy + dxdy
D D1 D2

Z 3 Z x Z 27/4 Z x
A(R) = dydx + dydx
4 4 2
2 x
3 27
x
Z 3 Z 27/4
x
= y| 4 dx + y|x4 x2 dx
x 27
2 3
Z 3  Z 27/4  
4 4 2
= x dx + x x dx
2 x 3 27
 2 3  2 27/4
x x 4
= 4 ln x + x3
2 2 2 81 3
5 3 729 9 4 273 4
= 4 ln + 3
+ 33
2 2 32 2 81 4 81
3 729 243 4
= 2 4 ln + +
2 32 16 3
665 3
= 4 ln
96 2
Si proyectamos sobre eje y
3
 
2 4 4
DI = (x, y) IR / x 3y, y 2
y 2 3

 
2 3 27
DI = (x, y) IR / y x 3y, 2 y
2 4
ZZ ZZ ZZ
A(R) = dxdy = dxdy + dxdy
D D1 D2

3 3
Z 2 Z
2
3y Z 27/4 Z
2
3y
A(R) = dxdy + dxdy
4 4
3 y
2 y
Z 2 Z 27/4  
hp i 3p
= 3y 4 ln y dy + 3y y dy
4
3
2 2
2 27/4
 p 2 p 
3 4 y
= 3y 3 + 3y 3
2 y 4 2 2
3
8 3 9 27 729
= 4 ln + +2
3 2 8 32
665 3
= 4 ln
96 2

359
Problema 7

Encontrar el volumen de la region acotada por los tres planos coordenados


y el plano x + 2y + 3z = 6 Solucion.

Figura 4.29: Region determinada por los tres planos coordenados y el plano x +
2y + 3z = 6

Usando integrales dobles y proyectando la region sobre el plano xy ten-


emos:  
RR 6 x 2y 2 6x
V = D dxdy , D = (x, y) IR / 0 x 6, 0 y
3 2

Z Z 6x
1 6 2
V = (6 x 2y) dydx
3 0 0
1 6
Z
 6x
= (6 x)y y 2 0 2 dx
3 0
1 6 (6 x)2 (6 x)2
Z  
= dx
3 0 2 4
Z 6
1
= (6 x)2 dx
12 0
 6
1 3
= (6 x) =6
36 0

Usando integrales dobles y proyectando la region sobre el plano yz ten-


emos:  
RR 2 6 2y
V = R (6 3z 2y) dzdy , R = (y, z) IR / 0 y 3, 0 z
3

360
62y
Z 3 Z
3
V = (6 2y 3z) dzdy
0 0
Z 3
  62y
3 2 3
= (6 2y)z z dy
0 2 0
Z 3
(6 2y)2 (6 2y)2

= dy
0 3 6
1 3
Z
= (6 2y)2 dy
6 0
3
1 (6 x)3

= =6
12 3 0

4.4.2. Cambios de orden de Integracion


Problema 1
Invierta el orden de integracion y evalue la integral resultante .
Z 1 Z 2
2
I= ey dydx
0 2x

Solucion.
El dominio de integracion dado es D = {(x, y) IR2 / 0 x 1, 2x y 2} .
Si se invierte el orden de integracion tenemos que modificar la particion
del n y o
dominio. D = (x, y) IR2 / 0 x , 0 y 2 ,entonces la inte-
2
gral
se puede escribir.

y
Z 1 Z 2 Z 2 Z
2
y2 2
I = e dydx = ey dxdy
Z0 2 2x
y
0 0
2y2
= xe dy
0
Z0 2
y y2 1 2 4
= e dy = ey
0 2 4 0
1 16 
= e 1
4

361
Problema 2
Invierta el orden de integracion y evalue la integral resultante .
Z 2Z 4

I= y cos ydydx
0 x2

Solucion.
El dominio de integracion dado es D = {(x, y) IR2 / 0 x 2, x2 y 4} .
Si se invierte el orden de integracion tenemos que modificar la particion
del 
dominio, D = (x, y) IR2 / 0 x y, 0 y 4 ,entonces la inte-
gral
se puede escribir


2 4 4 y

Z Z Z Z
y cos ydydx = y cos ydxdy
0 x2 0 0
4

Z
= y cos(y)x|0 y dy
Z0 4
= y cos(y)dy
0

Integrando esta ultima integral por partes se tiene:

Z 4 Z 4
y cos(y)dy = ysen(y)|40 sen(y)dy
0 0
= ysen(y)|40 + cos(y)|40
= 4sen(4) + cos(4) 1

Problema 3
Invierta el orden de integracion y evalue la integral resultante .
Z eZ ln x
I= ydydx
1 0

Solucion.
El dominio de integracion dado es D = {(x, y) IR2 / 1 x e, 0 y ln x} .
Si se invierte el orden de integracion tenemos que el dominio,
D = {(x, y) IR2 / ey x e, 0 y 1} ,entonces la integral
se puede escribir

362
Z eZ ln x Z 1 Z e
ydydx = ydxdy
1 0 0 ey
Z 4 e
= y x dy

y e
Z0 4
= y(e ey )dy
0
 2 4
y
= e ey [y ey ]40
2 0
= 8e 4e4 1

4.4.3. Cambios de variables: Coordenadas polares


Problema 1
ZZ
x2 + y 2 dxdy si D = {(x, y) IR2 / x2 + y 2 1} ,usando

Calcular
D
coordenadas polares. Vease figura 4.25
Solucion.
A partir de la coordenadas polares tenemos:
x = rcos, y = rsen = x2 + y 2 = r2
El
valor absoluto del Jacobiano de transformacion a polares es:
(x, y)
(r, ) = r

Reemplazando terminos en la integral, produce


ZZ ZZ
2 (x, y)
2 2

x + y dxdy = r drd
D D (r, )

Z 1 Z 2 Z 1 Z 2
= 3
r ddr = r3 |2
0 dr
0 0 0 0
1 4 1
Z
r
= 2 r3 dr = 2 =
0 4 0
2

Las coordenadas polares dieron una solucion mas simple del problema.
La simplicidad depende de la naturaleza del problema y de la simetria que
presenta el dominio.

363
Problema 2

Calcular el area de la region interior a la circunferencia x2 + y 2 = 8y y


exterior a la circunferencia x2 + y 2 = 9.
Solucion.
Determinemos el centro y radio de la circunsferencia
x2 + y 2 = 8y = x2 + y 2 8y = 0 = x2 + (y 4)2 = 16
ZZ
El area de la region D es: A (D) dxdy Vease figura 4.30
D

Figura 4.30: Region interior a la circunferencia x2 + y 2 = 8y y exterior a la


circunferencia x2 + y 2 = 9

Por simetra, podemos calcular el area de la region D en el primer cuad-


rante
y multiplicar por 2.
A fin de conocer los lmites de integracion en coordenadas polares
necesitamos conocer el angulo que forma la recta OT con el eje x.
x2 + y 2 = 8y = r2 = 8rsen = r = 8sen
x2 + y 2 = 9 = r = 3
Como T pertenece a ambas circunferencias se cumple
3
8sen = 3 = = arcsen
8  
3
Luego, la mitad de la region D = (r, ) /3 r 8sen; arcsen
8 2
ZZ ZZ
(x, y)
dxdy = drd
D (r, )

D

364
/2 8sen /2
8sen
r2
Z Z Z
2 rdrd = 2 d
arcsen 38 arcsen 38 2 3

3
Z /2    /2
sen2 9
64sen2 9 d =

64
arcsen 38 2 4 2 arcsen 3
8
 /2
55
= 16sen2
2 arcsen 83
 
55 55 3 3
= arcsen + 16sen(2arcsen )
4 2 8 8
38, 42

Problema 3
x2 + y 2
ZZ
Calcular p dxdy , si D es el interior del cardioide r =
D x + x2 + y 2
a (1 + cos )

Figura 4.31: D es la region interior del cardioide r = a (1 + cos )

Solucion.
Cambiando a cordenadas polares, tenemos:

x2 + y 2 r2
ZZ ZZ
(x, y)
p dxdy = drd
D x + x2 + y 2 D r cos + r (r, )
r2
ZZ
= rdrd
D r cos + r

365
2 a(1+cos )
r2
Z Z
= drd
0 0 1 + cos
2
a(1+cos )
r3
Z
1
= d
0 1 + cos 3 0
Z 2
3
a
= (1 + cos )2 d
3 0
a3 2
Z
1 + 2 cos + cos2 d

=
3 0
2
a3

sen2
= + 2sen + +
3 2 4 0
3
= a

Observacion si deseamos se rigurosos debemos hacer notar que la integral


es
impropia cuando x 0, e y = 0, pues en tal caso el denominador es cero.
Luego:

a(1+cos ) Z 2 Z a(1+cos )
r2 r2
Z Z
I = lm drd + lm drd
0 1 + cos + 1 + cos
0 0
3 Z 3 Z 2
a a
= lm (1 + cos )2 d + lm (1 + cos )2 d
3 0 + 3
3
a3
   
a 3 sen2 3 sen2
= lm + 2sen + + lm 3 2sen
3 2 4 + 3 2 4
3
= a

Problema 4
Calcular el volumen V del solido acotado por las graficas z = 9 x2 y 2
y z = 5.
Solucion.
Como el solido es simetrico, basta encontrar su volumen en el primer
octante y multiplicar su resultado por cuatro.
Usando integrales dobles y proyectando la region sobre el plano xy ten-
emos: Z Z
9 x2 y 2 5 dxdy
 
V =4
D

366
Figura 4.32: solido acotado por las graficas z = 9 x2 y 2 y z = 5

D = {(x, y) IR2 / x 0, y 0, 0 x2 + y 2 4}
A partir de la  coordenadas polares, obtenemos:
x = rcos
= f (x, y) = 4 x2 y 2 = 4 r2
y = rsen

0 x2 + y 2 = r2 4 0 r 2 y 0
2

n o
D = (r, ) / 0 r 2, 0
2
El
valor absoluto
del Jacobiano de transformacion a polares es:
(x, y)
(r, ) = r

Reemplazando terminos en la integral, produce:


Z Z
4 r2 rdrd
 
V = 4
D
Z /2 Z 2
4 r2 rdrd
 
= 4
0 0
Z /2  2
4 2 1 4
= 4 r r d
0 2 4 0
= 8

4.4.4. Cambios de variables. Coordenadas curvilneas


Problema 1
ZZ
Calcular I = 3xydxdy, donde D es la region acotada por las rectas
D
x 2y = 0, x 2y = 4
(1)
x + y = 4, x+y =1
Solucion.
Podemos usar el cambio de variables

367
Figura 4.33: D region acotada por x 2y = 0, x 2y = 4, x + y = 4,
x+y =1

1
u = x 2y
 x= (2u + v)
(1) = 3 (2)
v =x+y 1
y = (u v)
3
Asi,x 2y = 4 se transforma en u = 4

x 2y = 0 se transforma en u = 0

x + y = 1 se transforma en v = 1

x + y = 4 se transforma en v = 4

(x, y)
Para calcular el Jacobiano tenemos dos posibilidades.
(u, v)

La primera, es usar la transformacion inversa (2) x e y en terminos de u


yv.

(u, v)
La segunda, mucho mas simple, es calcular a partir de (1) y
(x, y)
luego
(u, v) 1
 
(x, y)
usar la propiedad = .
(u, v) (x, y)

(u, v) 1 2 (x, y) 1
En efecto = = 1 + 2 = 3 = =
(x, y) 1 1 (u, v) 3

Por lo tanto, del teorema del cambio e variables se deduce que:

368
ZZ ZZ  
1 1 (x, y)
I = 3xydxdy = 3 (2u + v) (u v) dudv
D D 3 3 (u, v)
Z 4Z 0
1
2u2 uv v 2 dvdu

=
1 4 9
Z 4 0
1 2 uv 2 v 3
= 2u v du
9 1 2 3 4
1 4
Z  
2 64
= 8u + 8u du
9 1 3
4
1 8u3

2 64 164
= + 4u u du =
9 3 3 1 9

Problema 2
Calcular el area de la region D, que esta acotada por las curvas
x2 y 2 = 1, x2 y 2 = 9
(1)
x + y = 4, x+y =6

Figura 4.34: D region acotada por x2 y 2 = 1, x2 y 2 = 9, x+y = 4, x+y = 6

Solucion.
Teniendo en cuenta el cambio de variables que transforma la region D en
la region D 
u = x2 y 2
(1) =
v =x+y
La imagen D de la region D esta acotada por la rectas verticales;
x2 y 2 = 1 se transforma en u = 1
x2 y 2 = 9 se transforma en u = 9
y las rectas horizontales
x + y = 4 se transforma en v = 4

369
x + y = 6 se transforma en v = 6
Es decir, D = {(u, v) /1 u 9, 4 v 6}
(x, y) (u, v)
Vamos a calcular a partir de (1)
y usar la propiedad
(u, v) (x, y)
 1
(x, y)
= (u, v)


(u, v) (x, y) .

(u, v) 2x 2y (x, y)
En efecto = = 2 (x + y) = 2v = = 1
(x, y) 1 1 (u, v) 2v
El teorema del cambio variables afirma que:

ZZ ZZ
(x, y)
A (D) = dxdy = dudv
D (u, v)

D
Z 9Z 6
1
= dvdu
1 4 3v
Z 9
1
= [ln v]64 du
2 1
 Z 9
1 6
= ln du
2 4 1
1 3 9 3
= ln [u]1 = 4 ln
2 2 2

Problema 3
x3 + y 3
ZZ
Calcular I = dxdy, donde D es la region del primer cuad-
D xy
rante acotada por:
y = x2 , y = 4x2
(1)
x = y 2 , x = 4y 2
Solucion.
El calculo de I sera bastante complejo si usamos coordenadas cartesianas
por la simetra que tiene el dominio.Sin embargo, una cambio de variables
simplifica la region D y la transforma en D .
x2 y2
Sean u = , v =
y x
Luego D esta acotada por la rectas verticales;
y = x2 se transforma en u = 1.
1
y = 4x2 se transforma en u = .
4
y las rectas horizontales
x = y 2 se transforma en v = 1.

370
Figura 4.35: D region acotada por y = x2 , y = 4x2 , x = y 2 , x = 4y 2

1
x = 4y 2 se transforma en v = .
4

 
1 1
Es decir, D = (u, v) /1 u , 1 v
4 4


(x, y)
Para calcular tenemos dos posibilidades, la primera es despejar
(u, v)
x

e y en terminos de u y v a partir de (1) .

(u, v) 1
 
(u, v) (x, y)
La segunda, es calcular y usar la propiedad
(u, v) = (x, y)
.
(x, y)

2
2x x


(u, v) 2 (x, y) 1
En efecto = y y = 4 1 = 3 =
(u, v) = 3

(x, y) y 2 2y
2
x x

Calculemos ahora la integral

371
x3 + y 3
ZZ  2
y2
ZZ 
x
I = dxdy = + dxdy
D xy D y x
Z 1 Z 1
1
= (u + v) dvdu
1/4 1/4 3
Z 1  1
v2

1
= uv + du
3 1/4 2 1/4
1 1 3
Z  
15
= u+ du
3 1/4 4 32
 1  
1 3 2 15 1 3 15 15 3
= u + u = +
3 8 32 1/4 3 8 16 32 4
15
=
64

Problema 4
ZZ
Evaluar la integral I = [x + y]2 dxdy, donde D es la region del plano
D
xy acotado por las curvas
x + y = 2, x + y = 4,
(1)
y = x, x2 y 2 = 4,

Figura 4.36: D region acotada por x + y = 2, x + y = 4, y = x y x2 y 2 = 4

Solucion. Observese que las ecuaciones de la curvas de la frontera de


D solo incluyen a x e y en las combinaciones de x y,y el integrando incluye
solamentenlas mismas combinaciones. Aprovechando estas simetras, sean las
coordenadas
u = x + y, v = x y
Luego, la imagen D de la region D esta acotada por las curvas;

372
x + y = 2 se transforma en u = 2.
x + y = 4 se transforma en u = 4.
A su vez
x y = 0 se transforma en v = 0.
x2 y 2 = (x + y)
 (x y) = 4 se transforma en  uv = 4.
4
Es decir, D = (u, v) / 2 u 4, 0 v
u
(u, v) 1
 
(x, y)
El jacobiano de la transformacion es

= (x, y)
.
(u,
v)
(u, v) 1 1 (x, y) 1
En efecto = = 2 = (u, v) = 2

(x, y) 1 1
Entonces:
ZZ ZZ
2 1
[x + y] dxdy = u2 dudv
D 2 D
1 4 4/u 2
Z Z
= u dvdu
2 2 0
1 4 2 4/u
Z
= u v|0 du
2 2
1 4
Z
= 4udu
2 2
4
4 u2
= = 12
2 2 2

4.4.5. Calculo de integrales triples en coordenadas rectangu-


lares cartesianas
Problema 1
1
Sea R la region en IR3 acotada por: z = 0, z = y, x = 0; x = 1, y =
2
0, y = 2 ZZZ
Calcular (x + y z) dxdydz.
R

Solucion.
Del grafico de la region (Vease figura 4.37) , tenemos que 0 z
1
y.Proyectando la region R
2
sobre el plano xy. As D = {(x, y) IR2 / 0 x 1, 0 y 2} .
Por lo tanto;

373
1
Figura 4.37: R region acotada por z = 0, z = y, x = 0; x = 1, y = 0, y = 2
2

1
ZZZ ZZ Z
2
y
(x + y z) dxdydz = ( (x + y z) dz)dxdy
R D 0

1 2 1
y 1 2 1y
z2 2
Z Z Z Z  Z
2
( (x + y z) dz)dydx = xz + yz dydx
0 0 0 0 0 2 0
1Z 2 Z 1Z 2
y2
Z  
1 1 3 2
(x + y)y dydx = xy + y dydx
0 0 2 8 0 0 2 8
Z 1 2 Z 1  1
1 2 1 3 1 2
xy + y dx = [(x + 1)] dx = x + x
0 4 8 0 0 2 0
3
=
2

Tambien es posible resolver el problema anterior proyectando la region R


sobre el plano xz.En tal caso, 2z y 2 y
D = {(x, z) IR2 / 0 x 1, 0 z 1}

ZZZ Z 1 Z 1 Z 2
(x + y z) dxdydz = ( (x + y z) dy)dzdx
R 0 0 2z

374
1 1 2 1 1
y2
Z Z  Z Z
xy + zy dzdx = 2 [x + 1 z xz] dzdx
0 0 2 2z 0 0
1 2 1 Z 1
z2
Z   
z 1 x
2 xz + z x dx = 2 x+1 dx
0 2 2 0 0 2 2
Z 1  1
1 2 3
[(x + 1)] dx = x +x =
0 2 0 2

Una tercera posibilidad de solucion consiste en proyectar la region R


sobre el plano yz.
Esta se deja como ejercicio.

Problema 2
ZZZ
Calcular x2 dxdydz si D es la region acotada por y 2 + z 2 = 4ax,
D
y 2 = ax, x = 3a

Figura 4.38: D region acotada por y 2 + z 2 = 4ax, y 2 = ax, x = 3a

Solucion.
La superficie y 2 + z 2 = 4ax corresponde a un paraboloide de revolucion
como el bosquejado en la figura.
En dos variables el grafico de y 2 = ax es una parabola, pero es tres
variables es la superficie de un manto parabolico.
Finalmente, el grafico x = 3 es un plano paralelo al plano xz a la distancia
3a.
Luego el grafico de la region es
La proyeccion
n de la region sobre el plano xy es: o
p p
3 2 2
D = (x, y, z) IR /D1 D2 , 4ax y z 4ax y

375
Por simetra se tiene:

ZZZ ZZ Z 4axy2
I = x2 dxdydz = 2 x2 dzdxdy
D D1 4axy 2
Z 3a Z

2 ax Z 4axy2
= 2 x2 dzdydx
0 ax 4axy 2

3a 2 ax  2 4axy2
Z Z
= 2
xz 2
dydx
0 ax 4axy

Z 3a Z 2 ax p
= 4
x2 4ax y 2 dydx
0 ax

De una tabla de integrales obtenemos


Z
1 u
a2 u2 du = (u a2 u2 + a2 arcsen )
2 a
As al integrar la expresion:
Z 2ax p   p 2ax
1 y

4ax y 2 dy = y 4ax y 2 + 4ax arcsen
ax 2 2 ax ax
1
 
1
= 2ax arcsen (1) ax 3ax + 4ax arcsen
2 2
1
= 2ax + ax 3 2ax
2 2 6
2 3
= ax + ax
3 2
Por lo tanto al sustituir en la integral anterior, queda

Z 3a
" # " ! #3a
2 3 2 3
4 + ax3 dx = + ax4
0 3 2 3 2
0
!
3 3
= 27a5 2 +
2

Problema 3

Calcular el volumen del solido acotado por la superficie y = x2 y los


planos y + z = 4 ; z = 0.

376
Figura 4.39: acotado por la superficie y = x2 y los planos y + z = 4, z = 0

Solucion.
Consideremos que la region esta acotada inferiormente por la frontera
z = 0 y superiomente por z = 4 y.
Si Proyectamos la region sobre el plano xy, se tiene:
= {(x, y, z) IR3 / (x, y) D, 0 z 4 y}
D = {(x, y) IR2 / 2 x 2, x2 y 4}
Luego el volumen de la region es

ZZZ Z 2 Z 4 Z 4y
V () = dxdydz = dzdydx
2 x2 0
2 4 4 2
y2
Z Z  Z
= (4 y) dydx = 4y dx
2 x2 2 2 x2
Z 2
x4

2
= 8 4x + dx
2 2
2
4 3 x4

256
= 8x x + =
3 10 2 15

4.4.6. Coordenadas esfericas


Problema 1
ZZZ p
x2 + y 2 + z 2 e(x +y +z ) dxdydz si D es la region de
2 2 2
Resolver I =
D
IR3 limitada por las superficies x2 + y 2 + z 2 = a2
x2 + y 2 + z 2 = b2 con 0 < b < a anillo esferico.
Solucion.
Por la simetra del dominio y la forma del integrando

377
Figura 4.40: D es la region de IR3 limitada por las superficies x2 + y 2 + z 2 = a2
x2 + y 2 + z 2 = b2 con 0 < b < a

usaremos coordenadas esfericas:


b 2 x 2 + y 2 + z 2 a2 = bra
x = rsen cos y
y = rsensen = tg = = 0 = 0
zy
z = r cos

tg = = 0 = 0 2
x
Recordando que el valor absoluto del Jacobiano a esfericas es :

(x, y, z) 2
(r, , ) = r sen se tiene:

2 a
Z Z Z
r2
(x, y, z)
I = re (r, , ) drdd

0 0 b
Z 2 Z Z a
2
= r3 er sen drdd
Z0 2 Z0 b a
1 2 r2 r2
= r e e sen dd
0 0 2 b
  Z 2 Z
1 2 b2 1 b2 1 2 a2 a2
= be + e ae e sen dd
2 2 2 0 0
  Z 2
1 2 b2 1 b2 1 2 a2 a2
= be + e ae e cos |0 d
2 2 2
  0Z 2
1 2 b2 1 b2 1 2 a2 2
= 2 be + e ae ea d
2 2 2 0
 
1 2 b2 1 b2 1 2 a2 a2
= 4 be + e ae e
2 2 2

378
Problema 2

Encontrar el volumen de la region determinada por x2 + y 2 + z 2 16 , z 2


x2 + y 2 .

Figura 4.41: Region determinada por x2 + y 2 + z 2 16 , z 2 x2 + y 2

Solucion.
x2 + y 2 + z 2 = 16 es una esfera con centro en el origen y radio 4
z 2 = x2 +y 2 es un cono con vertice en el origen y eje de simetra coincidente
con el eje z.
Como z 0 , solo debemos considerar solo la region sobre el plano xy.
La interseccion de laesfera con el cono
se obtiene mediante el sistema:
x2 + y 2 + z 2 = 16 z= 8
=
x2 + y 2 = z 2 x2 + y 2 = 8

Usaremos coordenadas esfericas:


0 x2 + y 2 +z 2 16 = 0r4
x = rsen cos y 8
y = rsensen = tg = = = 1 = 0
z 8 4
z = r cos
y
tg = = 0 = 0 2
x
Recordando que el valor absoluto del Jacobiano a esfericas es :

(x, y, z) 2
(r, , ) = r sen se tiene:

379

ZZZ Z 2 Z
4
Z 4
V = dxdydz = r2 sen drdd
D 0 0 0
Z 2 Z 3 4
4 r
V = sen dd
0 0 3 0
43 2
Z
V = cos |04 d
3 0
! !
43 2 43
Z
2 2
V = 1 d = 1 2
3 0 2 3 2

Otra opcion para resolver este problema es usar coordenadas cilndricas,en


tal caso
x = r cos x2 + y 2 + z 2 = 16 = z = 16 r2 .
2 2 2
y = rsen = x +y =z = z = r2
z=z

Tenamos que el Jacobiano de transformacion a cilndricas es:
(x, y, z)
(r, , z) = r luego:


ZZZ Z 2 Z 8 Z 16r2
V = dxdydz = rdzdrd
D 0 0 r2

Z 2 Z 8
2
= rz|r216r drd
0 0

Z 2 Z 8  
2 2
= r 16 r r drd
0 0
Z 2  3
 8
1p r
= (16 r2 )3 d
0 3 3 0
2  3 3  2  
= 2 8 16 = 64 32 2
3 3

4.4.7. Coordenadas Cilndricas


Problema 1
Usando integrales triples calcular el volumen de la region acotada por
z = x2 + y 2 y z = 27 2x2 2y 2 .
Solucion.
Por la simetra del volumen los resolveremos usando coordenadas cilndri-
cas.

380
Figura 4.42: Region acotada por z = x2 + y 2 y z = 27 2x2 2y 2

x = r cos z = x2 + y 2 = z = r2 .
y = rsen = z = 27 2x2 2y 2 = z = 27 2r2
z=z x2 + y 2 = 9 = r = 3.

Como el Jacobiano
de transformacion a cilndricas es:
(x, y, z)
(r, , z) = r se tiene:

ZZZ Z 2 Z 3 Z 272r2
V = dxdydz = rdzdrd
D 0 0 r2
Z 2 Z 3
2
= r z|272r
r2 drd
Z0 2 Z0 3
r 27 3r2 drd

=
0 0
Z 2  3
27 2 3 4
= r r d
0 2 4 0
243 2
Z
243 243
= d = 2 =
4 0 4 2

Problema 2
Calcular el volumen de la region acotada por la esfera x2 + y 2 + z 2 = 13
y el cono (z 1)2 = x2 + y 2 , z 1
Solucion.
El volumen pedido es
ZZZ
V = dxdydz
R

donde la region R esta dada por

381
Figura 4.43: Region acotada por la esfera x2 + y 2 + z 2 = 13 y el cono (z 1)2 =
x2 + y 2 , z 1

n p p o
3 2 2 2 2
R = (x, y, z) IR / (x, y) D; 1 + x + y z 4 x y

D corresponde a la proyeccion de R sobre el plano xy.


D = {(x, y, z) IR2 /x2 + y 2 13}
Por la simetra del volumen conviene usar coordenadas cilndricas.

x = r cos
y = rsen = x2 + y 2 + z 2 r2 + z 2 13 ,
z=z

Determinemos la imagen R de R

(z 1)2 = x2 + y 2 z 1 + r = 1 + r z 13 r2
Luego

R = (r, , z) IR3 / (r, ) D; 1 + r z 13 r2


La region R al ser proyectada sobre el plano xy. produce


z = 0 = x2 + y 2 = 13
n o
D1 = (r, ) IR3 / r 2 ;
2 2
Como el Jacobiano de transformacion a cilndricas es:

(x, y, z)
(r, , z) = r se tiene:

382

ZZZ Z 2 Z 2 Z 13r2
V = dxdydz = rdzddr
R 0 0 1+r
Z 2 Z 2
2
13r
= rz1+r ddr
Z0 2 Z0 2  
= r 13 r2 (1 + r) ddr
0
Z 02 

= 2 r 13 r2 r + r2 dr
0
 2 2
r3

1 2 3/2
 r
= 2 13 r +
3 2 3
   0
1 4 8
133/2 73/2

= 2 +
3 2 3

Problema 3

Calcular utilizando coordenadas cilndricas el volumen de la region R ,


donde R es el interior a la esfera x2 + y 2 + z 2 = 4, z 0,y exterior al cilindro
(x 1)2 + y 2 = 1.

Figura 4.44: R, region interior a la esfera x2 + y 2 + z 2 = 4, z 0,y exterior al


cilindro (x 1)2 + y 2 = 1

Solucion.
La region
n R se describe en coordenadas cartesianas mediante
o
p
R = (x, y, z) IR3 / (x, y) D; 0 z 4 x2 y 2
donde D es la proyeccion de R sobre el plano xy.

383
D = {(x, y) IR3 /x2 + y 2 4 ; (x 1)2 + y 2 1}

Transformemos la region R a coordenadas cilindricas definidas por



x = r cos
y = rsen = x2 + y 2 + z 2 = r2 (cos2 + sen2 ) + z 2 4
z=z


0 z 4 r2

La region R al ser proyectada sobre el plano xy da origen a dos subregiones

3
x2 + y 2 r2 4 0 r 2 si
2 2

(x 1)2 + y 2 1 r 2 cos y r 2 si -
2 2

Entonces, la region R puede describirse mediante



R = (r, , z) / (r, ) D = D1 D1 ; 0 z 4 r2


n o
D1 = (r, ) IR3 /2 cos r 2 ;
2 2
 
3
D2 = (r, ) IR3 /0 r 2 ;
2 2

Ademas, el Jacobiano de la transformacion a cilndricas es:



(x, y, z)
(r, , z) = r

384
En consecuencia la integral puede describirse por
ZZZ
I = (r) drddz
R

Z /2 Z 2 Z 4r2 Z 3/2 Z 2 Z 4r2
= rdzdrd + rdzdrd
/2 2 cos 0 /2 0 0
Z /2 Z 2 Z 3/2 Z 2
4r2 2
= r [z]0 drd + r [z]0 4r drd
/2 2 cos /2 0
Z /2 Z 2 Z 3/2 Z 2
= r 4 r2 drd + r 4 r2 drd
/2 2 cos /2 0
Z /2
 2 Z 3/2  2
1 2 3/2
 1 2 3/2

= 4r d + 4r d
/2 3 2 cos /2 3 0
8 /2 8 3/2
Z Z
2
3/2
= 1 cos d + d
3 /2 3 /2
8 /2 8 3/2
Z Z
3
= sen d + d
3 /2 3 /2
/2
cos3

8 8 8
= cos + + =
3 3 /2 3 3

Problema 4
x2 y 2 z 2
ZZZ  
Calcular I = 2
+ 2 + 2 dxdydz.
 a
D b c
2
y2 z2

3 x
En la region D = (x, y, z) IR / 2 + 2 + 2 1 a > 0, b > 0, c >
a b c
0

x2 y 2 z 2
Figura 4.45: D es la region interior al elipsoide + 2 + 2 =1
a2 b c

385
Solucion.
La region de integracion es un elipsoide de semieejes a,b,c.
Efectuemos un primer cambio de variables:
x = au, y = bv, z = cw.
Con ello, D se transforma en la bola.
D = {(u, v, w) /u2 + v 2 + w2 1} y el valor absoluto del Jacobiano que-
da :
(x, y, z) a 0 0

(u, v, w) = 0 b 0 = abc

0 0 c
Luego, aplicando el teorema del cambio de variables y obtenemos la inte-
gral

x2 y 2 z 2
ZZZ  
I = + 2 + 2 dxdydz.
a2 b c
Z Z ZD
2 (x, y, z)
2 2

= u +v +w dudvdw
(u, v, w)
Z Z ZD
2 (x, y, z)
2 2

= u +v +w dudvdw
(u, v, w)
Z Z ZD
= (u2 + v 2 + w2 ) (abc) dudvdw
D

Ahora, transformamos a coordenadas esfericas.


0 u2 + v 2 + w2 1 = 0 r 1
u = rsen cos v
v = rsensen = tg = = 0
wv
w = r cos

tg = = 0 2

u
Quedando, la region D = {(r, , ) /0 r 1, 0 , 0 2}

ZZZ Z 2 Z Z 1
2 2 2
r2 r2 sen drdd

abc (u + v + w )dudvdw = abc
D 0 0 0
2
1
r5
Z Z
= abc sen dd
0 0 5 0
Z 2
abc
= cos |0 d
5 0
2abc 2
Z
4abc
= d =
5 0 5

Observacion

386
Es claro que la integracion se podra haber efectuado usando directamente
la trasformacion compuesta.

x = arsen cos
(x, y, z)
y = brsensen = = abcr2 sen
(r, , )
z = cr cos

Problema 5
ZZZ
dxdydz.
Calcular I = q ,
D 2 2 2
(x a) + (y b) + (z c)
en la region D = {(x, y, z) IR3 /x2 + y 2 + z 2 R2 } , (a, b, c) es un
punto fijo
no peteneciente a la esfera x2 + y 2 + z 2 R2 .

Figura 4.46: D es la region interior a la esfera x2 + y 2 + z 2 = R2

Solucion.

Si usamos coordenadas cartesianas los lmites de integracion son dificul-


tosos, pues en tal caso tendramos.

ZZZ
dxdydz.
I = q
D (x a)2 + (y b)2 + (z c)2
Z r Z r2 x2 Z r2 x2 y2
dzdydx.
I = q
r r2 x2 r2 x2 y 2 (x a)2 + (y b)2 + (z c)2

Es claro que si usamos este camino las cosas no seran faciles.


Sin embargo , dada la simetria esferica del dominio y observando que el
integrando no es nada mas que el reciproco de la distancia desde (a, b, c)
/D
hasta (x, y, z) D,nos damos cuenta que el resultado no puede depender
mas que de la distancia d entre dichos puntos.Por ello, el resultado no puede

387
variar si ubicamos el eje z pasando por el punto (a, b, c). Si (0, 0, d) son las
nuevas coordenadas del punto fijo tenemos.
ZZZ
dxdydz.
I= q
D x2 + y 2 + (z d)2

Observacion
El razonamiento anterior es muy usado el calculo de integrales que apare-
cen aplicaciones a la Fsica pues en dicha Ciencia son comunes las leyes en
que aparece una distacia o el cuadrado de una distancia en el denominador
del integrando.
Para calcular I en (*) usamos coordenadas esfericas. Obtenemos:

R 2
r2 sen dddr
Z Z Z
I =
0 0 0 r2 + d2 2dr cos
R
r2 sen ddr
Z Z
= 2
0 0 r2 + d2 2dr cos

Para calcular

r2 sen ddr
Z
J=
0 r2 + d2 2dr cos
podemos hacer
s = r2 + d2 2dr cos
ds = 2drsend
Ademas, = 0 = s = r2 + d2 2dr = (d r)2
= = s = r2 + d2 + 2dr = (d + r)2
Reemplazando en la integral anterior produce

Z (d+r)2
r r (d+r)2
J = s1/2 ds = 2s1/2 (dr)2
2d (dr)2 2d
r
= [2 (d + r) 2 (d r)]
2d
r 2r2
= [4r] =
2d d

Por lo tanto

388
R
2r2
Z
I = 2 dr
0 d
3 R

4 r
I =
d 3 0
4 3
I = R
3d

4.5. Ejercicios propuestos integrales dobles y


triples
4.5.1. Integrales dobles
Integrales dobles en coordenadas cartesianas
1.- Resolver las integrales:
Z 1Z 1
a)
2x3 dxdy
0 y
y+2
Z 4 Z
9
b)
dxdy
5 2 4y

Z 2 Z 4x2
c)
xydydx
2 4x2

Respuestas
Z 1Z 1
3 1
a)
2x dxdy =
0 y 3
Z 4 Z y+2
3 9
b)
dxdy =
5 2 4y 2
Z 2 Z 4x2
c)
xydydx = 0
2 4x2
ZZ
2.- Calcular la integral xydxdy, donde D es la region acotada
D
por y 2 = x, y 2 = 3x 18, y 0
ZZ
135
Respuesta: xydxdy = Vease Figura 4.47
D 2

389
Figura 4.47: Region acotada por y 2 = x, y 2 = 3x 18, y 0

x2
ZZ  
x 2 2
3.- Calcular la integral dxdy, donde D = (x, y) IR /1 x 2, yx
D x2 + y 2 2

Figura 4.48: Region acotada por y 2 = x, y 2 = 3x 18, y 0

ZZ
x 3 1 7
Respuesta: I = dxdy = arctan 2 ln 5 + ln 2
D x2 +y 2 2 4 2
Vease Figura 4.48

Z 8 Z 2
y
4.- Evalue la integral resultante: dxdy

3 16 + x7
0 y

Z 2 Z x3
y 8
Respuesta dydx = Vease Figura 4.49
0 0 16 + x 7 7

390
Figura 4.49: Region acotada por y 2 = x, y 2 = 3x 18, y 0

5.- Calcule el volumen del solido acotado por las graficas de:
x2 + y 2 = 9, y 2 + z 2 = 2.

Figura 4.50: Solido acotado por las graficas de x2 + y 2 = 9, y 2 + z 2 = 2.

Z 3 Z 9y2 p
Respuesta V (R) = 8 9 y 2 dxdy
0 0

6.- Calcule el volumen del solido acotado por las graficas de las
ecuaciones dadas por z = x2 + 4, y = 4 x2 , x + y = 2; z = 0.
423
Respuesta V (R) =
20

391
Figura 4.51: Solido acotado por las graficas de las ecuaciones dadas por z = x2 +4,
y = 4 x2 , x + y = 2 z = 0.

4.5.2. Calculo de Integrales dobles usando transforma-


cion de coordenadas
ZZ
y
1.- Calcular dxdy,donde R es a region plana determinada
R x2
+ y2
por los puntos del primer cuadrante que son interiores a la circun-
fererencia x2 4x + y 2 = 0 y exteriores a la circunferencia x2 + y 2 = 4.

Figura 4.52: Region plana determinada por los puntos del primer cuadrante que
son interiores a la circunfererencia x2 4x + y 2 = 0 y exteriores a la circunferencia
x2 + y 2 = 4

ZZ
y 1
Respuesta: I = dxdy =
R x2 +y 2 2

392
ZZ
1/2
(x y)2 + 2 (x + 2) + 1

2.- Calcular la integral dxdy, donde R
R
es la region plana acotada por y = 0, x = 2, y = x.
a) Utilice la transformacion u = x y, v = x + y.

Figura 4.53: R es la region plana acotada por y = 0, x = 2, y = x y las transfor-


maciones u = x y, v = x + y. y x = u (1 + v) , y = v (1 + u)

b) Utilice la sustitucion x = u (1 + v) , y = v (1 + u) .
ZZ
1/2 1
(x y)2 + 2 (x + 2) + 1

Respuesta. I = dxdy = 2 ln 2
D 2

3.- Resolver las integrales usando las coordenadas mas convenientes


Z 2a Z 2axx2
x2 + y 2 dydx

a)
0 0
Z 2 Z 4y2 p
b) x2 + y 2 dxdy
2 0
Z
Z y 2
sen x2 + y 2 dxdy

c)
y 2

Z 2 Z 2+ 4x2 p
d)
x2 + y 2 dxdy
4x2
Z 2
Z 2
y
e) e x dxdy,
R

si R esta acotado por x = a, x = b, y = px, y = qx, con a < b, p < q.

393
Respuestas.
Z 2a Z 2axx2
3
x2 + y 2 dydx = a2

a)
0 0 4
Z 2 Z 4y2 p
4
b) x2 + y 2 dxdy =
2 0 3

Z Z y2
2 2

c) sen x + y dxdy = 2
y 2


Z 2 Z 2+ 4x2 p
64
d)
x2 + y 2 dxdy = (3 4)
2 4x2 9
Z 2
Z
y 1 2
b a2 (eq ep )

e) e x dxdy =
R 2

4.5.3. Integrales triples


4.5.4. Integrales triples iteradas
1.- Resolver las integrales triples por iteraciones:


4xy 2
Z 2 Z 2 x Z
2
a) xdzdydx
0 0 0
Z a Z
a2 x2 Z a2 x2 y2 p
b) a2 x2 y 2 z 2 dzdydx
0 0 0
Z 1 Z
1x2 Z 12 x2 y2
x2 + y 2 dzdydx

c)
1 1x2 0

Respuestas

4xy 2
2 2 x
4
Z Z Z
2
a) xdzdydx = 2
0 0 0 3
aZ Z a2 x2 y2 p

a2 x2
2
Z
b) a2 x2 y 2 z 2 dzdydx = a2
0 0 0 8
Z 1 Z 1x2 Z 12 x2 y2
4
x2 + y 2 dxdy =

c)
1 1x2 0 15

394
2.- Calcular las areas de las regiones D R2 acotadas por:

a) xy 4, y x, 27y 4x2 .

Figura 4.54: D es la region plana acotada por xy 4, y x, 27y 4x2

b) x2 + y 2 9, y x + 3, y x.

Figura 4.55: D es la region plana acotada por x2 + y 2 9, y x + 3, y x

Respuestas

2 3
a) A (D) = + 4 ln .
3 2
9
b) A (D) = (3 + 2) .
8
395
4.5.5. Integrales triples en coordenadas rectangulares
cartesianas.
ZZZ
1.- Calcular la integral triple xyzdxdydz, siendo el dominio de
D
integracion D el tetaedro delimitado por los planos coordenados y
y z
el plano de ecuacion x + + = 1.
2 3
ZZ
1
Respuesta. I = xyzdxdydz = .
D 20
ZZZ
2.- Calcular la integral triple x2 dxdydz, siendo el dominio de
D
integracion D acotado por y 2 + z 2 = 4ay, y 2 = ax, x = 3a,con a > 0
ZZZ !
2 2 3 3
Respuesta. I = x dxdydz = 27a 2
D 2

ZZZ
3.- Calcular la integral triple xdxdydz, si D es la region de IR3
D
acotada por las superficies 2x + 3y = 6, 2x + 3z = 6, y = 3 + 2x x2 ,
z=0

Figura 4.56: D es la region de IR3 acotada por las superficies 2x+3y = 6, 2x+3z =
6, y = 3 + 2x x2 , z = 0

 3
1 6 5 34 4 42 3 2
Respuesta. I = x x + x + 9x
9 5 4 3 1/3

396
4.5.6. Calcular las integrales dadas usando las coorde-
nadas adecuadas:
ZZZ
a) (x + y + z)3 dxdydz,si D esta acotado por x = 0, y = 0, z = 0,
D
x + y + z = 0.

Figura 4.57: D acotado por x = 0, y = 0, z = 0, x + y + z = 0


ZZZ
b) (x+y+z)2 dxdydz,si D esta acotado por 2 z = x2 +y 2 , x2 +y 2 +z 2 = 3
D

Figura 4.58: D acotado por 2 z = x2 + y 2 , x2 + y 2 + z 2 = 3

397
ZZZ
c) z 2 dxdydz,si D esta limitado por x2 + y 2 + z 2 = a2 , x2 + y 2 + z 2 = 2az
D

Figura 4.59: D esta limitado por x2 + y 2 + z 2 = a2 , x2 + y 2 + z 2 = 2az

x2 y 2 z 2
ZZZ
d) zdxdydz,si D esta acotado por z = 0, 2 + 2 + 2 = 1
D a b c

x2 y 2 z 2
Figura 4.60: D esta acotado por z = 0, + 2 + 2 =1
a2 b c

398
ZZZ
e) zdxdydz,si D esta limitado por z = h, z 2 = h2 (x2 + y 2 )
D

Figura 4.61: D esta limitado por z = h, z 2 = h2 x2 + y 2




Respuestas
ZZZ
1 5
a) (x + y + z)3 dxdydz = ln 2
Z Z ZD 2 16 
97
b) (x + y + z)2 dxdydz = 18 3
ZZZD 5 6
59
c) z 2 dxdydz = a5
Z Z ZD 480
abc2
d) zdxdydz =
ZZZD 4
h2
e) zdxdydz =
D 4

4.5.7. Resolver las integrales usando coordenadas esferi-


cas:
Z a Z a2 x2 Z a2 x2 y2
x2 + y 2 dzdydx.

a)
Za Z Z a2 x2 0
x2 + y 2 dxdydz, si D es la region limitada por

b)
D

r x + y 2 + z 2 R2 , z > 0.
2 2

Respuestas:

399
Z a Z a2 x2 Z a2 x2 y2
4
x2 + y 2 dzdydx = a5 .

a)
ZaZ Z a2 x2 0 15
4
x2 + y 2 dxdydz = R5 r 5 .
 
b)
D 15

Calcular los volumenes de las regiones D R3 acotado por:


a) z = x, z = x2 + y 2

Figura 4.62: Region limitada por z = x, z = x2 + y 2

b) x2 + y 2 = 4, x2 + y 2 z 2 = 1

Figura 4.63: Region limitada por x2 + y 2 = 4, x2 + y 2 z 2 = 1

c) z 2 = x2 + y 2 , y = 0, y = x, x = a.

400
Figura 4.64: Region limitada por z 2 = x2 + y 2 , y = 0, y = x, x = a

d) z = x + y, z = 0, x2 + y 2 = 1, x = 1, y = 1.
Respuestas.

Figura 4.65: Region limitada por z = x + y, z = 0, x2 + y 2 = 1, x = 1, y = 1


a) V (D) =
32
b) V (D) = 4 3
2 
c) V (D) = a 3 2 + ln 1 + 2
1
d) V (D) =
3

4.6. Aplicaciones integrales dobles y triples


4.6.1. Volumenes de cuerpos en el espacio
Si la funcion z = f (x, y) es continua tal que f (x, y) 0, entonces la
integral doble: Z Z
V () = f (x, y) dA
R

401
define el volumen bajo la superficie de la grafica de la funcion f (x, y) sobre
la region R IR2 Problema 1
Calcular el volumen acotado por el paraboloide z = x2 + y 2 entre los
planos z = 1 y z = 4.

Figura 4.66: Volumen acotado por el paraboloide z = x2 + y 2 y los planos z = 1


yz=4

Solucion. Es claro que el volumen a calcular esta dado por


Z Z
V () = (x2 + y 2 )dA
R
donde R = {(x, y) IR /1 x + y 2 1}
2 2

Cambiando a  coordenadas polares, queda


x = r cos
= 1 x2 + y 2 4 = 1 r 2, 0 2
y = rsen
Luego , el dominio de integracion se transforma en:
R = {(r, ) /1 r 2, 0 2}

(x, y)
Ademas, el jacobiano de transformacion es =r
(r, )
Reemplazando terminos en la ultima integral se tiene

Z 2 Z 2
V = r2 (r) drd
0 1
Z 2  4 2 Z 2  4 
r 2 1
= d = d
0 4 1 0 4 4
15 15
= (2) =
4 2
402
Problema 2
Calcular el volumen acotado por z 36 3x2 y 2 , y z x2 + 3y 2 .

Figura 4.67: Volumen acotado por el paraboloide z 363x2 y 2 , y z x2 +3y 2

Solucion. Ambas ecuaciones z = 36 3x2 y 2 , y z = x2 + 3y 2


representan paraboloides de revolucion.
La region de integracion esta definida por

R = (x, y, z) IR3 / x2 + 3y 2 z 36 3x2 y 2




.
Determinemos la proyeccion de la interseccion de ambas superficies sobre el
plano xy. 
z = 36 3x2 y 2
= 363x2 y 2 = x2 +3y 2 = 32 = 4x2 +4y 2
z= x2 + 3y 2
Por tanto, x2 + y 2 = 9, corresponde a una circunferencia centrada en el
origen de radio 3.
En consecuencia, el volumen de la region es

Z Z
36 3x2 y 2 (x2 + 3y 2 ) dxdy
  
V =
Z ZD
36 4x2 4y 2 dxdy
 
=
D

donde D = {(x, y) IR2 / 0 x2 + y 2 9}


Como la funcion del integrando es simetrica con respecto al plano xy,
podemos calcular el volumen en el primer octante y multiplicar por cuatro.

403
Z Z
36 4x2 4y 2 dxdy
 
V =4
D+

donde D = (x, y) IR2 / 0 x 3, 0 y

9 x2 .


Z 3 Z 9x2
36 4x2 4y 2
 
V = 4 dydx
0 0

Z 3 Z 9x2
4 9 x2 y 2 dydx
 
= 4
0 0

Para simplificar
 los calculos usemos coordenadas polares
x = r cos
= 0 x2 + y 2 9 = 0 r 3, 0
y = rsen 2
Luego , el n
dominio de integracion se transforma en:
o
D = (r, ) /0 r 3, 0
2
(x, y)
Ademas, el jacobiano de transformacion es =r
(r, )
Reemplazando terminos en la ultima integral se tiene


Z
2
Z 3
9 r2 (r) drd
 
V = 16
0 0
Z  3 Z  4
9 2 r4 34

2 2 3
= 16 r d = 16 d
0 2 4 0 0 2 4
 4
3
= 16 = 162
4 2

4.6.2. Area de figuras planas.


Sea D una region acotada en el plano xy. El area de D se define por:
ZZ
A (D) = dxdy
D

Si se tiene una region D de tipo I,

D = (x, y) IR2 / a x b, 1 (x) y 2 (x)




404
obtendramos que
ZZ Z bZ 2 (x)
A (D) = dxdy = dydx
D a 1 (x)
Z b
= (2 (x) 1 (x))dx
a

Problema 1

Calcular el area D de la region situada en el primer cuadrante, acotada


por:
y = x2 , y = 4x2
(1)
x = y 2 , x = 4y 2

Figura 4.68: area D de la region situada en el primer cuadrante, acotada por


y = x2 , y = 4x2 , x = y 2 y x = 4y 2

Solucion. El calculo del area sera bastante engorroso si usamos una


region de tipo I en coordenadas cartesianas por la simetra que tiene la region.
Sin embargo, un cambio de variables simplifica la region D y la transforma
en D .
x2 y2
Sean u = , v =
y x
Entonces las curvas que acotan la region se tranforman en
y = x2 = u = 1.
1
y = 4x2 = u = .
4
Luego D esta acotada por la rectas verticales; y las rectas horizontales
x = y 2 = v = 1.

405
1
x = 4y 2 = transforma en v = .
4
 
1 1
Es decir, D = (u, v) /1 u , 1 v
4 4

(x, y)
Para calcular tenemos dos posibilidades, la primera es despejar
(u, v)
(u, v)
x e y en terminos de u y v a partir de (1) . La segunda, es calcular
(x, y)
(u, v) 1
 
(x, y)
y usar la propiedad = .
(u, v) (x, y)
2
2x x


(u, v) 2 (x, y) 1
En efecto = y y = 4 1 = 3 =
(u, v) = 3

(x, y) y 2 2y
2
x x
Calculemos ahora el area pedida:

ZZ 1Z Z
1
A (D) = dxdy = dvdu
D D 3
1 1 1 1 1
Z Z Z
= dvdu = [v]1 du
3 1/4 1/4 3 1/4 1/4
1 1 3
= [u]1/4 =
4 16

4.6.3. Momentos y centros de masa para placas planas


delgadas

Definicion: Sea L una lamina con la forma de una region R del plano xy.
Si la densidad en (x, y) es (x, y) , y es continua en R, entonces la masa M y
los primeros momentos de la lamina con respecto al eje x e y respectivamente
son Mx , My y los centros de masa (x, y) se definen:

406
Z Z
M = (x, y) dA
Z ZR
Mx = y (x, y) dA
Z ZR
My = x (x, y) dydx
R RR
My x (x, y) dA
x = = R RR
M (x, y) dA
R RR
Mx y (x, y) dA
y = = R RR
M R
(x, y) dA
Si una partcula de masa m se encuentra en el punto (x, y, z) , entonces su
distancia al eje z es
Sea L una lamina con la forma de una region R del plano xy. Si la densidad
en (x, y) es (x, y) , y es continua en R, entonces los segundos momentos
de la lamina o momentos de inercia con respecto al eje x e y respectivamente
son Ix , Iy y los momentos de inercia con respecto a una recta L y respecto al
origen se definen:

Z Z
Ix = y 2 (x, y) dA
Z ZR
Iy = x2 (x, y) dydx
Z ZR
IL = r2 (x, y) (x, y) dA
Z ZR
I0 = (x2 + y 2 ) (x, y) dA = Ix + Iy
R

4.6.4. Centroide de figuras geometricas


Cuando (x, y) es igual a uno, la posicion del centro de masa x, y se
convierte en una caraterstica de la forma del objeto y no del material que
esta hecho y se define como el centroide de una placa.
Problema 1
Dada una placa delgada que cubre la region triangular acotada por el eje
x y las rectas x = 1, y = 2x en el primer cuadrante si la densidad de la placa
en en cada punto de la region esta dada por (x, y) = 6x + 6y + 6.Calcular:

407
a) la masa de la placa.
b) los primeros momentos
c) el centro de masa.
d) momentos de inercia(segundos momentos) con respecto a los ejes x e
y
e) radios de giro con respecto a los ejes x e y respectivamente
f) radio de giro con respecto al origen

Figura 4.69: Region triangular acotada por el eje x y las rectas x = 1, y = 2x en


el primer cuadrante

Solucion.
Consideremos una particion de la region que delimita la placa como una
region de tipo I: R = {(x, y) / 0 x 1, 0 y 2x} .
a) La masa de la placa es

Z 1 Z 2x
M = (x, y) dydx
0 0
Z 1 Z 2x
= (6x + 6y + 6) dydx
0 0
Z 1 2x
6xy + 3y 2 + 6y 0 dx

=
Z0 1
24x2 + 12x dx
 
=
0
1
= 8x + 6x2 0 = 14
 3

b) El primer momento con respecto al eje x es

408
Z 1 Z 2x
Mx = y (x, y) dydx
0 0
Z 1 Z 2x
6xy + 6y 2 + 6y dydx

=
Z0 1 0
2x
3xy 2 + 2y3 + 6y 2 0 dx

=
Z0 1
28x3 + 12x2 dx
 
=
0
1
= 7x + 4x1 0 = 11
 4

Por otra parte, el primer momento con respecto al eje y es:

Z 1 Z 2x
My = x (x, y) dydx
0 0
Z 1 Z 2x
6x2 + 6xy + 6x dydx

=
Z0 1 0
 2 2x
= 6x y + 3xy 2 + 6xy 0 dx
Z0 1
12x3 + 12x3 + 12x2 dx
 
=
0
1
= 6x + 4x3 0 = 10
 4

c) As, las coordenadas del centro de masa son:

My 10 5
x = = =
M 14 7
Mx 11
y = =
M 14

d) El momento de inercia con respecto al eje x es

409
Z 1 Z 2x
Ix = y 2 (x, y) dydx
Z0 1 Z0 2x
6xy 2 + 6y 3 + 6y 2 dydx

=
0 0
Z 1 2x
3 3 4 3
= 6xy + y + 2y dx
0 2 0
Z 1
40x4 + 116x3 dx
 
=
0
1
= 8x + 4x4 0 = 12
 5

Por otra parte, el momento de inercia con respecto al eje y es

Z 1 Z 2x
Iy = x2 (x, y) dydx
Z0 1 Z0 2x
6x3 + 6x2 y + 6x2 dydx

=
Z0 1 0
2x
6x3 y + 3x2 y 2 + 6x2 y

= 0
dx
0
Z 1
12x4 + 12x4 + 12x3 dx
 
=
0
 1
24 5 4 39
= x + 3x =
5 0 5
Los radios de giros con respecto a los ejes x e y son
r r r
Ix 12 6
Rx = = =
M 14 7
r r
Iy 39
Ry = =
M 70
El radio de giro con respecto al origen es
r
I0
R0 =
M
donde el momento de inercia con respecto al origen esta dado por:
Z Z
I0 = (x2 + y 2 ) (x, y) dxdy = Ix + Iy

410
s
99
5
R0 =
14
Problema 2

Calcular el momento de inercia con respecto al eje x de la lamina plana


R situada en el primer cuadrante, acotada por las curvas
y 2 = x, y 2 = 2x, x2 = y, x2 = 3y, si la densidad en cada punto de la
region es (x, y) = x3 y

Figura 4.70:

Solucion: El momento de inercia con respecto al eje x en coordenadas


cartesianas queda
ZZ
Ix = y 2 (x, y) dxdy
Z ZR
= y 2 (x3 y)dxdy
Z ZR
= (xy)3 dxdy
R
Sin embargo, conviene hacer un cambio de variables para simplificar la
geometra de la
region
y2
u=
(u, v) 1

(x, y)

x2
x = J = (u, v) = (x, y) =

v=
y
y 2 2y 1


x2 x
1
= 2x x2 =

3
y y2

411
Examinemos ahora como se transforma la region
y2 y2
y 2 = x, y 2 = 2x = = 1; =2
x x
x2 x2
x2 = y, x2 = 3y = = 1; =3
y y
Luego, R = {(u, v) / 1 u 2, 1 v 3} .
y 2 x2
Por otra parte, el integrando se puede expresar uv = = xy
x y

ZZ
Ix = (xy)3 dxdy
Z 2RZ 3  
3 1
= (uv) dvdu
1 1 3
1 2 3
Z Z 3
= u du v 3 dv
3 1 1
 4 2  4 3
1 u v
=
3 4 1 4 1
  
1 1 81 1
= 4
3 4 4 4
1 15
= 20 = 25
3 4

4.6.5. Momentos y Centros de masa de un solido

Sea Q un solido una region del espacio en IR3 . Si la densidad en (x, y, z)


es (x, y, z) , y es continua en Q, entonces la masa M y los primeros mo-
mentos del solido con respecto a los planos xy, xz e yz respectivamente son
Mxy , Mxz , Myz y las coordenadas del centro de masa (x, y, z) se definen:

412
Z Z Z
M = (x, y, z) dV
Q
Z Z Z
Mxy = y (x, y) dV
Q
Z Z Z
Mxz = y (x, y, z) dV
Q
Z Z Z
Myz = x (x, y, z) dV
Q
RRR
Mxy Q
z (x, y, z) dV
x = = RRR
M (x, y, z) dV
R R RQ
Mxz Q
y (x, y, z) dV
y = = RRR
M (x, y, z) dV
R R RQ
Mxy Q
x (x, y, z) dV
z = = RRR
M Q
(x, y, z) dV

Sea Q un solido una region del espacio en R3 . Si la densidad en (x, y, z) es


(x, y, z) , y es continua en Q, entonces los segundos momentos del solido o
momentos de inercia con respecto al eje x, al eje y,y al eje z respectivamente
son Ix , Iy , Iz se definen:

Z Z Z
Ix = (y 2 + z 2 ) (x, y, z) dV
Z Z ZQ
Iy = (x2 + z 2 ) (x, y, z) dV
Z Z ZQ
Iz = (x2 + y 2 ) (x, y, z) dV
Q

4.6.6. Masa de un solido


Problema 1

La densidad en un cubo de lado a es proporcional al cuadrado de la dis-


tancia respecto a uno de los vertices ( al que denotaremos por 0). Demostrar
que la masa del cubo es igual a su volumen multiplicada por la densidad en
un vertice adyacente a 0.

413
Solucion. Si R es una region cerrada y acotada en IR3 y la densidad
en el punto (x, y, z) R es (x, y, z) , entonces la masa de R esta dada por
Z Z
M= (x, y, z) dV
R

Situando el cubo con el vertice en el origen del sistema de coordenadas y


tres de sus caras coincidiendo con los planos coordenados, se tiene:
(x, y, z) = x2 + y 2 + z 2 y por lo tanto
Z Z
x2 + y 2 + z 2 dxdydz

M=
R

donde R = {(x, y, z) IR3 /0 x a; 0 y a; 0 z a} . Luego

a a a aa a
z3
Z Z Z Z Z 
2 2 2 2 2

M = x + y + z dzdydx = (x + y )z + dydx
0 0 0 0 0 3 0
Z aZ a Z a a
a3 y 3 a3

2 2 2
= x a+y a+ dydx = ax y + a + y dx
0 0 3 0 3 3 0
Z a 4
  3 4
a
2a x 2a
= a2 x 2 + dx = a3 + x = a5
0 3 3 3 0

Por otra parte la distancia entre 0 y cualquier vertice adyacente a el es a.


Entonces la densidad de un vertice adyacente es (a, 0, 0) = a2 , (0, a, 0) =
a2 , (0, 0, a) = a2 .
Por lo tanto, queda V (R) (a, 0, 0) = a3 a2 = a5 = M
Problema 2

Un solido tiene forma de cilindro circular recto con radio de la base R


y altura h. Si la densidad en un punto es directamente proporcional a la
distancia a una de las bases. Encontrar:
a) las coordenadas del centro de masa
b) el momento de inercia con respecto al eje axial de simetra del
solido cilndrico
Soluciones
La region del solido cilndrico esta acotado por
Q = {(x, y, z) IR3 / 0 x2 + y 2 R2 , 0 z h}
Calculemos en primer lugar la masa total del cilndro, usando coorde-
nadas cilndricas , la region del solido se transforma en
Q = {r, , z)/ 0 r R, 0 2, 0 z h}

414
ZZZ
M = (x, y, z) dV
Q
ZZZ Z R Z 2 Z h
= kzrdrddz = kzrdzddr
Q 0 0 0
R 2 h R 2
z2 h2
Z Z  Z Z 

= k ddr = k rddr
0 0 2 0 0 0 2
2 Z R  2
R
h r
= k []02 rdr = kh2
2 0 2 0
1
= kh2 R2
2

El momento de inercia del solido con respecto al plano xy es

ZZZ ZZZ
Mxy = z (x, y, z) dV = z (kz) dV
Q Q
ZZZ Z R Z 2 Z h
2
= kz (r) drddz = kz 2 rdzddr
Q 0 0 0
Z R Z 2  3
h 3 Z R
z h
= k ddr = k []2
0 rdr
0 0 3 0 3 0
 2
R
2 r 1
= kh3 = kh3 R2
3 2 0 3

Finalmente, las coordenadas del centro de masa son:

My
x = =0
M
Mx
y = =0
M
1
Mxy kh3 R2 2
z = = 3 = h
M 1 3
kh2 R2
2
415
b) El momento de inercia con respecta al eje de simetra z es:
ZZZ ZZZ
2 2
Iz = (x + y ) (x, y, z) dV = (r2 )kz (r) drddz
Q Q
R 2 h R Z 2
h2
Z Z Z Z
= (r3 )kzdzddr = k (r3 )ddr
0 0 0 2 0 0
Z R  4 R
r
= kh2 (r3 )dr = kh2
0 4 0
1
= kh2 R4
4
Problema 3
Calcular el momento respecto al eje de simetra de un cono homogeneo aco-
tado por las graficas x2 y 2 + z 2 = 0 e y = 3, si tiene una densidad
(x, y, z) constante

Figura 4.71:

Solucion Sea la region del cono circular recto :


n p p o
Q = (x, y, z) IR3 / y x y, 0 y 3, y 2 x2 z y 2 x2

Sea (x, y, z) = k, la densidad es constante. Luego, el momento con


respecto al eje y es:
ZZZ
Iz = (x2 + y 2 ) (x, y, z) dV
Q

416
Usando coordenadas cilndricas tenemos:
x = r cos
y= y Q = {(r, , y) / 0 r 3, 0 2, r y 3}
z = rsen
El Jacobiano de transformacion es

(x, y, z)
=r
(r, , z)
Reemplazando en la expresion anterior se tiene

ZZZ Z 3 Z 2 Z 3
2
Iz = (r )k (r) drddy = k (r3 )dyddr
Q 0 0 r
Z 3 Z 2 Z 3
= k (r3 ) [y]3r ddr = k
(r3 ) [3 r] []2
0 dr
0 0 0
Z 3  3
3 4 3 4 1 5
= 2k (3r r )dr = 2k r r
0 4 5 0
11 5
= 3 k
10

Problema 4

Calcular el volumen y el centroide de la region acotada arriba por la esfera



= a, y abajo por el cono = 0 , donde 0 < 0 < , que tiene una densidad
2
(x, y, z) = 1
Solucion.
Sea la region
2 p

a p
Q = (x, y, z) IR3 / 0 x2 + y 2 , y 2 + x2 z a2 y 2 x2
2
El volumen de esta region en coordenadas esfericas es

Z 2 Z 0 Z a Z 2 Z 0
2 1 3
V = senddd = a senddd
0 3
Z 02 0 0 0
1 3 2
= a [ cos ]00 d = a3 [1 cos 0 ]
3 0 3

Por simetra, el centroide esta en el eje z, luego determinemos el momento


del solido respecto al plano xy

417
ZZZ
Mxy = z (x, y, z) dV
Q
ZZZ Z 2 Z 0 Z a
2
Mxy = ( cos ) senddd = 3 cos senddd
Q 0 0 0
2 0 2 0
sen2
Z Z Z 
1 4 1
= a cos sendd = a4 d
0 0 4 4 0 2 0
Z 2
1 4 1
= a sen2 0 d = a4 sen2 0
8 0 4
As, las coordenadas del centro de masa son:
Myz
x = =0
M
Mxz
y = =0
M
Mxy 3
z = = a (1 + cos 0 )
M 8
Problema 5

Se tiene una region solida que se encuentra al exterior de la esfera x2 +


y 2 +z 2 = a2 y dentro de la esfera x2 +y 2 +z 2 = b2 ,suponiendo que la densidad
de un punto es directamente proporcional al cuadrado de la distancia de P
al centro de las esferas.
a) Calcule la masa del solido
b) Calcule su momento de inercia con respecto a una recta que pasa por
el
centro
Solucion.
a) La region solida esta acotada por Q = {(x, y, z) IR3 / a2 x2 + y 2 + z 2 b2 }
y la densidad del punto es (x, y, z) = k (x2 + y 2 + z 2 ) .
La masa total del casquete esferico en coordenadas cartesianas es

ZZZ
M = (x, y, z) dV
Q
ZZZ
k x2 + y 2 + z 2 dxdydz

=
Q

Para calcular la masa usaremos coordenadas esfericas:

418
Figura 4.72: Region solida que se encuentra al exterior de la esfera x2 + y 2 + z 2 =
a2 y dentro de la esfera x2 + y 2 + z 2 = b2


x = sen cos
(x, y, z)
y = sensen = = 2 sen
(, , )
z= cos

La region de integracion queda:


Q = {(, , ) / a b, 0 , 0 2}
Al sustituir en la expresion anterior tenemos que la masa total es

Z 2 Z Z b
M = (k2 )2 senddd
0 0 a
Z 2 Z  5 b

= k sendd
0 0 5 a
 2
Z
1
= 5
k b a 5
[ cos ]0 d
5
Z0 2
2
k b5 a5

= d
5 0
4
k b4 a4

=
5

Por la simetra, el momento de inercia con respecto al eje z es

419
ZZZ
x2 + y 2 (x, y, z) dV

Iz =
Q
Z 2 Z Z b
2 sen2 (k2 )2 senddd

=
0 0 a
Z 2 Z Z b
= k6 sen3 ddd
0 0 a
Z 2 Z  7 b

= k sen3 dd
0 0 7 a
Z 2 Z
1 7 7
sen 1 cos2 dd
 
= k b a
7
Z0 2 0 
1 7 7
 1 3
= k b a cos + cos d
7 0 3 0
Z 2
4
k b 7 a7

= d
21 0
8 7
= k(b a7 )
21
Problema 6

Calcular el momento de inercia con respecto al eje z del solido acotado


entre las esferas x2 + y 2 + z 2 a2 , x2 + y 2 + z 2 9a2 . Si la densidad en
x2 + y 2 + z 2
cada punto de la region es (x, y, z) = .
x2 + y 2

Figura 4.73: Solido acotado entre las esferas x2 +y 2 +z 2 a2 , x2 +y 2 +z 2 9a2 .

Solucion
Por la simetra, el momento de inercia con respecto al eje z es

420
ZZZ
x2 + y 2 (x, y, z) dV

Iz =
Q
 x2 + y 2 + z 2
ZZZ
2 2
= x +y dV
Q x2 + y 2
ZZZ
x2 + y 2 + z 2 dxdydz

= =
Q

donde la region solida esta acotada por


Q = {(x, y, z) IR3 / a2 x2 + y 2 + z 2 9a2 }.
Para calcular la ultima
integral usaremos coordenadas esfericas:
x = sen cos
(x, y, z)
y = sensen = = 2 sen
(, , )
z= cos

La region de integracion queda:
Q = {(, , ) / a 3a, 0 , 0 2}
Al sustituir en la expresion anterior tenemos que Iz queda:

Z 2 Z Z 3a
Iz = (2 )(2 sen)ddd
Z0 2 Z0 Za 3a
= 4 senddd
0 0 a
Z 2 Z  5 3a

= sendd
0 0 5 a
 2
Z Z
1 5 5
= (3a) a sendd
5 0 0
8 5  2
Z
= a [ cos ]0 d
5
Z0
16 5  2
= a d
5 0
32 5
= a
5
Problema 7

Encontrar el centro de masa del solido homogeneo , cuya densidad es


(x, y, z) = 1, que esta acotado por arriba por z 2 = 64 + x2 + y 2 y por
abajo z 2 = 2( x2 + y 2 ), z 0.

421
Figura 4.74:

Solucion.
Como la densidad es constante (x, y, z) = 1,y de acuerdo a la simetra
,que presenta el problema las coordenadas del centro de masa son

Myz
x = =0
M
Mxz
y = =0
M RRR
Mxy Q
zdV
z = =
M V (Q)
Para representarel problema, usaremos coordenadas cilndricas.
x = r cos
(x, y, x)
y = rsen = = r es el Jacobiano de transforma-
(r, , z)
z= z

cion.
Primero determinemos  la funcion interseccion de las superficies:
z 2 = 64 + x2 + y 2
= x2 + y 2 = 64
z 2 = 2(x2 + y 2 )
Pero en cilndricas describen: x2 + y 2 = r2 = 0 r2 64 = 0
r8
Por otra parte 0 2
Ahora determinemos las cotas para z
z 2 = 64 + x2 + y 2 = 64 r2 = z = 64 2
+ r ,con z 0
2 2 2 2
z = 2(x + y ) = 2r = z = 2r
Luegola region de integracion queda:
Q = (r, , z) / 0 r 8, 0 2, 2r z 64 + r2

Calculemos el volumen de la region:

422
ZZZ

V (Q ) = rdrddz
Q

Z 2 Z 8 Z 64+r2 Z 2 Z 8
2
=
r dzdrd = [z]64+r
2r
rdrd
0 0 2r 0 0
Z 2 Z 8 h i
= 64 + r2 2r rdrd
Z0 2 Z
0
8 Z 8 2

= 2
r 64 + r dr 2 r dr d
0 0 0
Z 2 "  8

 3 8 ! #
1 2 r
= (64 + r2 )3/2 2 d
0 2 3 0 3 0
218/2 83
 21/2 
2
= 2 2
3 3 3
2 h i
V (Q ) = 29 23/2 1 229

3
Por otro lado, el momento de inercia es

ZZZ
M = (x, y, z) zdV
Q
ZZZ
= z rdrddz
Q

Z 2 Z 8 Z 64+r2
=
r zdzddr
0 0 2r
2 64+r2 8
z2
Z Z 
= r drd
0 0 2 2r
1 2 8 
Z Z
64 + r2 2r2 rdrd

=
2 0
Z 2 Z0 8
1
64r r3 drd
 
=
2 0 0
Z 2  8
1 2 r4
= 32r d
2 0 4 0
1  2 
= 2 8 (32 16) = 1024
2
Por tanto, la tercera coordenada del centro de masa del solido es:

423
RRR
zdV 1024  
Q
z= =  =3 2+1
V (Q) 1024 2 1
3

4.6.7. Determinacion del centroide dee un solido


Problema 1

Encontrar el centroide del solido encerrado por el cilindro x2 + y 2 = 4


, acotado por arriba por el paraboloide z = x2 + y 2 , y por abajo por el
plano XY. Para efectos de calculo suponga que la densidad es (x, y, z) = 1
constante.

Figura 4.75: Solido encerrado por el cilindro x2 + y 2 = 4 , acotado por arriba por
el paraboloide z = x2 + y 2 , y por abajo por el plano XY

Solucion
El centroide del solido (x, y, z) ,esta en su eje de simetra,en este caso
el eje z,. Esto hace que x = y = 0.
Para hallar z, dividimos el primer momento Mxy por la masa M del solido,
es decir

Mxy
z=
M
RRR
Donde el primer momento RRRse define M xy =
z (x, y, z) dxdydz y la
masa del solido es M =
(x, y, z) dxdydz siendo
= {(x, y, z) IR3 / x2 + y 2 4, 0 z x2 + y 2 }

424
Por la simetra del dominio, es necesario hacer un cambio de variables a
coordenadas cilndricas

x = r cos
y = rsen = el jacobiano de transformacion es
z= z


cos rsen 0
(x, y, z)
= sen r cos 0 = r
(r, , z) 0 0 1
Entonces la region se tranforma en
= {(r, , z) / 0 r 2, 0 2, 0 z r2 }
El valor de Mxy es:

ZZZ
Mxy = z (x, y, z) dxdydz

ZZZ Z 2 Z 2 Z r2
= zrdrddz = zrdzdrd
0 0 0
2 2
r 2 2 2
z 2 r5
Z Z Z Z
= rdrd = drd
0 0 2 0 0 0 2
2 6 2 Z 2
Z
r 16
= d = d
0 12 0

0 3
32
=
3
El valor de la masa M es

ZZZ
M = (x, y, z) dxdydz

ZZZ Z 2 Z 2 Z r2
= rdrddz = rdzdrd
0 0 0
Z 2 Z 2 Z 2 Z 2
2
= z|r0 rdrd = r3 drd
0 0 0 0
2
2 Z 2
r4
Z
= d = 4d
0 4 0 0
= 8

Por lo tanto
Mxy 4
z= =
M 3
425
 
4
En consecuencia, el centroide se localiza en 0, 0, .
3
Problema 2

Calcular el momento de inercia del solido con respecto al eje z , que


esta acotado por arriba por la esfera solida x2 + y 2 + z 2 = a2 y por abajo
por el cono = 3 ,y cuya densidad es constante = 1

Solucion:
En coordenadas rectangulares, el momento de inercia con respecto al eje
z es
ZZZ
x2 + y 2 (x, y, z) dV

Iz =
Q

En coordenadas esfericas, queda


Z 2 Z /3 Z 1
2 sen2 2 senddd

Iz =
0 0 0

2 /3 1 2 /3 1
5
Z Z Z Z Z 
4 3
sen ddd = sen3 dd
0 0 0 0 0 5 0
Z 2 Z /3  /3 Z 2
1 2 1  1 3
sen 1 cos dd = cos + cos d
5 0 0 5 0 3 0
1 2 5
Z
1
= d = (2)
5 0 24 24

Iz =
12

4.7. Autoevaluacion Integrales dobles y triples


Tiempo 2 horas
Pregunta 1
R 2 R 4x2 xe2y
a) Dada la integral 0 0
dydx, dibuje la region de integracion y
4y
evalue la integral.
b) Calcular el volumen del solido limitado por las superficies

x2 + y 2 + z 2 = 1, x2 + y 2 + z 2 = 4, z 2 = x2 + y 2 , con z 0

Pregunta 2

426
RRR
Calcular R
(x2 + y 2 )dxdydz, siendo R un cono recto de revolucion de
altura h, base situada en el plano XY, eje de simetra en z, dado por
la ecuacion a2 (h z)2 = h2 (x2 + y 2 ).
Pregunta 3 RRR
Calcular la integral I = R
xyzdxdydz, en la region del primer octante
limitada por los planos coordenados y el plano x + y + z = 1.

Pauta de Correccion
Pregunta 1
a) Por la forma del integrando es conveniente cambiar el orden de inte-
gracion,
en efecto


2 4x2 4 4y
xe2y xe2y
Z Z Z Z
dydx = dxdy
0 0 4y 0 0 4y
Z 4  2 2y 4y
xe
= dy
0 4 (4 y) 0
Z 4 2y
e
= dy
0 2
 2y 4
e e8 1
= =
4 0 4

b) El volumen del solido esta dado por


Z Z Z
V (D) = dxdydz
R

Usando coordenadas esfericas, tenemos

x = r sin cos
y = r sin sin
z = r cos

(x, y, z)
que tiene por Jacobiano de la transformacion = r2 sin .
(r, , )
Entonces esta region R corresponde en el espacio r a la region R dada
por R = (r, , ) /1 r 2; 0 4 ; 0 2 .

427
Des esta manera


Z 2 Z
4
Z 2
V (D) = r sin drdd
0 0 1
 3 2
Z 2 Z
4 r
= sin dd
0 0 3 1
Z 2
7
= [ cos ]04 d
3 0
!
7 2
= 1 2
3 2
!
14 2
= 1
3 2

Pregunta 2
Queremos calcular la integral la integral de la funcion f (x, y) = y 2 + z 2
sobre
la region R limitada por el cono a2 (h z)2 = h2 (x2 + y 2 ).
Este es un caso tpico que por la simetra de la region y del integrando,
bajo cualquier punto de vista conviene hacer el cambio a coordenadas
cilndricas para evaluar la integral. Sea

x = cos
y = sin
z = z

(x, y, z)
que tiene por Jacobiano de la transformacion = .
(, , z)
Observese que la region de integracion R se describe, proyectando sobre
el
plano XY z = 0 = (x2 + y 2 ) a2 = 0 a y 0 2. De
modo que
la proyecion de la region de integracion R se ve en el plano (, ) como
el
rectangulo 0 a y 0 2.
Por otra parte, sustituyendo en a2 (h z)2 h2 (x2 + y 2 ) =
h
0 z (a ) . De esta manera, la region de integracion en el nuevo
a
428
sistema queda como
 
h
R = (, , z) /0 a ; 0 2; 0 z (a )
a
La funcion a integrar f (y, z) = x2 + y 2 se reduce a f ( sin , z) = 2 .Se
tiene
entonces que

Z Z Z Z Z Z
2 2
2 () dddz

(x + y )dxdydz =
R R
h
Z a Z 2 Z a
(a)
= 3 dzdd
Z0 a Z0 2 0
h
= 3 (a ) dd
0 a
Z0
2h a
a3 4 d

=
a 0
a
2h 4 5

= a
a 4 5 0
h 4
= a
10

Pregunta 3.
El plano corta a los ejes coordenados en los puntos (1, 0, 0) , (0, 1, 0) y
(0, 0, 1) .
Si se proyecta la region sobre el plano XY, se tiene z = 0 = x + y = 1,
luego Rxy = {(x, y) /0 x 1, 0 y 1 x} . Por consiguiente, una
particion
de la region esta dada por

R = {(x, y, z) /0 x 1, 0 y 1 x, 0 x 1 x y}

429
Luego la integral se puede expresar:
Z Z Z Z 1 Z 1x Z 1xy
xyzdxdydz = xyzdzdydx
R 0 0 0
Z 1 Z 1x
1
= xy (1 x y)2 dydx
2
Z0 1 Z0 1x
1
xy y(1 x)2 2(1 x)y 2 + y 3 dydx

=
0 0 2
Z 1 " #
1 (1 x)4 (1 x)4 (1 x)4
= x 2 + dx
2 0 2 3 4
1 1
Z
1
= x(1 x)4 dx =
2 0 720

Autoevaluacion No 2
Tiempo 2 horas
Pregunta 1
Encontrar el volumen del solido, en el primer octante, acotado por los
planos
coordenados, el cilndro x2 + y 2 = 4 y el plano z + y = 3.
Pregunta R R2.R 1
Calcular R 1+(x2 +y 2 +z 2 )3/2 3/2
dxdydz, siendo R la esfera
[ ]
x 2 + y 2 + z 2 = a2 .
Pregunta 3.
y
Utilice la sustitucion u = , v = y + x2 para evaluar la integral
x
y + 2x2
Z Z
dydx
x2 + xy
Rxy

en donde Rxy es la region del primer cuadrante acotada por las curvas
y = 3 x2 , y = 8 x2 , e y = 2x.  1
(x, y) (u, v)
Nota: Usar la propiedad = en un dominio en que
(u, v) (x, y)
ambos Jacobianos son distintos de cero.

Pauta de Correccion.

430
Pregunta 1
El volumen de la region R esta dado por
Z Z
V = f (x, y)dxdy
R

En nuestro caso f (x, y) 


= z = 3y queda definida en
la region
del primer
2
octante dada por R = (x, y) /0 x 2, 0 y 4 x . Entonces,
al sustituir terminos en el integrando, obtenemos


Z 2 Z 4x2
V = (3 y) dydx
0 0
Z 2  2
4x2
y
= 3y dx
0 2 0
2 4 x2
Z   
= 3 4 x2 dx
0 2
2
3 x3
 x
= 2
4 x + 6arcsen 2x +
2 2 6 0
  8 9 8
= 6 4+ =
2 6 3
Pregunta 2.
1
Calculemos la integral triple de la funcion f (x, y, z) = 3/2
[1+(x2 +y2 +z2 )3/2 ]
sobre la region R limitada por la esfera x + y + z = a2 .
2 2 2

Aunque el problema se puede plantear en coordenadas directamente en


coordenadas cartesianas , por la forma de la region y del integrando, sera
dificl resolverlo directamente. En consecuencia, lo resolveremos en
coordenadas esfericas. Sea

x = r sin cos
y = r sin sin
z = r cos

(x, y, z)
que tiene por Jacobiano de la transformacion = r2 sin .
(r, , )
Entonces esta region R corresponde en el espacio r a la region R
dada por R = {(r, , ) /0 r a; 0 ; 0 2}
1
El integrando f (x, y, z) = 3/2 en las nuevas coordenadas
[1+(x +y +z2 )3/2 ]
2 2

431
1
queda f (r sin cos , r sin sin , r cos ) = ,de modo que la
(1 + r3 )3/2
integral por calcular toma la forma:

(r2 sin )
Z Z Z Z Z Z
1
i3/2 dxdydz = drdd
[1 + r3 ]3/2
h
R
1 + (x2 + y 2 + z 2 )3/2 R

a
2 a 2
r2 sin 2
Z Z Z Z Z
drdd = sin dd

0 0 0 [1 + r3 ]3/2 0 0 3
(1 + r )1/2
0

!Z !
2
1 1
2 1 1/2
cos |0 d = 4 1 1/2
|2
0
(1 + a) 0 (1 + a)
!
1
= 8 1
(1 + a)1/2
Pregunta 3.
R R y + 2x2
Vamos a evaluar la integral 2
dydx, en la region Rxy del primer
Rxy x + xy
cuadrante acotada por las curvas y = 3 x2 , y = 8 x2 , e y = 2x.
y
Usando el cambio de variables u = , v = y + x2 , la region Rxy se
x
transforma en la region del primer cuadrante acotada por las curvas:
y + x2 = 3 = v = 3
y + x2 = 8 = x = 8,
y
= 2 = u = 2;
x
y
= 0 = u = 0
x
Luego, Ruv = {(u, v) /0 u 2, 3 v 8} .
Calculemos ahora el Jacobiano
 de la transformacion usando
la propiedad
1
y2 1 1 2

(x, y) (u, v) = x
(x, y)
= = x x =
(u, v) (x, y) 2x 1 (u, v) y + 2x2
Sustituyendo terminos en la integral, obtenemos
y + 2x2
Z Z Z Z
1
2
dydx = dudv
x + xy 1+u
Rxy Ruv
Z 2 Z 8 Z 2
1 5
= dudv = du
0 3 1+u 0 1+u
= 5 ln |1 + u|20 = 5 ln 3

432
Autoevaluacion No 3

Tiempo 2 horas
Pregunta 1
R1R1 2
Considere la integral I = 0 y ex dxdy, dibuje la region de integracion,
cambie el orden de integracion y calculela.
Pregunta 2
R R ( xy )
Utilice la sustitucion u = x+y, y = xy para evaluar I = R
e x+y dA,
donde R es la region triangular acotada por las rectas x = 0, y = 0 y
x = 1 y.

Pregunta 3
En la esfera x2 + y 2 + z 2 = a2 los planos y = x , y = 3x determinan
un
sector esferico D, con x 0, y 0, z 0. Calcular el volumen de D

Pauta Autoevaluacion
Pregunta 1 R1R1 2
La integral I = 0 y ex dxdy esta definida sobre la region de tipo II
dada
por RII = {(x, y) R2 /y x 1, 0 y 1} . Invirtiendo la particion
de la region a una de tipo I, tenemos:

RI = (x, y) R2 /0 x 1, 0 y x


entonces
Z 1 Z 1 Z 1 Z x
x2 2
e dxdy = ex dydx
0 y 0 0
Z 1 h
x 2 x
i 1h x2
i1
ye dx = e
0 0 2 0
 
1 1
= 1
2 e
Pregunta 2

Usando la sustitucion

u= x+y
(1)resolviendo x, y en terminos de u, v produce
v = xy

433
u+v

x= 2
uv (2) determinemos como tranforma la region triangular
y= 2

R = (x, y) R2 /0 x 1, 0 y 1 x


Reemplazando valores en (2)



x+y =1 u =1
x =0 = u + v = 0
y =0 v =u

Luego, la proyeccion de la region en el plano (u, v) es
R = {(u, v) R2 /0 u 1, u v u}
Calculemos
el Jacobiano
de la transformacion, nos queda
(x, y) 1 1 1 1
(u, v) = 1 1 = 2 = 2
2 2
2 2
La funcion a integrar es, en terminos de las nuevas variables

Z Z Z Z  
( xy
x+y )
1v
( u )
e dA = e dudv
R R 2
Z 1Z u
1 1h
  Z
v
( u ) 1 v
iu
e dudv = ue( u ) du
0 u 2 2 0 u
 1
(e1 e) 1 (e e1 ) u2
Z
udu =
2 0 2 2 0
1
(e e )
=
4
Pregunta 3
El volumen de la region R esta dado por
Z Z Z
V (D) = dxdydz
R

Usando coordenadas esfericas, tenemos

x = r sin cos
y = r sin sin
z = r cos

(x, y, z)
que tiene por Jacobiano de la transformacion = r2 sin .
(r, , )
Entonces esta region R corresponde en el espacio r a la region R

434
dada por R = (r, , ) /0 r a; 0 2 ; 4 3 .


De esta manera


Z
3
Z
2
Z a
V (D) = r2 sin drdd

4
0 0

a3
Z Z
3 2
= sin dd

4
0 3

3 Z
a 3
= d
3
4

a3   a3
= =
3 3 4 36

435
Captulo 5

Integral de Linea

Es una forma de generalizar la integral de Riemann en una variable,


conocida en el curso anterior, a una integral definida sobre una curva del
espacio bi o tridimensional.
Planteamiento

Supongamos que tenemos:


a) f : U IR3 IR funcion escalar continua en U y
b)
c : [a, b] IR3 ,
c (t) = (x(t), y(t), z(t)) una trayectoria de clase
C 1,
es decir una curva en IR3 .
La integral de la funcion f calculada sobre la trayectoria descrita por la
curva C se define de la forma dada a continuacion.
La integral de trayectoria de f (x, y, z) a lo largo de la trayectoria
c ,

3 1
esta definida cuando c : [a, b] IR es de clase C y ademas la funcion
compuesta f (t) = f (x(t), y(t), z(t)) es continua en I = [a, b]; por:
Z Z b
f ds = f (x(t), y(t), z(t)) k
c 0 (t)k dt


c a

Alternativamente se denota como:


Z Z Z b
f ds = f (x, y, z)ds = f (
c (t)) k
c 0 (t)k dt


c

c a

Observaciones

i) Si f (x, y, z) = 1, tenemos la longitud del arco.

436
En efecto, como f (x, y, z) = 1,
Z Z Z b
f (x, y, z) ds = ds = kc 0 (t)k dt


c
c a
Z bq
= (x0 (t))2 + (y 0 (t))2 + (z 0 (t))2 dt
a

ii) Si C la curva definida por c (t) es de clase C 1 por tramos o f (


R
c (t))
es continua por tramos, entonces calculamos
c
f ds en segmentos sobre los
Rb
0
cuales a f ( c (t)) k c (t)k dt es continua y sumamos las integrales resultantes.
Por ejemplo si c = c
1
c
2
c con a lo sumo sus extremos en comun
3

Z Z Z Z
f (x, y, z)ds = f (x, y, z)ds + f (x, y, z)ds + f (x, y, z)ds


c

c1

c2

c2

Algunos ejemplos donde aplicamos esta definicion; son los siguientes.


Ejemplo 1
Sea la funcion escalar f (x, y, z) = x2 + y 2 + z 2 continua en R y

c : [0, 2] IR3 , definida por c (t) = (cos t, sin t, t) una trayectoria de
R
clase C 1 . Calcular
c
f (x, y, z)ds

Solucion:
En este caso tenemos la ecuaciones parametricas :
x(t) = cos t, y(t) = sin t, z(t) = t
Luego la funcion compuesta es f (x(t), y(t), z(t)) = cos2 t + sin2 t + t2 =
1 + t2
Ademas c 0 (t)k = 2
c 0 (t) = ( sin t, cos t, 1) = k
Por lo tanto, sutituyendo terminos en la integral, queda

Z Z 2
f (x, y, z)ds = f (x(t), y(t), z(t)) k
c 0 (t)k dt


c
Z0 2

= 1 + t2 2dt
0
2
t3

2 t+
3 0

2 2
3 + 4 2

=
3

Ejemplo 2

437
R
Calcular la siguiente integral de trayectoria
c
f (x, y, z)ds donde
a) f (x, y, z) = x + y + z y c (t) = (cos t, sin t, t), t [0, 2]
Solucion:
En este caso se tiene que x(t) = cos t, y(t) = sin t, z(t) = t
Que produce la funcion compuesta f ( c (t)) = cos t + sent + t,

0
Ademas c (t) = ( sin t, cos t, 1) y kc 0 (t)k = 2. Entonces
Z Z 2
f (x, y, z)ds = 2(cos t + sent + t)dt
c 0

2
t2

= 2 sent cos t +
2
Z 0
4 2 2
= f (x, y, z)ds =
c 2

R Como taller adicional, evaluar las siguientes integrales de trayectoria




c
f (x, y, z)ds
donde:
b) f (x, y, z) = cos z y c (t) = (cos t, sin t, t), t [0, 2]
c) f (x, y, z) = x cos z y c (t) = (t, t2 , 0), t [0, 1]

Ejemplo 3
a) Demostrar que la integral de lnea de f (x, y), a lo largo de una trayec-
toria dada en coordenadas polares por r = r(), con 1 2 es:
s  2
Z 2
2
dr
I= f (r cos , r sin ) r + d
1 d

b) Calcular la longitud de arco de r = 1 + cos , con 0 2


Solucion:

a) En primer lugar, determinemos la ecuaciones parametricas de la


trayectoria en coordenadas polares

x = r cos = x = r() cos


=
c () = (r() cos , r () sin )
y = r sin = y = r () sin

Derivando la ecuacion de la trayectoria, queda

438

c 0 () = (r0 () cos r () sin , r0 () sin + r() cos )

Lo que implica

q
k
c 0 ()k = (r0 () cos r () sin )2 + (r0 () sin + r() cos )2
q
= r ()2 + r0 ()2

Por tanto
Z Z 2 q
I= f (x, y) ds = f (r cos , r sin ) r ()2 + r0 ()2 d


c 1

b) En este caso se tiene f (x, y) = 1, y como r = 1 + cos , con


0 2 q q
Obtenemos r () + r () = (1 + cos )2 + ( sin )2
2 0 2

Entonces:
Z 2 q
s = (1 + cos )2 + ( sin )2 d
Z0 2
= 2 + 2 cos d
0
Z

= 2 | cos |d
0 2
Z Z   

= 2 cos d + cos d =
0 2 2
8

Es util tener en cuenta que la integral de trayectoria se puede aplicar a


problemas de fsica e ingeniera
1. Si f (x, y, z) representa la masaR por unidad de longitud de un alambre
delgado con la forma de C, entonces c
f ds es la masa total del alambre.
2. Si f (x,
R y, z) es la componente tangencial de una fuerza en (x, y, z) de
C, entonces c
f ds representa el trabajo realizado por la fuerza al mover una
partcula a lo largo de la curva C.
3.- Si f : I IR2 IR funcion escalar continua en I = [a, b] y
f (x, y)ds = a f (x(t), y(t)) k
c 0 (t)k dt representa el
R Rb
f (x, y) 0,entonces
c
area de una pared cuya base es la imagen de c y la altura f (x, y).

439
Figura 5.1: Pared zigzagueante

Ejemplo 4

a) Calcular analticamente la masa M de un resorte que tiene la forma


de una helice dada por la ecuacion:

r (t) = (5cost; 5sent; 4t); t [0; 2] si la
densidad en cada punto (x, y, z) es el cuadrado de la distancia en cada punto
al origen de coordenadas.

Figura 5.2: Helice circular

b) Si el centro de masa de un resorte con forma de alambre delgado de


masa M, se define por (XM ; YM ; ZM ) con

440
Z
1
XM = x(x, y, z)ds
M
Zr
1
YM = y(x, y, z)ds
M
Zr
1
ZM = z(x, y, z)ds
M
r

Calcular la componente ZM del resorte helicoidal


Solucion.
a) La formula para el calculo de la masa del resorte es:
Z Z b
M= (x, y, z)ds = (

r (t)) k

r 0 (t)k dt


r a

donde se tiene que:



r (t) = (5cost; 5sent; 4t); t [0; 2] = r 0 (t)

0
2
= (5 sin t; 5 cos t; 4)
2
= k r (t)k = 25 sin t + 25 cos t + 16 = 41 y la densidad
(x; y; z) = x2 + y 2 + z 2 = ( r (t)) = (25 + 16t2 )
Reemplazando en el integrando, queda
Z Z 2
M = (x, y, z)ds = (25 + 16t2 ) 41dt

r 0
2

  
16 3 128 3
= 41 25t + t = 41 50 +
3 0 3

b) La componente ZM del centro de masa es


Z Z b
1 1
ZM = z(x, y, z)ds = z(t)(r (t)) k

r 0 (t)k dt
M r
M a
 2
1
Z 2
2
41 25 2 4
= t(25 + 16t ) 41dt = t + 4t
M 0 M 2 0
2
41 [50 + 64 ]
50 2 + 64 4 = 
 
= 
M 128 2
50 +
3

5.1. Campos vectoriales




Sea F = (F1 , F2 , F3 ) un campo vectorial en R3 continuo definida sobre
la trayectoria
c : [a, b] R3 con
c perteneciente a C 1 .

441


Definimos la integral de lnea de F a lo largo de c por
Z b



Z


F ds = F (c (t))
c 0 (t)dt


c a

Esta definicion se puede generalizar para campos vectoriales en Rn

Figura 5.3: Campo vectorial radial

Figura 5.4: Campo circular tangencial

Otras
R observaciones.



1) c
F ds tambien se puede definir si F (c (t))
c 0 (t) es continua
por tramos, expresando en este caso la integral como una suma de integrales
del tipo anterior.
2) Si C es trayectoria tal que c 0 (t) 6=

0 , podemos usar el vector
tangente unitario T para calcular la integral de lnea como sigue.
b

442
Figura 5.5: Flujo de un fluido en un tubo cilndrico largo


c 0 (t)
Tb =
k
c 0 (t)k
Z


Z b


c 0 (t) 


F ds =
F ( c (t))
0
kc 0 (t)k k c (t)k dt
c a
Z bh
F ( c (t)) T k

c 0 (t)k dt
i
= b
Za

b
= F T ds


c

Por lo tanto


b
Z Z
F d
s = F T ds


c

c


Si es el angulo entre F y Tb


b





Z Z
F T = F T cos = F cos =

F ds = F cos ds


c

c



Si F es un campo de fuerza , entonces esta ultima integral es precisa-
mente la definicion de trabajo realizado por el campo de fuerza sobre una
partcula de masa unitaria que se mueve desde el punto inicial hasta un
punto terminal a lo largo de la curva C descrita por la trayectoria c (t).
3) Otra manera usual de escribir la integral de lnea es:




Z Z

F ds = F1 dx + F2 dy + F3 dz donde F = (F1 , F2 , F3 )


c

c

443
As:
Z b 


Z Z
dx dy dz
F d

s = F1 dx + F2 dy + F3 dz = F1 + F2 + F3 dt


c

c a dt dt dt
Z b
dx dy dz
= (F1 , F2 , F3 ) ( , , )dt
a dt dt dt
b


Z
= F (c (t))
c 0 (t) dt
a

Ejemplo 1:
R
Evaluar I = c x2 ydx + (x2 y 2 ) dy , si C es el arco de la parabola
y = 3x2 desde (0, 0) hasta (1, 3) .

Figura 5.6: Grafico de la curva y = 3x2

Solucion:
Al parametrizar la trayectoria se tiene C : x = t, y = 3t2 con 0 t 1.
luego
c (t) = (t, 3t2 ) =
c 0 (t) = (1, 6t)
Sustituyendo terminos en el integrando, obtenemos
Z 1
(3t4 , t2 9t4 ) (1, 6t) dt
  
I =
Z0 1
3t4 + t2 9t4 6t dt
  
=
0
 1
3 5 6 4 54 6 69
= t + t t =
5 4 6 0 10

444
Ejemplo 2
R
Evaluar c yzdx + xzdy + xydz donde C esta formada por los segmentos
de rectas que unen A (1, 0, 0) a B (0, 1, 0) a C (0, 0, 1)
Solucion.


En este caso C = AB + BC + CA por lo cual
Z Z Z
yzdx + xzdy + xydz = yzdx + xzdy + xydz + yzdx + xzdy + xydz

c AB BC
Z
+ yzdx + xzdy + xydz
CA

Pametrizando cada segmento de trayectoria, produce



AB : x = 1 t = dx = dt
y=t = dy = dt
z=0 = dz = 0
Z Z 1


yzdx + xzdy + xydz = 0dt = 0
AB 0


BC : x = 0 = dx = 0
y =1t = dy = dt
z=t = dz = dt
Z Z 1



yzdx + xzdy + xydz = 0dt = 0
BC 0


CA : x = t = dx = dt
y=0 = dy = 0
z =1t = dz = dt
Z Z 1


yzdx + xzdy + xydz = 0dt = 0
CA 0

Por tanto Z
yzdx + xzdy + xydz = 0


c

Ejemplo 3
R

Evaluar I =
c
F d
s , donde F (x, y) = (xy, x2 y) y

c : [1, 1] IR2 tal que
c (t) = (t, |t|).
c
/ C1 .
Solucion

445
La curva C es continua por tramos.
As
c : [1, 0] IR2 ,
1
c (t) = (t, t)
1 = dx = dt, dy = dt

2
c2 : [0, 1] IR , c 2 (t) = (t, t) = dx = dt, dy = dt



De modo que: c = c 1 + c 2





Z Z Z
F d

s = F d

s + F d
s


c

c1

c2
Z Z
2
= xydx + x ydy + xydx + x2 ydy


c1

c2
Z 0 Z 1
= (t2 + t3 )dt + (t2 + t3 )dt = 0
1 0

Por tanto


Z
F d

s =0


c

5.2. Cambio de parametrizacion


La pregunta crucial que surge al pensar en una curva es cambiara el
valor de la integral si se cambia la parametrizacion de la curva sobre la que
se integra?.
En primer lugar, ilustremos esto con el siguiente ejemplo.

Ejemplo 1
Si C es la mitad inferior del crculoR unitario en el plano xy que une los
puntos (1, 0) con (1, 0) . Calcular C (1 + xy)ds parametrizando de dos
maneras diferentes esta curva.
Solucion.

 


0 t
Sea c (t) = (t, 1 t ) = c (t) = 1,
2 =
1 t2
c 0 (t)k = 1
k
1 t2
entonces
Z Z 1 1
(1 + xy)ds = (1 t 1 t2 ) dt

c 1 1 t2
Z 1 
1
= t =
1 1 t2

446
Por otro lado, si ponemos c (t) = (cos t, sin t) con t 2

c 0 (t) = ( sin t, cos t) = k c 0 (t)k = 1
entonces, al reemplazar los terminos del integrando queda
Z 1 2
sin2 t
Z 
(1 + xy)ds = (1 + sin t cos t) dt = t + =

c 1 2
Por lo tanto, hemos obtenido el mismo valor para la integral a
lo
largo de estas dos trayectorias diferentes que tienen la misma
traza.
En segundo lugar, enunciemos un resultado general con el siguiente teo-
rema.
Teorema 5.2.1. Si
c (t) y

p (t) son dos parametrizaciones distintas suaves
o suaves por tramos, que conecta los puntos P0 y P1 , que tienen la misma
traza y direccion, y si f (x, y, z) esta definida y es continua sobre la traza,
entonces Z Z
f (x, y, z)ds = f (x, y, z)ds


c

p

5.2.1. Reparametrizacion
Sea h : I1 I una funcion de clase C 1 con valores reales que sea biyectiva
entre I = [a, b] y I1 = [a1 , b1 ]. Si
c : [a, b] IR3 una trayectoria C 1 por
tramos. Entonces a la composicion

p = c h : [a , b ] IR3
1 1

La llamamos reparametrizacion de c
El valor de la integral de lnea no vara por cambios de parametrizacion
de la curva c , excepto por el signo si hay cambios de orientacion, el teorema
correspondiente es:


Teorema 5.2.2. Sea F un campo vectorial continuo y c : [a, b] IR3 ,
1
3
trayectoria de clase C , y sea p : [a1 , b1 ] IR una reparametrizacion de

c . Entonces
i) Si
p conserva la orientacion:



Z Z
F d
s = F d
s


c

p

ii) Si

p invierte la orientacion



Z Z

F ds = F d
s


p

c

447
Figura 5.7:

Ejemplo 1:


Sea F (x, y, z) = (yz, xz, xy) campo vectorial y c : [5, 10] IR3

2 3
trayectoria dada por c (t) = (t, t , t ) .
Una trayectoria opuesta: c : [5, 10] R3 se puede construir:
op


c (t) = c(a + b t)
op

en este caso queda


c (t) =
op
c (5 t) = ((5 t), (5 t)2 , (5 t)3 ) .
R

F d F d

R
Evaluar
c
s y
c op
s.

Solucion:
10  


Z Z
dx dy dz
F d
s = F1 + F2 + F3 dt


c 5 dt dt dt
Z 10
= yzdx + xzdy + xydz
5
Z 10
5
t + 2t5 + 3t5 dt

=
5
= 984,375

por otro lado


10


Z Z
F d
s = yzdx + xzdy + xydz


c op 5

448
Z 10
(5 t)5 2(5 t)5 3(5 t)5 dt
 
=
5
Z 10 10
6(5 t)5 dt = (5 t)6 5

=
5
= 984,375

5.3. Independencia de trayectoria


R
Cuando el valor de una integral
c
F d
s sobre una trayectoria contin-
ua por tramos que conecta dos puntos de una region R depende solamente
de los puntos extremos, se dice que la integral es independiente de la
trayectoria.

Ejemplo. Independencia de trayectoria en una integral de lnea


 2 

y 2y
Si F (x, y) = , ; A = (1, 1); B = (4, 2)
x2 x


Calcular el trabajo realizado por el campo de fuerza F al llevar un objeto
desde A hasta B, por:
a) un camino compuesto de un tramo horizontal seguido de un tramo
vertical;
b) un camino compuesto de un tramo vertical seguido de un tramo hori-
zontal.
Solucion.
a) Si llamamos C a la curva, la podemos subdividir en las curvas C1 y
C2
Tendremos




Z Z Z

F ds =
F ds + F d
s


c

c1

c1

Calculamos
 ambas integrales por separado
x= 1+t
C1 = , 0t3
y= 1


y2
Z Z
2y
= F d
s = 2
dx dy

c1 c1 x

x
Z 3  3
1 1 3
= 2
dt = =
0 (1 + t) (1 + t) 0 4

449

x= 4
C2 = , 0t3
y = 1t
3

y2 2(1 t)
Z Z Z
2y
= F d
s = 2
dx dy = dt


c2

c2 x x 0 4
 3
1 1 3 9 3
= t t2 = =
2 4 0 2 4 4
Con lo que resulta
y2
Z  
2y 3 3
2
dx dy = + =0


c x x 4 4

b) Llamemos C * a este otro camino, tambien lo podemos separar en dos


tramos C3 y C4 . Tendremos




Z Z Z

F ds =
F ds + F d
s


c

c3

c4

Calculamos
 ambas integrales por separado haciendo parametrizaciones:
x= 1
C3 = , 0t3
y = 1t


y2
Z Z
2y
= F d
s = 2
dx dy

c c3 x x
Z 33
2(1 t) 3
(1)dt = 2t t2 0 = 3

=
0 1

x= 1+t
C4 = , 0t3
y = 2



Z Z
2y
= F d
s = dyy2
x 2 dx

c4 x

c4
Z 3 3
(2)2

4
= dt = =3
0 (1 + t)2 (1 + t) 0
Sumando se obtiene
y2
Z
2y
2
dx dy = 3 + 3 = 0


c x x
Por ambos caminos dio el mismo resultado.
Intente otro camino para esta integral que lleve desde A hasta B.

450
5.4. Campos Conservativos
Ciertos campos de fuerza importantes en la fsica provienen de un poten-
cial escalar. Si existe una funcion definida en una region R y si al campo


de fuerza F tiene la propiedad de que


F = = grad


entonces se dice que es un potencial de F .

5.4.1. Campo gradiente


Un campo vectorial continuo el cual se obtiene como el gradiente de una
funcion escalar se llamara campo gradiente y una funcion de la cual se
obtiene, la funcion potencial


F =


Teorema 5.4.1. (Teorema fundamental) Sea F un campo gradiente con
potencial definida en una region R y sean P0 y P1 puntos cualesquiera de
R, entonces:


Z
F d s = (P2 ) (P1 )


c
donde c : [a, b] IR3 tal que
c (a) = P , y
1
c (b) = P , y
2
c es una
1
trayectoria de clase C .
Demostracion



Z Z

F d
s = dx + dy + dz


c
c x y z
Z b 
dx dy dz
= + + dt
a x dt y dt z dt
Z b

= (c (t)) dt = [ (
c (t))]b
a
a t
= (P2 ) (P1 )
Por tanto


Z
F d
s = (P2 ) (P1 )


c

451
Adicionalmente podemos afirmar que


Teorema 5.4.2. Si F es un campo gradiente en una region R, entonces
R


c
F d
s es independiente de la trayectoria.


Comentario: Si F es campo gradiente existe definida en R tal que

F =, y el teorema fundamental garantiza la independencia de trayectoria.


Observacion: No es dificl ver que el recproco de este teorema tambien
es cierto.


Teorema 5.4.3. Si F es un campo vectorial continuo sobre una region R y
R

si c
F d

s es independiente de la trayectoria, entonces F es un campo
gradiente.

Curva simple C, se define como la imagen de una aplicacion c : I IR3


1
de C que sea uno a uno en el intervalo I. Una curva simple es aquella que
no se intersecta a si misma, c (a) y c (b) se llaman punto inicial y punto
final de la curva.
Curva cerrada simple, se define como c : [a, b] IR3 de C 1 tal que
1) es uno a uno en [a, b]
2) c (a) =
c (b) Observacion:
i) Si la curva satisface solo (2) es curva cerrada.
ii) Las curvas simples cerradas tienen dos direcciones de movimiento posi-
ble.
Integrales de lnea sobre curvas simples orientadas y curvas cerradas sim-
ples C.
Sea C una curva simple orientada imagen de c : [a, b] IR3 entonces
definimos:



Z Z Z Z

F ds =
F ds y f ds = f ds
C

c C

c
De las curvas cerradas y la independencia de trayectoria se tiene el siguiente
teorema.
R

Teorema 5.4.4. C F d
s es independiente de la trayectoria en R si y
R

solo si C F d s = 0 para toda curva cerrada C contenida en R.

Un dominio D es conexo si dos puntos cualesquiera de D se pueden unir


con un segmento de recta y todos los puntos de la recta pertenecen a D
Una region es simplemente conexa y abierta si ella es un conjunto abierto
y tal que toda curva simple cerrada en D encierra puntos que solo estan en
el interior de D. Si un campo de fuerza tiene la propiedad de que el trabajo

452
realizado sobre una partcula en movimiento conforme se mueve de un punto
a otro es independiente de la trayectoria, se llama campo conservativo.
De los teoremas anteriores se puede formular lo siguiente:Una condicion
necesaria y suficiente para que un campo de fuerzas sea consevativo es que
sea un campo gradiente

Teorema 5.4.5. Sean M (x, y) y N (x, y) funciones con derivadas parciales


continuas en un conjunto abierto y conexo D.


Entonces el campo vectorial F (x, y) = (M (x, y), N (x, y))es conservativo
M N
si y solo si = en D
y x

Ejemplo 1


Sea F (x, y) = (2xy, (x2 y))
a) Probar que es campo conservativo.
b) Hallar funcion potencial.
Solucion.
N
= 2x


x


a) = Campo conservativo en IR2
M

= 2x

x
b) Existe Potencial (x, y) tal que
Z
(x, y)
= 2xy (x, y) = 2xydx + h1 (y)
x
Z
(x, y) 2
x2 y dx + h2 (x)

= x y (x, y) =
y
(x, y) = x2 y + h1 (y) 2 y2
2 (x, y) = x y +C
(x, y) = x2 y y2 + h2 (x) 2


(Rotacional) Sea F (x, y, z) = (P (x, y, z), Q(x, y, z), R(x, y, z)) funcion



vectorial, se define el rotacional de F ,denotado por rot F (x, y, z) o

F (x, y, z) por:
bi j
b k
b



F (x, y, z). =
x y z

P Q R

453


Teorema 5.4.6. Si F (x, y, z) = (P (x, y, z), Q(x, y, z), R(x, y, z)) funcion
vectorial donde P, Q, R son funciones con derivadas parciales continuas en
un conjunto abierto y conexo D. Entonces



F es un campo conservativo F (x, y, z) = 0 en D
P Q P R Q R
es decir si y solo si = , = , =
y x z x z y

Ejemplo 2


Verifique que F (x, y, z) = (yz, xz, xy) es un campo conservativo en IR3
Solucion.



bi j
b k
b


F (x, y, z). =
x y z
yz xz xy
= ((x x) , (y y) , (z z))
= (0, 0, 0)



o sea F (x, y, z) = 0 en IR3

Ejemplo 3


Dado el campo vectorial F (x, y, z) = (3y 2 z+yex , 6xyz+ex , 3xy 2 ).Verifique
que es campo conservativo y calcular potencial.

Solucion:
R Q
= 6yz + ex = 6yz + ex
y z
R P
= 3y 2 = 3y 2
x z
Q P
= 6yz + ex = 6yz + ex
x y



Luego F (x, y, z) = 0 en IR3 ,es entonces claramente un campo
vectorial
conservativo, entonces existe (x, y, z) tal que
Z
(x, y, x) 2 x
3y 2 z + yex dx + h1 (y, z)

= 3y z + ye (x, y, z) =
x
= 3xy 2 z + yex + h1 (y, z)

454
Z
(x, y, x)
= 6xyz + ex (x, y, z) = (6xyz + ex ) dx + h1 (x, z)
y
= 3xy 2 z + yex + h1 (x, z)

Z
(x, y, x)
= 3xy 2 (x, y, z) = 3xy 2 dx + h1 (x, z)
z
= 3xy 2 z + h1 (x, y)

De aqu se deduce que

(x, y, z) = 3xy 2 z + yex

R
Ademas, si se pide calcular I = C
F d

s y C es la curva descrita
por

c (t) = (cos t, sent, t), t [0, 2] , entonces



Z
F d
3
s = 3xy 2 z + yex 0

I =
C
= (1, 0, 2) (1, 0, 0) = 0 0 = 0

(Divergencia de un campo vectorial)




Si F (x, y, z) = (P (x, y, z), Q(x, y, z), R(x, y, z)) tal que P, Q y R tienen


derivadas parciales en alguna region D. La divergencia de F se denota por:



P (x, y, z) Q(x, y, z) R(x, y, z)
div F = F = + +
x y z



Si F es un campo vectorial, entonces la div F da informacion acerca del

flujo en un punto P(x,y,z):





i) Si div F < 0 en un punto P(x,y,z) , entonces el flujo del campo F se
orienta hacia el punto y se dice que hay un sumidero en P.



ii) Si div F > 0 en un punto P(x,y,z) , entonces el flujo del campo F se
orienta desde el punto y se dice que hay una fuente en P.



iii) Si div F = 0 en un punto P(x,y,z) , entonces el flujo del campo F
es nulo.

455
Introduccion Teorema de Green
El teorema de Green, que enunciaremos a continuacion, dice que bajo
ciertas condiciones las integrales de lnea pueden expresarse y calcularse con
integrales dobles.
Comencemos por definir una region compacta R descrita simultaneamente
por las desigualdades

1 (x) y 2 (x), a x b

denominada region orientada en direccion del eje y

1 (y) x 2 (y), c y d
denominada region orientada en direccion del eje x donde las funciones
1 y 2 son continuas y seccionalmente suaves en [a, b] y las funciones
1 y 2 son continuas y seccionalmente suaves en [c, d] .
En esta region cerrada R con una frontera continua seccionalmente suave
consideramos definidas las funciones M y N continuamente
diferenciables ( o de clase C1 ) definidas en R.
La curva C frontera de la region R, se dice que esta orientada
positivamente ,si al caminar sobre ella la region R estara a su izquierda.
En este marco de ideas y condiciones se plantea el teorema de Green.

5.4.2. Teorema de Green


Sea R una region cerrada y acotada con frontera C,orientada en sentido
M N
positivo. Si M, N, y son continuas en R. Entonces
y x
Z Z Z  
N M
M dx + N dy = dA
C R x y

456
Figura 5.8: Region cerrada de tipo a, frontera orientada positiva

Demostracion.
Supongamos que R esta descrita por

1 (x) y 2 (x), a x b

Z Z Z
M dx = M dx + M dx
C C1 C2
Z b Z a
= M (x, 1 (x))dx + M (x, 2 (x))dx
a b

Z Z b
= M dx = [M (x, 1 (x)) M (x, 2 (x))] dx
C a
Por otra parte
Z Z Z b Z 2 (x) Z b
M M
dA = dA = [M (x, 2 (x)) M (x, 1 (x))] dx
R y a 1 (x) y a

Luego Z Z Z
M
M dx = dA
C R y
De manera simlar y considerando ahora

1 (y) x 2 (y), c y d

Se establece que Z Z Z
N
N dy = dA
C R x

457
Figura 5.9: Region cerrada de tipo b, frontera orientada positiva

Por lo tanto
Z Z Z  
N M
M dx + N dy = dA
C R x y

Ejemplo 1

Evaluar la integral
Z
y 2 + senx2 dx + cos y 2 x dy
 
I=
C

donde C es la frontera de la region cuadrada 0 x 1, 0 y 1

Figura 5.10: Frontera de la region cuadrada 0 x 1, 0 y 1

458
Solucion.
Usando el Teorema de Green podemos afirmar que
M
M (x, y) = y 2 + senx2 = = 2y
y
N
N (x, y) = cos y 2 x = = 1
x
Por lo tanto aplicando el teorema
Z 1Z 1 Z 1
1
(1 2y)dy = y y 2 0 = 2

I= (1 2y)dxdy =
0 0 0

Comentario. Sin usar el teorema de Green este ejercicio resulta muy


complicado de resolver, vale la pena que lo intente.

459
Ejemplo 2

I
ey dx + (xey + 2y)dy con D = (x; y) IR2 /x2 + y 2 1

Calcular I =
C

Figura 5.11: D = (x; y) IR2 /x2 + y 2 1




Solucion
Por el Teorema de Green podemos afirmar que

M
M (x, y) = ey = = ey
y
N
N (x, y) = xey + 2y = = xey + 2y
x
Por lo tanto aplicando el teorema
Z Z   Z Z
N M
I= dxdy = (ey ey )dxdy = 0
D x y

Corolario 5.4.1. Aplicando el teorema de Green a M = y y N = x se


deduce que I
1
A(D) = xdy ydx
2 C
donde A(D) es el area de la region D contenida en el plano XY , y C es el
borde de D recorrrido en sentido positivo. Esta formula nos permite calcular
un area en terminos de una integral de lnea.

460
x2 y 2
Figura 5.12: Region + 2 1
a2 b

Ejemplo 3
x2 y 2
Consideremos la region 2 + 2 1, cuyo borde es una elipse de semi-ejes
a b
dados por a y b.Calcular el area de la region D
Solucion
Podemos parametrizar la elipse usando x(t) = acost , y(t) = bsent,
t [0, 2], y luego se obtiene


r (t) = (acost, bsent) =
r 0 (t) = (asent, b cos t)

1 2
Z
A(D) = (xy 0 yx0 )dt
2 0
1 2
Z
= (ab cos2 t (absen2 t))dt
2 0
= ab

5.5. Aplicaciones de la integral de trayectoria


A continuacion se resumen las formulas para el calculo de la masa y el
momento de resortes, varillas delgadas y alambres a lo largo de una curva
suave C en el espacio.
R
Masa: M = C (x, y, z) ds, donde = (x, y, z) es la densidad y ds un
elemento diferencial de arco.
MomentosR con respecto a los planos R coordenados: R
Myz = C x (x, y, z) ds; Mxz = C y (x, y, z) ds; Mxy = C z (x, y, z) ds
Coordenadas del centro de Masa
Myz Mxz Mxy
x= ; y= ; z=
M M M
Momentos
R 2 de inercia con respecto a los
R ejes2 y otras rectas
2 2
Ix = R C (y + z ) (x, y, z) ds, Iy = C (x + z ) (x, y, z) ds;
Iz = C (y 2 + z 2 ) (x, y, z) ds;

461
R
IL = C r2 (x, y, z) ds donde r (x, y, z) es la distancia del punto (x, y, z)
a la recta L.
Radio r de giro con respecto a una recta L.
IL
RL =
M

Problema 1
Dado el resorte de densidad constante = 1 con forma helicoidal


r (t) = cos 4tbi + sen4tb
j + tb
k, 0 t 2.

Encontrar la masa del resorte y el momento de inercia con respecto al eje z


Solucion
La masa del resorte esta dada por
Z
M = (x, y, z) ds
ZC

= (1)ds
C
Z 2
= |

r 0 (t)| dt
0
q
Encontremos primero |r 0 (t)| = (4sen4t)2 + (4 cos 4t)2 + 1 = 17.
Luego evaluemos la integral
Z 2
M = |

r 0 (t)| dt
Z0 2
= 17dt
0

M = 2 17

El momento de inercia con respecto al eje z se define por:

Z
x2 + y 2 ds

Iz =
ZC2
cos2 4t + sen2 4t (1) 17dt

=
Z0 2
= 17dt = 2 17
0

462
Problema 2
Sea un arco de metal con forma de semicircunferencia y 2 +z 2 = 1, z 0.
Si la densidad en el punto (x, y, z) del arco es (x, y, z) = 2 z , determinar
las coordenadas del centro de masa del arco.
Solucion.
Por simetra sabemos que x = 0; y = 0; dado que el arco esta el el plano
yz con la masa distribuida simetricamente con respecto al eje z.
Para encontrar z , parametizamos el arco de semicircunferencia
q como:

r (t) = cos tb
j+sentb

k, 0 t = | r (t)| = (sent) + (cos t)2 =
0 2

1
Por otra parte, (x, y, z) = 2 z = ( r (t)) = (2 sent)
Ahora, determinemos la masa utilizando la definicion de integral de trayec-
toria

Z Z
M = (x, y, z) ds = (

r (t)) |

r 0 (t)| dt
ZC 0

= (2 sent) (1) dt = [2t + cos t]0


0
M = 2 2

A continuacion calculemos el momento de inercia Mxy


Mxy 8
Luego, z = = 0, 57.
M 2 (2 2)
Finalmente, la coordenada del centro de masa es (0; 0; 0, 57) .

Problema 3
Un alambre tiene la forma de una semicircunferencia de radio a. La densi-
dad lineal de masa en un punto P es directamente proporcional a la distancia
de P a la recta que pasa por los extremos del alambre, cual es la masa total
del alambre?.
Solucion
En primer lugar situar el alambre en el plano xy de sistema coordenado,
como en la figura:
Ahora, la densidad puede interpretarse como un campo escalar, ya que,
en
cada punto P = (x, y) de la curva que describe el alambre, la densidad es
(x, y) = ky.
Calcular la masa total equivale a sumar la densidad sobre todos los puntos

463
de la curva, y as lo que queremos determinar es
Z Z b
M= (x, y, z) ds = (

r (t)) |

r 0 (t)| dt
C a

donde C es la curva que describe el alambre, la semicircunferencia de


radio a.
Una parametrizacion para C es: r (t) = (a cos t, asent) , t [0, ]
entonces r (t) = (asent, a cos t) = |

0 r 0 (t)| = a
Lo que implica que la masa del alambre es

Z Z
2
M = (kasent) (a) dt = ka sentdt
0 0
= ka2 [ cos t]0 = 2ka2

Problema 4

Un alambre tiene forma de circunferencia, x2 + y 2 = a2 . Determine su


masa y su momento de inercia respecto de un diametro si la densidad en un
punto (x, y) del alambre esta dada por la funcion (x, y) = |x| + |y| .
Solucion
La masa del alambre viene dada por la expresion:

Z Z b
M= (x, y, z) ds = (

r (t)) |

r 0 (t)| dt
C a

siendo C la curva cuya trayectoria representa la forma del alambre, en


este
caso una circunferencia que parametrizamos por:



r (t) = (acost, asent); t [0, 2]

que es de clase C1 .

r 0 (t) = (asent, a cos t) = k

r 0 (t)k = 1
Por lo tanto la masa es igual a

464
Z 2 Z 2
M = (

r (t)) |

r 0 (t)| dt = (|a cos t| + |asent|)adt
0 0
Z /2 Z
2 2
= a (cos t + sent) dt + a ( cos t + sent) dt +
0 /2
Z 3/2 Z 2
2 2
+a ( cos t sent) dt + a (cos t sent) dt
3/2
/2
= a2 [sent cost]0 + a2 [sent cost]/2 +
+a2 [sent + cost]3/2
+ a2 [sent + cost]2
3/

= 8a2

Para calcular el momento de inercia respecto de un diametro necesitamos


la distancia de un punto cualquiera (x, y) a dicho diametro. Para simplificar,
tomaremos como eje el eje OX, por tanto, la funcion que da la distancia de
un punto al eje es r(x, y) = |y|. Teniendo en cuenta la definicion del momento
de inercia respecto de un eje se tiene:
Z Z
2
IL = r (x, y, z) ds = y 2 (|x| + |y|) ds
C C
Z 2
= a4 sen2 t (|sent| + |cost|) dt
0
Z /2 Z
4 2 4
= a sen t (sent + cost) dt + a sen2 t (sent cost) dt
0 /2
Z 3/2 Z 2
4 2 4
+a sen t (sent cost) dt + a sen2 t (sent + cost) dt
3/2
4
= 4a

5.5.1. Area de una pared


Problema 1
h i


La base de una pared en el primer cuadrante es la trayectoria c : 0,

2
2 3 3
IR ,dada por c (t) = (3 cos t, 3sen t) y su altura es en cada punto (x, y) es
y
f (x, y) = 1 + .Calcular el area de la pared.
3
Solucion

465
El area de una pared esta dada por la integral de trayectoria
Z Z b
f (x, y) ds = f (
c (t)) |
c 0 (t)| d
C a

donde f (x, y) 0, (x, y) C


En este caso tenemos que
c (t) = (3 cos3 t, 3sen3 t) , t 0,

h i
2

c 0 (t) = (9 cos2 tsent, 9sen2 t cos t) = k
c (t)k = 9sent cos t
f (c (t)) = 1 + sen3 t
As, la integral es
Z h Z /2
yi
1 + sen3 t (9sent cos t) dt

1+ ds =
C 3 0
Z /2
sent + sen4 t cos tdt

= 9
0
/2
sen2 t sen5 t

63
= 9 + =
2 5 0 10

Problema 2

La base de una pared en el primer cuadrante es la trayectoria c :[0, 1]


IR 2
2
,dada por c (t) = (t, t ) y su altura es en cada punto (x, y) es f (x, y) =
1 + 4y.
a) Determinar la longitud de la base de la pared.
b) Calcular el area de la pared.
Solucion
Tenemos que

c (t) = (t, t2 ) = c 0 (t)k = 1 + 4t4
c 0 (t) = (1, 2t) = k
La longitud de la pared se determina por
Z 1 Z 1
l = k
c 0 (t)k dt = 1 + 4t2 dt
0 0
1

1   1
= t 1 + 4t2 + ln 2t + 1 + 4t2
2 2 0

 
1 1  
= 5 + ln 2 + 5
2 2

466
El area de una pared esta dada por la integral de trayectoria
Z Z 1
f (x, y) ds = f (
c (t)) k
c 0 (t)k dt
C
Z0 1    
= 1 + 4t 4 4
1 + 4t dt
0
Z 1  1
4
 4 5
= 1 + 4t dt = t + t
0 5 0
9
=
5

5.6. Aplicaciones de la integral de lnea




Definicion . Sea C una curva simple y regular, y sea F : R3 R3
un campo vectorial continuo. Definimos la integral de trabajo (o integral de


lnea) de F sobre la curva C D por

b



Z Z
W = F d
r = F (
r (t))

r 0 (t) dt
C a

donde

r : [a, b] IR3 es una parametrizacion regular de C
Cuando la curva C es cerrada, entonces se puede escribir



I
W = F d
r
C




y esta integral recibe el nombre de circulacion de F a lo largo de C. Si F
representa el campo de velocidades de un fluido, la circulacion es la integral
de la componente tangencial de la velocidad a lo largo de la curva cerrada
C, proporcionando la cantidad neta de giro del fluido alrededor de C.

(Campo Conservativo)


En general, se dice que un campo vectorial F : D R3 R3 es conservativo
en D si existe un potencial


: D R3 R tal que F = sobre D. En efecto, si C es una curva

467
regular parametrizada por

r : [a, b] IR3 , entonces tenemos que

b



Z Z
F d
r = F (
r (t))

r 0 (t) dt
C a
Z b
= (

r (t))

r 0 (t) dt
a
Z b
d
= [ (
r (t))] dt
a dt
= (

r (b)) (

r (a))




Proposicion 5.6.1. Sea F = F (x, y, z) un campo vectorial continuo sobre


un abierto conexo de IR3 . Entonces las propiedades sobre F siguientes, son
equivalentes, para que sea campo vectorial conservativo:



i) F = 0
ii) Para toda curva C D cerrada y regular por pedazos se tiene



I
F d
r =0
C

iii) Para cualquier par de curvas regulares, C1 D y C2 D, con iguales


puntos inicial y final, se tiene




Z Z
F d
r = F d
r
C1 C2

Problema 1


Calcular el trabajo ejercido por el campo vectorial F (x, y, z) = (3x
2y, y + 2z, x2 )sobre una partcula que se mueve segun la curva C dada por
x = z 2 , z = y 2 desde el punto (0, 0, 0) hasta el (1, 1, 1).
Solucion
En primer lugar parametricemos la curva C: r (t) = (t4 , t, t2 ), t [0, 1]


De modo que r 0 (t) = (4t3 , 1, 2t) y F (
r (t)) = (3t4 2t, t + 2t2 , t8 )

468
Calculando directamente tenemos
Z b



Z
W =
F dr = F ( r (t))
r 0 (t) dt
ZC1 a

= (3t4 2t, t + 2t2 , t8 ) (4t3 , 1, 2t)dt


Z0 1
= (12t7 8t4 + t + 2t2 2t9 )dt
0
 1
3 8 8 5 1 2 2 3 1 10
= t t + t + t t
4 5 2 3 5 0
13
=
15

Problema 2



Un campo de fuerza gravitacional F (x, y, z) esta dado por F (x, y, z) =
k

3 r . Calcular el trabajo que realiza la fuerza gravitacional que actua
krk
sobre una partcula que se mueve a lo largo del eje x desde el P0 (1, 0, 0)
hasta P(2, 0, 0) .
Solucion.
Si parametrizamos la trayectoria tenemos:
r (t) = (t, 0, 0) 1 t 2

k
Ahora, determinemos la funcion compuesta: F ( r (t)) = 3 (t, 0, 0).
t
Aplicando la definicion de trabajo tenemos

2



Z Z
W = F d
r = F (
r (t))

r 0 (t) dt
ZC2 1
k
= (t, 0, 0) (1, 0, 0) dt
1 t3
Z 2  2
k k k
= dt = =
1 t2 t 1 2

Problema 3
Si una partcula es atrada hacia el origen por una fuerza cuya magnitud
es proporcional a la distancia k r k de la partcula al origen, que trabajo se
hace cuando se mueve la partcula desde el punto (0, 1) hasta el (1, 2) a lo
largo de y = 1 + x2 suponiendo un coeficiente de roce entre la partcula y
la trayectoria?. Desprecie la fuerza de gravedad.

469
Solucion.
Formulemos el problema usando el parametro longitud de arco s
Z s2



Z
W =
F dr = F (r (s))

r 0 (s) ds
ZCs2 s1
Z s2


= F ( r (s)) b
t (s) ds = FT (r (s)) ds
s1 s1

La fuerza que trabaja es la componente tangencial a la curva. Entonces,


determinemos esta componente a partir de la informacion.
Sea el angulo que forma la tangente a la curva en un punto P (x, y)
cualquiera con el eje x.
Sea el angulo que forma el radio vector P con el eje y.
Sea el angulo entre la tangente y el radio vector en P.
Al mover la partcula a lo largo de la trayectoria, hay que realizar trabajo
contra dos fuerzas que tienen componente tangencial, a saber, la componente
tangencial de la fuerza central:
Ft = F cos = kr cos y la fuerza de rozamiento

Fr = Fn = F sen = krsen.

Donde Fn es la componente de la fuerza central que es perpendicular a la


trayectoria y que actua para mantener la partcula en su trayectoria.
Por tanto la fuerza tangencial resultante es:
FT = kr cos + krsen
Aplicando, el teorema del angulo externo de la geometra plana se tiene:
=
Reemplazando en la expresion anterior, queda
FT = kr cos ( ) + krsen ( ) .
Luego, el trabajo queda
Z s2
W = FT (

r (s)) ds
s1
Z L
= (kr cos ( ) + krsen ( )) ds
0
Z L Z L
= k r cos ( ) ds + k rsen ( ) ds
0 0
Z L
= k r [cos cos + sensen] ds +
0
Z L
+k r [sen cos ds sen cos ] ds
0

470
Ahora bien, el trabajo expresemos el trabajo en coordenadas cartesianas
x = r cos dx = cos ds
y = rsen dy = sends
Por lo tanto, sustituyendo estas en la ultima expresion para W, se tiene:
Z Z
W =k xdx + ydy + k ydx xdy
C C

Parametizando la curva C, queda: r (t) = (t, t2 + 1), 0 t 1 =

r 0 (t) = (1, 2t), 0 t 1


Sustituyendo en la integral anterior, obtenemos

Z 1 Z 1
2
W = k (t, t + 1) (1, 2t)dt + k (t2 + 1, t) (1, 2t)dt
Z0 1 Z 1 0

= k (2t3 + 3t)dt + k (1 t2 )dt


0 0
 1  1
1 4 3 2 1 3
= k t + t + k t t
2 2 0 3 0
3
W = 2k + k
2
Es el trabajo total efectuado durante el movimiento.

Problema 4


Sea F (x, y) el campo de fuerzas definido en IR2 por


F (x, y) = (2x + ycos(xy), xcos(xy))


Calcular el trabajo realizado por F sobre cualquier curva cerrada con-
tenida en IR2
Solucion
Si el campo es conservativo, el trabajo realizado por el campo sobre
cualquier curva cerrada sera nulo. Por tanto, supongamos que existe : IR2


IR diferenciable tal queF = . Entonces, se debe satisfacer

x
= 2x + ycos(xy)
(1)
y
= xcos(xy)
Integrando la primera ecuacion respecto de x obtenemos,

(x, y) = x2 + sen(xy) + h(y).

471
Si derivamos respecto de y , luego comparamos con la segunda ecuacion
de (1) obtenemos que

xcos(xy) + h0 (y) = xcos(xy)

lo que implica
h0 (y) = 0 = h (y) = c


Por lo tanto, la funcion (x, y) = x2 + sen(xy) + c verifica que F =
Finalmente


I
W = F d
r =0
C

Problema 5
Para que valores de a IR el campo vectorial


F (x, y, z) = (axy z 3 , (a 2)x2 , (1 a)xz 2 )

es conservativo? Para esos valores, calcular la funcion potencial.


Solucion


Para cualquier valor de a el campo F es de clase C1 en R3 y sera
conservativo si su rotacional es cero (x; y; z) R3 . Calculemos
el rotacional


i j k



F =

x y z


axy z 3 (a 2)x2 (1 a)xz 2
= (0, 3z 2 (1 a)z 2 , 2x(a 2) ax)

que se anula si se cumplen las ecuaciones:

(1 a)z 2 + 3z 2 = 0
2x(a 2) ax = 0



Por tanto, para a = 4 el campo F es conservativo y : IR3 IR tal
que


F = = (4xy z 3 , 2x2 , 3xz 2 ),entonces

472

= 4xy z 3
x

= 2x2
y

= 3xz 2
z
Integrando la primera ecuacion respecto de x, queda
Z
(x, y, z) = (4xy z 3 )dx = 2x2 y xz 3 + h(y, z)

Por tanto
(x, y, z) = 2x2 y xz 3 + g(z)
derivando la funcion con respecto a z y luego comparamos con la tercera
ecuacion tenemos


= 3xz 2 + g 0 (z) = 3xz 2 = g 0 (z) = 0 = g (z) = c
z


En consecuencia, la funcion potencial del campo F es

(x, y, z) = 2x2 y xz 3 + c

Problema 6
Una masa M en el origen en IR3 ejerce una fuerza sobre una masa m lo-
mM
calizada en r = (x, y, z) con magnitud G 2 y dirigida hacia el origen.
r
Aqu G es la constante gravitacional, que depende de las unidades de medi-

r

p
2 2 2
cion y r = | r | = x + y + z ,si recordamos que es un vector unitario
r
dirigido hacia el origen, entonces podemos escribir el campo de fuerza como

mM
F = G 3 r
r
a) Mostrar que el campo vectorial es conservativo.


b) Hallar un potencial escalar para F .


c) Hallar el trabajo realizado por F al trasladar la partcula de masa m
desde un punto P1 hasta P2


d) Cual es el trabajo realizado por la fuerza F al mover la partcula
de masa m desde el infinito hasta una distancia r del origen?.

473
Solucion
a) En primer lugar calculemos
 

mM
F = G 3 r
r
   
1
1

= GmM 3 r + 3 r
r r
Puesto que de las identidades basicas del analisis vectorial, tenemos




i) F = () F + ( F )
ii) (rn ) = nrn2
r
Entonces, queda:
 

3
1

F = GmM r r + 3 r
r r
De modo que el primer termino se anula pues r r =0
Luego queda aun por calcular el segundo termino

i j k


r = x y z = (0, 0, 0)
x y z

Por tanto




F = 0 para r 6= 0


b) Luego existe tal que F = .(En fsica se define que el campo de
fuerza proviene del valor negativo del gradiente de la funcion escalar).
Aprovechando la simetra radial del campo de fuerza expresemos el
gradiente en coordenadas cilndricas
Z
mM
mM
= G 2 = ( r ) = G 2 dr
r r r
GmM
(

r) = +C
r
Donde c es una constante de integracion
c) Como el campo es conservativo, el trabajo depende de la posicion de
los
puntos inicial y final y el trabajo es igual a


Z
F dr = (
r (2)) (
r (1))
C
 
1 1
= GmM
r1 r2

474
donde r1 y r2 son las distancias radiales de los puntos P1 y P2
respectivamente al origen.

d) Si r1 y r2 = r se tiene que

GmM
W =
r

Problema 7


Dado el campo de fuerzas F (x, y) = (y 3 + 1, 3xy 2 + 1).


a) Es F conservativo? en caso que lo sea, hallar la funcion potencial de

F.
b) Hallar el trabajo realizado al mover un objeto desde el punto (0, 0) al
(2, 0),a lo largo de la semicircunferencia (x 1)2 + y 2 = 1 con y 0.
c) Hallar el trabajo realizado al mover el objeto a lo largo de la circun-
ferencia
completa.
Solucion


Examinemos el rotor de F

i j k

= 0, 0, 3y 2 3y 2

F = x y
0
y 3 + 1 3xy 2 + 1 0
= (0, 0, 0)

Por tanto, estamos en presencia de un campo conservativo, luego existe


una
funcion escalar (x, y) tal que


= y3 + 1
x

= 3xy 2 + 1
y

Integrando la primera ecuacion respecto de x, queda


Z
(x, y) = (y 3 + 1)dx = xy 3 + x + h(y)

Para calcular la funcionh(y),calculamos la derivada parcial de respecto


de

475
y luego comparamos con la segunda ecuacion:

= 3xy 2 + h0 (y) = 3xy 2 + 1 = h0 (y) = 1 = h(y) = y + c
y


En consecuencia, la funcion potencial del campo F es

(x, y, z) = xy 3 + x + y + c


b) Como F es conservativo la integral es independiente del camino,
unicamente depende de los puntos inicial y final. El trabajo realizado al
mover el objeto desde (0, 0) hasta (2, 0) sera:



Z
F d
r = (2, 0) (0, 0)
C
= (2 + c) c = 2

c) A lo largo de la circunferencia tenemos una curva cerrada,lo que


implica que el trabajo sera nulo:


I
W = F d
r =0
C

Problemas propuestos para reflexionar


1.- Considere dos curvas que parten del punto P = (0, 2) y finalizan en
Q = (, 0); la braquistocrona dada por la ecuacion:


r (t) = (t + sen(t + ); 1 cos(t + )); t [0; ];
y el segmento rectitlneo que va de P a Q. Pruebe que si se suelta una bola
que se desliza por cada curva, bajo el efecto de la gravedad, entonces llega
antes la bola de la braquistocrona que la del segmento rectilneo. Tengase
en cuenta que el tiempo total, para una curva cualquiera, es la integral a lo
largo de dicha curva del campo escalar
1
(x, y) = p
2g (h0 y)
donde g es la gravedad y h0 la altura inicial de la bola.
2.- Calcular el trabajo realizado por una fuerza proporcional al vector
dirigido hacia el origen, sobre el recorrido del primer cuadrante de la elipse
x2 y 2
+ 2 = 1, desde el punto (a, 0) hasta el (0, b).
a2 b
476
k 2
Solucion: (a b2 ) con k > 0.
2
3.- Dado h > 0, sea C la curva que se encuentra sobre la superficie defini-
z2
da por x2 + y 2 = 2 , de forma tal que la altura z = z() satisface la ecuacion
h
dz
diferencial = z; z(0) = h donde z y representan las coordenadas cilndri-
d
cas.
(a) Parametrice la ecuacion de la curva C r () .Calcule la curvatura
y la torsion  

1 1 1
(b) Considere el campo vectorial F (x, y, z) = , ,
h i x y z2
Sea C0 la restriccion de C a , .Calcule el trabajo realizado por el

6 3
campo F al desplazar una partcula a traves de C0 .
Solucion
2
a)

r () = e cos , e sen, he , curvatura = ; torsion =
e (2 + h2 )
h
e (2 + h2 )

 
R
/3 1 1 1
b) C F d r = ( r ())|/6 = +
3 h e/3 e/6

5.7. Problemas Resueltos


Problemas 1
x3
ds, donde

R
Calcular la integral de trayectoria

r
r es la trayectoria
y
x2
y= entre los puntos (0, 0) y (2, 2) .
2
Solucion
Primero, determinemos
la ecuacion parametrica de la trayectoria
x= t  2
t
t2

t [0, 2] r (t) = t,
y= 2
2
Derivando la expresion anterior,queda

r 0 (t) = (1, t) = k r 0 (t)k = 1 + t2
Ademas
x3
f (x, y) = = f ( r (t)) = 2t
y
A partir de la definicion de integral de trayectoria tenemos

477
Z
x3
Z 2
ds = 2t 1 + t2 dt


r y 0
 2
2 2 3/2

= 1+t
3 0
2  
= 75 1
3

Problema 2
Dada la funcion escalar f (x, y) = 2xy, calcular la integral de trayectoria
x2 y 2
a lo largo de la curva elipse + = 1 desde el punto (3, 0) hasta (0, 2).
9 4
Solucion
Observemos que r es el segmento de elipse que esta en el primer
cuadrante.Entonces
 al parametrizar la curva queda
x = 3 cos t
t 0, 2
 
r (t) = (3 cos t, 2sent)
y = 2sent
Derivando la trayectoria

r 0 (t) = (3 sin t, 2 cos t) k
r 0 (t)k = 5sen2 t + 4
Calculemos la funcion escalar f sobre la trayectoria
f (x, y) = 2xy = f (x (t) , y (t)) = 6 cos tsent
Calculemos la integral

Z Z /2
2xyds = 12 cos tsent 5sen2 t + 4dt


r 0
 /2
4 3/2
= 5sen2 t + 4
5 0
76
=
5

Problema 3
xydx + x2 dy, donde
R
Calcular la integral de lnea
r
r es la trayectoria
2 2
x + 4y = 4, x > 0.
Solucion
Primero, escribamos la ecuacion parametrica de la trayectoria orientada
positivamente
x = 2 cos t
t 2 , 2 =
 
r (t) = (2 cos t, sent)
y = sent

478



F (x, y) = xy, x2 = F (x (t) , y (t)) = 2 cos tsent, (2 cos t)2
 

Determinemos el vector

r 0 = (2sent, cos t)
Calculemos la integral

Z Z /2
2
2 cos tsent, (2 cos t)2 (2sent, cos t) dt

xydx + x dy =


r /2
Z /2
= (4sen2 t cos t + 4 cos3 t)dt
/2
Z /2
= (8sen2 t cos t + 4 cos t)dt
/2
  2
8 3 8
= sen t + 4sent =
3 3
2

Problema 4

y 2 dx + xdy, donde

R
Calcular la integral de lnea

r
r es la trayectoria
y 2 = 2x x2 , tal que x > 1, y > 0.
Solucion
Observemos que

r es el segmento de circunferencia:
y 2 = 2x x2 (x 1)2 + y 2 = 1 tal que x > 1, y > 0.
Entonces:

x = 1 + cos t
t 0, 2 =

 
r (t) = (1 + cos t, sent)
y= sent
=
r 0 (t) = (sent, cos t)


Calculemos el campo vectorial F sobre la trayectoria



F (x, y) = (y 2 , x) = F (x (t) , y (t)) = (sen2 t, 1 + cos t)

479
Calculemos la integral

Z Z /2
2
sen3 t + cos2 t + cos t dt

y dx + xdy =


r 0
Z /2  
2
 1 + cos 2t
= 1 cos t sent + ( ) + cos t dt
0 2
/2
cos3 t t sen2t

= cos t + + + + sent
3 2 4 0
1
= 1 + +1
3 4
5
= +
3 4

Problema 5

(8x + z)dx + 2xz 2 dy 4y 2 dz, siendo



R
Calcular la integral de lnea
r
r la
curva definida por las ecuaciones: z = 9 2x2 4y 2 , z = 1.
Solucion
Observemos, que la curva contenida en el plano z = 1, es la elipse

2x2 + 4y 2 = 8,con semi ejes a = 2 y b = 2, que se parametriza
mediante.
x = 2 cos t
t [0, 2] = 
y= 2sent r (t) = 2 cos t, 2sent, 1 t
z= 1

[0, 2]


Calculemos el campo vectorial F sobre la trayectoria



F (x, y, z) = (8x + z, 2xz 2 , 4y 2 ) = F (x (t) , y (t)) = (16 cos t + 1, 4 cos 1t, 1)
Evaluemos el vector


r 0 (t) = 2sent, 2 cos t, 0 luego, obtenemos



F (x (t) , y (t))

r 0 (t) = (16 cos t + 1, 4 cos t, 1) 2sent, 2 cos t, 0


Entonces la integral de lnea es

Z
2 2
Z 2  
(8x+z)dx+2xz dy4y dz = 32sent cos t 2sent + 4 2 cos2 t dt


r 0

480
2
Z  
2 1 + cos 2t
= 16sen2 t + 2 cos t 0 + 4 2

dt
0 2
t sen2t 2
 
= 4 2 +
2 4 0

= 4 2

Problema 6


Calcular el trabajo producido por campo de fuerzas dado por F = (3x +
4y, 2x+3y 2 ), a lo largo de la circunferencia C de radio 2 centrada en el origen
y recorrida con orientacion positiva.

Solucion
Definimos el trabajo mediante la integral de lnea
b



Z Z
F d
r = F (
r (t))

r 0 (t) dt


r a

Luego, parametrizando la trayectoria tenemos:




r (t) = (2cost, 2sent), t [0, 2].
=
r 0 (t) = (2sent, 2 cos t)
Reemplazando el integrando, queda

I
W = (3x + 4y, 2x + 3y 2 , 0) (dx, dy, dz)
Z 2
= (6cost + 8sent, 4cost + 12sen2 t, 0) (2sent, 2cost, 0)dt
Z0 2
= [16sen2t + 8cos2t]dt
0
= 16 + 8 = 8.

Trabajo negativo significa que el campo de fuerza disipa energa.

5.7.1. Campo conservativo


Problemas 1



Sea el campo vectorial F : IR3 IR3 dado por F (x, y, z) = (x, y, z).
R
Calcular la integral

r
F d
r

481
a) Si C es la circunferencia x2 + y 2 = 4 recorrida en el sentido positivo.
b) Si C es la recta que une P = (1, 0, 0) con Q = (1, 0, 4).
c) Si C es la helicoide

r (t) = (cos(4t), sen(4t), 4t), t [0, 1] que une
P = (1, 0, 0) con Q = (1, 0, 4).
Solucion
a) Las ecuacion parametrica de la circunferencia de radio 2 centrada en
el
origen y recorrida en sentido positivo, es

r (t) = (2 cos t, 2sent, 0) t [0, 2] =

r 0 (t) = (2sent, 2 cos t, 0)
Entonces, la integral de linea queda



Z Z
F d
r = (x, y, z) (dx, dy, dz)


r

Z r2
= (2 cos t, 2sent, 0) (2sent, 2 cot s, 0) dt
0
Z 2
= 8 cos tsentdt = 0
0

b) La ecuacion parametrica de la recta que une P = (1, 0, 0) con Q =


(1, 0, 4)
es:




r (t) = P + ( Q P )t = t(1, 0, 4) t [0, 1] =

r 0 (t) = (1, 0, 4) .
Entonces

1


Z Z
F d
r = (t, 0, 4t) (1, 0, 4) dt


r 0
Z 1  1
= 16tdt = 8t2 0 = 8
0

c) A partir de la ecuacion de la helicoide se obtiene



r 0 (t) = (4sen(4t), 4 cos(4t), 4)
Sustituyendo terminos en el integrando, queda

1


Z Z
F d
r = (cos(4t), sen(4t), 4t) (4sen(4t), 4 cos(4t), 4)dt


r 0
Z 1
= (8sen(4t) cos(4t) + 4t)dt
0
Z 1  1
= 16tdt = 8t2 0 = 8
0

482
El valor de la integral de lnea es el mismo por ambas trayectorias.

Problema 2

2x cosydx x2 senydy, donde


R
Calcular la integral
r
r : [1, 2] IR2

definida por r (t) = et1 , sen .
t
Solucion.
Determinemos si el campo vectorial es conservativo, de modo que calcu-
lamos

i j k


F = = (0, 0, 0)
x y z
2x cos y x2 seny 0




Como hallamos que F = 0 , entonces F tiene una funcion potencial
(x, y) tal que


(x, y) = 2x cos y
x

(x, y) = x2 seny
y
Integrando la primera ecuacion parcialmente con respecto a x, se tiene
0
(x, y) = x2 cos y + h (y) = (x, y) = x2 seny + h (y) = x2 seny
y
0
h (y) = 0 h (y) = c


En consecuencia, la funcion potencial (x, y) para F (x, y) es
(x, y) = x2 cos y + c
Entonces , podemos afirmar que
Z Z
2
2x cos ydx x senydy = d

r


r

r
= (

r (2)) (

r (1))
donde


 
( r (2)) = e, sen = e2 + c
2
(
r (1)) = (1, sen) = 1 + c
Por tanto. obtenemos:
Z
2x cos ydx x2 senydy = e2 1


r

483
Problema 3


Considere el campo vectorial F (x, y, z) en IR3 definido por :
 

yz xz xy
F (x, y, z) = , ,
1 + x y z 1 + x y z 1 + x2 y 2 z 2
2 2 2 2 2 2

yzdx + xzdy + xydx


,donde

R
Evaluar

r
r es:
1 + x2 y 2 z 2
a) el segmento rectlineo entre (0, 0, 0) y (1, 1, 1) .
b) la interseccion de x2 + y 2 + (z 1)2 = 1, con x2 + y 2 + z 2 = 1.
Solucion
Se tiene que las componentes del campo vectorial son continuas
(x, y, z) IR3
Primero, verifiquemos
si el campo vectorial es conservativo
o no.

i j k



F = = (0, 0, 0)

x y z

yz xz xy

1 + x2 y 2 z 2 1 + x2 y 2 z 2 1 + x2 y 2 z 2
Puesto que
xy xz yz xy
2 2 2
= 2 2 2
, 2 2 2
= , etc.
y 1 + x y z z 1 + x y z z 1 + x y z x 1 + x2 y 2 z 2
= (0, 0, 0)




Como hallamos que F = 0 , entonces F tiene una funcion potencial
(x, y) tal que:

yz

(x, y) =

x 1 + x2 y 2 z 2


xz
(x, y) =
y 1 + x2 y 2 z 2
xy



(x, y) =

z 1 + x2 y 2 z 2
Integrando la primera ecuacion parcialmente con respecto a x, se tiene

(x, y) = arctg (xyx) + h (y, z) =


xz 0 xz
(x, y) = 2 2 2
+ h (y, z) =
y 1+x y z 1 + x2 y 2 z 2
0
h (y, z) = 0 h (y, z) = g (x)

484


En consecuencia, la funcion potencial (x, y) para F (x, y)es

(x, y) = arctg (xyx) + g (z) =


xy 0 xy
(x, y) = 2 2 2
+ g (x) =
1+x y z 1 + x2 y 2 z 2
0
g (x) = 0 g (x) = c

Entonces , podemos concluir que

(x, y) = arctg (xyx) + c

En este caso hallamos , que el valor de la integral

Z Z
yzdx + xzdy + xydx
= d
r = (1, 1, 1) (0, 0, 0)


r 1 + x2 y 2 z 2
r
= arctg (1) arctg (0)

=
4
Si

r es la interseccion de dos esferas la curva resultante es cerrada, en

consecuencia
Z Z
yzdx + xzdy + xydx
= d

r =0


r 1 + x2 y 2 z 2

r

5.7.2. Teorema de Green


Problema 1


Verificar el teorema de Green para el campo vectorial F (x, y, z) = (2(x2 + y 2 ), (x + y)2 ) ,donde
las curvas frontera de la region D corresponden al contorno del triangulo con
vertices en los puntos (1, 1) , (2, 2) ,y (1, 3) orientado positivamente.
Solucion


Como el campo vectorial F (x, y) es de clase C 1 , y la region D conexa,
entonces el teorema de Green afima que:
Z Z Z  

P dx + Qdy = Q P dxdy
C D x y
Identificando terminos, tenemos que
P
P (x, y) = 2(x2 + y 2 ) = = 4y
y

485
Q
Q (x, y) = (x + y)2 = = 2(x + y)
x
Entonces calculemos
ZZ   ZZ
Q P
dxdy = 2 (x y) dxdy
D x y D

donde D = {(x, y) IR2 : 1 x 2, x y 4 x} . Luego

ZZ Z 2 Z 4x
2 (x y) dxdy = 2 (x y) dydx
D 1 x
Z 2 4x
y2
= xy dx
1 2 x
Z 2
2x(4 x) (4 x)2 2x2 + x2 dx

=
1
3 2
Z 2 " #
(x 2)
= 4 (x 2)2 dx = 4
1 3
1
4
=
3

Calculemos directamente la integral de lnea, segmentando la frontera


en tres curvas:

Z Z Z
P dx + Qdy = P dx + Qdy + P dx + Qdy
C C1 C2
Z
+ P dx + Qdy
C3

Parametricemos los segmentos de curvas que unen los puntos (1, 1) y


(2, 2) ;
(2, 2) y (1, 3) ; (1, 3) y (1, 1)

Aca debe ir grafico

Sea C1 la recta y = x, 1 x 2 =
r (t) = (t, t) , t [1, 2]

0
= r (t) = (1, 1) , t [1, 2] entonces:

486
Z Z 2
2 2 2
2 t + t2 + (2t)2 dt
 2  
2(x + y )dx + (x + y) dy =
C1 1
Z 2  3 2
2 t
= 8t dt = 8
1 3 1
56
=
3

Sea C2 la recta y = 4 x, 1 x 2 =
r (t) = (4 t, t) , t [2, 3]

0
= r (t) = (1, 1) , t [2, 3] , entonces:

Z Z 3
2 2 2
2 (4 t)2 + t2 (1) + (4)2 dt
  
2(x + y )dx + (x + y) dy =
C2 2
Z 3
2 16 8t + 2t2 + 16 dt
  
=
2
Z 3 Z 3
[t 2]2 dt
2 
= 4 t 4t + 4 dt = 4
2 2
" #3
(t 2)3 4
= 4 =
3 3
2

Sea C3 la recta x = 1, 1 y 3 = r (t) = (1, 3 t) , t [0, 2]



0
= r (t) = (0, 1) , t [0, 2] , entonces:

Z Z 2
2 2 2
2(x + y )dx + (x + y) dy = (4 t)2 (1)dt
C3 0
" #2
(4 t)3 8 64 56
= = =
3 3 3 3
0

Por lo tanto, al sumar los tres terminos tenemos:


Z
56 4 56 4
2(x2 + y 2 )dx + (x + y)2 dy = =
C 3 3 3 3

Lo que muestra la validez de la formula del teorema de Green.

487
Problema 2
H
Verificar el teorema de Green para C x2 ydx + xy 2 dy, donde C es la
frontera de la region R en el primer cuadrante, limitada por las graficas de
y = x, y 3 = x2 .
Solucion.
Primero, calculemos la integral de lnea considerando la orientacion pos-
itiva
de la frontera, dividiendola en dos segmentos C1 y = x y C2 y 3 = x2 .
Determinemos  los puntos que se intersectan ambas curvas:
y= x
= x3 = x2 = x2 (x 1) = 0 x = 0 y x = 1
y 3 = x2
Luego, ambas curvas se intersectan en los puntos (0, 0) y (1, 1) .En
consecuencia
 la region R queda delimitada por
R = (x, y) IR2 /0 x 1, x y x2/3
Parametrizando el segmento de curva C1 tenemos:
C1 : r 1 (t) = (t, t) , t [0, 1] =
r 01 (t) = (1, 1)
Calculemos el campo vectorial sobre la curva C1


F (r 1 (t)) = t3 , t3 =



F (r 1 (t))

r 01 (t) = t3 , t3 (1, 1) = t3 + t3 = 2t3
 

Para C2 encontramos  


2
C2 : r 2 (t) = 1 t, (1 t)2/3 , t [0, 1] = r 01 (t) = 1, (1 t)1/3

3
Luego, la funcion compuesta para el campo sobre C2 es:


F (r 2 (t)) = (1 t)8/3 , (1 t)7/3 =

 


0 8/3 7/3
 2 1/3
F ( r 1 (t)) r 1 (t) = (1 t) , (1 t) 1, (1 t)
3
2
= (1 t)8/3 (1 t)2
3
Entonces la integral de lnea queda
I Z 1 Z 1
2 2 3 2
x ydx + xy dy = 2t dt ((1 t)8/3 + (1 t)2 )dt
C 0 0 3
 4 11
t 3(1 t)11/3 2(1 t)3
= + +
4 11 9 0
1 3 2
=
4 11 9
1
=
198
488


Por otra parte el campo vectorial F (x, y) de clase C 1 , es decir campo con-
tinuo con primera derivada continua,definido en la region R conexa,acotado
por una frontera cerrada, entonces podemos aplicar el teorema de Green que
afima:
I Z Z  
2 2 2 2
x ydx + xy dy = (x y) (xy ) dxdy
C R x y

donde R = (x, y) IR2 /0 x 1, x y x2/3 ,entonces:

Z Z   Z 1 Z x2/3
2 2
 2
y x2 dydx

(x y) (xy ) dxdy =
R x y 0 x
Z 1 3 x2/3
y 2
= x y dx
0 3 x
Z 1 2 
x 8/3 2 3
= 8x + x dx
0 3 3
 3 1
x 3 11/3 x4
= x +
9 11 6 0
1 3 1 1
= + =
9 11 6 198
Con esto,verificamos el teorema de Green en este caso particular.

Problema 3
H y x
Calcule la integral C
dx + 2 dy a lo largo de la curva C
2x2
+ 3y 2 2x + 3y 2
formada por los lados del cuadrado con vertices en (1, 1) , (1, 1) , (1, 1) , (1, 1) .
Solucion.


Claramente vemos que el campo vectorial F (x, y) en la region acotada
por
C no es continuo, con primeras derivadas parciales continuas en el origen
(0, 0) .
Luego, vamos a excluir el origen de la region . Dado que la region envuelta
por la curva C, que excluye la singularidad, no es simplemente conexa, se
tiene que:  
R R RR Q P
C
P dx + Qdy + C1 P dx + Qdy = D dxdy
x y
donde la curva C1 es la elipse con ecuacion 2x2 + 3y 2 = r2 , orientada en
el sentido horario, con normal apuntando hacia fuera de la region D.

489
Por otra parte.
3y 2 2x2

Q
=
 
x (2x2 + 3y 2 )2 Q P

= =0
P 3y 2 2x2 x y
=


(2x2 + 3y 2 )2

y
Entonces, tenemos
Z Z ZZ  
Q P
P dx + Qdy + P dx + Qdy = dxdy = 0
C C1 D x y

Z Z
= P dx + Qdy = P dx + Qdy
C C1
Z Z
= P dx + Qdy = P dx + Qdy
C C1

 
r r
Parametrizando C1 como

r (t) = cos t, sent con 0 t 2
2 3
Se obtiene
Z Z 2     
rsent r r r
P dx + Qdy = sent + cos t cos t dt
C1 0 3r2 2 2r2 3
1
= (2)
6
r
2
=
3

Por lo tanto, la integral


Z r
2
P dx + Qdy =
C 3

Problema 4
Sea C una curva cerrada simple que encierra una region
2
y2
 
2 x
D = (x, y) IR / + =1
4 5

Calcular el area del interior de la elipse usando el teorema de Green.


Solucion.

490
A partir del teorema de Green tenemos
I
1
A (D) = xdy ydx
2
C

Parametricemos la ecuacion de la elipse , mediante



r (t) = (2cos(t), 5sen(t)), 0 t 2.
Entonces x(t) = 2cos(t), y(t) = 5sen(t), luego
dx = 2sen(t), dy(t) = 5 cos(t)
Reemplazando terminos en el integrando

1
I
1
Z 2
xdy ydx = (2 5cos2 (t) + 2 5sen2 (t))dt
2 2 0
C
Z 2
= 5 dt
0

= 2 5

Problema 5
Considere la region R del plano x2 + (y a)2 a2 ; x2 + y 2 2a2 y usando
el teorema de Green, verifique que el area de dicha region coincide con el area
de un cuadrado de lado a.
Solucion
La curva C1 descrita por la ecuacion x2 + (y a)2 = a2 ,corresponde a
la circunferencia con centro en (0, a) y radio a y la curva C2 es la

ecuacion
x2 + y 2 = 2a2 de la circunferencia con centro en (0, 0) y radio
a 2.
Calculemos los puntos de interseccion de ambas curvas, igualando

ambas ecuaciones, produce 2a2 2ay = 0 = y = a


Sustituyendo este resultado en la segunda ecuacion, obtenemos
x2 = a2 x = a
Por lo tanto, los puntos de interseccion de ambascurvas son P1 =
(a, a)
y P2 = (a, a) que tienen coordenadas polares (a 2, 3/4) y (a 2, /4)
respectivamente.

En consecuencia, la curva cerrada C que forma la frontera de R es la


union

491
de la curva C1 parametrizada por:

x(t) = acost, y(t) = a + asent, donde t [0, ]

y de la curva C2 parametrizada por



x(t) = a 2cost, y(t) = a 2sent, dondet [/4, 3/4].

El teorema de Green afirma que


I
1
A (R) = xdy ydx
2
C

donde la orientacion de C es positiva.


I I
1 1
A (R) = xdy ydx + xdy ydx
2 2
C1 C2

Las orientaciones de C1 y C2 , van en sentido opuesto a los punteros del


reloj,
para que C tenga orientacion positiva. Entonces

1 2 1 /4 2
Z Z
2
A (R) = (a sent + a )dt + (2a )dt
2 0 2 3/4
1 2 /4

= a [ cos t + t]0 + 2a2 [t]3/4
2
1
2a2 + a2 a2 = a2

=
2
Resultado que verifica que el area de la region R es igual a la de un
cuadrado de lado a.

5.8. Problemas propuestos


5.8.1. Integral de trayectoria
1.- Calcular el area de la superficie lateral cuya base esta sobre la curva

r del plano XY y la altura en cada (x,y) es z = f (x, y) ,donde:


a) f (x, y) = xy ,
r : x2 + y 2 = 1 desde (1, 0) hasta (0, 1) .
b) f (x, y) = h ,
r : y = 1 x2 desde (1, 0) hasta (0, 1) .


c) f (x, y) = xy, r : y = 1 x2 desde (1, 0) hasta (0, 1) .
Respuestas

492
1
a) A(S) = ,
2
h 
b)A(S) = 2 5 + ln 2 + 5 ,
4
1  
c)A(S) = 25 5 11
120 R
2.- Evaluar las siguientes

integrales de trayectoria
r
f (x, y, z) ds
z


a) f (x, y, z) = e , r : [0, 1] IR dada por r (t) = (1, 2, t2 ) .
3

b) f (x, y, z) = yz,
r : [1, 3] IR3 dada por
r (t) = (t, 3t,
 2t) . 
x+y 3
2 3/2
c) f (x, y, z) = , r : [1, 2] IR dada por r (t) = t, t , t .
y+z 3
d) f (x, y, z) = z, r : [0, 2] IR3 dada por
r (t) = (tcost, tsent, t) .
Respuestas
R z
a) e ds = 2,
Rr
b) r
yzds = 52 14,
R x+y 16
c) r
ds = 2 3
y+z h 3
R 1 p i
d) zds = 2 3
(2 + 4 ) 2 2
r
3 R
3.- Evaluar las siguientes integrales de trayectoria
r
f (x, y) ds
2 2/3 2/3
a) f (x, y) = 8y 5x, la curva C dada por x + y = 1.
b) f (x, y) = x + y, siendo C un triangulo de vertices (0, 0), (1, 0) y (0,
1), recorrido en sentido antihorario.
Respuestas
R
a) C (8y 2 5x)ds = 12
R
b) C (x + y)ds = 1 + 2

5.8.2. Integral de lnea


4.-R Evaluar las siguientes integrales de lnea:
a)
r
xdx + ydy + zdz, donde r : [1, 2] IR3 dada por
r (t) =
2 3
(t , 3t, R2t ) .
b) r
2xyzdx + x2 zdy + x2 ydz, donde r es una curva simple orientada
que
uneR (1, 1, 1) con (1, 2, 4).
c) r
x2 zdx + 6ydy + yz 2 dz, donde r : [1, 3] IR3 dada por

r R(t) = (t, t2 , ln t) .
d) C xydx + (x y)dy, donde C esta formado por los segmentos de recta
que van
desde
R (0, 0) a (2, 0) y de (2, 0) a (3, 2) .
2
e) C (x+2)dx+3zdy+y dz,siendo C la curva interseccion de las superficies
x2 + y 2 + z 2 = 1; z = x 1

493
Respuestas.
R
a) R
r
xdx + ydy + zdz = 147
b)R r
2xyzdx + x2 zdy + x2 ydz = 7
c)
r
x2 zdx + 6ydy + yz 2 dz = 249, 49
R 17
d) C xydx + (x y)dy =
3
R 3 2
e) C (x + 2)dx + 3zdy + y 2 dz =
4


5.- Evaluar la integral de lnea de F a lo largo de cada una de las siguientes
trayectorias.


a) F (x, y) = (x2 2xy, y 2 2xy),a lo largo de la parabola y = x2 desde
(1, 1) a (1, 1)


b) F (x, y, z) = (x, y, xz y) sobre el segmento de recta desde el punto
(0, 0, 0) hasta el punto (1, 2, 4).


c) F (x, y) = xx+y x+y 2
2 +y 2 , x2 +y 2 a lo largo de la circunferencia x + y
2
= a2
recorrida en sentido positivo.
x2 y 2


y
d) F (x, y) = 2 2 , 2 2 sobre el cuarto de elipse 2 + 2 = 1
x
1+x +y 1+x +y a b
situado en el primer cuadrante, desde el punto (a, 0) hasta el punto (0, b).


e) F (x, y, z) = (yz, xz, xy) a lo largo de la helice r (t) = (a cos t, asent, bt)
entre los puntos (a, 0, 0) y (a, 0, 2b).
Respuestas
R 14
a) F dr =
C 15
R 23
b) F dr =
C 6
R
c) C F d r = 0


R
d) F dr =
C
1 + b 2 1 + a2
R
e) F dr = 0
C

5.8.3. Campos conservativos


R
6.- Pruebe que la integral (6xy 2 y 3 )dx + (6x2 y 3xy 2 )dy ; es inde-
C
pendiente del camino que une los puntos (1, 2) con (3, 4).
a) Calcule el valor de la integral parametrizando el segmento.
b) Verifique el resultado anterior utilizando la funcion potencial.
Respuestas
R1
a) 0 F (r (t)) .

r 0 (t) dt = 236


b) F = (x, y, z) = 3x2 y 2 xy 3 + C

494


7.- Para que valores de a IR el campo vectorial F (x, y, z) = (axy
z 3 ; (a 2)x2 ; (1 a)xz 2 ) es conservativo? Para esos valores, calcule la funcion
potencial.
Respuesta


Si a=4 la funcion potencial del campo F es (x; y; z) = 2x2 y xz 3 + C.


8.- En los casos en los que F sea conservativo, halle la correspondiente
funcion potencial.


a) F (x, y, z) = (10xz 3 + 1, 6y 2 , 15x2 z 2 ).


b) F (x, y, z) = (2xy 3 , x2 z 3 , 3x2 yz 2 ) .
Respuestas


a) F es conservativo, funcion potencial: (x, y, z) = 5x2 z 3 + x 2y 3 + C


b) F no es conservativo.


9.- Sea F (x, y, z) = (z 3 + 2xy, x2 , 3xz 2 ).


a) Muestre que F es un campo conservativo.


b) Calcule directamente la integral de F alrededor del contorno del cuadra-
do convertices (1, 1) .
c) Determine la funcion potencial.

5.8.4. Teorema de Green


10.-R Usar el teorema de Green para hallar el valor de la integral de lnea:
a) r
xy 2 dx + x2 ydy, donde r : [0, 2] IR2 dada por r (t) =
(4 cos Rt, 2sent)
b) r
xydx + x2 dy, donde r es la frontera que limita la region
2
yR x ,yx 2
c) C (arctgx + y )dx + (ey x2 )dy, donde C es la frontera que encierra
la
2 2
R 1 x + y2 9, y 0
region
d) C 2xydx + (x + 2x)dy, donde Ces la frontera que limita la region
interior
x2 y 2
a la elipse + = 1 y exterior a la circunferencia x2 + y 2 = 1
H 9 4
e) C (x2 + y 2 ) dx + (2x + y 2 ) dy, donde C es la frontera del cuadrado
con vertices (0, 0) , (k, 0) , (0, k) , (k, k) .

Respuestas
R
a)
r
xy 2 dx + x2 ydy = 0
R 1
b)
r
xydx + x2 dy =
12
495
R 104
c)
r
(arctgx + y 2 )dx + (ey x2 )dy =
R 3
d)H

r
2xydx + (x2 + 2x)dy = 10
e) C (x2 + y 2 ) dx + (2x + y 2 ) dy = k 2 (2 k)
H y x
10.- Demuestre que C
dx + 2 dy = 2, donde C es cualquier
+y 2 x2 x + y2
curva cerrada , simple , que incluya el origen

11.- Calcular el area de una elipse con semi ejes a, b usando el teorema de
Green.
Respuesta
A (R) = ab.
12.- Hallar el area acotada por el arco de hipocicloide x = a (t sent) , y =
a (1 cos t) ,donde a > 0, y 0 t 2, y el eje x.

Respuesta
A (R) = 3a2

5.9. Autoevaluaciones
Autoevaluacion No 1
Tiempo 2 horas
Pregunta 1



Sea F (x, y) el campo de fuerzas definido por F (x, y) = (x2 + 7y, x + y sin y 2 ) .Calcular
H

F d
r sobre la frontera del triangulo de vertices (0, 2) , (0, 0) y (1, 0) us-
C
ando la definicion de integral de lnea.
Pregunta 2 H 
Evalue la integral 1 + e x dx + (cos y 2 + x2 ) dy, siendo C una curva
C
cerrada, ubicada en el primer cuadrante, formada por los arcos de
circunferencia de radios 1 y 2 respectivamente y por los segmentos
rectos 1 x 2, 1 y 2.
Pregunta 3
R


Calcular la integral I = F d r ,si F = (x, y, z) es un campo vectorial
C
definido en R3 C es dada por y 2 + z 2 = 1, x = z entre A = (1, 0, 1) y
B = (0, 1, 0) .

Pauta de Correccion

496
Pregunta 1
La circulacion a lo largo de la frontera del triangulo es la suma de las
circulaciones en cada uno de los lados





I Z Z Z

F dr =
F dr +
F dr + F dr
C C1 C2 C3

Parametrizando cada uno de los lados del triangulo se tiene


C1 : y = 0,
r 1 (t) = (t, 0) , t [0, 1] =



r 1 (t) = (1, 0) y F (
0
r 1 (t)) = t2 , t


C2 y = 2 (1 x) ,
:
r 2 (t) = (t, 2 (1 + t)) , t [1, 0] =



r 2 (t) = (1, 2) y F ( r 1 (t)) = t2 + 14 (1 + t) , t + 2 (1 + t) sin (2 (1 + t))2
0


C3x = 0,
:
r 3 (t) = (0, t) , t [2, 0] =



r 03 (t) = (0, 1) y F ( r 1 (t)) = 7t, t sin (t)2


Sustituyendo terminos en los integrandos , obtenemos


Z 1 Z 0


I

F dr = 2
t dt + t 14 (1 + t) + 2t + 4 (1 + t) sin (2 (1 + t))2 dt
 2 
0 1
C
Z 0
+ t sin t2 dt
2
 3 1 " 3 2 0
#
t t cos (2 (1 + t))
= + 14t 7t2 + t2
3 0 3 2
1
 2
0
cos t
+ = 8
2 2

Pregunta 2
De acuerdo con el teorema de Green tenemos
I  
ZZ  

x 2 2
 2 2
  x
1+e dx + cos y + x dy = cos y + x 1+e dxdy
x y
C
ZDZ
= 2xdxdy
D

497
Cambiando a coordenadas polares ,queda
ZZ ZZ
2xdxdy = 2 (r cos ) rdrd
D D
Z /2 Z 2
r2 cos drd

= 2
0 1
Z /2  3 2
r
= 2 cos d
0 3 1
14 14
= [sin ]/2
0 =
3 3

Pregunta 3


Como el rotacional
del campo F :
i j k


F = x y z = (0, 0, 0) es nulo el campo es conservativo =

x y z


existe (x, y, z) tal que F = (x, y, z)

Es decir = x, = y, = z. Integrando parcialmente la primera
x y z
x2
componente con respecto a x, obtenemos (x, y, z) = + h (y, z) =
2
2
h y
(x, y, z) = = y = h (y, z) = + g (z)
y y 2
x2 y 2
De modo que (x, y, z) = + + g (z) = (x, y, z) = g 0 (z) = z
2 2 z
x2 y 2 z 2
Por tanto, el potencial es (x, y, z) = + +
2 2 2
y la integral de lnea queda



Z Z
F d
r = xdx + ydy + zdz = (B) (A)
C C
1 1
= 1=
2 2

Autoevaluacion No 2

Tiempo 2 horas
Pregunta 1.

498


Calcular el trabajo realizado por el campo de fuerza F (x, y, z) = (x, y, z)
al mover una partcula a lo largo de la parabola y = x2 , z = x,desde x = 0
hasta x = 2.

Pregunta 2.


Verifique el teorema de Green para el campo F : IR2 IR2 dado por


F (x, y) = (x2 y, xy 2 ) sobre la region R x2 + y 2 a2 .
Pregunta 3.
R 2 x2 y2
Calcular 2xydx + x dy si C es la curva = 1 entre los puntos
4 9
C
A = 4, 3 3 y B = (2, 0) .

Pauta de Correccion

Pregunta 1.


El trabajo realizado por el campo de fuerza F se define por la
integral de trayectoria



Z
W = F d
r
C

donde C es la trayectoria, que en este caso se describe por la


ecuacion parametrica

r (t) = (t, t2 , t) t [0, 2] =

r 0 (t) = (1, 2t, 1). Ademas al evaluar


el campo sobre la trayectoria queda F ( r (t)) = (t, t2 , t)
Entonces, el valor del trabajo es

2 2


Z Z
F (
r (t))

r 0 (t)dt = t + 2t3 + t dt

0 0
= 12

Pregunta 2.
La frontera de R orientada positivamente es la curva C, dada por la
circunferencia x2 + y 2 = a2 , que puede parametrizar como:
x = a cos t, y = a sin t, 0 t 2
Entonces, por una parte se tiene

499


I I
F d
r = x2 ydx + xy 2 dy
C C
Z 2
= (a4 cos2 t sin2 t + a4 cos2 t sin2 t)dt
Z0 2
= (2a4 cos2 t sin2 t)dt
Z0 2
a4
= sin2 2tdt
0 2
a4
=
2
Por otra parte

(xy 2 ) (x2 y)
Z Z   Z Z
dxdy = (y 2 + x2 )dxdy
R x y R
Z 2 Z a
= r2 rdrd
Z0 2 0 4 a
r
= d
0 4 0
a4
=
2
Pregunta 3
Metodo 1
Como f (x, y) = 2xy y g (x, y) = x2 verifican condiciones de continuidad
g f
y derivabilidad , y = 2x = , entonces existe potencial (x, y) = x2 y+C
x y
tal que:
Z
2xydx + x2 dy = (B) (A)
C
 
= (0 + C) 48 3 + C

= 48 3

Metodo 2
Parametrizando la curva como x (t) = 2 cosh t, y (t) = 3 sinh t

500
Autoevaluacion No 3
Tiempo 2 horas
PreguntaR 1 3
Calcular (2xy + yz)dx + (3x2 y 2 + xz)dy +xydz si C es determinada
C
por
a)
r (t) = (t, t2 , t3 ) desde A = (1, 1, 1) hasta B = (2, 4, 8) .
b) x2 + y 2 + z 2 = 2, z = x2 + y 2 .
Pregunta 2
a) Probar que el area de una region D limitada H por curva regular ( o
curvas regulares por tramos) es dada por A (D) = xdy.
C
b) Aplicar la formula anterior para calcular el area de D limitada por las
curvas y = x +2, y = x2
Pregunta H 3
Calcular zdx + xdy + ydz si C es la curva resultante de la interseccion
C
de las superficies x2 + y 2 + z 2 = 4, z = 1.

Pauta de Autoevaluacion

Pregunta 1


Las componentes del campo vectorial F son:
f (x, y, z) = 2xy 3 + yz; g (x, y, z) = 3x2 y 2 + xz; h (x, y, z) = xy
continuas con primeras derivadas continuas


Como el rotacional
del campo F , tiene derivadas parciales continuas y
i j k


F = x y z = (0, 0, 0) es nulo entonces es
2xy 3 + yz 3x2 y 2 + xz xy
conservativo


= existe (x, y, z) tal que F = (x, y, z)

Es decir = 2xy 3 + yz, = 3x2 y 2 + xz, = xz.
x y z
De la primera R de estas expresiones se obtiene que
3
(x, y, z) = (2xy + yz)dx + h (y, z)
(x, y, z) = x2 y 3 + xyz + h (y, z) =
Derivando esta expresion con respecto a y y usando las dos expresiones
restantes para , tenemos
h
(x, y, z) = 3x2 y 3 + xz + = 3x2 y 3 + xz = h (y, z) = g (z)
y y
De modo que (x, y, z) = x2 y 3 + xyz + g (z) =

501
Derivando esta expresion con respecto a z

(x, y, z) = xy + g 0 (z) = xy = g (z) = C
z

Entonces una funcion potencial para F es (x, y, z) = x2 y 3 + xyz + C
Finalmente la integral
Z
(2xy 3 + yz)dx + (3x2 y 2 + xz)dy + xydz = (B) (A)
C
= 318

b) En este caso
I
(2xy 3 + yz)dx + (3x2 y 2 + xz)dy + xydz = 0
C

pues el campo es conservativo y la curva C es cerrada


Pregunta 2
a) El teorema de Green asegura que
I ZZ  
Q (x, y) P (x, y)
P (x, y) dx + Q (x, y) dy = dxdy
x y
C D

Sea P (x, y) = 0 y Q (x, y) = x en D ,usando la expresion anterior se


obtiene
I ZZ
xdy = (1) dxdy = A (D)
C D

a) Calculemos la circulacion , siendo C = C1 C2 la curva que limita a


region D.
Parametrizando las curvas se tiene

C1 :

r 1 (t) = t, t2 , 1 t 2 =

0
y (t) = t2 y y (t) = 2t

C2 : r 2 (t) = (t, t + 2) , 1 t 2 =
0
y (t) = t + 2 y y (t) = 1

502
Aplicando la formula anterior

I
A (D) = xdy
C
Z 2 Z 1
= t (2t) dt + tdt
1 2
2 1
 2  
2 3 t
= t +
3 1 2 2
9
=
2

Pregunta 3
Si C es la curva resultante de la interseccion de las superficies x2 +y 2 +z 2 =
4, z = 1 = x2 + y 2 = 3.
Determinemos la ecuacion parametrica de la trayectoria

C :

r (t) = 3 cos t, 3 sin t, 1 , 0 t 2




0
r (t) = 3 sin t, 3 cos t, 0

Calculemos la integral de lnea

I Z 2
zdx + xdy + ydz = ( 3 sin t + 3 cos2 t + 0)dt
0
C
Z 2
= 3 cos2 tdt = 3
0

503
Captulo 6

Integrales de superficie

La integral de superficie generaliza la integral doble, tal como la integral


de lnea generaliza la integral de Riemann vista en el primer curso de Calculo.
Para ver el significado de esta integral sigamos el siguiente razonamiento: S
es una superficie definida por

z = g(x, y)

y R la region proyeccion de S en el plano xy.


Supongamos que g, gx , gy son continuas sobre R. Sea P una particion de
R tal que cada subrectangulo Ri generado por la particion es la proyeccion
un Si porcion de la superficie S generada por la particion P. Sea 4Si el
area de Si
Sea ademas f : S IR una funcion continua. Definimos en terminos
de las sumas intermedias de Riemann la integral de superficie de f sobre S
como ZZ n
X
f (x, y, z)dS = lm f (xi , yi , zi )4Si
kpk0
S i,j=1

Para el calculo de esta integral:


Sea S superficie definida por z = g(x, y) y R region proyeccion de S en
el plano xy.
Suponiendo g, gx , gy son continuas sobre R y f funcion continua sobre
S, entonces
ZZ ZZ q
f (x, y, z)dS = f (x, y, g(x, y)) 1 + [gx ]2 + [gy ]2 dA
S Rxy

De manera similar.
Si S se expresa de la forma x = g(y, z), (y, z) Ryz entonces

504
ZZ ZZ q
f (x, y, z)dS = f (g(y, z), y, z) 1 + [gy ]2 + [gz ]2 dA
S Ryz

Si S se expresa de la forma y = g(x, z), (x, z) Rxz entonces


ZZ ZZ q
f (x, y, z)dS = f (x, g(x, z), z) 1 + [gx ]2 + [gz ]2 dA
S Rxz

Ejemplo 1:
ZZ
Evalue xydS S : z = 9 x2 , 0 x 2, 0 y x
S

Figura 6.1:

Solucion:

ZZ ZZ
xydS = xy 1 + 4x2 dA
S Rxy
Z 2 Z x
= xy 1 + 4x2 dydx
0 0
No por aqu!

505
ZZ ZZ
xydS = xy 1 + 4x2 dA
S Rxy
Z 2 Z 2
= xy 1 + 4x2 dxdy
0 y

391 17 + 1
=
240

Observacion. ZZ ZZ q
1.- Si f (x, y, z) = 1 = Area de S = dS. = 1 + [gx ]2 + [gy ]2 dA
S R
2.- Si f (x, y, z) = (x,
Z Z y, z) densidad de una lamina ,entonces
Masa de lamina = (x, y, z)dS
S

Ejemplo 2:
Calcule el area lateral del cono z 2 = x2 + y 2 entre los planos z = 1 y
z = 4 usando integral de superficie.

Figura 6.2: Cono z 2 = x2 + y 2 acotado por los planos z = 1 y z = 4

Solucion. q
En primer lugar calculemos el integrando 1 + [gx ]2 + [gy ]2 donde
x2 y2
1 + [gx ]2 + [gy ]2 = 1 + 2 + =2
x + y 2 x2 + y 2
entonces ZZ ZZ
Area de S = dS = 2dA
S R

506
Cambiando a coordenadas polares, se tiene:
ZZ 4
Z 2 4 Z 2
r2
Z 
2dA = 2 rdrd = 2 d
0 1 0 2 0
R
Z 2
= 8 2 d = 16 2
0

6.1. Superficie orientada


Decimos que una superficie es orientada si se puede definir un vector


normal unitario N en todo punto de la superficie que no pertenezca a la
frontera de forma tal que lo vectores normales varan de forma continua
sobre la superficie S.

Figura 6.3: Superficie orientada

Sea S una superficie descrita por z = g(x, y) . Hay dos vectores normales
unitarios a S en (xo , yo , g(xo , y0 )) a saber, si ponemos G(x, y, z) = z g(x, y)
entonces


G
gxbi gybj+bk
N = o sea N = q vector normal unitario
kGk 2 2
1 + [gx ] + [gy ]

Podemos orientar todas estas superficies tomando el lado positivo de S




como el lado desde el cual apunta N . As, el lado positivo de dicha superficie


esta determinado por la normal unitaria N con componente positiva b k.
Por otra parte, si ponemos G(x, y, z) = g(x, y) z

507

G gxbi + gybjb k
N = =q
kGk
1 + [gx ]2 + [gy ]2


Corresponde, al lado negativo de la superficie con normal unitaria N con
componente b
k hacia abajo.

6.1.1. Integral de flujo.




Si F es un campo vectorial perteneciente a C 1 definido sobre una super-
ficie orientada S, con vector unitario N
b , entonces la integral de superficie de

F sobre S es.


b
ZZ ZZ
F dS = F N dS
S S

La que se llama integral de flujo. El nombre es debido a su aplicacion a un


problema fsico de un fludo a traves de una superficie.
Supongamos que una superficie S esta inmersa en un fluido que tiene
un campo de velocidades continuo F.
Entonces

b
ZZ
Volumen de fluido a traves de S = F N dS
(por unidad de tiempo)
S

S es superficie orientada mediante N b, y


F (x, y, z) = (P (x, y, z), Q(x, y, z), R(x, y, z))
campo vectorial tal que P, Q, R son funciones escalares con primeras derivadas
parciales continuas en S.
Ademas si = (x, y, z) representa la densidad del fluido en cada (x, y, z)de
S,entonces

b
ZZ
Masa del fludo a traves de S = F N dS
(por unidad de tiempo)
S

Ejemplo
Sea S la parte del paraboloide z = 4 (x2 + y 2 ) Sobre el plano XY,
orientado hacia arriba. Un fludo de densidad constante = k pasa a traves
de S segun el campo de velocidades.


F (x, y, z) = (x, y, z)

508
Hallar la razon de flujo de masa a traves de S
Solucion:

2xbi + 2yb
j+bk
G(x, y, z) = z 4 + x2 + y 2 = G = (2x, 2y, 1) = N b=p
1 + 4x2 + 4y 2


b
ZZ
Masa del fludo a traves de S = kF N dS
S


b
ZZ ZZ
(2x, 2y, 1) p
x, y, 4 x2 + y 2 p

k F N dS = k 1 + 4x2 + 4y 2 dA
1 + 4x2 + 4y 2
S Rxy

ZZ ZZ
 2
2x + 2y 2 + 4 x2 + y 2 dA = k
 2
x + y 2 + 4 dxdy
 
= k
Rxy Rxy
Z 2 Z 2 Z 2
4 + r2 rdrd = k

= k 12d = 24k
0 y 0

Por lo tanto

Masa del fluido a traves de S = 24k


(por unidad de tiempo)

En base a lo observado en el desarrollo de este ejercicio se tiene el siguiente


planteamiento:
Si S es una superficie definida por z = g(x, y) y Rxy denota la region
proyeccion de S en el plano xy, entonces

b

ZZ ZZ
F N dS = F (gxbi gybj+b k)dA
S Rxy

orientada hacia arriba, y



b

ZZ ZZ
F N dS = F (gxbi + gyb
jb
k)dA
S Rxy

orientada hacia abajo.

509
6.1.2. Superficies Parametrizadas.
En el tratamiento anterior de este tema hemos supuesto que una superficie
viene definida por la ecuacion z = g(x, y) , sin embargo se debe aclarar que
hay superficies que no se pueden definir de esta forma, extenderemos la idea
definiendo la forma parametrica de dar una superficie.


Una superficie parametrizada es una funcion : D IR2 IR3 donde


D es alguna dominio de IR2 La superficie S que corresponde a es su


imagen , es decir S = (D) Podemos escribir


(u, v) = (x (u, v) , y (u, v) , z (u, v)) , (u, v) D

Vease figura 6.4 Si cada una de las funciones componentes son diferenciables
de clase C 1 , entonces llamamos a S una superficie diferenciable o de clase
C 1.

Figura 6.4: Superficie parametrizada

6.1.3. Vector normal a S :




Definimos en cada punto (uo , vo )


x(u0 , v0) b y(u0 , v0) b z(u0 , v0) b
Tu= i+ j+ k
u u u
es vector tangente a la curva t (t, v0 ) en (uo , vo )


x(u0 , v0) b y(u0 , v0) b z(u0 , v0) b
Tv= i+ j+ k
v v v
es vector tangente a curva t (uo , t) en (uo , vo )



Tu Tv es vector normal a la superficie S.

510
Ahora el concepto de superficie suave se da en la siguiente definicion.




Decimos que la superficie S es suave en (uo , vo ) si T u Tv 6= 0 en
(uo , vo ) D. Diremos que una superficie es suave si es suave en todos los


puntos (uo , vo ) S.

Ejemplo 1
Sea S la superficie descrita por x2 + y 2 + z 2 = r2 , z 0
a) Parametrizar usando coordenadas esfericas.
b) Determine el vector normal a la superficie S.

Figura 6.5: semisfera de ecuacion x2 + y 2 + z 2 = r2 , z 0

Solucion:
a) La superficie x2 + y 2 + z 2 = r2 , z 0 corresponde a una semiesfera con
centro el el origen (0, 0, 0) radio r.
Usando coordenadasesfericas tenemos:
x = r sin u cos v
y = r sin u sin v
z = r cos u

Luego la superficie parametrizada es


(u, v) = (r sin u cos v, r sin u sin v, r cos u) , (u, v) D
n o
donde D = (u, v) /0 u , 0 v 2 .
2
b) Determinemos los vectores tangentes a la superficie S



T u = u = (r cos u cos v, r cos u sin v, r sin u)



T v = v = (r sin u sin v, r sin u cos v, 0)
Calculemos el producto vectorial
i j k



T u T v = r cos u cos v r cos u sin v r sin u

r sin u sin v r sin u cos v 0

511
Por lo tanto, el vector normal a la esfera es



T u T v = r2 sin2 u cos v, sin2 u sin v, sin u cos v .


Ejemplo 2
Sea S la superficie descrita por z 2 = x2 + y 2 , z 0.
a) Parametrizar usando coordenadas cilndricas.
b)Es diferenciable esta superficie?
c) Probar que no es suave en (0, 0, 0).

Figura 6.6: cono con vertice en el origen

Solucion:
a) La superficie z 2 = x2 + y 2 , z 0 corresponde a un cono su vertice en
(0, 0, 0) .
Usando coordenadas
cilndricas tenemos:
x = u cos v
= z 2 = x2 + y 2 = u2 cos2 u + sin2 v

y = u sin v
z= u

z = u, u 0
Luego la superficie parametrizada queda



(u, v) = (u cos v, u sin v, u) , u 0

b) Como cada una de las componentes es diferenciable como funcion de


u, v
entonces S es una superficie diferenciable.
c) Veamos si es o no una superficie suave

512



T u = (cos v, sin v, 1) = T u (0, 0) = (cos 0, sin 0, 1)


T u (0, 0) = (1, 0, 1)




T v = (u sin v, u cos v, 0) = T v (0, 0) = (0 ( sin 0) , 0 cos 0, 0)



T v = (0, 0, 0) = 0



As Tu T v = 0 , de modo que la superficie no es suave en (0, 0, 0).
Para efectos de calculo, consideraremos solo superficies suaves por


pedazos que sean uniones de imagenes parametrizadas i : Di IR2
IR3 ,para las cuales:
i) Di es una region elemetal del plano.


ii) i es de clase C1 y uno a uno, excepto quizas en la frontera D.


iii) Si es la imagen de i es suave , excepto quizas en un numero finito
de
puntos.

6.1.4. Area de una superficie parametrizada


Definimos el area de una superficie parametrizada S , denotada por A (S) ,mediante:


ZZ
A (S) = Tu T v dudv

D





donde Tu T v es la norma del vector normal T u T v .

Propiedades 6.1.1. i) Si S es la union de superficies parametrizadas S =


Si entonces
X
A (S) = A (Si )
i=1
s 2  2  2

(x, y) (y, z) (z, x)
ii) Tu T v = + +

(u, v) (u, v) (u, v)

513
Figura 6.7: Area de una superficie parametrizada

Figura 6.8: manto del paraboloide z = x2 + y 2 , 0 z a2

Ejemplo 1
Determine el area del manto del paraboloide z = x2 + y 2 , 0 z a2 .
Solucion:
Usando coordenadas cilndricas tenemos


(u, v) = u cos v, u sin v, u2 , (u, v) D


donde D = {(u, v) /0 u a, 0 v 2}

Como el area del manto esta dado por



ZZ
A (S) = Tu T v dudv

D

514
Calculemos el integrando
(x, y) cos v u sin v
= = u,
(u, v) sin v u cos v
(y, z) sin v u cos v 2
=
(u, v) 2u 0 = 2u cos v

(z, x) 2u 0
= 2u2 sin v
=
(u, v) cos v u sin v
As el integrando del area es
2


Tu T v = u + 4u4 cos2 v + 4u4 sin2 v = u 1 + 4u2



Es evidente que Tu T v se anula para u = 0 y (0, v) = (0, 0, 0)

para
cualquier v. Luego (0, 0, 0) es el unico punto donde la superficie no es
suave.
Entonces
Z 2 Z a


ZZ
Tu T v dudv = u 1 + 4u2 dudv

D 0 0
Z 2
1 h 3/2 ia
= 1 + 4u2 dv
12 0 0
h 3/2 i
= 1 + 4a2 1
6

Area de una superficie en coordenadas cartesianas.


Una susperficie S dada de la forma z = g (x, y) donde (x, y) D, admite
la parametrizacion


(x, y) = (x, y, g (x, y)) , (x, y) D
C 1 , esta parametrizacion
Si f esde clase    es suave, entonces
 

g
g

g g
T x = 1, 0, y T y = 0, 1, = T x T y = , , 1
x y x y
Entonces: s 
2  2

g g
T x T y = + + 1.

x y
De este modo la formula para el area de la superficie se reduce a



ZZ
A (S) = Tx T y dxdy

D
s 
ZZ 2  2
g g
= + + 1 dxdy
D x y

515
Resultado que coincide con lo propuesto en la primera parte.

6.1.5. Integral de una funcion escalar sobre una su-


perficie.
Si f : D IR3 IR,dada por f (x, y, z) es una funcion continua definida
sobre una superficie parametrizada S, definimos, la integral de f sobre S como:
 

Z Z ZZ
f dS = f (u, v) Tu T v dudv

S D

Si S es la union de superficies parametrizadas S = Si entonces


Z Z XZ Z
f dS = f dS
S i=1 Si

Ejemplo
Z Z
Evaluar z dS, donde S es el hemisferio superior de radio r de la
p S
esfera z = r2 x2 y 2 .
Solucion.
Para este problema es conveniente representar el hemisferio
parametricamente por:


(u, v) = (r sin u cos v, r sin u sin v, r cos u) , (u, v) D

sobrenla region D del plano uv dada omediante



D = (u, v) /0 u , 0 v 2
2
De las ecuaciones parametricas obtenemos, los vectores tangentes a la
superficie S.



T u = u = (r cos u cos v, r cos u sin v, r sin u)



T v = v = (r sin u sin v, r sin u cos v, 0)
Calculemos el producto vectorial
i j k



T u T v = r cos u cos v r cos u sin v r sin u

r sin u sin v r sin u cos v 0
Luego, el vector normal a la esfera es

516



r2 sin2 u cos v, sin2 u sin v, sin u cos v

Tu Tv =



= u T v = r2 sin u
T

De modo que

Z Z Z 2 Z /2
(r cos u) r2 sin u dudv

z dS =
S 0 0
Z 2 Z /2
= r3 cos u sin ududv
0 0
2 /2
sin2 u
Z 
3
= r dv
0 2 0
= r3

6.1.6. Integral de Superficie de campos vectoriales


Sean S una superficie regular orientable, N b : S IR3 un campo de


normales continuo sobre S, y F : IR IR3 un campo vectorial
3

continuo definido sobre un abierto que contiene a S. Definimos la integral




de F sobre la superficie S orientada segun N
b mediante


b


ZZ ZZ
F N dS = F u v dudv

D



donde : D IR2 IR3 es una parametrizacion regular de S compatible
con la orientacion, esto es, tal que



u v
N=
b


u v

Para interpretar correctamente el valor de la integral de flujo es necesario


especificar el campo de normales N b . Por ejemplo, si orientamos un casquete
esferico usando la normal exterior N b = rb, la integral de flujo corresponde
al flujo neto que sale de la esfera a traves del casquete. Si este valor fuese
negativo, significa que lo que el flujo entra a la esfera.

517
Ejemplo 1:

Sea S es la esfera unitaria x2 + y 2 + z 2 = 1 parametrizada en coordenadas




esfericas por (, ) = (sin cos , sin sin , cos ) , 0 , 0 2.

RR b
Sea F (x, y, z) = (x, y, z) radio vector del origen. Calcular F N dS


Solucion:
En este caso D es region del plano definida por: 0 , 0
2.


T = (cos cos , cos sen, sin )



T = ( sin sin , sin cos , 0)




T T = cos sin2 bi sin sin2 b
 
j + (cos sin ) b
k

Notese que el vector normal apunta hacia el lado positivo de la superficie




F T T = sin


b
ZZ ZZ
F N dS = sin dd

D

Z 2 Z
= sin dd
0 0
= 4

Observacion
Si S es una superficie regular orientada segun un campo de normales Nb

y si S es la misma superficie pero con la orientacion opuesta N , entonces
b
se tiene que



ZZ ZZ
F dS = F dS
S S

518
6.1.7. Aplicacion al campo de la fsica:
Calcular el flujo del campo electrico producido por una carga Q en el
origen, a traves del manto de la esfera S de radio R orientado segun la normal
exterior.
El campo electrico producido por la carga Q viene dado por


Q rb
E =
4o r2
donde o una constante universal . De esta forma se obtiene el siguiente
flujo
electrico



ZZ ZZ
Q rb
E dS = rbdS
4o R2
S S

Parametrizando la superficie en coordenadas esfericas se tiene




(, ) = (R sin cos , R sin sin , R cos ) , 0 , 0 2


= = R2 sin

ZZ Z 2 Z
Q rb Q 1 2
rbdS = R sin dd
4o R2 0 0 4o R2
S
Q
=
o

6.2. Teoremas de Gauss y de Stokes

Ahora dos teoremas de gran importancia en el campo de las aplicaciones,


particularmente en la fsica, el primero de ellos es el teorema de Gauss o
teorema de la divergencia.

6.2.1. Divergencia


En IR3 ,sea F un campo vectorial,anteriormente hemos definimos



P Q R
div F = F = + +
x y z

519
Ejemplo.

2
Sea F (x, y, z) = (xy, y 2 +exz , sen(xy)), calcule la divergencia del campo
vectorial 2
xy (y 2 + exz ) sen(xy)
divF = + + = 3y
x y z
El foco de atencion lo ponemos ahora en regiones R solidas simples (
por ejemplo cajas rectangulares, esferas, elipsoides etc). La frontera S es
una superficie cerrada y convenimos que la orientacion positiva es hacia
afuera, representada por el vector normal N b hacia afuera de la superficie.
En este contexto enunciamos el teorema.

6.2.2. Teorema de la divergencia de Gauss.


Sea R una region simple solida limitada por una superficie cerrada S


orientada por un vector normal unitario dirigido al exterior de R. Si F es
un campo vectorial cuyas funciones componentes P, Q y R tienen derivadas
parciales continuas en la region R, entonces

b

ZZ ZZZ
F N dS = div F dV
S R

Demostracion.
Solo daremos algunas ideas generales para indicar un procedimiento para
la
demostracion


Sea F (x, y, z) = (P (x, y, z), Q(x, y, z), R(x, y, z))


P Q R
= div F = F = + +
x y z
Integrando


ZZZ ZZZ ZZZ ZZZ
P Q R
div F dV = dV + dV + dV
x y z
R R R R

Por otro lado, si N


b es el normal unitario de S hacia afuera la integral de
superficie es


b
ZZ ZZ ZZ ZZ ZZ
F dS = F N dS = P i N dS +
b b Qj N dS +
b b RbkNb dS
S S S S S

520
Siguiendo el procedimiento de la demostracion del teorema de Green basta
probar las siguientes tres igualdades:

ZZZ ZZ
P
dV = Pbi N
b dS
x
Z ZRZ Z SZ
Q
dV = jN
Qb b dS
y
Z Z ZR Z SZ
R
dV = kN
Rb b dS
z
R S

Detalles y comentarios los puede consultar en el texto Calculo de James


Stewart, Tercera edicion (Thomson), pagina 935

Ejemplo 1:
ZZ
2 2 2
Sea S esfera unitaria x +y +z = 1. Demostrar que (xy + yz + xz) dS =
S
0
Solucion:
Claramente xbi + yb
j + zb
k es vector normal a la esfera en todo (x, y, z) .

xbi + yb
j + zbk
N
b = p
x2 + y 2 + z 2
= xbi + ybj + zbk, ya que x2 + y 2 + z 2 = 1

consideramos ahora el siguiente arreglo para identificar

xy + yz + xz = (y, z, x) (x, y, z)


= (y, z, x) N


= F = (y, z, x)


P Q R
F = + + =0+0+0=0
x y z
Aplicando Teorema de Divergencia

521


ZZ ZZZ
(xy + yz + xz) dS = F dV
S
Z ZRZ
= 0dV = 0
R

Ejemplo 2

b

ZZ ZZZ
Verificar el teorema de la divergencia F N dS = div F dV para
S R


F (x, y, z) = (2x y)bi (2y z)b
j + zb
k S: limitado por los planos
coordenados
y x + 2y + z = 6

Figura 6.9: S limitado por los planos coordenados y x + 2y + z = 6

Solucion.
En este caso S = S1 + S2 + S3 +S4 donde S1 sera el lado de la figura en
el plano x + 2y + z = 6, S2 el lado de la figura en el plano coordenado
yz, S3 el lado de la figura en el plano coordenado xz, S4 el lado
correspondiente al plano coordenado xy.
Tenemos


b
b
b
b
b
ZZ ZZ ZZ ZZ ZZ
F N dS = F N1 dS + F N2 dS + F N3 dS + + F N4 dS
S S1 S2 S3 S4

522

b

ZZ ZZ
F N1 dS = F (x, y, z) (gx (x, y), gy (x, y), 1)dA
S1 Rxy


ZZ ZZ
= F (x, y, z) (1, 2, 1)dA = (2x y, (2y z), z) (1, 2, 1)dA
Rxy Rxy
ZZ ZZ
= ((2x y) 2(2y z) + z)dA = (2x 5y + 3(6 x 2y)dxdy
Rxy Rxy
Z 6Z 21 x+3 Z 6
7 9 9
= (x 11y + 18)dydx = ( x2 + x + )dx = 45
0 0 0 8 2 2


b

ZZ ZZ ZZ
F N2 dS = F (bi)dS = (y 2x)dS
S2 S S2
Z Z2
= (y 2(6 2y z)dydz
Ryz
Z 3 Z 62y
= (5y + 2z 12) dzdy = 27
0 0


b

ZZ ZZ
F NdS = F (b
j)dS
S3 S
Z Z3
6xz
ZZ
= (2y z)dS = (2( ) z)dxdz
2
S3 Rxz
Z 6 Z 6x
= (6 x 2z) dzdx = 0
0 0


b

ZZ ZZ
F N4 dS = F (b
k)dS
S4 S
Z Z4
= 0dxdy = 0
Rxy

De estos calculos se tiene que



b
ZZ
F N dS = 45 27 + 0 + 0 = 18
S

523


Por el otro lado div F = 2 2 + 1 = 1, la integral triple es

6 x2 +3 6x2y


ZZZ ZZZ Z Z Z
div F dV = dV = dzdydx
0 0 0
R R
6 Z x2 +3 6
x2
Z Z  
= (6 x 2y)dydx = 3x + 9 dx = 18
0 0 0 4

6.2.3. Teorema de Stokes.


Este teorema da la relacion entre una integral de superficie sobre una
superficie orientada S y una integral de lnea a lo largo e una curva cerrada
siempre seccionalmente suave C con orientacion positiva que acota a S.

Teorema 6.2.1. Sea S una superficie limitada por una curva C ,orienta-


da positivamente, seccionalmente suave. Si F es un campo vectorial cuyas
componentes tienen derivadas parciales continuas en una region abierta que
contiene a S y C, entonces.





I ZZ  ZZ 

F ds = rot F d S = F dS
C S S

Recuerde que la integral de lnea se puede expresar:



Z Z
P dx + Qdy + Rdz o F Tbds
C C

Ejemplo 1



I
Use el teorema de Stokes para evaluar F d
s , donde F (x, y, z) =
C
C
(3z, 5x, 2y) y C es la interseccion del plano z = y + 3 con el cilindro
x2 + y 2 = 1. Oriente la elipse en sentido contrario al movimiento de las
agujas del reloj, vista desde arriba.
Solucion.
La interseccion del plano con el cilindro permite visualizar que S en este
caso es la porcion del plano que queda al interior del cilindro y que C es la
elipse interseccion

524


 b
I ZZ 
F d
s = F N dS
C S

bi b
j k
b



F = x y z
= (2bi + 3bi + 5b
k)
3z 5x 2y

zy+3=0 b = 1 (b
= N j + k)
2
Calculando

 b 1
F N k) (b
= (2bi + 3bi + 5b j + k)
2
2
= = 2
2
y entonces la integral sera
ZZ ZZ


I

F ds = 2dS = 2 dS
C S S

= 2 Area(S) = 2 (1 2) = 2

S es el interior de una elipse de semi ejes 1 y 2.

6.3. Problemas Resueltos


Problema 1
Sea la funcion
r : D IR2 , definida por

r (r, ) = (r cos , r senv, r)
donde D = {(r, ) /0 r 1, 0 2} una parametrizacion de un cono
S. Hallar su area de superficie.

Solucion Z Z
El area de una superficie parametrizada se define A(S) = k

r u

r v k dudv
D
Calculemos el integrando
s de area usando la propiedad:
2  2  2


(x, y) (y, z) (z, x)
k r r r k = + +
(r, ) (r, ) (r, )
siendo los Jacobianos

(x, y) cos rsen
= =r
(r, ) sen r cos

525
Figura 6.10: Representacion parametrica de un cono

(y, z) sen r cos
= = r cos
(r, ) 1 0
(z, x) 1 0
= = rsen
(r, ) cos rsen
As que el integrando
de area es



k r r r k = r + r2 cos2 + r2 sen2 = r 2
2

Por tanto,el area de la superficie S es

Z Z Z 2 Z 1
A(S) = k
r r
r k dudv = 2rdrd
D 0 0
Z 2  2 1
r
= 2 d
0 2 0
Z 2
2
= d
2
0
= 2

Observacion, k r r r kse anula para r = 0,pero


r (0, ) = (0, 0, 0) .
As, (0, 0, 0) es el unico punto donde la superficie no es suave.

Problema 2

Calcular el area S de la region del cono definido por x2 = y 2 + z 2 , interior


al cilindro x2 + y 2 = a2 , acotado en el octante x 0, y 0, z 0.
Solucion
Parametrizando la ecuacion del cono, usando coordenadas cartesianas
(y, z),se
tiene que

526
Figura 6.11: region del cono definido por x2 = y 2 +z 2 interior al cilindro x2 +y 2 =
a2 , acotado en el octante el primer octante



p 
r (y, z) = 2 2
y + z , y, z , (y, z) D
La superficie S esta al interior del cilindro , por lo que

x2 + y 2 = 2y 2 + z 2 a2 .

Entonces, la region D esta definida sobre el plano yz por


D = {(y, z) IR2 /2y 2 + z 2 a2 , y 0, z 0} .
El area de la superficie es:
Z Z
A(S) = k

r y
r z k dydz
D

luego ,si

! !

y z
ry = p , 1, 0 ,

rz=
p , 0, 1 =
y2 + z2 y2 + z2

y z
r y

rz = (1, p , p )
2
y +z 2 y + z2
2

= k
r
y

r k= 2
z

Z Z
A(S) = 2dydz
D
2 2 2
Observemos que: 2y + z a , y 0, z 0

527
y2 z2
2 + 1, y 0, z 0 es el area de la cuarta parte de la elipse
a2

a

2
Usando cambio de variables tenemos:
a a cos a rsen

x = r cos a2
2 = (x,y) = 2 2 = r

(r,)
y = arsen asen ar cos 2

y2 z2
2 + 1 0 r2 1 = 0 r 1 y 0
a2

a

2
Entonces la region D = {(r, ) /0 r 1 y 0 } .Luego, si

Z Z Z Z a2
A(S) = 2dydz = 2 rdrd
D D 2
Z /2 Z 1 Z /2 2 1
r
= a2 rdrd = a2 d
0 0 02 0
2
a
=
4
Problema 3
Calcular el area S de la region del manto del cilindro x2 + y 2 = 2y
comprendida entre y + z = 2 y z = 0.

Figura 6.12: Region del manto del cilindro x2 + y 2 = 2y comprendida entre


y + z = 2 y z = 0. D, parametrizacion de esta region

Solucion
El area de la superficie es:
Z Z
A(S) = k

r u

r v k dudv
D

528
Parametrizando la superficie del cilindro, x2 +y 2 = 2y x2 +(y1)2 =
1
Usando coordenadas cilindricas (u, v),se tiene que



r (u, v) = (cos u, 1 + senu, v) , (u, v) D

donde D = {(u, v) /0 u 2, 0 v 2 (1 + senu)}


Determinemos el integrando de area

i j k


r u

rv =

senu cos u 0 = (cos u, senu, 0)

0 0 1
= k
r
u
r k=1 v

Por tanto,el area de la superficie S es

Z Z
A(S) = dudv
D
Z 2 Z 1senu
= dvdu
0 0
Z 2
= (1 senu) du
0
= 2

6.3.1. Integrales de superficie


Problema 1
Sea S el octante positivo de la superficie x2 + y 2 + z 2 = 1.Calcular la
RR 1
integral de superficie S
p dS
x2 + y 2 + (z 1)2
Solucion
Parametricemos la Superficie S usando coordenadas esfericas, es decir



r (u, v) = (senu cos v, senu senv, cos u)

onde D = {(u, v) /0 u /2, 0 v /2}

Calculemos el valor del integrando sobre S

529
1 1
p = p
x2 + y 2 + (z 1)2 sen2 u cos2 v + sen2 u sen2 v + (cos2 u 1)2
1
=
sen2 u + cos2 u + 1 2 cos u
1
=
2 2 cos u

Calculemos el producto vectorial


i j k


r u

rv

= cos u cos v cos u senv senu

senu senv senu cos v 0
= sen2 u cos v, sen2 u senv, senu cos v


Luego , obtenemos la norma del producto :


k
r u 1/2
r v k = (sen4 u + sen2 u cos2 u) = |senu| = senu porque 0
u /2
Finalmente, apliquemos la definicion de integral de superficie y calculemos

/2 /2
k

r r vk
Z Z Z Z
1
p dS = u dudv
S x2 + y 2 + (z 1)2 0 0 2 2 cos u
Z /2 Z /2
senu
= dudv
0 0 2 2 cos u

Z /2
1 /2
= 2 1 cos u 0 dv
2 0
Z /2
2
= dv
2 0

=
2
ProblemaZZ 2
Evaluar z 2 ds, donde S es el paraboloide z = x2 + y 2 , comprendido
S
entre z = 1 y z = 4.
Solucion
Para este problema tenemos dos opciones para representar parametrica-
mente la corona del paraboloide.

530
Figura 6.13:

Usando coordenadas cartesianas rectangulares

r (x, y) = (x, y, x2 + y 2 ) , donde D = {(x, y) IR2 /1 x2 + y 2 4}


Calculemos el vector normal a la superficie S, dado por

i j k

r x
r y = 1 0 2x = (2x, 2y, 1)
0 1 2y

Luego , obtenemos la norma del producto :


k

r p
x r k = 4x2 + 4y 2 + 1
y

Aplicando la definicion de integral de superficie

ZZ Z Z
2
z ds = z 2 k

r y

r z k dxdy
S Z ZD p
= (x2 + y 2 ) 4x2 + 4y 2 + 1dxdy
D

Usando coordenadas polares para evaluar la integral doble,

531
Z Z
2 2
p Z 2 Z 2
(x + y ) 4x2 + 4y 2 + 1dxdy = r2 4r2 + 1rdrd
D 0
"1 #
2 3/2 2 2
Z Z
2 1 2
2
3/2
= r 4r + 1 r 4r2 + 1 dr d
0 12
1 12 1
Z 2 " 2 2 #
1 3/2 1 5/2
r2 4r2 + 1 4r2 + 1

= d
0 12
1 120
1
 
1 1 3/2
= 173/2 53/2 175/2 + 5
6 10 10

6.3.2. Integral de Flujo de un campo vectorial


Problema 1


Calcular el flujo para F (x, y, z) = (3x, 3y, z) en la superficie z = 9
x2 y 2 tal que z 0

Figura 6.14:

Solucion
Usando coordenadas cartesianas rectangulares, la parametrizacion de la
superficie S es

r (x, y) = (x, y, 9 x2 y 2 ) , donde D = {(x, y) IR2 /x2 + y 2 9, z 0}
Calculemos el vector normal a la superficie S, dado por

i j k



r x r y = 1 0 2x = (2x, 2y, 1)
0 1 2y

Aplicando la definicion de integral de superficie al flujo a traves de S, se


obtiene

532



ZZ ZZ
F n
bds = F (
r (x, y)) (

r x

r y ) dxdy
S Z ZD
(3x, 3y, z) (2x, 2y, 1)dxdy
Z ZD
6x2 + 6y 2 + z dxdy

=
Z ZD
6x2 + 6y 2 + 9 x2 y 2 dxdy

=
Z ZD
5x2 + 5y 2 + 9 dxdy

=
D

Cambiando a coordenadas polares

ZZ Z 2 Z 3
2 2
5r2 + 9 rdrd
 
5x + 5y + 9 dxdy =
D 0 0
567
=
2

Alternativamente, podemos parametrizar directamente la superficie S en


coordenadas cilndricas.



r (r, ) = r cos , rsen, 9 r2 , 0 r 3, 0 2


Calculemos el vector normal a la superficie y el campo vectorial sobre la


superficie:


i j k




n = r r r = cos

sen 2r
rsen r cos 0
= = 2r2 cos , 2r2 sen, r


Aplicando la definicion de integral de superficie al flujo sobre S, se obtiene

533



ZZ ZZ
F n
bds = F (
r (r, )) (

r r

r ) drd
S Z ZD
3r cos , 3 rsen, 9 r2 2r2 cos , 2r2 sen, r drd
 
=
Z ZD
6r3 (cos2 + sen2 ) + 9r r3 drd

=
Z ZD
5r3 + 9r drd

=
D
Z 2 Z 3
567
5r3 + 9r drd =

=
0 0 2

Problema 2



Sea F el campo vectorial F (x, y, z) = (y, x, xz) . Calcular la integral de
RR

flujo ( F ) n
bdS, donde nb el campo normal exterior al paraboloide, y S
S
el sector de la superficie del paraboloide z = x2 + (y 1)2 , interior al cilindro
x2 + (y 2)2 = 3.

Figura 6.15: S, sector de la superficie del paraboloide z = x2 + (y 1)2 , interior


al cilindro x2 + (y 2)2 = 3

Solucion
Tenemos que calcular la integral de flujo




ZZ ZZ
( F ) n
bdS = ( F )
n dS
S D

534


En primer lugar, calculamos el rotor del campo F ,

i j k



F = = (0, z, 0),
x y z
y x xz

Parametricemos el sector de la superficie del paraboloide z = x2 +(y 1)2


interior al cilindro x2 + (y 2)2 = 3 usando coodenadas cilindricas con centro
en el punto (0,2,0) y radio 3.Entonces


(u, v) = (ucosv, 2 + usenv, u2 + 2usenv + 1) , 0 u 3, 0 v 2.
Determinemos los vectores tangentes a la superficie del paraboloide


u (u, v) = (cosv, senv, 2u + 2senv)


v (u, v) = (usenv, ucosv, 2ucosv)
Dado que en el vertice (0, 1, 0) del paraboloide el vector normal apunta
hacia el exterior , tenemos que la normal exterior tiene componentes (0,0,-1),
entonces

i j k




n = v (u, v) u (u, v) = usenv ucosv 2ucosv
cosv senv 2u + 2senv
= 2u2 cosv, 2u2 senv + 2u, u


En consecuencia, la integral queda



ZZ Z Z
F
n dS = (0, u2 + 2usenv + 1, 0) (2u2 cosv, 2u2 senv + 2u, u)dudv
D
D

Z 3 Z 2
= (2u4 senv + 4u3 sen2 v + 6u2 senv + 2u3 + 2u)dvdu
0 0
2
Z 3   
3 v sen2v 3
= 4u + (2u + 2u)v du
0 2 4 0

Z 3
 3
8u3 + 4u du = 2u4 + 2u2 0
 
=
0
= 24

Problema 3


Calcular el flujo de salida del campo vectorial F (x, y, z) = (2y, zy, 3z) a
traves de la superficie cilndrica x2 + y 2 = 4 acotada en el primer octante por
los planos x = 0, y = 0, z = 0, z = 5.

535
Figura 6.16: Superficie cilindrica x2 + y 2 = 4, acotada en el primer octante por
los planos coordenados

Solucion
Tenemos que calcular




 


ZZ ZZ
F n
bdS = F (, z) z ddz
S D

Parametricemos la superficie cilndrica



(, z) = (2 cos , 2 sin , z)

donde D = {(, z) /0 /2, 0 z 5}.


Determinemos los vectores tangentes a la superficie


(, z) = (2 sin , 2 cos , 0)


z (, z) = (0, 0, 1)
El vector normal que apunta al exterior de la superficie es

i j k



(, z) z (, z) = 2sen 2 cos 0 = (2cos, 2sen, 0)
0 0 1

536


Entonces, el flujo de F a traves de esta superficie es


  

ZZ
F (, z) z drdz
Z ZD
= (4 sin , 2z sin , 3z) (2 cos , 2 sin , 0) ddz
D
Z /2 Z 5
4 5 sin 2 + z sin2 dzd

=
0 0
Z /2   
25 1 cos 2
= 4 sin 2 + d
0 2 2
 /2  /2
cos 2 sin 2
= 20 + 50
2 0 2 4 0
25
= 20 +
2

6.3.3. Teorema de la divergencia de Gauss


Problema 1 Z Z



Calcular el flujo ndS del campo vectorial F (x, y, z) = (x2 + senz, xy + cos z, ey )
F b
S
a traves de la frontera S limitada por la superficie cilndrica x2 + y 2 = 2y y
los planos z = 0, y + z = 2,

Figura 6.17: S limitada por la superficie cilndrica x2 + y 2 = 2y y los planos z = 0,


y + z = 2,

Solucion

537


Puesto que el campo vectorial F es continuo, con primeras derivadas
parciales continuas en IR3 y la region V IR3 encerrada es conexa y S es una
superficie cerrada suave, entonces es aplicable el teorema de la divergencia
de Gauss.




ZZ ZZZ
F nbdS = F dV
S V

Para resolver la ultima integral necesitamos describir el solido V. Comple-


tando el cuadrado, la ecuacion de la superficie cilndrica dada puede escribirse
como:
x2 + y 2 = 2y x2 + (y 1)2 = 2y.
Luego, la proyeccion del solido sobre el plano XY es la circunsferencia
de centro (0, 1) y radio 1. Por otra parte z esta acotado entre los dos planos
dados 0 z 2 y.
Por tanto,
n el solido esta dado por o
p p
3 2 2
V = (x, y, z) IR / 2y y x 2y y , 0 y 2, 0 z 2 y
Calculemos la divergencia del campo vectorial:

y
F = (x2 + senz) + (xy + cos z) + (e ) = 3x
x y z
Entonces:

ZZZ Z 2 Z 2yy2 Z 2y
3xdxdydz = 3xdzdxdy
V 0 2yy 2
0

Z 2 Z 2yy2
= 3x (2 y) dxdy
0 2yy 2
Z 2   2yy2
3 2
= (2 y) x dy = 0
0 2 2yy 2

Problema 2
Verificar el teorema de la divergencia para el campo vectorial

F (x, y, z) = (x, y, z) sobre la esfera x2 + y 2 + z 2 = a2 .


Solucion
El teorema de la divergencia afirma que.




ZZ ZZZ
F n bdS = F dV
S V


La divergencia de F es

538


F = (x) + (y) + (z) = 3.
x y z
de modo que.



ZZZ ZZZ
F dV = 3 dV
V V
 
4 3
= 3 a
3
= 4a3

Calculemos ahora el Flujo sobre la superficie.




ZZ ZZ
F nbdS = F (r (u, v)) (

r u

r v ) dudv
S D

Parametricemos la superficie usando coordenadas esfericas.




r (u, v) = a (sin u cos v, sin u sin v, cos u) , con D = {(u, v) /0 u , 0 v 2}
Calculemos el producto vectorial

i j k


2

r u r v = a cos u cos v cos u sin v sin u

sin u sin v sin u cos v 0
= a2 sin2 u cos v, sin2 u sin v, sin u cos u




Por consiguiente F ( r (u, v)) (

r u
r v ) = a3 sin u
entonces la integral de flujo queda



ZZ ZZ
F nbdS = a3 sin ududv
S D
Z 2 Z
= a3 sin ududv
0
Z 20
= a3 [ cos u]0 dv
Z0 2
= 2a3 dv
0
= 4a3

Por tanto, comparando estos resultados hemos verificado el teorema de


Gauss.

Problema 3

539
Figura 6.18: Region acotada por la frontera x2 + y 2 = 4, z = 0, y z = 3



Verificar el teorema de la divergencia para F (x, y, z) = (x3 , y 3 , z 3 ) en
R, la region acotada por la frontera x2 + y 2 = 4, z = 0,y z = 3.
Solucion RR RRR

El teorema de la divergencia afirma que S
F n
b ds = R
F dV
Calculemos el flujo que produce el campo sobre la frontera:






ZZ ZZ ZZ ZZ
F n
bds = F n
bds + F n
bds + F n
bds
S S1 S2 S3

Si definimos parametrizacion de las superficies inferior y superior que


limitan el cilindro tenemos:
S1 :
r (u, v) = (u, v, 0) D = {(u, v) /0 u 2, 0 v 2, 0}
El vector normal
con orientacion
positiva es
i j k

r u
r v = 1 0 0 = (0, 0, 1)
0 1 0
As , obtenemos:




ZZ ZZ
F n
bds = F (
r (u, v)) (

r u

r v ) dudv
S1
Z ZD
u3 , v 3 , 0 (0, 0, 1) dudv = 0

=
D

Analogamente para:

540
S3 :

r (u, v) = (u, v, 0) donde D = {(u, v) /0 u 2, 0 v 2}

i j k



r u r v = 1 0 0 = (0, 0, 1)
0 1 0
Luego:




ZZ ZZ
F n
bds = F (
r (u, v)) (

r u

r v ) dudv
S3
Z ZD
u3 , v 3 , 33 (0, 0, 1) dudv

=
D
Z 2 Z 2
= 33 dudv
0 0
= 108
Si definimos la parametrizacion para el manto del cilndro se tiene:
S :
r (u, v) = (2 cos u, 2senu , v)
2

donde D = {(u, v) /0 u 2, 0 v 3}

El vector normal con orientacion positiva es,



i j k



r u r v = 2senu 2 cos u 0 = (2 cos u, 2senu, 0)

0 0 1
En consecuencia,




ZZ ZZ
F n
bds = F (
r (u, v)) (

r u

r v ) dudv
S2
Z ZD
(2 cos u)3 , (2senu)3 , v 3 (2 cos u, 2senu, 0) dudv

=

ZD 3 Z 2
cos4 u + sen4 u dudv

= 16
Z0 3 Z0 2
1
= 16 (6 + 2 cos 4u) dudv
0 0 8
Z 3  2
1 2
= 16 6u + sen4u dv
0 8 4 0
Z 3
= 2 (12) dv
0
= 72

541
Por lo tanto, sumando las integrales de flujo se tiene:


ZZ
F nbds = 102 + 72 = 180
S


x3 y 3 z 3
Por otra parte, como F = + + = 3 (x2 + y 2 + z 2 ) ,
x y z
entonces


ZZZ ZZZ
3 x2 + y 2 + z 2 dV

F dV =
R R
Usando coordenadas cilndricas para evaluar la integral triple, tenemos

x = r cos
y = rsen = 0 x2 + y 2 4 0 r2 4 0 r 2
z= z

Entonces la region R = {(r, , z) /0 r 2 , 0 2, 0 z 3}

ZZZ Z 2 Z 2 Z 3
2 2 2
r2 + z 2 rdzdrd
 
3 x +y +z dV = 3
R 0 0 0
Z 2 Z 2  3
2 r3
= 3 r z+ rdrd
0 0 3 0
Z 2 Z 2
 2 
= 3 3r + 9 rdrd
0 0
Z 2  2
3 4 9 2
= 3 r + r d
0 4 2 0
Z 2
= 3 30d
0
= 90|2
0 = 180

Problema 4


Calcular el flujo del campo F (x, y, z) = (x + y 3 , 2y ez , 3z 1) en la
R la region acotada por la frontera S x2 + y 2 + 3z 2 = 1.
Solucion:
Usemos el teorema de la divergencia de Gauss para calcular el flujo, que
afirma que,



ZZ ZZZ
F nbds = F dV
S R
Calculemos la divergencia del campo vectorial en la region R.

542


F = (x + y 3 ) + (2y ez ) + (3z 1) = 1 + 2 3 = 0
x y z


Entonces el flujo del campo vectorial F a traves de S verifica



ZZ ZZZ
F n
bds = 0 dV = 0
S R

6.3.4. Teorema de Stokes


Problema 1


Verificar el teorema de Stokes para F (x, y, z) = (y 3 , x3 , z 3 ) donde S
es la porcion del plano x + y + z = 1 al interior del cilindro x2 + y 2 = 1.

Figura 6.19: S es la porcion del plano x + y + z = 1 al interior del cilindro


x2 + y 2 = 1

Solucion.
H


El teorema de Stokes afirma que C F d
RR
r = S F n
bdS
Calculemos

i j k



F = = (0, 0, 3x2 + 3y 2 ),
x3 y3 z3

y x z


y N = (x + y + z 1) = (1, 1, 1)

543



ZZ ZZ
F n
bdS = F N dxdy
S Z ZD
= 3(x2 + y 2 ) dxdy
D

donde D = {x, y) IR2 /x2 + y 2 1} .


Usando coordenadas polares :

x = r cos cos rsen
= | (x, y) (r, )| = =r
y = rsen sen r cos
x2 + y 2 1 0 r2 1 = 0 r 1 y 0 2

Entonces la region D = {(r, ) /0 r 1 y 0 2} .Luego, si

ZZ Z 1 Z 2
2 2
3(x + y ) dxdy = 3 r3 ddr
D 0 0
Z 1
= 6 r3 dr
0
3
=
2
Problema 2

Verificar el teorema de Stokes para evaluar la integral de lnea


Z
xdx + yz 2 dy + xzdz
C

donde C es la interseccion de la semiesfera x2 + y 2 + z 2 = 1, z 0 y el


cilindro x2 + y 2 = y.

Solucion
El teorema de Stokes afirma que



I ZZ

F dr = F n
bdS
C S

donde S es la region de la superficie de la semiesfera cuya frontera es la curva


C.
Calculemos directamente la integral de lnea, parametrizando la curva C
mediante coordenadas cilndricas x = r cost, y = r sent, z = z.

544
Figura 6.20:

Al sustituir enla ecuaciones de las superficies que definen la curva C,


tenemos que z = 1 r2 , r = sent donde 0 t .
Luego, la ecuacion de la curva es


 p 
r (t) = sin t cos t, sin2 t, 1 sin2 t , 0 t

Usando identidades trigonometricas sin2 t = 1


2
(1 cos 2t), cos2 t = 1
2
(1 +
cos 2t) se tiene:

 

1 1
r (t) = sin 2t, (1 cos 2t), cos t , 0 t
2 2

0
r (t) = (cos 2t, sin 2t, sin t) , 0 t

Evaluemos la integral de linea:

Z Z  
2 1 1 2 1
xdx + yz dy + xzdz = sin 2t cos 2t (1 cos 2t) cos tsen2t sin 2t cos t sin t
C 0 2 2 2
Z  
1 1 2 1 2
= sin 2t cos 2t (1 cos 2t) sin 2t sin 2t dt
0 2 4 4
Z   
1 1 2 1 1 cos 4t
= sin 2t cos 2t (1 cos 2t) sin 2t dt
0 2 4 4 2
 
1 2 1 1 3 1 sin 4t
= sin 2t + cos 2t cos 2t t +
4 8 24 8 32 0

=
8
545
Por otra parte, debemos evaluar



ZZ ZZ
F n bdS = F N dA
S D
Como S es la region de la superficie de la semiesfera cuya frontera es la
curva C, una parametrizacion de S viene dada por


 
2
T (r, ) = r cos , rsen, 1 r , 0 r sen, 0
A continuacion calculemos el vector normal a la superficie:

i j k



r
N = Tr T=
cos sin
1r 2
r sin r cos 0
 2
r cos r2 sin

= , ,r
1 r2 1 r2
La orientacion del vector normal a la superficie es compatible con la
orientacion de su frontera C.
Determinemos el rotor del campo vectorial

i j k



F = = (2yz, z, 0),
x y z

x yz xz

Por lo que la funcion compuesta queda



   
2 2
F T (r, ) = 2r sin 1 r , 1 r , 0
Entonces, estamos en condiciones de calcular la integral de superficie,

sin


ZZ Z Z
2r3 sin cos r2 sin drd

F N dA =
D 0 0
Z  4 sin
r r3
= sin cos + sin d
0 2 3 0
Z  5
sen4

sin
= cos + d
0 2 3
 6 
1 4
Z
sin
= sin d
12 0 3 0
1 4
Z
= sin d
3 0

546
Puesto que el integrando queda

 2
4 1 cos 2 1
1 2 cos 2 + cos2 2

sin = =
2 4
 
1 1 + cos 4
= 1 2 cos 2 +
4 2
 
1 3 cos 4
= 2 cos 2 +
4 2 2
Podemos calcular


1 1 3
Z Z  
1 4 cos 4
sin d = 2 cos 2 + d
3 0 3 0 4 2 2
 
1 3 sin 4
= sin 2 +
3 2 2 0

=
8
Problema 3



I
Sea F (x, y, z) = (x, y, z), calcular la integral de lnea F dr, siendo C
C
la curva interseccion de las superficies S1 dada por ecuacion z = x2 + 2y 2 y
S2 por la ecuacion z = 4 x2 .

Figura 6.21: C la curva interseccion de las superficies S1 dada por ecuacion z =


x2 + 2y 2 y S2 por la ecuacion z = 4 x2

Solucion
El teorema de Stokes afirma que



I ZZ

F dr = F n
bdS
C S

547
donde la curva C es la frontera de la superficie S dada por

S = {(x, y, z) IR3 : x2 + y 2 2, z = 4 x2 }
Calculemos

i j k



F = = (0, 0, 0).
x y z
x y z
Por lo tanto


ZZ ZZ
F n
bdS = (0, 0, 0) n
bdS
S S
Lo que indica que estamos frente a un campo conservativo.
Problema Z Z4



Calcular F n bdS, siendo F = (xz + yz 2 + x, xyz 3 + y, x2 z 4 ) y
S
S = S1 S2 donde S1 es la superficie cilndrica x2 + y 2 = 1, 0 z 1 y S2
es la semiesfera x2 + y 2 + z 2 = 1, z 0.

Figura 6.22: Superficie S = S1 S2 donde S1 es la superficie cilndrica x2 +y 2 = 1,


0 z 1 y S2 es la semiesfera x2 + y 2 + z 2 = 1, z 0

Solucion.

El teorema de Stokes afirma que





ZZ I
F n bdS = F d
r
S C

donde la curva C es la frontera de la superficie S dada por x2 + y 2 = 1


Parametizando la circunferencia, queda:


r (t) = (cos t, sin t, 0) , t [0, 2]


r 0 (t) = ( sin t, cos t, 0) , t [0, 2] .

548


F (r (t)) = (cos t, sin t, 0)
Calculemos la integral
Z 2


I

F dr = (cos t, sent, 0) ( sin t, cos t, 0) dt
C 0
= 0
Problema 5


Determine la circulacion del campo F = (x2 y, 4z, x2 ) alrededor de la
curva Cpdada por la inteseccion del plano z = 4 y el cono
z = x2 + y 2 ,recorrida en sentido positivo.

p
Figura 6.23: Plano z = 4 y cono z = x2 + y 2

Solucion
El teorema de Stokes afirma que



I ZZ

F dr = F n
bdS
C S

donde la curva C es la frontera de la superficie S dada por

S = {(x, y, z) IR3 : x2 + y 2 z 2 , z 4}
Parametrizando el cono se tiene


T (r, ) = (r cos , r sin , r) ,
donde D = {(r, ) /0 r 2, 0 2}
A continuacion calculemos el vector normal a la superficie:

i j k




N = T r T = cos
sin 1
r sin r cos 0
= (r cos , r sin , r)

549
La orientacion del vector normal a la superficie es hacia el exterior del
cono
Ademas


i j k


F = = (4, 2x, 1).
2x y z
x y 4z x2



Luego la funcion compuesta F T (r, ) es




F T (r, ) = (4, 2r cos , 1)

Por lo tanto



ZZ ZZ
F nbdS = F N drd
S Z ZD
= (4, 2r cos , 1) (r cos , r sin , r) drd
D
Z 2 Z 2
= (4r cos + r sin + r) ddr
0 0
= 4

6.4. Ejercicios Propuestos


6.4.1. Area de una superficie
1. Problema
Calcule el area de la superficie de x2 + y 2 + z 2 = a2 , a > 0, z 0.
Respuesta
A (R) = 2a2

2. Problema
Calcule el area de la superficie del sector del plano ax + by + cz + d = 0
que queda dentro del cilindro x2 + y 2 = r2 (c 6= 0, r > 0) .
Respuesta

r2 a2 + b2 + c2
A (R) =
c

550
Figura 6.24: Superficie del sector del plano ax + by + cz + d = 0 que queda
dentro del cilindro x2 + y 2 = r2 (c 6= 0, r > 0)

Figura 6.25:

3. Problema
Calcule el area del helicoide

r (u, v) = (u cos v, u sin v, v) para 0 u
1, 0 v 2
Respuesta

A (R) = (ln( 2 + 1) + 2

4. Problema
Hallar el area de la superficie S descrita por la parametrizacion: r :D
R2 R3 ;
r (u; v) = (u cos v; u sin v; u2 ) ; donde D = {(u, v) IR2 /0 u 4, 0 v 2} .
Respuesta
1 h 3/2
i
I= (65) 1
12
551
Figura 6.26:

5. Problema
Obtener el area de la superficie S , frontera de
= {(x, y, z) IR3 : (z 1)2 x2 + y 2 y x2 + y 2 + z 2 1}

Figura 6.27:

Respuesta

Area de la superficie S = Sesf Scono = 2 + 2
6. Problema
Calcule el area de la porcion de superficie conica z 2 = x2 + y 2 situada
por encima del plano z = 0 y limitada por la esfera x2 + y 2 + z 2 = 2ax
Respuesta

2 2
I= a
4

552
Figura 6.28:

7. Problema
Calcule el area de la superficie S de una pelota de rugby formada por la

rotacion de la curva x = cos z, y = 0, z alrededor del eje z.
2 2
Respuesta
Ecuacion parametrica de la superficie r (u, v) = (cos u cos v, cos u sin v, u)
 
A = 2 2 + ln 1 + 2

6.4.2. Integrales de funciones escalares sobre superficie


1. Problema Z
Calcule la integral x2 zdS ; siendo S la superficie externa de
S
x2 + y 2 = a2 comprendida entre z = 2 y
z = 2.

Figura 6.1:

Respuesta

553
Z
x2 zdS = 0
S
2. Problema Z
Calcule la integral de superficie (x4 y 4 + y 2 z 2 x2 z 2 + 1)dS . Si
S
la superficie S es la hoja superior del cono z 2 = x2 + y 2 que esta en el
interior del cilindro (x 1)2 + y 2 = 1.

Figura 6.2: Superficie S, hoja superior del cono z 2 = x2 +y 2 que esta en el interior
del cilindro (x 1)2 + y 2 = 1

Respuesta

A = 2

554
3. Problema Z
Calcule la integral de superficie I = zdS, donde S es la superficie
S
parametrizada por
r (x, y) = (x, y, 2x + 3y + 2a), (x, y) D tal que
D = {(x, y) IR2 /x2 + y 2 a2 } .

Figura 6.3: Aca voy

Respuesta

I = 2 14a3
4. Problema Z p
Calcule la integral de superficie I = 1 x2 y 2 dS, donde S es una
S
pelota de rugby formada por la rotacion de la curva x = cos z, y =

0, z alrededor del eje z.
2 2
Respuesta
Superficie parametrizada

r (u, v) = (cos u cos v, cos u sin v, u) ,
n o
D = (u, v) / u , 0 v 2
2 2
4 
I= 2 21
3

6.4.3. Integral de Flujo


1. Problema


Calcule la integral del campo vectorial F (x, y, z) = (x, y, z) a traves de
la superficie lateral del paraboloide x2 + y 2 = 2az , con 0 z 2a.
Respuesta
I = 4a3
2. Problema


Calcule la integral del campo vectorial F (x, y, z) = (yz, x, z 2 ) a traves
de la superficie del cilindro parabolico y = x2 , con 0 x 1, 0 z
4,donde la primera componente de n es positiva.

555
(a) Campo vectorial (b) paraboloide

Figura 6.1:

(a) Campo vectorial (b) sector de cilindro


parabolico

Figura 6.2:

Respuesta
Se tiene que

n = (2x, 1, 0)
I=2

3. Problema



Calcule la integral del campo vectorial F (x, y, z) = (0, yz, z 2 ) a traves
de la superficie S cortada del cilindro y 2 + z 2 = 1, por los planos
x = 0; x = 1.
Respuesta
Se tiene que

n = (0, y, z)
I=2

556
(a) Campo vectorial (b) Cilindro cortado por
dos planos

Figura 6.3:

6.4.4. Teorema de la divergencia de Gauss


1. Problema



ZZ
2
Sea F (x, y, z) = (xz, yz, z ). Calcule F n
bds ; siendo S la cara
S
externa del paraboloide x2 + y 2 = 3z; entre
z = 0 y z = 1.

Figura 6.1: Paraboloide x2 + y 2 = 3z limitado por los planos z = 0 y z = 1

a) Directamente.
b) Aplicando el teorema de Gauss.
Respuesta.
I = 3
2. Problema



ZZ
Calcular el flujo F n
bds, del campo vectorial F = (x, y, z) a traves
S
de la superficie S del elipsoide ax2 + by 2 + cz 2 = 1

557
Respuesta.


ZZ
2
bds =
F n
S abc
3. Problema



ZZ
Calcular el flujo bds,del campo vectorial F = (2xy 2 , z 3 , x2 y)
F n
S
a traves la superficie S formada por el hemisferio superior de la esfera
x2 + y 2 + z 2 = a2 y el cilindro x2 + y 2 = a2

(a) Campo vectorial (b) Superficie formada por el hemisfe-


rio superior de la esfera y el cilindro

Figura 6.2:

Respuesta.


ZZ
4
bds = a5
F n
S 15

558
4. Problema



ZZ

Calcular el flujo nds, del campo vectorial F = 0, tan z + esin xz , y 2
F b
S
a traves de la superficie S del semi-elipsoide 2x2 + 3y 2 + z 2 = 6, z 0,
con la normal apuntando
hacia el exterior.

(a) Campo vectorial (b) Semi-elipsoide

Figura 6.3:

Respuesta.
r


ZZ
3
F n
bds =
S 2

559
5. Problema



ZZ
(x, y, z)
Calcular el flujo F b
nds, del campo vectorial F =
S (x2 + y 2 + z 2 )3/2
a traves de la superficie S dada por el paraboloide z = 2x2 y 2 ,tales
z 0, con la normal apuntando hacia el exterior.
Pendiente
Respuesta.
ZZ

 
F n bds = 2 2
S
6. Problema

Sea S la superficie del paraboloide z = x2 + (y 1)2 , interior al cilindro





x2 + (y 2)2 = 3. SeaZ Z F el campo vectorial F (x, y, z) = (y, x, xz)


. Calcular la integral F n bds,utilizando el teorema de Gauss,
S
donde n
b es la normal exterior al paraboloide.

(a) Campo vectorial (b) Paraboloide interior al


cilindro

Figura 6.4:

Respuesta
Colocando una tapa T a la superficie S, de modo que S = S T sea
S
una superficie cerrada, podemos aplicar el teorema de Gauss



ZZ ZZ
F n bds = F nbds = 2
S T
7. Problema
Sea S la porcion del paraboloide z + 1 = x2 + y 2 , situada debajo del



ZZ
plano z = 1, y sea F (x, y, z) = (0, x 2yz, x2 ). Hallar F nbds,
S

560
donde nb es la normal exterior al paraboloide, utilizando el teorema de
Gauss.

(a) Campo vectorial (b) Paraboloide situada de-


bajo del plano

Figura 6.5:

Respuesta
Colocando una tapa T a la superficie S, de modo que S = S T sea
S
una superficie cerrada, podemos aplicar el teorema de Gauss



ZZ ZZ
F n bds = F nbds = 2
S T

6.4.5. Teorema de Stokes


1. Problema
Sea S el sector del paraboloide z = x2 + y 2 , situado en el primer
octante , limitado por el plano z = 1 y los planos principales, y sea



I
F = (y z, z x, x y) . Calcular la integral de lnea F d
r.
C
Respuesta


I
8
F dr.=
C 3 2
2. Problema


Para el campo vectorial F = (x2 + yez , y 2 + zex , z 2 + xey ) definida so-


bre S : x2 + y 2 + z 2 = a2 con z 0.Calcular el flujo del rotacional de F
a traves de S.
Respuesta



I ZZ

F dr = F n bds = a2
C S

561
(a) Campo vectorial (b) Sector de
paraboloide, limita-
do por planos

Figura 6.1:

(a) Campo vectorial (b) hemisferio superior de la


esfera

Figura 6.2:

3. Problema


Dado el campo vectorial F (x, y, z) = (y + sin x, z 2 + cos y, x3 ) y la curva


r (t) = (sin t, cos t, sin 2t); t [0; 2]; halle


I
F dr utilizando el teorema de Stokes.
C
Respuesta
I = 2

562
(a) Campo vectorial (b) paraboloide hiperbolico

Figura 6.3:

4. Problema


Calcule el flujo del campo F (x, y, z) = (x, y, z) a traves de la superficie
del solido limitado por x2 + y 2 = 9, z = 0, y z = 0.
a) Directamente.
b) Aplicando el Teorema de Stokes
Respuesta
I = 81

5. Problema I
Calcule la integral y 2 dx + xydy + xzdz, siendo C la curva interseccion
C
del cilindro x2 + y 2 = 2y y el plano y = z.
a) Directamente.
b) Aplicando el teorema de Stokes.

Figura 6.4:

Respuesta

563
I=0
6. Problema I
Calcule la integral I = (y 1)dx + z 2 dy + ydz, aplicando el teorema
C
z2
de Stokes, siendo C la curva interseccion de las superficies x2 + y 2 = ,
2
z =y+1

Figura 6.5:

Respuesta

I = 2

6.5. Aplicaciones
En esta seccion presentamos algunas aplicaciones simples que ilustran la
utilizacion de las, integrales de lineas, integral de flujo y de los teoremas de
Gauss , Stokes y Green .

6.5.1. Aplicaciones Integral de Flujo


Problema 1
Calcular el flujo del campo electrico producido por una carga Q en el
origen, a traves del manto de la esfera S de radio R orientado segun la normal
exterior.

Solucion
El campo electrico producido por la carga Q viene dado por

564

Q rb
E =
4o r2
donde o una constante universal . De esta forma se obtiene el siguiente flujo
electrico



ZZ ZZ
Q rb
E dS = rbdS
4o R2
S S
Parametrizando la superficie en coordenadas esfericas se tiene

(, ) = (R sin cos , R sin sin , R cos ) , 0 , 0 2




= = R2 sin
ZZ Z 2 Z
Q rb Q 1 2
rbdS = R sin dd
4o R2 0 0 4o R2
S
Q
=
o
Por tanto


ZZ
Q
E dS =
o
S
Corresponde a la formulacion integral de la Ley de Gauss, que se aplica
tambien a distribuciones de carga mas generales.

Problema 2
Calcular el flujo del campo electrico generado por una carga Q en el
origen, sobre el plano infinito x = 1.
Solucion
En este caso la normal es constante Nb = bi . El campo electrico producido
por una carga Q en coordenadas cartesianas, esta dado por


Q rb Q (x, y, z)
E = =
4o r 2 4o [x2 + y 2 + z 2 ]3/2
El flujo del campo viene dado por


b
ZZ ZZ
Q (1, y, z)
E idS = (1, 0, 0) dS
4o [x2 + y 2 + z 2 ]3/2
S S
Z Z
Q dxdy
=
4o [1 + y 2 + z 2 ]3/2

565
Utilizando el cambio de variables a coordenadas polares, tenemos
y = rcos, z = rsen = y 2 + z 2 = r2
(y, z)
y el jacobiano de transformacion es =r
(r, )
Reemplazando el integrando, obtenemos

Z Z 2 Z
Q rddr Q rdr
=
4o 0 0 [1 + r2 ]3/2 2o 0 [1 + r2 ]3/2
Qh i
2 1/2

= 1+r
o 0
Q
=
o

Problema 3


Si E (x, y, z) = (2x, 2y, 2z) es un campo electrico, encontrar el flujo de

E que sale a traves de la superficie cerrada S que consta de la semiesfera


x2 + y 2 + z 2 = 1, z 0, y su base.
Solucion
Como los puntos de la circunferencia x2 + y 2 = 1, z = 0, son singulares,
debemos descomponer la superficie en dos partes, de modo que llamaremos
S1 a la semiesfera y S2 al crculo que forma la tapa inferior.
Una parametrizacion de S1 viene dada por la funcion


r 1 (u, v) = ( sin u cos v, sin u sin v, cos u)
n o
donde D1 = (u, v) IR3 /0 u , 0 v 2
2
El vector normal exterior a la superficie es



T u T v = (sin2 u cos v, sin2 u sin v, cos u sin u)
De este modo, el flujo a traves de S1 es


ZZ ZZ
E nbdS = 2(cos2 v sin3 u + sin2 v sin3 u + sin u cos2 u)dudv
S1 D
Z 2 Z /2
= 2 sin ududv
0 0
= 4
Analogamente, si parametrizamos la superficie S2 por

r (u, v) = (ucosv, u sin v, 0)
2

566
donde D2 = {(u, v) IR2 /0 u 1, 0 v 2}

i j k



r u r v = cos v
sin v 0 = (0, 0, u)
u sin v u cos v 0
El flujo a traves de la superficie S2 es



ZZ ZZ
E n
bdS = 2(u cos v, u sin v, 0) (0, 0, u) dudv
S2 D
ZZ
= 0 dudv = 0
D

Por lo tanto, la suma de ambos flujos da como resultado




ZZ
E nbdS = 4
S

Problema 4


Si F (x, y, z) = (0, y, 0) representa la velocidad de un fluido. Calcular
la razon del flujo de dicho fluido que atraviesa la superficie
S = {(x, y, z) : x2 + z 2 = y, 0 y 1}.

Figura 6.6: Superficie S = {(x, y, z) : x2 + z 2 = y, 0 y 1}

Solucion

567
En primer lugar, consideremos la siguiente parametrizacion de la superficie:
r(u, v) = (u cos v, u2 , u sin v), 0 u 1, 0 v 2.
El vector normal a la superficie viene dado por:


i j k



Tu Tv = cos v 2u sin v =

u sin v 0 u cos v
= (2u2 cos v, u, 2u2 sin v)

Entonces el flujo esta dado por,

2 1


Z Z Z Z
F n
bdS = (0, u, 0) (2u2 cos v, u, 2u2 sin v)dudv
S 0 0
Z 2 Z 1 Z 2  3 1
2 u
= (u )dudv = dv
0 0 0 3 0
2
=
3
El signo negativo del resultado es debido a que el vector normal tiene sentido


opuesto al del movimiento del fluido (la componente b j de F es positiva pero
la de n
b es negativa). As pues, la razon de flujo que entra a la superficie es
2
de .
3

6.5.2. Aplicacion del teorema de Gauss


Problema 1
Suponga que la temperatura de un punto de una esfera S es proporcional
al cuadrado de la distancia de dicho punto al centro de la esfera. Se sabe que


el campo estacionario de temperaturas J asociado a este potencial queda


definido por la expresion J = T , donde k > 0 representa la conductivi-
dad termica del medio y T la temperatura.
Calcular directamente la razon total del flujo de calor que atraviesa la
esfera, suponiendo que tiene centro el origen y radio a.
Solucion
Debemos calcular





Z Z Z Z
J n
bdS =
J ( r (u, v)) T u T v dudv
S D

Consideremos la parametrizacion de S dada por:

568


r (u, v) = (a sin u cos v, a sin u sin v, a cos u)
donde D = {(u, v) IR3 /0 u , 0 v 2}
Determinemos el flujo de calor. Como T (x, y, z) = C(x2 + y 2 + z 2 ), entonces

J (x, y, z) = kT (x, y, z) = kC(2x, 2y, 2z)


El vector normal exterior a la superficie es



T u T v = (a2 sin2 u cos v, a2 sin2 u sin v, a cos u sin u)
y la razon de flujo a traves de la esfera viene dada por la integral
2




Z Z Z Z
J n
bdS = kCT T u T v dudv
S
Z0 2 Z0
= 2kCa3 sin ududv
0 0
3
= 8kCa

Problema 2
Suponga que la temperatura en una region de IR3 esta dada por
T (x, y, z) = C(x2 + y 2 + z 2 ).


Se sabe que el campo estacionario de temperaturas J asociado a este


potencial queda definido por la expresion J = T .Suponga que la region
contiene a la esfera S centrada en el origen y de radio a.
Calcular el flujo de calor que sale a traves del casquete esferico S,usando
el teorema de Gauss , dada la conductividad termica > 0 del material.
Solucion
Sea un abierto de IR3 de frontera regular S orientada segun la normal
exterior N b
El flujo de calor producido por el campo estacionario de temperaturas es


b

ZZ ZZZ
J N dS = J dV
S V
ZZZ
= (T ) dV
ZV Z Z
= 2 T dV
V

569
Calculemos el Laplaciano de temperaturas

2 2 x2 2 y 2 2 z 2
T = + + = C(2 + 2 + 2) = 6C
x2 y 2 z 2
Reemplazando el integrando, queda


ZZ ZZZ
J d S = 6C dV
S
 
4 3
= 6C R
3
= 8CR3

Problema 3
El hecho de que la temperatura decrece hacia el origen hace que el gra-
diente apunte en esa direccion de modo que el flujo neto de calor que entra
a la esfera es negativo.
El filtro de una n maquina de lavar ropa tiene la forma
o conica descrita
p
3 2 2
por el conjunto = (x, y, z) IR / x + y z 3 , por el circula una
corriente de agua, cuyo campo de velocidades esta dado por la expresion

F (x, y, z) = (2yz cos (y 2 ) , 2x cos (x2 ) , 1) .


a) Muestre que la cantidad de agua en el interior del filtro se mantiene
constante, suponiendo que la densidad el agua es = 1 gr/cm3 .
b) Usando el teorema de Stokes, calcule el flujo total de agua traves de
las paredes del filtro

Solucion
Por el teorema de la divergencia de Gauss, se tiene que el flujo total de
agua a traves de las paredes del filtro es



ZZ ZZZ
F nbds = F dV
S

Calculemos la divergencia del campo de velocidades

 

2
 2

F = 2yz cos y + 2x cos x + (1) =
x x x


F = 0

570
n p o
Figura 6.7: = (x, y, z) IR3 / x2 + y 2 z 3



Como F = 0,el flujo total de agua a traves de las paredes del filtro es
nulo. Por lo tanto la cantidad de agua que entra al filtro es igual a la que sale,
entonces la cantidad de agua en el interior del filtro se mantiene constante.


b) Sea S la pared del manto de .Como F es un campo de divergencia


nula en IR3 , entonces F es un campo rotacional, es decir, existe un campo




vectorial R tal que F = R .


En consecuencia, se busca R que satisfaga la condicion anterior.


Sea R = (R1 , R2 , R2 ) tal que

R3 R2
= 2yz cos y 2


y z
R1 R3
= 2xz cos x2


z x
R2 R1
= 1
x y

Este problema tiene muchas soluciones.Formulemos un modelo en que


R1 = 0, entonces

R3 R2
= 2yz cos y 2


y z
R3
= 2xz cos x2


x
R2
= 1
x
Integrando parcialmente las dos ultimas ecuaciones se obtiene

571
R3 R2
= 2yz cos y 2


y z
R3 = zsen x2 + f (x, y)


R2 = x + g (y, z)

Se tiene mas de una solucion, consideremos g (x, y) = 0, luego queda


R3
= 2yz cos y 2

y
R3 = zsen x2 + f (x, y)


R2 = x

Derivando la segunda ecuacion parcialmente con respecto a y y se iguala con


la primera

f (x, y) = 2yz cos y 2

y
Integrando parcialmente con respecto a y esta ultima expresion produce

f (x, y) = zsen y 2 + h (z)




Para simplificar consideremos h (z) = 0,por consiguiente

R3 = zsen x2 + zsen y 2
 



Finalmente, el campo R queda:


R = 0, x, zsen x2 + zsen y 2
 

El teorema de Stokes afirma que





ZZ I
R n
bdS = R d
r
S C


El flujo que produce el campo R a traves del manto es igual a la integral
de lnea sobre la frontera C que limita la supercicie S.
C = {(x, y, z) IR3 / x2 + y 2 = 9, z = 3}
Parametrizando la curva se tiene

r (t) = (3 cos t, 3set, 3) , t [0, 2]


Z 2



I
R dr = R (
r (t))

r 0 (t) dt
C 0

572
Z 2
0, 3 cos t, 27sen3 t 27sent cos2 t (3sent, 3 cos t, 0) dt

=
0
Z 2 Z 2    2
2 1 + cos 2t 9 sen (2t)
= 9 cos tdt = 9 dt = +t
0 0 2 2 2 0
= 9

6.5.3. Aplicacion teorema de Stokes


Problema 1


Calcular el trabajo producido por la fuerza F (x, y, z) = (y 3 , x3 , z 3 )
,sobre la trayectoria recorrida en el sentido positivo, dada por la interseccion
de las superficies x + y + z = a, x2 + y 2 = a2 .

Figura 6.8:

Solucion
Como el campo es continuo y la trayectoria es cerrada, podemos calcular
el trabajo aplicando el teorema de Stokes.




I ZZ 
W = F d
r = F n
b dS
ZCZ  S


n
= F
k

n k dA
D knk

Al parametrizar en coordenadas cartesianas el plano que contiene a la


curva obtenemos


(x, y) = (x, y, a x y), donde (x, y) D tal que x2 + y 2 a2 .

573
Calculemos el integrando del lado derecho de la formula anterior
i j k



F = = (0, 0, 3x2 + 3y 2 )
x3 y3 z3

y x z
El vector normal al plano esta dado por


n = = (1, 1, 1) , donde = x + y + z a = 0
En consecuencia, al sustituir terminos en al ultima integral queda



ZZ  ZZ
F 0, 0, 3x2 + 3y 2 (1, 1, 1) dxdy

n dxdy =
D ZDZ
= 3 (x2 + y 2 )dxdy
D

Si hacemos cambio a coordenadas polares obtenemos

2 a 2 a
4
Z Z Z 
2

3 () dd = 3 d
0 0 0 4 0
Z 2
4
a
= 3 d
4 0
6a4
=
4
Corresponde al trabajo total realizado por el campo de fuerzas.

Problema 2
Calcular la circulacion del campo de velocidades de un fludo dado por

F (x, y, z) = (arctan(x2 ), 3x, e3z tan (z)) a lo largo de la interseccion de la


esfera x2 + y 2 + z 2 = 4, con el cilindro x2 + y 2 = 1, con z > 0.
Solucion.
La circulacion de un campo vectorial es su integral de lnea a lo largo de
una curva cerrada.
Vemos que el campo vectorial es bastante complejo, por lo que calcular
directamente la integral de linea puede resultar engorroso.
Se espera que al ocupar el teorema de Stokes se simplifique el calculo.
Entonces




I ZZ 

F dr = F n b dS
C S

574
Figura 6.9:

Calculemos el integrando del lado derecho de la formula anterior




i j k



F = = (0, 0, 3)
x y z
arctan(x2 ) 3x e3z tan(z)

Luego, queda



I ZZ
F d
r = (0, 0, 3)

n dA
C D

Para determinar el vector normal parametricemos la superficie en coor-


denadas cilndricas
x = cos


 p 
y = p sin 2
= r (, ) = cos , sin , 4 donde
z= 4 2

(, ) D tal que 0 1, 0 2
Luego


i j k !
2 cos 2 sin



n = (

r

r ) = cos

sin p

= ,p ,
4 4
p


4 4 4 4
sin cos 0
Reemplazando terminos en el integrando se tiene

575
!

2 cos 2 sen
I ZZ
F d
r = (0, 0, 3) p ,p , dd
C D 4 4 4 4
Z 2 Z 1 Z 2  2 1

= 3dd = 3 d
0 0 0 2 0
3 2
Z
= d
2 0
= 3

6.5.4. Aplicacion teorema de Green


Problema 1
Usando el teorema de Green determinar el momento de inercia de una
arandela homogenea de densidad superficial ,que tiene masa M, radio inter-
no a y radio externo b; respecto de uno de sus diametros.

Figura 6.10: region R = (x, y) IR2 / a2 x2 + y 2 b2




Solucion.
Determinaremos el momento de inercia con respecto al diametro que es
colineal con el eje x.
El momento de inercia con respecto a este eje esta dado por
ZZ
Ix = y 2 dA
R

donde es la densidad superficial.

576
La region R = {(x, y) IR2 / a2 x2 + y 2 b2 } , no es simplemente
conexa, sin embargo, se puede extender el teorema de Green, sumando inte-
grales de lneas sobre fronteras separadas, de un numero finito de subregiones
(simplemente conexas), para construir una integral sobre una sola frontera.

ZZ   Z Z
Q P
dxdy = P dx + Qdy P dx + Qdy
R x y C1 C2
Z Z
= P dx + Qdy + P dx + Qdy
C1 C2

Buscamos P y Q, tales que


Q P
= y2
x y
P y3
Consideremos en este caso Q = 0 = = y 2 = P =
y 3
Asi, el momento de inercia queda:
y3 y3
ZZ Z Z 
2
Ix = y dA = dx dx
R C1 3 C2 3
3 3
Z Z 
y y
= dx + dx
C1 3 C2 3

Parametrizamos
 ambas curvas:

x = b cos t dx = b sin tdt
C1 : = con 0 t 2
y = b sin t dy = b cos tdt
 
x = a cos t dx = a sin tdt
C2 : = con 0 t 2
y = a sin t dy = a cos tdt
Reemplazando terminos en la ultima integral, queda:

2 Z 2 3 3
b3 sin3 t
Z 
a sin t
Ix = (b sin t)dt + (a sin t)dt
0 3 0 3
(b4 a4 ) 2 4
Z
= sin tdt
3 0
(b4 a4 ) 2 2
Z
sin t 1 cos2 t dt

=
3 0

Aplicando las identidades trigonometricas:


(1 + cos 2t) (1 cos 2t)
cos2 t = , sen2 t = , obtenemos
2 2
577
(b4 a4 ) 2 1
Z
Ix = [(1 cos 2t)(1 cos 2t)] dt
3 0 4
2
(b4 a4 ) 3t sen2t sen4t

= +
3 8 4 32 0
4 4
(b2 + a2 )
 
(b a ) 6
= = (b2 a2 )
3 8 4
2 2
M (b + a )
=
4

Problema 2
Un fluido de densidad constante gira alrededor del eje z con velocidad

v = w(ybi+xb j) donde w es una constante positiva llamada rapidez angular,


muestre que la circulacion del campo de velocidades es
I
v d

r = 2R2 w
c

Solucion
La circulacion de un campo vectorial es su integral de lnea a lo largo de
una curva cerrada. Por el teorema de Stokes, la circulacion de
v alrededor
de la circunferencia C de radio R que acota a un disco S en el plano xy , esta
dado por
Calculemos
i j k



v = = (0, 0, (w (w)) = (0, 0, 2w)
x y z

wy wx 0
Luego, ( v )b
kdS = 2wb kb
kdS = 2wdxdy.Entonces
I ZZ


v d

r = (

v )b
k dS
C Z ZS
= 2wdxdy
S
= 2w R2


Problema 3

x2
Sea el campo de fuerzas F (x, y) = (2xy 2 + y, 2x2 y + + x), demostrar
2
que en cualquier camino cerrado simetrico con respecto al eje y, la circulacion

578
es cero.
Solucion
x2
Tenemos que F (x, y) = (2xy 2 + y, 2x2 y + + x) es un campo vectorial
2
continuo con derivadas parciales
Q

= 4xy + x + 1
x , continuas definidas en la region de integracion.
P
= 4xy + 1
y
Luego, aplicando el teorema de Green se tiene

ZZ  


I I
Q P
F d
r = (P dx + Qdy) = dxdy
C R x y
ZZ
= xdxdy
R

donde R = {(x, y) IR2 / a x a, 1 (x) y 2 (x)}


Particionemos la Region R en dos subregiones simetricas con respecto al
eje y, tal que:
R = R1 R2 y R1 R2 = .Entonces
ZZ ZZ ZZ
xdxdy = xdxdy + xdxdy
R R1 R2

Figura 6.11: Region R = (x, y) IR2 / a x a, 1 (x) y 2 (x) sub-




dividida en dos regiones R1 y R2

Se tiene que x es una funcion impar definida en un intervalo simetrico,


entonces
Z 0 Z a
xdx + xdx = 0
a 0

579
Por lo tanto ZZ ZZ
xdxdy + xdxdy = 0
R1 R2
En consecuencia


I
F d
r =0
C

6.5.5. Aplicaciones al electromagnetismo


Problema 1



Sean E (t, x, y, z)y B (t, x, y, z) los campos electrico y magnetico respec-
tivamente que dependen de la posicion y del tiempo, en el espacio. Sea S una
superficie con frontera C. Definimos


I
Voltaje alrededor de C = E d r
C


ZZ
Flujo magnetico a traves de S = B n
bdS
S
La ley de Faraday afirma que el voltaje alrededor de C es igual a la razon
de cambio negativo del flujo magnetico a traves de S, es decir



I ZZ
1
E dr = B nbdS (1,1)
C c t S
Utilice el teorema de Stokes para deducir la Ley de Faraday a partir de
la ecuacion diferencial de Maxwell



1B
E =
c t
Solucion
Supongamos que se cumple



1B
E =
c t
Sea C una curva cerrada, estacionaria en las coordenadas x, y, z, y S una


superficie limitada por C . Si B (t, x, y, z) es el campo magnetico medido en
x, y, z en el instante t, entonces tomando la integral de superficie sobre S se
tiene.


ZZ ZZ
1
( E ) n
bdS = B nbdS
S c S t
Transformando el lado izquierdo por medio del teorema de Stokes:


Z ZZ
1
E dr = B n bdS
C c S t

580
Aplicando linealidad al lado derecho obtenemos



Z ZZ
1
E dr = B n
bdS
C c t S
que es la ley de Faraday.

Problema 2
La ley de Ampere afirma que si la densidad de corriente electrica esta de-



scrita por un campo vectorial J ,y el campo magnetico es B , entonces la


circulacion B alrededor de la frontera C que limita una susperficie S es igual


a la integral de J sobre S es decir



Z ZZ
4
B dr = J nbdS
C c S
Pruebe que la ley de Ampere se deduce a partir de la ecuacion de Maxwell
del estado estacionario

4

B = J
c
Solucion
Supongamos que se cumple

4

B = J
c
Tomando la integral de superficie sobre S se tiene.



ZZ ZZ
4
( B) n
bdS = J n
bdS
S c S
Aplicando el teorema de Stokes al lado izquierdo de la ecuacion



Z ZZ
4
B dr = J nbdS
C c S
que es la ley de Ampere.
Esta Ley tambien se puede expresar como


Z
4
B d r = I
C c
donde


ZZ
I= J nbdS
S
es la intensidad de la corriente electrica en una superficie S limitada por una
curva cerrada C.

581
Problema 3


Hallar el campo magnetico B producido por un alambre recto infinito, a
una distancia r del alambre que lleva una corriente estacionaria I.
Solucion
Considere un alambre recto extendido sobre el eje z desde - hasta
+.Como hay simetra cilindrica, el alambre coincide con el eje axial del
cilindro,se escoge una trayectoria circular con con un punto del eje z como
centro, con radio r


Por la simetra, el vector B no es solamente constante azimutal sino




tambien tienen la disma direccion que d r y su magnitud B es constante
alrededor de la trayectoria C. Por consiguiente



Z
4
B dr = I
C c


Z
4
B kdr k cos 0 = I
C c


Z 4
B kd rk = I
C c

4

B (2r) = I
c
Finalmente queda

2
B = Ibe
cr

Problema 4




Sean E (t, x, y, z)y B (t, x, y, z) los campos electrico y magnetico respec-
tivamente que dependen de la posicion y del tiempo, en el espacio. que sat-
isfacen la ecuaciones de Maxwell




1B
E =
c t



1E 4

B = + J
c t c


E = 4


B = 0

582
que estan escritas para los campos en el vaco, en presencia de una densidad
de carga electrica y de corriente,es decir,cargas en movimiento de densidad

J
a) Deducir la forma integral de la ecuacion de Maxwell





Z ZZ ZZ
1 4
B dr = E nbdS + J nbdS
C c t S c S

donde C representa una espira de alambre por el que fluye corriente en el


sentido contrario al de los punteros del reloj con respecto al vector unitario
normal a la superficie n
b. La integral de superficie del primer termino del lado
derecho de la ecuacion es llamada flujo electrico y el segundo corresponde a
la intensidad de corriente I. Si S cualquier superficie orientada con frontera
C.


b) Suponga que B (t, x, y, z) es un campo solenoidal, es decir,


( B ) = 0, que satisface ecuacion (2) de Maxwell. Demostrar que la
densidad de corriente satisface la ecuacion de continuidad.
 
4
J + =0
c t
Solucion
b) Consideremos la ecuacion de Maxwell.



1E 4

B = + J
c t c
Tomando la integral de superficie sobre S se tiene.





ZZ ZZ ZZ
1 E 4
( B) n
bdS = n
bdS + J n
bdS
S c S t c S

Aplicando el teorema de Stokes al lado izquierdo de la ecuacion







Z ZZ ZZ
1 E 4
B d
r = n
bdS + J n
bdS
C c S t c S

Aplicando linealidad al primer termino del lado derecho, obtenemos





Z ZZ ZZ
1 4
B dr = E nbdS + J n
bdS
C c t S c S

que es la forma integral de la ecuacion de Maxwell.


c) Consideremos la ecuacion

583



1E 4

B = + J
c t c
Tomemos la divergencia en los dos lados .


1
4

( B ) = E + J
c t c


Aplicando la condicion de solenoidal y reemplazando E por (3)

1 4

0= (4) + J
c t c
Por lo tanto, se tiene
 
4

J + =0
c t

Problema 5


Sean E y B dos vectores con derivadas parciales continuas de segundo


orden con respecto a la posicion y el tiempo . Supongase ademas que E y

B satisfacen las ecuaciones de Maxwell en el vaco, en ausencia de densidad




de cargas y densidad de corriente J :



E = 0 (6.5.1)


B = 0 (6.5.2)



1B
E = (6.5.3)
c t



1E
B = (6.5.4)
c t
donde c es una constante que corresponde a la velocidad de la luz en el vaco


a) Demostrar que E y B satisfacen la ecuacion de onda


2
1 2
= 2 2
c t


b) Demostrar que E y B satisfacen la ecuacion :
 
1 2 2

( E + B ) + c ( E B ) = 0
t 2

584
c) Integre la expresion anterior sobre un volumen encerrado dentro de
una superficie , y demostrar que
ZZZ  
1 2 2

ZZ

( E + B ) dV + c (E B ) n
bdS = 0
t 2 S

Solucion
a) Si tomamos el rotacional de la ecuacion 6.5.3 tenemos:

1

( E ) = ( B )
c t


Eliminamos a B por medio de la ecuacion 6.5.4, entonces



1 1E
( E ) = ( )
c t c t



1 2 E
( E ) = 2 2
c t
Aplicando al rotacional la identidad:




( E ) = ( E ) 2 E
se tiene:


2
1 2 E
( E ) E = 2 2
c t
Por tanto, aplicando la ecuacion 6.5.1 al primer termino se tiene el campo


electrico E satisface la ecuacion de onda


2
1 2 E
E = 2 2
c t
Por otra parte, si tomamos el rotacional de la ecuacion 6.5.4 tenemos:

1

( B ) = ( E )
c t


Eliminamos a E por medio de la ecuacion 6.5.3, entonces



1 1B
( B ) = ( )
c t c t



1 2 B
( B ) = 2 2
c t
Aplicando al rotacional la identidad:

585




( B ) = ( B ) 2 B
se tiene:


2 1 2 B
( B ) B = 2 2
c t
En consecuencia, aplicando la ecuacion 6.5.2, al primer termino se tiene


que el campo electrico B tambien satisface la ecuacion de onda


2 1 2 B
B = 2 2
c t


b) Si hacemos en la ecuacion 6.5.3 el producto interno por H queda


1
B ( E ) = B B
c t


Analogamente , si hacemos en la ecuacion 6.5.4 el producto interno por E
se tiene




1 E
E ( B ) = E
c t
Restamos la segunda ecuacion de la primera, entonces obtenemos







1 1 E
B ( E ) E ( B ) = B B E
c " t c t #






1
E
B ( E ) E ( B ) = B B +E
c t t

Ahora, reemplazando por las identidades:








B ( E ) E ( B ) = ( E B )

 


E 1 2
2
B B +E = (B + E )
t t t 2
La ecuacion anterior se convierte en
 

1 1 2
2
(E B ) = (B + E )
c t 2
por lo tanto, multiplicando por la constante c, obtenemos:
 
1 2 2

( E + B ) + c ( E B ) = 0
t 2

586
c) Si integramos la ultima expresion sobre el volumen dentro de una
superficie cerrada S, obtenemos
ZZZ    
1 2 2

( E + B ) + c ( E B ) dV = 0
t 2
Aplicando linealidad queda:
ZZZ  
1 2 2

ZZZ

( E + B ) dV + c ( E B )dV = 0
t 2

Usando el teorema de la divergencia en el segundo termino de la ecuacion


queda:
ZZZ  
1 2 2

ZZ

( E + B ) dV + c (E B ) n
bdS = 0
t 2 S

6.6. Auto evaluaciones


Autoevaluacion No 1
Tiempo 2 horas
Problema 1


Dado el campo vectorial F (x, y, z) = (0, 0, z) .
RR

a) calcular F n
bdS, siendo S la superficie descrita por la ecuacion
S
parametrica:

r (u, v) = (sin u cos v, sin u sin v, cos u sin u) , donde

D = {(u, v) /0 u , 0 v 2}

b) determine el volumen de la region acotada por la superficie S.

Problema 2

Sea

r = (x, y, z) en una superficie cerrada regular S. Obtener
RR

r n
bdS en terminos del volumen V que encierra S.
S
Problema RR 3.


Calcular F n
bdS, siendo el campo vectorial
S



F (x, y, z) = x2 + y 3 + z 4 , x2 + y 3 + z 4 , x3 + y 4 + z 4


y S la superficie formada por las seis caras de paraleleppedo

= (x, y, z) IR3 /3 x 5, 1 y 2, 0 z 1


587
Pauta de Correccion

Problema 1


a) Calculemos directamente el flujo del campo F sobre la superficie S.




ZZ ZZ
F n
bdS = F (r (u, v))

n dudv
S D


Se tiene que F (
r (u, v)) = (0, 0, cos u sin u) y el vector normal exterior
a la
superficie es

i j k




n = r u r v = cos u cos v cos u sin v sin u 2

sin u sin v sin u cos v 0
= sin3 u cos v, sin3 u sin v, cos u sin u




F (
r (u, v))

n = (0, 0, cos u sin u) sin3 u cos v, sin3 u sin v, cos u sin u


Reemplazando en el integrando obtenemos:


2
1 2 2
Z Z Z Z
2 2
cos u sin ududv = sin 2ududv
0 0 4 0 0
1 2 1 cos 4u
Z Z  
= dudv
4 0 0 2

1 2 2u sin 4u
Z 
= dv
4 0 4 0
1 2 2
Z
= dv =
4 0 2 4


el flujo del campo F sobre la superficie S.
b) Sea la region cuyo volumen esta acotado por la superficie S. De
acuerdo
con el teorema de la divergencia de Gauss, se tiene




ZZ ZZZ
F nbdS = F dV
S



Como al calcular F = 0 + 0 + 1 = 1,obtenemos

588


ZZ ZZZ
F n
bdS = dV = V ()
S

Por tanto, el volumen pedido es

2
V () =
4

Problema 2.

De acuerdo con el teorema de la divergencia de Gauss se tiene

ZZ ZZZ


r n
bdS =

r dV
S
Z
ZZ
= 3dV

= 3V

donde V es el volumen encerrado por S

Problema 3
Sea la region cuyo volumen esta acotado por la superficie S. De acuerdo
con
el teorema de la divergencia de Gauss, se tiene




ZZ ZZZ
F n
bdS = F dV
S



Como al calcular F = 2x + 3y 2 + 4z 3 ,obtenemos

589


ZZZ ZZZ
2x + 3y 2 + 4z 3 dxdydz

F dV =

Z 5 Z 2 Z 1
2x + 3y 2 + 4z 3 dzdydx

=
Z3 5 Z1 2 0
1
2xz + 3y 2 z + z 4 0 dydx

=
Z3 5 Z1 2
2x + 3y 2 + 1 dydx

=
Z3 5 1
2
2xy + y 3 + y

= 1
dx
3
Z 5
(2x + 8)dx = x2 + 8x = 32
 
=
3

Autoevaluacion No 2
Tiempo 2 horas
Problema 1. RR


Calcular la integral bdS, en donde F (x, y, z) = (xy 2 , x2 y, y) , y S
F n
S
es la superficie del cilindro x2 + y 2 = 1 acotado por los planos:
z = 1 y z = 1.

Problema 2.


Sea F (x, y, z) = 0, y, 0 el campo de velocidades de un fluido. Calcular
RR

el flujo ndS, que atraviesa la superficie S = {(x, y, z) IR3 /x2 + y 2 = y, 0 y 1} .
F b
S
Problema 3.


Verificar el teorema de Stokes para F (x, y, z) = (3y, xz, yz 2 ) , siendo S
la superficie del paraboloide 2z = x2 + y 2 , limitada por z = 2.Considere que
n
b es la normal exterior a la superficie S.

Pauta de Autocorrecion

Problema 1
De acuerdo con el teorema de la divergencia de Gauss,se tiene



ZZ ZZZ
F n
bdS = F dV
S

590
siendo la region limitada por la superficie S, es decir el cilindro macizo
dado por la ecuacion = {(x, y, z) IR3 /x2 + y 2 1, 1 z 1} .
Teniendo en cuenta que el integrando en cordenadas cartesianas


F = y 2 + x2 + 0,queda:



ZZZ ZZZ
x2 + y 2 dxdydz

F dV =

Por la simetria del integrando y del dominio conviene aplicar el cambio


de
coordenadas cilndricas x = cos , y = sin , cuyo jacobiano es
J (, , z) = . Entonces la region transforma a


= {(, , z) /2 1, 1 z 1} y el integrando produce F =
2 ,luego.

ZZZ ZZZ
2 2
2 dddz
 
x +y dxdydz =

Z1 Z 2 Z 1
= 3 dddz
1 0 0
1 2 1
4
Z Z 
= dddz
1 0 4 0
=

Problema 2.
En primer lugar consideremos la siguiente parametrizacion de la Superfi-
cie


r (u, v) = (u cos v, u2 , u sin v) , donde = {(u, v) , 0 u 1, 0 v 2}
Determinemos el vector normal a la superficie


r u (u, v) = (cos v, 2u, sin v)


r v (u, v) = (u
sin v, 0, u cos v)
i j k



r u r v = cos v


2u sin v = (2u2 cos v, u, 2u2 sin v)
u sin v 0 u cos v
Entonces

591



ZZ ZZ
F n
bdS = F (
r (u, v)) (

r u

r v ) dudv
S
Z 2 Z 1
(0, u, 0) 2u2 cos v, u, 2u2 sin v dudv

=
Z0 2 Z0 1
= u2 dudv
0 0
2
=
3

Problema 3.
De acuerdo con el teorema de Stokes, debemos verificar que




I ZZ

F dr = F n bdS
C S

H

En primer lugar, calculemos F d
r
C
Determinemos la curva C que limita la Superficie abierta del paraboloide:
si z = 2 = x2 + y 2 = 4
Parametricemos la curva C, con una orientacion positiva, se tiene
r (t) =


(2 cos t, 2 sin t, 2) t [2, 0] y evaluemos el campo sobre la curva F ( r (t)) =
(6 sin t, 4 cos t, 8 sin t) , luego
0


I Z
F d 12 sin2 t + 8 cos2 t dt

r =
2
C
Z 2
8 + 4 sin2 t dt

=
Z0 2   
1 cos 2t
= 8+4 dt
0 2
 2
sin 2t
= 16 + 2 t
2 0
= 20

592
Calculemos, ahora



ZZ ZZ
F nbdS = F (
r y

r x ) dxdy
S R

Las ecuacion
 parametrica  de la superficie S en coordenadas cartesianas
2 2
x + y
es

r (x, y) = x, y, ,y el vector normal exterior es
2
i j k

n = r y
r x = 0 1 y = (x, y, 1)
1 0 x
Ademas
i j k


F = /x /y /z = (z + x, 0. (z + 3))
3y xz yz 2
Finalmente R es la proyeccion sobre el plano xy, R = {(x, y) IR2 /x2 + y 2 = 4} ,entonces

Z Z " 2 #
x2 + y 2 x2 + y 2
+ x, 0, 3 (x, y, 1) dxdy
2 2
R
ZZ "  2 2 #
x + y2 2 x2 + y 2
= x +x + + 3 dxdy
2 2
R

Haciendo un cambio a coordenadas polares x = cos , y = sin


Z 2 Z 2  
1 5 2 1 2
cos + cos + + 3 dd
0 0 4 2
Z 2 Z 2 Z 2
128 2
= cos d + 4 cos d + 8 d
28 0 0 0
Z 2
1 + cos 2
= 0+4 d + 16 = 4 + 16
0 2
= 20

Autoevaluacion No 3
Tiempo 2 horas
Pregunta 1
RR

Usar el teorema de Stokes para resolver, F nbdS , si la superficie
S


S esta dada por z = 1 x2 y 2 con z 0 y F = (y, z, x) .

593
Pregunta 2
Determinar el area de la superficie x2 + y 2 = 4, limitada por el plano
z = 0 y por la superficie z = 2x2 + y 2

Pauta Correcciones
Pregunta 1
El teorema de Stokes asegura que



ZZ I
F n bdS = F d
r
S C

donde C es la curva cerrada que limita S.


Sea C: 1 x2 y 2 = 0; z = 0 = x2 + y 2 = 1 corresponde a una
circunferencia en el plano xy centrada en el origen de radio 1.
Al parametrizar la curva considerando una orientacion positiva, produce

r (t) = (cos t, sin t, 0) t [2, 0] =

0
r (t) = ( sin t, cos t, 0) t [2, 0] =


F (
r (t)) = (sin t, 0, cos t)


Calculemos entonces la integral de lnea F de sobre la curva C

2


I Z
F d
r = [sin t ( sin t) + 0 + 0] dt
0
C
2 2  
1 cos 2t
Z Z
2
= sin tdt = dt
0 0 2
 2
t sin 2t
= =
2 4 0
Pregunta 2
Metodo 1
En primer lugar, consideremos la Superficie cilndrica parametrizada por


r (u, v) = (2 cos u, 2 sin u, v) , donde = {(u, v) , 0 u 2, 0 v 4 cos2 u + 4}
Determinemos el vector normal a la superficie


r u (u, v) = (2 sin u, 2 cos u, 0)


r v (u, v) = (0,
0, 1)
i j k


r u
r v = 2 sin u 2 cos u 0 = (2 cos u, 2 sin u, 0)
0 0 1

594
|

r u
r v| = 2
El area de la superficie es entonces
Z Z
A (S) = |

r u

r v | dudv

Z 2 Z 4 cos2 u+4
= 2dvdu
0 0
Z 2
4 cos2 u + 4 du

= 2
0
= 24

Metodo 2
El area de la superficie se puede determinar con la integral curvilnea
Z
A (S) = zds
C

Sea C: x2 + y 2 = 4 es a una circunferencia en el plano xy centrada en


el origen de radio 2. Al parametrizar la curva considerando una orientacion
positiva, produce

r (t) = (2 cos t, 2 sin t) , t [2, 0] =

0
r (t) = (2 sin t, 2 cos t) , t [2, 0] =
z (

r (t)) = 4 cos2 u + 4


Luego, el area de la superficie es

Z 2
4 cos2 u + 4 2dt

A (S) =
0
= 24

595
Bibliografa

[1] Tom M. Apostol, Calculus, Vol I, 2a edicion, Ed. Reverte, 1982


[2] George Arfken, Mathematical Methods for Physicists, Academic Press,
1985
[3] R.V. Churchill, Fourier Series and Boundary Value Problems, 2o ed.
Nueva York, Mac Graw Hill,1963
[4] Erwing Kreyszing, Matematicas Avanzadas para Ingeniera, Vol I y II,
Limusa, 1995
[5] R. Larson ,R.Hostetler, B. Edwars, Calculo con Geometra Analtica,
Vol 2, Mc Graw Hill, 1999.
[6] J.E. Marsden, A.J. Tromba, Calculo Vectorial, Addison Wesley Long-
man, 1998
[7] Peter V. ONeil, Matematicas Avanzadas para Ingeniera, Thomson,
2005
[8] Claudio Pita Ruiz, Calculo Vectorial, Prentice-Hall Hispanoamerica, SA,
1995
[9] George B. Thomas Jr., Calculo varias variables, Addison-Wesley, Pear-
son 2000S

596

También podría gustarte